Sie sind auf Seite 1von 133

TAXATION II ATTY.

ACOSTA -
CAJUSTIN

TOPIC: GROSS INCOME COMPENSATION it is the amount they deem is subject to tax. The excess is to be treated as
expense of the company.
FRINGE BENEFITS
2) The entrance fee should not be considered income since it is an expense of
[G.R. No. L-12954. February 28, 1961.] his employer, and membership therein is merely incidental to his duties of
increasing and sustaining the business of his employer.

COLLECTOR OF INTERNAL
REVENUE, petitioner, vs. ARTHUR 3) His wife merely accompanied him to New York on a business trip as his
HENDERSON, respondent. secretary, and at the employer-corporations request, for the wife to look at
details of the plans of a building that his employer intended to construct. Such
must not be considered taxable income.
[G.R. No. 13049. February 28, 1961.] The Collector of Internal Revenue merely allowed the entrance fee as
nontaxable. The rent expense and travel expenses were still held to be taxable.
ARTHUR HENDERSON, petitioner, vs. COLLECTOR The Court of Tax Appeals ruled in favor of the taxpayers, that such expenses
OF INTERNAL REVENUE, respondent. must not be considered part of taxable income. Letters of the wife while in New
York concerning the proposed building were presented as evidence.
ISSUE: Whether or not the rental allowances and travel allowances furnished
Rental allowances and travel allowances by a company are not part of and given by the employer-corporation are part of taxable income?
taxable income.
FACTS: HELD: NO. Such claims are substantially supported by evidence.
Sps. Arthur Henderson and Marie Henderson filed their annual income tax
These claims are therefore NOT part of taxable income. No part of the
with the BIR. Arthur is president of American International Underwriters for
allowances in question redounded to their personal benefit, nor were such
the Philippines, Inc., which is a domestic corporation engaged in the business
amounts retained by them. These bills were paid directly by the employer-
of general non-life insurance, and represents a group of American insurance
corporation to the creditors. The rental expenses and subsistence allowances
companies engaged in the business of general non-life insurance.
are to be considered not subject to income tax. Arthurs high executive position
and social standing, demanded and compelled the couple to live in a more
The BIR demanded payment for alleged deficiency taxes. In their spacious and expensive quarters. Such subsistence allowance was a
computation, the BIR included as part of taxable income: 1) Arthurs SEPARATE account from the account for salaries and wages of employees. The
allowances for rental, residential expenses, subsistence, water, electricity and company did not charge rentals as deductible from the salaries of the
telephone expenses 2) entrance fee to the Marikina Gun and Country Club employees. These expenses are COMPANY EXPENSES, not income by
which was paid by his employer for his account and 3) travelling allowance of employees which are subject to tax.
his wife
The taxpayers justifications are as follows:
SYLLABUS
1) as to allowances for rental and utilities, Arthur did not receive money for the
allowances. Instead, the apartment is furnished and paid for by his employer-
corporation (the mother company of American International), for the 1. TAXATION; INCOME TAXES; ALLOWANCES FOR BUSINESS
employer corporations purposes. The spouses had no choice but to live in the EXECUTIVE'S HOUSING EXPENSES; CASE AT BAR. The taxpayers in the
expensive apartment, since the company used it to entertain guests, to case at bar, are childless and there are only the two of them in the family.
accommodate officials, and to entertain customers. According to taxpayers, Although the quarters they occupied exceeded their personal needs, the
only P 4,800 per year is the reasonable amount that the spouses would be exigencies of husband-taxpayer's high executive position demanded and
spending on rental if they were not required to live in those apartments. Thus, compelled them to live in more spawning and pretentious quarters like the

1
TAXATION II ATTY. ACOSTA -
CAJUSTIN
ones they had occupied. They had to entertain and put up house-guests in their
apartments. This is the reason why the husband-taxpayer's employer- Amount subject to tax P29,605.83
corporation had to grant him allowance for rental and utilities in addition to 1952:
his annual basic salary to take care of those extra expenses for rental and Net Income P36,780.11
utilities in excess of their personal needs. The fact that the taxpayers had to live Less: Personal Exemption 3,000.00
or did not have to live in the apartment's chosen by the husband-taxpayer's
employer-corporation is of no moment, for no part of the allowances in Amount subject to tax P33,780.11
question redounded to their personal benefit or was retained by them. Their (Exhibits 1, 3, 5, 7, 9, A, F, J, N, R.). In due time the taxpayers received from
bills for rental and utilities were paid directly by the employer-corporation to the Bureau of Internal Revenue assessment notices Nos. 15840-48, 25450-
the creditors. Nevertheless, the taxpayers are entitled only to a ratable value of 49, 15255-50, 25705-51 and 22527-52 and paid the amounts assessed as
the allowances in question. Only the reasonable amount they would spent for follows:
house rental and utilities such as light, water, telephone, etc., should be subject
to tax. The excess should be considered as expenses of the corporation. 1948:
14 May 1949, O.R. No 52991,
PADILLA, J p:
Exhibit B P2,068.12
These are petitions filed by the Collector of Internal Revenue (G. R. No. L-12954)
and by Arthur Henderson (G. R. No. L-13049) under the provisions of Section 12 September 1949, O.R. No. 160473,
18, Republic Act No. 1125, for review of a judgment dated 26 June 1957 and a Exhibit B-1 2,068.11
resolution dated 28 September 1957 rendered and adopted by the Court of Tax
Appeals in Case No. 237. Total Paid P4,136.23
1949:
The spouses Arthur Henderson and Marie B. Henderson (later referred to as the 13 May 1950, O.R. No. 232366,
taxpayers) filed with the Bureau of Internal Revenue returns of annual net income Exhibit G P2,314.95
for the years 1948 to 1952, inclusive, where the following net incomes, personal 15 September 1950, O.R. No. 247918,
exemptions and amounts subject to tax appear: Exhibit G-1 2,314.94

1948: Total Paid 4,629.89
Net Income P29,573.79 1950:
Less: Personal Exemption 2,500.00 27 April 1951, O.R. No. 323173,
Exhibit K P7,273.00
Amount subject to tax P27,073.79 1951:
1949: Amount withheld from salary and
Net Income P31,817.66 paid by employer P5,780.40
Less: Personal Exemption 2,500.00 15 May 1952, O.R. No. 33250,
Exhibit O 360.50
Amount subject to tax P29,317.66 15 August 1952, O.R. No. No. 383318,
1950: Exhibit O-1 361.20
Net Income P34,815.74
Less: Personal Exemption 3,000.00 Total Paid P6,502.10
1952:
Amount subject to tax P31,815.74 Amount withheld from salary and
1951: paid by employer P5,660.40
Net Income P32,605.83 18 May 1953, O.R. No. 438026,
Less: Personal Exemption 3,000.00 Exhibit T 1,160.30

2
TAXATION II ATTY. ACOSTA -
CAJUSTIN
13 August 1953, O.R. No. 443483, Tax due thereon P8,292.21
Exhibit T-1 1,160.00 Less: tax already assessed paid
per OR Nos. 232366 & 247918 4,629.89
Total paid P7,981.00
Deficiency tax due P3,662.23
(Should be) 3,662.32
On 28 November 1953, after investigation and verification, the Bureau of
1950:
Internal Revenue reassessed the taxpayers' income for the years 1948 to
Net Income per return P34,815.74
1952, inclusive, as follows:
Add:
1948: Rent, electricity, water allowances 8,373.73
Net income per return P29,573.79
Add: Net Income per investigation P43,189.47
Rent expense 7,200.00 Less: Personal exemption 3,000.00
Additional bonus for 1947 received
May 13, 1948 6,500.00 Net taxable income P40,189.47
Other income: Tax due thereon P10,296.00
Manager's residential expense Less: Tax already paid per OR. #323173 7,273.00
(2/29/48 a/c/ 4.51) 1,400.00
Manager's residential expense Deficiency tax still due & assessable P3,023.00
(refer to 1948 P & L 1,849.32 =========
Entrance fee Marikina Gun & 1951:
Country Club 200.00 Net income per return P32,605.83
Add house rental allowance from AIU 5,782.91
Net income per investigation P46,723.11
Less: Personal exemption 2,500.00 Net income per investigation P38,388.74
Less: Personal exemptions 3,000.00
Net taxable income P44,223.11
Tax due thereon P8,562.47 Amount of income subject to tax P35,388.74
Less: Amount of tax already paid Tax due thereon P 8,560.00
per OR #52991 & 160473 4,136.23 Less: tax already assessed and paid
per OR Nos. A33250 & 383318 6,502.00
Deficiency tax still due & assessable
1949: Deficiency tax due P2,058.00
Net income per return P31,817.66 =========
Add disallowances 1952:
Capital loss (no capital gain) 3,248.84 Net income per return P36,780.11
Undeclared bonus 8,857.75 Add:
Rental allowance from A.I.U 1,800.00 Withholding tax paid by company 600.00
Subsistence allowance Traveling allowances 3,247.40
from A.I.U 6,051.50 P14,958.09 Allowances for rent, telephone,
water, electricity, etc. 7,044.67
Net income per investigation P46,775.75
Less: Personal exemption 2,500.00 Net income per investigation 47,672.18
Less: Personal exemption 3,000.00
Amount of income subject to tax P43,275.75

3
TAXATION II ATTY. ACOSTA -
CAJUSTIN
Net taxable income P44,672.18 deficiency taxes assessed under Official Receipts Nos. 451841, 451842, 451843,
Tax due thereon P12,089.00 451748 and 451844 (Exhibits C, I, M, Q and Y). After hearing conducted by the
Less: Tax already withheld P5,660.40 Conference Staff of the Bureau of Internal Revenue on 5 October 1954 (pp. 74-85,
Tax already paid per BIR rec.), on 27 May 1955 the Staff recommended to the Collector of Internal
O.R. Nos. 438026, 443484 2,320.00 7,981.00 Revenue that the assessments made on 28 November 1953, Exhibits 2, 4, 6, 8, 10)
be sustained except that the amount of P200 as entrance fee to the Marikina Gun
Deficiency tax still due & collectible P4,108.00 and Country Club paid for the husband-taxpayer's account by his employer in
======== 1948 should not be considered as part of the taxpayer's taxable income for that
year (pp. 95-107, BIR rec.). On 14 July 1955, in line with the recommendation of
the Conference Staff, the Collector of Internal Revenue denied the taxpayer's
(Exhibits 2, 4, 6, 8, 10) and demanded payment of the deficiency taxes on or
request for reconsideration, except as regards the assessment of their income tax
before 28 February 1954 with respect to those due for the years 1948, 1949,
due for the year 1948, which was modified as follows:
1950 and 1952 and on or before 15 February 1954 with respect to that due for
the year 1951 (Exhibits B-2, H, L, P, S).
In the foregoing assessments, the Bureau of Internal Revenue considered as part
Net income per return P29,573.79
of their taxable income the taxpayer-husband's allowances for rental, residential
Add: Rent expense 7,200.00
expenses, subsistence, water, electricity and telephone; bonus paid to him;
Additional bonus for 1947 received
withholding tax and entrance fee to the Marikina Gun and Country Club paid by
on May 13, 1948 6,500.00
his employer for his account; and travelling allowance of his wife. On 26 and 27
Manager's residential expense
January 1954 the taxpayers asked for reconsideration of the foregoing
(2/29/48 a/c No. 4.41) 1,400.00
assessments (pp. 29, 31, BIR rec.) and on 11 February 1964 and 28 February 1955
Manager's residential expense
stated the grounds and reasons in support of their request for reconsideration
(1948 profit and loss) 1,849.32
(pp. 36-38, 62-66, BIR rec.). They claim that as regards the husband-taxpayer's

allowances for rental and utilities such as water, electricity and telephone, he did
Net income per investigation P46,523.11
not receive the money for said allowances, but that they lived in the apartment
Less: Personal exemption 2,500.00
furnished and paid for by his employer for its convenience; that they had no

choice but live in the said apartment furnished by his employer, otherwise they
Net taxable income P44,023.11
would have lived in a less expensive one that as regards his allowances for rental
Tax due thereon P8,506.47
of P7,200 and residential expenses of P1,400 and P1,849.32 in 1948, rental of
Less: Amount already paid 4,136.23
P1,800 and subsistence of P6,051.50 (the latter merely consisting of allowances

for rent and utilities such as light, water, telephone, etc.) in 1949 rental, electricity
Deficiency tax still due P4,370.24
and water of P8,373.73 in 1950, rental of P5,782.91 in 1951 and rental, telephone,
water, electricity, etc. of P7,044.67 in 1952, only the amount of P3,900 for each
year, which is the amount they would have spent for rental of an apartment and demanded payment of the deficiency taxes of P4,370.24 for 1948,
including utilities, should be taxed; that as regards the amount of P200 P3,662.23 for 1949, P3,023 for 1950, P2,058 for 1951 and P4,108 for 1952,
representing entrance fee to the Marikina Gun and Country Club paid for him by 5% surcharge and 1% monthly interest thereon from 1 March 1954 to the date
his employer in 1948, the same should not be considered as part of their income of payment and P80 as administrative penalty for late payment, to the City
for it was an expense of his employer and his membership therein was merely Treasurer of Manila not later than 31 July 1955 (Exhibit 14). On 30 January
incidental to his duties of increasing and sustaining the business of his employer; 1956 the taxpayers again sought a reconsideration of the denial of their
and that as regards the wife-taxpayer's travelling allowance of P3,247.40 in 1952, request for reconsideration and offered to settle the case on a more equitable
it should not be considered as part of their income because she merely basis by increasing the amount of taxable portion of the husband-taxpayer's
accompanied him in his business trip to New York as his secretary and, at the allowances for rental, etc. from P3,000 yearly to P4,800 yearly, which "is the
behest of her husband's employer, to study and look into the details of the plans value to the employee of the benefits he derived therefrom measured by what
and decorations of the building intended to be constructed by his employer in its he had saved on account thereof' in the ordinary course of his life . . . for
property at Dewey Boulevard. On 15 and 27 February 1954, the taxpayers paid the which he would have spent in any case.' The taxpayers also reiterated their

4
TAXATION II ATTY. ACOSTA -
CAJUSTIN
previous stand regarding the transportation allowance of the wife taxpayer of I. The Court of Tax Appeals erred in finding that the herein
P3,247.40 in 1952 and requested the refund of the amounts of P3,477.18, respondent did not have any choice in the selection of the
P569.33, P1,294, P354 and P2,164, or a total of P7,858.51 (Exhibit Z). On 10 living quarters occupied by him.
February 1956 the taxpayers again requested the Collector of Internal
Revenue to refund to them the amounts allegedly paid in excess as income II. The Court of Tax Appeals erred in not considering the fact
taxes for the years 1948 to 1952, inclusive (Exhibit Z-1). The Collector of that respondent is not a minor company official but the
Internal Revenue did not take any action on the taxpayer's request for refund. President of his employer-corporation, in the appreciation of
respondent's alleged lack of choice in the matter of the
On 15 February 1956 the taxpayers filed in the Court of Tax Appeals a petition to selection of the quarters occupied by him.
review the decision of the Collector of Internal Revenue (C.T.A. Case No. 237).
After hearing, on 26 June 1957 the Court rendered judgment holding "that the III. The Court of Tax Appeals erred in giving full weight and
inherent nature of petitioner's (the husband-taxpayer) employment as president credence to respondent's allegation, a self-serving and
of the American International Underwriters of the Philippines, Inc. does not unsupported declaration that the ratable value to him of the
require him to occupy the apartments supplied by his employer-corporation;" living quarters and subsistence allowance was only P400.00
that, however, only the amount of P4,800 annually, the ratable value to him of the a month.
quarters furnished constitutes a part of taxable income; that since the taxpayers
did not receive any benefit out of the P3,247.40 travelling expense allowance IV. The Court of Tax Appeals erred in holding that only the
granted in 1952 to the wife-taxpayers and that she merely undertook the trip ratable value of P4,800.00 per annum, or P400.00 a month
abroad at the behest of her husband's employer, the same could not be considered constitutes income to respondent.
as income; and that even if it were considered as such, still it could not be subject V. The Court of Tax Appeals erred in arbitrarily fixing the
to tax because it was deductible as travel expense; and ordering the Collector of amount of P4,800.00 per annum, or P400.00 a month as the
Internal Revenue to refund to the taxpayers the amount of P5,109.33 with interest only amount taxable against respondent during the five tax
from 27 February 1954, without pronouncement as to costs. The taxpayers filed a years in question.
motion for reconsideration claiming that the amount of P5,986.61 is the amount
refundable to them because the amounts of P1,400 and P1,849,32 as manager's VI. The Court of Tax Appeals erred in not finding that
residential expenses in 1948 should not be included in their taxable net income travelling allowance in the amount of P3,247.40 constituted
for the reason that they are of the same nature as the rentals for the apartment, income to respondent and, therefore, subject to the income
they being mainly expenses for utilities as light, water and telephone in the tax.
apartment furnished by the husband-taxpayer's employer. The Collector of
Internal Revenue filed an opposition to their motion for reconsideration. He also VII. The Court of Tax Appeals erred in ordering the refund of
filed a separate motion for reconsideration of the decision claiming that his the sum of P5,109.33 with interest from February 17, 1954.
assessment under review was correct and should have been affirmed. The (G.R. No. L-12954.)
taxpayers filed an opposition to this motion for reconsideration of the Collector of
Internal Revenue; the latter, a reply thereto. On 28 September 1957 the Court The taxpayers have assigned the following errors allegedly Committed by the
denied both motions for reconsideration. On 7 October 1957 the Collector of Court of Tax Appeals:
Internal Revenue filed a notice of appeal in the Court of Tax Appeals and on 21 I. The Court of Tax Appeals erred in its computation of the
October 1957, within the extension of time previously granted by this Court, a 1948 income tax and consequently in the amount that should
petition for review (G.R. No. L-12954). On 29 October 1957 the taxpayers filed a be refunded for that year.
notice of appeal in the Court of Tax Appeals and a petition for review in this Court
(G.R. No. L-13049). II. The Court of Tax Appeals erred in denying our motion for
reconsideration as contained in its resolution dated
The Collector of Internal Revenue has assigned the following errors allegedly September 28, 1957. (G.R. No. L-13049.)
committed by the Court of Tax Appeals:
The Government's appeal:

5
TAXATION II ATTY. ACOSTA -
CAJUSTIN
The Collector of Internal Revenue raises questions of fact. He claims that the childless and are the only two in the family; that during the years 1948 to 1952,
evidence is not sufficient to support the findings and conclusion of the Court of they lived in apartments chosen by his employer; that from 1948 to the early part
Tax Appeals that the quarters occupied by the taxpayers were not of their choice of 1950, they lived at the Embassy Apartments on Dakota Street, Manila, where
but that of the husband- taxpayers employer; that it did not take into they had a large sala, three bedrooms, dining room, two bathrooms, kitchen and a
consideration the fact that the husband-taxpayer is not a mere minor company large porch, and from the early part of 1950 to 1952, they lived at the Rosaria
official, but the highest executive of his employer-corporation; and that the wife- Apartments on the same street where they had a kitchen, sala, dining room, two
taxpayer's trip abroad in 1952 was not, as found by the Court, a business but a bedrooms and bathrooms; that despite the fact that they were the only two in the
vacation trip. In Collector of Internal Revenue vs. Aznar, 102 Phil., 979; 56 Off. family, they had to live in apartments of the size beyond their personal needs
Gaz. 2386, this Court held that in petitions for review under Section 18,Republic because as president of the corporation, he and his wife had to entertain and put
Act No. 1125, it may review the findings of fact of the Court of Tax Appeals. up houseguests; that during all those years of 1948 to 1952, inclusive, they
entertained and put up houseguests of his company's officials, guests and
The determination of the main issue in the case requires a review of the evidence. customers such as the president of C. V. Starr & Company, Inc., who spent four
Are the allowances for rental of the apartment furnished by the husband- weeks in his apartment, Thomas Cocklin, a lawyer from Washington, D.C., and
taxpayer's employer-corporation, including utilities such as light, water, Manuel Elizalde, a stockholder of AIUPI; that were he not required by his
telephone, etc. and the allowance for travel expenses given by his employer- employer to live in those apartments furnished to him, he said his wife would
corporation to his wife in 1952 part of taxable income? Section have chosen an apartment only large enough for them and spend from P300 to
29, Commonwealth Act No. 466, National Internal Revenue Code, provides: P400 monthly for rental; that of the allowances granted to him, only the amount
of P4,800 annually, the maximum they would have spent for rental, should be
"Gross income" includes gains, profits, and income derived
considered as taxable income and the excess treated as expense of the company;
from salaries, wages, or compensation for personal service of
and that the trip to New York undertaken by his wife in 1952, for which she was
whatever kind and in whatever form paid, or from
granted by his employer-corporation travelling expense allowance of P3,247.40,
professions, vocations, trades, businesses, commerce, sales,
was made at the behest of his employer to assist its architect in the preparation of
or dealings in property, whether real or personal, growing out
the plans for a proposed building in Manila and procurement of supplies and
of the ownership or use of or interest, in such property; also
materials for its use, hence the said amount should not be considered as part of
from interest, rents, dividend, securities, or the transactions
taxable income. In support of his claim, letters written by his wife while in New
of any business carried on for gain or profit or gains, profits,
York concerning the proposed building, inquiring about the progress made in the
and income derived from any source whatever. (Emphasis
acquisition of the lot, and informing him of the wishes of Mr. C. V. Starr,
supplied.)
chairman of the board of directors of the parent-corporation (Exhibits U-1, U-1-A,
The Court of Tax Appeals found that the husband-taxpayer "is the president of the V, V-1 and W) and letter written by the witness to Mr. C. V. Starr concerning the
American International Underwriters for the Philippines, Inc., a domestic proposed building (Exhibits X, X-1) were presented in evidence.
corporation engaged in insurance business;" that the taxpayers "entertained
officials, guests and customers of his employer-corporation, in apartments
furnished by the latter and successively occupied by him as president thereof; that Mrs. Marie Henderson testified that for almost three years, she and her husband
"In 1952, petitioner's wife, Mrs. Marie Henderson, upon request of Mr. C.V. Starr, gave parties every Friday night at their apartment for about 18 to 20 people; that
chairman of the parent corporation of the American International Underwriters their guests were officials of her husband's employer-corporation and other
for the Philippines, Inc., undertook a trip to New York in connection with the corporations; that during those parties movies for the entertainment of the guests
purchase of a lot in Dewey Boulevard by petitioner's employer-corporation, the were shown after dinner; that they also entertained during luncheons and
construction of a building thereon, the drawing of prospectus and plans for said breakfast; that these involved and necessitated the services of additional servants;
building, and other related matters." and that in 1952 she was asked by Mr. C. V. Starr to come to New York to take up
problems concerning the proposed building and entertainment because her
Arthur H. Henderson testified that he is the President of American International
husband could not make the trip himself, and because "the woman of the family is
Underwriters for the Philippines, Inc., which represents a group of American
closer to those problems."
insurance companies engaged in the business of general insurance except life
insurance; that he receives a basic annual salary of P30,000 and allowances for The evidence presented at the hearing of the case substantially supports the
house rental and utilities like light, water, telephone, etc., that he and his wife are findings of the Court of Tax Appeals. The taxpayers are childless and are the only

6
TAXATION II ATTY. ACOSTA -
CAJUSTIN
two in the family. The quarters, therefore, that they occupied at the Embassy Mrs. Cresencia Perez Ramos, an examiner of the Bureau of Internal Revenue who
Apartments consisting of a large sala, three bedrooms, dining room, two examined the books of account of the American International Underwriters for
bathrooms, kitchen and a large porch, and at the Rosaria Apartments consisting the Philippines, Inc., testified that the total amount of P3,249.32 was reflected in
of a kitchen, sala, dining room, two bedrooms and a bathroom, exceeded their its books as "living expenses of Mr. and Mrs. Arthur Henderson in the quarters
personal needs. But the exigencies of the husband-taxpayer's high executive they occupied in 1948;" and that "the amount of P1,400 was included as
position, not to mention social standing, demanded and compelled them to live in manager's residential expense while the amount of P1,849.32 was entered as
a more spacious and pretentious quarters like the ones they had occupied. profit and loss account."
Although entertaining and putting up houseguests and guests of the husband-
taxpayer's employer-corporation were not his predominant occupation as Buenaventura Loberiza, acting head of the accounting department of the
president, yet he and his wife had to entertain and put up houseguests in their American International Underwriters for the Philippines, Inc., testified that
apartments. That is why his employer-corporation had to grant him allowances rentals utilities, water, telephone and electric bills of executives of the corporation
for rental and utilities in addition to his annual basic salary to take care of those were entered in the books of account as "subsistence allowances and expenses;"
extra expenses for rentals and utilities in excess of their personal needs. Hence, that there was a separate account for salaries and wages of employees and
the fact that the taxpayers had to live or did not have to live in the apartments officers; and that expenses for rentals and other utilities were not charged to
chosen by the husband-taxpayer's employer-corporation is of no moment, for no salary accounts.
part of the allowances in question redounded to their personal benefit or was
The taxpayers' claim is supported by the evidence. The total amount of P3,249.32
retained by them. Their bills for rental and utilities were paid directly by the
"for manager's residential expense" in 1948 should be treated as rentals for
employer- corporation to the creditors (Exhibits AA to DDD, inclusive; pp. 104,
apartments and utilities and should not form part of the ratable value subject to
170-193, t.s.n.). Nevertheless, as correctly held by the Court of Tax Appeals, the
tax.
taxpayers are entitled only to a ratable value of the allowances in question, and
only the amount of P4,800 annually, the reasonable amount they would have The computation made by the taxpayers is correct. Adding to the amount of
spent for house rental and utilities such as light, water, telephone, etc. should be P29,573.79, their net income per return, the amounts of P6,500, the bonus
the amount subject to tax, and the excess considered as expenses of the received in 1948, and P4,800, the taxable ratable value of the allowances, brings
corporation. up their gross income to P40,873.79. Deducting therefrom the amount of P2,500
for personal exemption, the amount of P38,373.79 is the amount subject to
Likewise, the findings of the Court of Tax Appeals that the wife- taxpayer had to
income tax. The income tax due on this amount is P6,957.19 only. Deducting the
make a trip to New York at the behest of her husband's employer-corporation to
amount of income tax due, P6,957.19, from the amount already paid, P8,562.47
help in drawing up the plans and specifications of a proposed building, is also
(Exhibits B, B-1, C), the amount of P1,605.28 is the amount refundable to the
supported by the evidence. The parts of the letters written by the wife-taxpayer to
taxpayers. Add this amount to P569.33, P1,294.00, P354.00 and P2,164.00,
her husband while in New York and the letter written by the husband- taxpayer to
refundable to the taxpayers for 1949, 1950, 1951 and 1952, and the total is
Mr. C. V. Starr support the said findings (Exhibits U-2, V-1, W-1, X). No part of
P5,986.61.
the allowance for travelling expenses redounded to the benefit of the taxpayers.
Neither was a part thereof retained by them. The fact that she had herself The judgment under review is modified as above indicated. The Collector of
operated on for tumors while in New York was but incidental to her stay there and Internal Revenue is ordered to refund to the taxpayers the sum of P5,986.61,
she must have merely taken advantage of her presence in that city to undergo the without pronouncement as to costs.
operation.
The taxpayers' appeal:
[G.R. No. 96016. October 17, 1991.]
The taxpayers claim that the Court of Tax Appeals erred in considering the
amount of P1,400 and P1,849.32, or a total of P3,249.32, for "manager's
residential expense" in 1948 as taxable income despite the fact "that they were of COMMISSIONER OF INTERNAL
the same nature as the rentals for the apartment, they being expenses for utilities, REVENUE, petitioner, vs. THE COURT OF APPEALS
such as light, water and telephone necessarily incidental to the apartment and EFREN P. CASTAEDA, respondents.
furnished to him by his employer."

7
TAXATION II ATTY. ACOSTA -
CAJUSTIN
The Court of Tax Appeals found for private respondent Castaeda and ordered
the Commissioner of Internal Revenue to refund Castaeda the sum of P12,557.13
SYLLABUS withheld as income tax. (Annex "C", petition). Cdpr
Petitioner appealed the above-mentioned Court of Tax Appeals decision to this
TAXATION; WITHHOLDING TAX; TERMINAL LEAVE PAY; NOT SUBJECT Court, which was docketed as G.R. No. 80320. In turn, we referred the case to the
THEREOF. The Court has already ruled that the terminal leave pay received Court of Appeals for resolution. The case was docketed in the Court of Appeals as
by a government official or employee is not subject to withholding (income) CA-G.R. SP No. 20482.
tax. In the recent case of Jesus N. Borromeo vs. The Hon. Civil Service On 26 September 1990, the Court of Appeals dismissed the petition for review
Commission, et al., G.R. No. 96032, 31 July 1991, the Court explained and affirmed the decision of the Court of Tax Appeals. Hence, the present
the rationale behind the employee's entitlement to an exemption from recourse by the Commissioner of Internal Revenue.
withholding (income) tax on his terminal leave pay as follows: . . .
commutation of leave credits, more commonly known as terminal leave, is The Solicitor General, acting on behalf of the Commissioner of Internal Revenue,
applied for by an officer or employee who retires, resigns or is separated from contends that the terminal leave pay is income derived from employer-employee
the service through no fault of his own. (Manual on Leave Administration relationship, citing in support of his stand Section 28 of the National Internal
Course for Effectiveness published by the Civil Service Commission, pages 16- Revenue Code; that as part of the compensation for services rendered, terminal
17). In the exercise of sound personnel policy, the Government encourages leave pay is actually part of gross income of the recipient. Thus
unused leaves to be accumulated. The Government recognizes that for most
public servants, retirement pay is always less than generous if not meager and " . . . . It (terminal leave pay) cannot be viewed as salary for
scrimpy. A modest nest egg which the senior citizen may look forward to is purposes which would reduce it. . . . there can thus be no
thus avoided. Terminal leave payments are given not only at the same time but 'commutation of salary' when a government retiree applies
also for the same policy considerations governing retirement benefits." for terminal leave -because he is not receiving it as salary.
What he applies for is a 'commutation of leave credits.' It is
an accumulation of credits intended for old age or separation
from service. . . . ."
The issue to be resolved in this petition for review on certiorari is whether or not
terminal leave pay received by a government official or employee on the occasion The Court has already ruled that the terminal leave pay received by a government
of his compulsory retirement from the government service is subject to official or employee is not subject to withholding (income) tax. In the recent case
withholding (income) tax. of Jesus N. Borromeo vs. The Hon. Civil Service Commission, et al., G.R. No.
96032, 31 July 1991, the Court explained the rationale behind the employee's
We resolve the issue in the negative. entitlement to an exemption from withholding (income) tax on his terminal leave
Private respondent Efren P. Castaeda retired from the government service as pay as follows:
Revenue Attache in the Philippine Embassy in London, England, on 10 December " . . . commutation of leave credits, more commonly known as
1982 under the provisions of Section 12 (c) of Commonwealth Act 186, as terminal leave, is applied for by an officer or employee who
amended. Upon retirement, he received, among other benefits, terminal leave pay retires, resigns or is separated from the service through no
from which petitioner Commissioner of Internal Revenue withheld P12,557.13 fault of his own. (Manual on Leave Administration Course for
allegedly representing income tax thereon. Effectiveness published by the Civil Service Commission,
Castaeda filed a formal written claim with petitioner for a refund of the pages 16-17). In the exercise of sound personnel policy, the
P12,557.13, contending that the cash equivalent of his terminal leave is exempt Government encourages unused leaves to be accumulated.
from income tax. To comply with the two-year prescriptive period within which The Government recognizes that for most public servants,
claims for refund may be filed, Castaeda filed on 16 July 1984 with the Court of retirement pay is always less than generous if not meager and
Tax Appeals a Petition for Review, seeking the refund of income tax withheld scrimpy. A modest nest egg which the senior citizen may look
from his terminal leave pay. forward to is thus avoided. Terminal leave payments are
given not only at the same time but also for the same policy
considerations governing retirement benefits."

8
TAXATION II ATTY. ACOSTA -
CAJUSTIN
In fine, not being part of the gross salary or income of a government official or finding sufficient evidence to overcome the prima facie correctness of the
employee but a retirement benefit, terminal leave pay is not subject to income questioned assessments. In a petition for review, the Court of Appeals affirmed
tax. cdphil the ruling of the CTA. Hence, the present petition. The issue is whether
ANSCOR's redemption of stocks from its stockholders as well as the exchange
ACCORDINGLY, the petition for review is hereby DENIED. of common shares can be considered as equivalent to the distribution of
SO ORDERED. taxable dividend making the proceeds thereof taxable under the provisions
Section 83 (B) of the 1939 Revenue Act. SHECcT
The Supreme Court modified the decision of the Court of Appeals in that
ANSCOR'S redemption of 82,752.5 stock dividends is herein considered as
essentially equivalent to a distribution of taxable dividends for which it is liable
TOPIC: GROSS INCOME DIVIDENDS for the withholding tax-at-source. While the Board Resolutions authorizing the
redemptions state only one purpose reduction of foreign exchange
[G.R. No. 108576. January 20, 1999.] remittances in case cash dividends are declared. Said purpose was not given
credence by the court in case at bar. Records show that despite the existence of
enormous corporate profits no cash dividends were ever declared by ANSCOR
COMMISSIONER OF INTERNAL from 1945 until the BIR started making assessments in the early 1970's.
REVENUE, petitioner, vs. THE COURT OF APPEALS, Although a corporation under certain exceptions, has the prerogative when to
COURT OF TAX APPEALS and A. SORIANO issue dividends, yet when no cash dividends are issued for about three decades,
CORP., respondents. this circumstance negate the legitimacy of ANSCOR's alleged purposes. With
regard to the exchange of shares, the Court ruled that the exchange of common
with preferred shares is not taxable because it produces no realized income to
the subscriber but only a modification of the subscriber's rights and privileges
which is not a flow of wealth for tax purposes. IHcTDA
SYNOPSIS

SYLLABUS
Don Andres Soriano, a citizen and resident of the United States formed in the
1930's the corporation "A Soriano Y Cia," predecessor of ANSCOR. On
December 30, 1964 Don Andres died. On June 30, 1968, pursuant to a Board 1.TAXATION; PRESIDENTIAL DECREE NO. 67; NOT BEING A TAXPAYER,
Resolution, ANSCOR redeemed 28,000 common shares from Don Andres' A WITHHOLDING AGENT LIKE THE PRIVATE RESPONDENT IS NOT
estate. By November 1968, the Board further increased ANSCOR's capital PROTECTED BY THE AMNESTY UNDER THE DECREE. An income
stock to P75M divided into 150,000 preferred shares and 600,000 common taxpayer covers all persons who derive taxable income. ANSCOR was assessed
shares. About a year later ANSCOR again redeemed 80,000 common shares by petitioner for deficiency withholding tax under Section 53 and 54 of the
from Don Andres' estate, further reducing the latter's common shareholdings. 1939 Code. As such, it is being held liable in its capacity as a withholding agent
ANSCOR's business purpose for both redemptions of stock is to partially retire and not in its personality as a taxpayer. In the operation of the withholding tax
said stocks as treasury shares in order to reduce the company's foreign system, the withholding agent is the payor, a separate entity acting no more
exchange remittances in case cash dividends are declared. In 1973, after than an agent of the government for the collection of the tax in order to ensure
examining ANSCOR's books of account and records Revenue Examiners issued its payments; the payer is the taxpayer he is the person subject to tax impose
a report proposing that ANSCOR be assessed for deficiency withholding tax-at- by law; and the payee is the taxing authority. In other words, the withholding
source, pursuant to Sections 53 and 54 of the 1939 Revenue Code for the year agent is merely a tax collector, not a taxpayer. Under the withholding system,
1968 and the second quarter of 1969 based on the transactions of exchange and however, the agent-payor becomes a payee by fiction of law. His (agent)
redemption of stocks. Subsequently, ANSCOR filed a petition for review with liability is direct and independent from the taxpayer, because the income tax is
the Court of Tax Appeals assailing the tax assessments on the redemptions and still impose on and due from the latter. The agent is not liable for the tax as no
exchange of stocks. In its decision, the CTA reversed the BIR's ruling after wealth flowed into him he earned no income. The Tax Code only makes the

9
TAXATION II ATTY. ACOSTA -
CAJUSTIN
agent personally liable for the tax arising from the breach of its legal duty to assuming arguendo, that those business purposes are legitimate, the same
withhold as distinguish from its duty to pay tax since: "the government's cause cannot be valid excuse for the imposition of tax. Otherwise, the taxpayer's
of action against the withholding agent is not for the collection of income tax, liability to pay income tax would be made to depend upon a third person who
but for the enforcement of the withholding provision of Section 53 of the Tax did not earn the income being taxed. Furthermore, even if the said purposes
Code,compliance with which is imposed on the withholding agent support the redemption and justify the issuance of stock dividends, the same
and not upon the taxpayer." Not being ataxpayer, a withholding agent, like has no bearing whatsoever on the imposition of the tax herein assessed
ANSCOR in this transaction is not protected by the amnesty under the decree. because the proceeds of the redemption are deemed taxable dividends since it
Codal provisions on withholding tax are mandatory and must be complied with was shown that income was generated therefrom. Thirdly, ANSCOR argued
by the withholding agent. The taxpayer should not answer for the non- that to treat as 'taxable dividend' the proceeds of the redeemed stock dividends
performance by the withholding agent of its legal duty to withhold unless there would be to impose on such stock an undisclosed lien and would be extremely
is collusion or bad faith. The former could not be deemed to have evaded the unfair to intervening purchasers, i.e. those who buys the stock dividends after
tax had the withholding agent performed its duty. This could be the situation their issuance. Such argument, however, bears no relevance in this case as no
for which the amnesty decree was intended. Thus, to curtail tax evasion and intervening buyer is involved. And even if there is an intervening buyer, it is
give tax evaders a chance to reform, it was deemed administratively feasible to necessary to look into the factual milieu of the case if income was realized from
grant tax amnesty in certain instances. In addition, a "tax amnesty, much like a the transaction. Again, we reiterate that the dividend equivalence test depends
tax exemption, is never favored nor presumed in law and if granted by a on such "time and manner" of the transaction and its net effect. The
statute, the terms of the amnesty like that of a tax exemption must be undisclosed lien may be unfair to a subsequent stock buyer who has no capital
construed strictly against the taxpayer and liberally in favor of the taxing interest in the company. But the unfairness may not be true to an original
authority." The rule on strictissimi juris equally applies. So that, any doubt in subscriber like Don Andres, who holds stock dividends as gains from his
the application of an amnesty law/decree should be resolved in favor of the investments. The subsequent buyer who buys stock dividends is investing
taxing authority. capital. It just so happen that what he bought is stock dividends. The effect of
its (stock dividends) redemption from that subsequent buyer is merely to
2.ID.; NATIONAL INTERNAL REVENUE CODE OF 1939; TAX ON STOCK return his capital subscription, which is income if redeemed from the original
DIVIDENDS; REDEMPTION AND CANCELLATION; PURPOSES INVOKED subscriber. After considering the manner and the circumstances by which the
BY PRIVATE RESPONDENT CORPORATION, UNDER THE FACTS OF THE issuance and redemption of stock dividends were made, there is no other
PRESENT CASE ARE NO EXCUSE FOR ITS TAX LIABILITY; REASON. conclusion but that the proceeds thereof are essentially considered equivalent
First, the alleged "filipinization" plan cannot be considered legitimate as it was to a distribution of taxable dividends. As "taxable dividend" under Section
not implemented until the BIR started making assessments on the proceeds of 83(b), it is part of the "entire income" subject to tax under Section 22 in
the redemption. Such corporate plan was not stated in nor supported by any relation to Section 21 of the 1939 Code. Moreover, under Section 29(a) of said
Board Resolution but a mere afterthought interposed by the counsel of Code, dividends are included in "gross income." As income, it is subject to
ANSCOR. Being a separate entity, the corporation can act only through its income tax which is required to be withheld at source. The 1997 Tax Code may
Board of Directors. The Board Resolutions authorizing the redemptions state have altered the situation but it does not change this disposition.
only one purpose reduction of foreign exchange remittances in case cash
dividends are declared. Not even this purpose can be given credence. Records
show that despite the existence of enormous corporate profits no cash dividend
was ever declared by ANSCOR from 1945 until the BIR started making 3.ID.; ID.; ID.; THE EXCHANGE OF COMMON WITH PREFERRED SHARES
assessments in the early 1970's. Although a corporation under certain IN CASE AT BAR IS NOT TAXABLE; IT PRODUCES NO REALIZED INCOME
exceptions, has the prerogative when to issue dividends, yet when no cash TO THE SUBSCRIBER BUT ONLY A MODIFICATION OF THE
dividends was issued for about three decades, this circumstances negates the SUBSCRIBER'S RIGHTS AND PRIVILEGES WHICH IS NOT A FLOW OF
legitimacy of ANSCOR's alleged purposes. Moreover, to issue stock dividends WEALTH FOR TAX PURPOSES. Both the Tax Court and the Court of
is to increase the shareholdings of ANSCOR's foreign stockholders contrary to Appeals found that ANSCOR reclassified its shares into common and
its "filipinization" plan. This would also increase rather than reduce their need preferred, and that parts of the common shares of the Don Andres estate and
for foreign exchange remittances in case of cash dividend declaration, all of Doa Carmen's shares were exchanged for the whole 150,000 preferred
considering that ANSCOR is a family corporation where the majority shares at shares. Thereafter, both the Don Andres estate and Doa Carmen remained as
the time of redemptions were held by Don Andres' foreign heirs. Secondly, corporate subscribers except that their subscriptions now include preferred
shares. There was no change in their proportional interest after the exchange.

10
TAXATION II ATTY. ACOSTA -
CAJUSTIN
There was no cash flow. Both stocks had the same par value. Under the facts Don Andres, after the other stockholders waived in favor of the former their pre-
herein, any difference in their market value would be immaterial at the time of emptive rights to subscribe to the new issues. 6 This increased his subscription to
exchange because no income is yet realized it was a mere corporate paper 14,963 common shares. 7 A month later, 8 Don Andres transferred 1,250 shares
transaction. It would have been different, if the exchange transaction resulted each to his two sons, Jose and Andres, Jr., as their initial investments in
into a flow of wealth, in which case income tax may be imposed. ANSCOR. 9 Both sons are foreigners. 10
Reclassification of shares does not always bring any substantial alteration in
the subscriber's proportional interest. But the exchange is different there By 1947, ANSCOR declared stock dividends. Other stock dividend declarations
would be a shifting of the balance of stock features, like priority in dividend were made between 1949 and December 20, 1963. 11 On December 30, 1964 Don
declarations or absence of voting rights. Yet neither the reclassification nor Andres died. As of that date, the records revealed that he has a total shareholdings
exchange per se, yields realize income for tax purposes. A common stock of 185,154 shares 12 50,495 of which are original issues and the balance of
represents the residual ownership interest in the corporation. It is a basic class 134,659 shares as stock dividend declarations. 13 Correspondingly, one-half of
of stock ordinarily and usually issued without extraordinary rights or privileges that shareholdings or 92,577 14 shares were transferred to his wife, Doa Carmen
and entitles the shareholder to a pro ratadivision of profits. Preferred stocks Soriano, as her conjugal share. The other half formed part of his estate. 15
are those which entitle the shareholder to some priority on dividends and asset A day after Don Andres died, ANSCOR increased its capital stock to P20M 16 and
distribution. Both shares are part of the corporation's capital stock. Both in 1966 further increased it to P30M. 17In the same year (December 1966), stock
stockholders are no different from ordinary investors who take on the same dividends worth 46,290 and 46,287 shares were respectively received by the Don
investment risks. Preferred and common shareholders participate in the same Andres estate 18 and Doa Carmen from ANSCOR. Hence, increasing their
venture, willing to share in the profits and losses of the enterprise. Moreover, accumulated shareholdings to 138,867 and 138,864 19 common shares each. 20
under the doctrine of equality of shares all stocks issued by the corporation
are presumed equal with the same privileges and liabilities, provided that the On December 28, 1967, Doa Carmen requested a ruling from the United States
Articles of Incorporation is silent on such differences. In this case, the Internal Revenue Service (IRS), inquiring if an exchange of common with
exchange of shares, without more, produces no realized income to the preferred shares may be considered as a tax avoidance scheme 21 underSection
subscriber. There is only a modification of the subscriber's rights and privileges 367 of the 1954 U.S. Revenue Act. 22 By January 2, 1968, ANSCOR reclassified its
which is not a flow of wealth for tax purposes. The issue of taxable dividend existing 300,000 common shares into 150,000 common and 150,000 preferred
may arise only once a subscriber disposes of his entire interest and not when shares. 23
there is still maintenance of proprietary interest.
In a letter-reply dated February 1968, the IRS opined that the exchange is only a
Petitioner Commissioner of Internal Revenue (CIR) seeks the reversal of the recapitalization scheme and not tax avoidance. 24 Consequently, 25 on March 31,
decision of the Court of Appeals (CA) 1which affirmed the ruling of the Court of 1968 Doa Carmen exchanged her whole 138,864 common shares for 138,860 of
Tax Appeals (CTA) 2 that private respondent A. Soriano Corporation's the newly reclassified preferred shares. The estate of Don Andres in turn,
(hereinafter ANSCOR) redemption and exchange of the stocks of its foreign exchanged 11,140 of its common shares, for the remaining 11,140 preferred
stockholders cannot be considered as "essentially equivalent to a distribution of shares, thus reducing its (the estate) common shares to 127,727. 26
taxable dividends" under Section 83(b) of the 1939 Internal Revenue Act. 3
On June 30, 1968, pursuant to a Board Resolution, ANSCOR redeemed 28,000
The undisputed facts are as follows: cdlex common shares from the Don Andres' estate. By November 1968, the Board
further increased ANSCOR's capital stock to P75M divided into 150,000 preferred
Sometime in the 1930s, Don Andres Soriano, a citizen and resident of the United shares and 600,000 common shares. 27 About a year later, ANSCOR again
States, formed the corporation "A. Soriano Y Cia", predecessor of ANSCOR, with a redeemed 80,000 common shares from the Don Andres' estate, 28 further
P1,000,000.00 capitalization divided into 10,000 common shares at a par value reducing the latter's common shareholdings to 19,727. As stated in the Board
of P100/share. ANSCOR is wholly owned and controlled by the family of Don Resolutions, ANSCOR's business purpose for both redemptions of stocks is to
Andres, who are all non-resident aliens. 4 In 1937, Don Andres subscribed to partially retire said stocks as treasury shares in order to reduce the company's
4,963 shares of the 5,000 shares originally issued. 5 foreign exchange remittances in case cash dividends are declared. 29
On September 12, 1945, ANSCOR's authorized capital stock was increased to In 1973, after examining ANSCOR's books of account and records, Revenue
P2,500,000.00 divided into 25,000 common shares with the same par value. Of examiners issued a report proposing that ANSCOR be assessed for deficiency
the additional 15,000 shares, only 10,000 was issued which were all subscribed by withholding tax-at-source, pursuant to Sections 53 and 54 of the 1939 Revenue

11
TAXATION II ATTY. ACOSTA -
CAJUSTIN
Code, 30 for the year 1968 and the second quarter of 1969 based on the ANSCOR, however, avers that it has no duty to withhold any tax either from the
transactions of exchange and redemption of stocks. 31 The Bureau of Internal Don Andres estate or from Doa Carmen based on the two transactions, because
Revenue (BIR) made the corresponding assessments despite the claim of the same were done for legitimate business purposes which are (a) to reduce its
ANSCOR that it availed of the tax amnesty under Presidential Decree (P.D.) foreign exchange remittances in the event the company would declare cash
23 32 which were amended by P.D.'s 67 and 157. 33However, petitioner ruled dividends, 40 and to (b) subsequently "filipinized" ownership of ANSCOR, as
that the invoked decrees do not cover Sections 53 and 54 in relation to Article allegedly envisioned by Don Andres. 41 It likewise invoked the amnesty
83(b) of the 1939 Revenue Act under which ANSCOR was assessed. 34 ANSCOR's provisions of P.D. 67.
subsequent protest on the assessments was denied in 1983 by petitioner. 35
Subsequently, ANSCOR filed a petition for review with the CTA assailing the tax
assessments on the redemptions and exchange of stocks. In its decision, the Tax We must emphasize that the application of Sec. 83(b) depends on the special
Court reversed petitioner's ruling, after finding sufficient evidence to overcome factual circumstances of each case. 42 The findings of facts of a special court
the prima facie correctness of the questioned assessments. 36 In a petition for (CTA) exercising particular expertise on the subject of tax, generally binds this
review, the CA, as mentioned, affirmed the ruling of the CTA. 37 Hence, this Court,43 considering that it is substantially similar to the findings of the CA
petition. cdll which is the final arbiter of questions of facts. 44The issue in this case does not
only deal with facts but whether the law applies to a particular set of facts.
The bone of contention is the interpretation and application of Section 83(b) of Moreover, this Court is not necessarily bound by the lower courts' conclusions of
the 1939 Revenue Act 38 which provides: law drawn from such facts. 45
"Sec. 83.Distribution of dividends or assets by corporations. AMNESTY:

We will deal first with the issue of tax amnesty. Section 1 of P.D. 67 46 provides:
(b)Stock dividends A stock dividend representing the
"1.In all cases of voluntary disclosures of previously untaxed
transfer of surplus to capital account shall not be subject to
income and/or wealth such as earnings, receipts, gifts,
tax. However, if a corporation cancels or redeems stock
bequests or any other acquisitions from any source
issued as a dividend at such time and in such manner as to
whatsoever which are taxable under the National Internal
make the distribution and cancellation or redemption, in
Revenue Code, as amended, realized here or abroad by
whole or in part, essentially equivalent to the distribution of
any taxpayer, natural or juridical; the collection of all
a taxable dividend, the amount so distributed in redemption
internal revenue taxes including the increments or penalties
or cancellation of the stock shall be considered astaxable
or account of non-payment as well as all civil, criminal or
income to the extent it represents a distribution of earnings
administrative liabilities arising from or incident to such
or profits accumulated after March first, nineteen hundred
disclosures under the National Internal Revenue Code, the
and thirteen." (Italics supplied).
Revised Penal Code, the Anti-Graft and Corrupt Practices
Specifically, the issue is whether ANSCOR's redemption of stocks from its Act, the Revised Administrative Code, the Civil Service laws
stockholder as well as the exchange of common with preferred shares can be and regulations, laws and regulations on Immigration and
considered as "essentially equivalent to the distribution of taxable dividend," Deportation, or any other applicable law or proclamation, are
making the proceeds thereof taxable under the provisions of the above- hereby condoned and, in lieu thereof, a tax of ten (10%) per
quoted law. centum on such previously untaxed income or wealth is
hereby imposed, subject to the following conditions:
Petitioner contends that the exchange transaction is tantamount to "cancellation'' (conditions omitted) [Emphasis supplied].
under Section 83(b) making the proceeds thereof taxable. It also argues that the
said Section applies to stock dividends which is the bulk of stocks that ANSCOR The decree condones "the collection of all internal revenue taxes including
redeemed. Further, petitioner claims that under the "net effect test," the estate of the increments or penalties or account of non-payment as well as all civil,
Don Andres gained from the redemption. Accordingly, it was the duty of ANSCOR criminal or administrative liabilities arising from or incident to" (voluntary)
to withhold the tax-at-source arising from the two transactions, pursuant to disclosures under the NIRC of previously untaxed income and/or wealth
Section 53 and 54 of the 1939 Revenue Act. 39 "realized here or abroad by any taxpayer, natural or juridical."

12
TAXATION II ATTY. ACOSTA -
CAJUSTIN
May the withholding agent, in such capacity, be deemed a taxpayer for it to avail "Section 4.Cases not covered by amnesty. The following
of the amnesty? An income taxpayer covers all persons who derive taxable cases are not covered by the amnesty subject of these
income. 47 ANSCOR was assessed by petitioner for deficiency withholding tax regulations:
under Sections 53 and 54 of the 1939 Code. As such, it is being held liable in its
capacity as a withholding agent and not in its personality as a taxpayer. xxx xxx xxx

In the operation of the withholding tax system, the withholding agent is the payor, (2)Tax liabilities with or without assessments, on withholding
a separate entity acting no more than an agent of the government for the tax at source provided under Sections 53 and 54 of
collection of the tax 48 in order to ensure its payments; 49 the payer is the the National Internal Revenue Code, as amended; 59
taxpayer he is the person subject to tax imposed by law; 50 and the payee is the
ANSCOR was assessed under Sections 53 and 54 of the 1939 Tax Code.Thus,
taxing authority. 51 In other words, the withholding agent is merely a tax
by specific provision of law, it is not covered by the amnesty.
collector, not a taxpayer. Under the withholding system, however, the agent-payor
becomes a payee by fiction of law. His (agent) liability is direct and independent TAX ON STOCK DIVIDENDS
from the taxpayer, 52 because the income tax is still imposed on and due from the General Rule
latter. The agent is not liable for the tax as no wealth flowed into him he earned
no income. The Tax Code only makes the agent personally liable for the Section 83(b) of the 1939 NIRC was taken from Section 115(g)(1) of the U.S.
tax 53 arising from the breach of its legal duty to withhold as distinguished from Revenue Code of 1928. 60 It laid down the general rule known as the
its duty to pay tax since: 'proportionate test' 61 wherein stock dividends once issued form part of the
capital and, thus, subject to income tax. 62 Specifically, the general rule states
"the government's cause of action against the withholding agent is not for the that:
collection of income tax, but for the enforcement of the withholding provision of
Section 53 of the Tax Code,compliance with which is imposed on the withholding "A stock dividend representing the transfer of surplus to capital account shall not
agent and notupon the taxpayer." 54 be subject to tax." cdasia

Not being a taxpayer, a withholding agent, like ANSCOR in this transaction, Having been derived from a foreign law, resort to the jurisprudence of its origin
is not protected by the amnesty under the decree. may shed light. Under the US Revenue Code, this provision originally referred to
"stock dividends" only, without any exception. Stock dividends, strictly speaking,
Codal provisions on withholding tax are mandatory and must be complied with by represent capital and do not constitute income to its recipient. 63 So that the
the withholding agent. 55 The taxpayer should not answer for the non- mere issuance thereof is not yet subject to income tax 64 as they are nothing but
performance by the withholding agent of its legal duty to withhold unless there is an "enrichment through increase in value of capital investment." 65 As capital,
collusion or bad faith. The former could not be deemed to have evaded the tax had the stock dividends postpone the realization of profits because the "fund
the withholding agent performed its duty. This could be the situation for which represented by the new stock has been transferred from surplus to capital and no
the amnesty decree was intended. Thus, to curtail tax evasion and give tax evaders longer available for actual distribution." 66 Income in tax law is "an amount of
a chance to reform, 56 it was deemed administratively feasible to grant tax money coming to a person within a specified time, whether as payment for
amnesty in certain instances. In addition, a "tax amnesty, much like a tax services, interest, or profit from investment." 67 It means cash or its
exemption, is never favored nor presumed in law and if granted by a statute, the equivalent. 68 It is gain derived and severed from capital, 69 from labor or from
terms of the amnesty like that of a tax exemption must be construed strictly both combined 70 so that to tax a stock dividend would be to tax a capital
against the taxpayer and liberally in favor of the taxing authority." 57 The rule increase rather than the income. 71 In a loose sense, stock dividends issued by the
on strictissimi juris equally applies. 58 So that, any doubt in the application of an corporation, are considered unrealized gain, and cannot be subjected to income
amnesty law/decree should be resolved in favor of the taxing authority. tax until that gain has been realized. Before the realization, stock dividends are
Furthermore, ANSCOR's claim of amnesty cannot prosper. The implementing nothing but a representation of an interest in the corporate properties. 72 As
rules of P.D. 370 which expanded amnesty on previously untaxed income capital, it is not yet subject to income tax. It should be noted that capital and
under P.D. 23 is very explicit, to wit: income are different. Capital is wealth or fund; whereas income is profit or gain or
the flow of wealth. 73 The determining factor for the imposition of income tax is
whether any gain or profit was derived from a transaction. 74

13
TAXATION II ATTY. ACOSTA -
CAJUSTIN
The Exception As qualified by the phrase "such time and in such manner," the exception was not
intended to characterize as taxable dividend every distribution of earnings arising
"However, if a corporation cancels or redeems stock issued as a dividend at such
from the redemption of stock dividends. 81 So that, whether the amount
time and in such manner as to make thedistribution and cancellation or
distributed in the redemption should be treated as the equivalent of a "taxable
redemption, in whole or in part, essentially equivalent to the distribution of a
dividend" is a question of fact, 8 2 which is determinable on "the basis of the
taxable dividend, the amount so distributed in redemption or cancellation of the
particular facts of the transaction in question." 83 No decisive test can be used to
stock shall be considered as taxable income to the extent it represents a
determine the application of the exemption under Section 83(b). The use of the
distribution of earnings or profits accumulated after March first, nineteen
words "such manner" and "essentially equivalent" negative any idea that a
hundred and thirteen." (Emphasis supplied).
weighted formula can resolve a crucial issue Should the distribution be treated
In a response to the ruling of the American Supreme Court in the case of Eisner as taxable dividend. 84 On this aspect, American courts developed certain
v. Macomber 75 (that pro rata stock dividends are not taxable income), the recognized criteria, which includes the following: 85
exempting clause above quoted was added because corporations found a loophole
1)the presence or absence of real business purpose,
in the original provision. They resorted to devious means to circumvent the law
and evade the tax. Corporate earnings would be distributed under the guise of its 2)the amount of earnings and profits available for the
initial capitalization by declaring the stock dividends previously issued and later declaration of a regular dividend and the
redeem said dividends by paying cash to the stockholder. This process of corporation's past record with respect to the
issuance-redemption amounts to a distribution of taxable cash dividends which declaration of dividends,
was just delayed so as to escape the tax. It becomes a convenient technical
strategy to avoid the effects of taxation. 3)the effect of the distribution as compared with the
declaration of regular dividend,
Thus, to plug the loophole the exempting clause was added. It provides that the
redemption or cancellation of stock dividends, depending on the "time" and 4)the lapse of time between issuance and redemption, 86
"manner" it was made, is "essentially equivalent to a distribution of taxable
dividends," making the proceeds thereof "taxable income" "to the extent it 5)the presence of a substantial surplus 87 and a generous
represents profits". The exception was designed to prevent the issuance and supply of cash which invites suspicion as does a
cancellation or redemption of stock dividends, which is fundamentally not meager policy in relation both to current earnings
taxable, from being made use of as a device for the actual distribution of cash and accumulated surplus, 88
dividends, which is taxable. 76 Thus, cdtai REDEMPTION AND CANCELLATION
"the provision had the obvious purpose of preventing a corporation from avoiding For the exempting clause of Section 83(b) to apply, it is indispensable that: (a)
dividend tax treatment by distributing earnings to its shareholders in two there is redemption or cancellation; (b) the transaction involves stock dividends
transactions a pro rata stock dividend followed by a pro rata redemption and (c) the "time and manner" of the transaction makes it "essentially equivalent
that would have the same economic consequences as a simple dividend." 77 to a distribution of taxable dividends." Of these, the most important is the
third. cda
Redemption is repurchase, a reacquisition of stock by a corporation which issued
Although redemption and cancellation are generally considered capital
the stock 89 in exchange for property, whether or not the acquired stock is
transactions, as such, they are not subject to tax. However, it does not
cancelled, retired or held in the treasury. 90 Essentially, the corporation gets
necessarily mean that a shareholder may not realize a taxable gain from such
back some of its stock, distributes cash or property to the shareholder in payment
transactions.78 Simply put, depending on the circumstances, the proceeds of
for the stock, and continues in business as before. The redemption of stock
redemption of stock dividends are essentially distribution of cash dividends,
dividends previously issued is used as a veil for the constructive distribution of
which when paid becomes the absolute property of the stockholder.
cash dividends. In the instant case, there is no dispute that
Thereafter, the latter becomes the exclusive owner thereof and can exercise
ANSCOR redeemed shares of stocks from a stockholder (Don Andres) twice
the freedom of choice. 79 Having realized gain from that redemption, the
(28,000 and 80,000 common shares). But where did the shares redeemed come
income earner cannot escape income tax. 80
from? If its source is the original capital subscriptions upon establishment of the
corporation or from initial capital investment in an existing enterprise, its

14
TAXATION II ATTY. ACOSTA -
CAJUSTIN
redemption to the concurrent value of acquisition may not invite the application consummation of a preconceived plan, not to reorganize a
of Sec. 83(b) under the 1939 Tax Code,as it is not income but a mere return of business or any part of a business, but to transfer a parcel of
capital. On the contrary, if the redeemed shares are from stock dividend corporate shares to a stockholder." 102
declarations other than as initial capital investment, the proceeds of the
redemption is additional wealth, for it is not merely a return of capital but a gain Depending on each case, the exempting provision of Sec. 83(b) of the 1939 Code
thereon. may not be applicable if the redeemed shares were issued with bona
fide business purpose, 103 which is judged after each and every step of the
It is not the stock dividends but the proceeds of its redemption that may be transaction have been considered and the whole transaction does not amount to a
deemed as taxable dividends. Here, it is undisputed that at the time of the last tax evasion scheme.
redemption, the original common shares owned by the estate were only
25,247.5. 91 This means that from the total of 108,000 shares redeemed from the ANSCOR invoked two reasons to justify the redemptions (1) the
estate, the balance of 82,752.5 (108,000 less 25,247.5) must have come alleged "filipinization" program and (2) the reduction of foreign exchange
from stock dividends. Besides, in the absence of evidence to the contrary, theTax remittances in case cash dividends are declared. The Court is not concerned
Code presumes that every distribution of corporate property, in whole or in part, with the wisdom of these purposes but on their relevance to the whole
is made out of corporate profits,92 such as stock dividends. The capital cannot be transaction which can be inferred from the outcome thereof. Again, it is the
distributed in the form of redemption of stock dividends without violating the "net effect rather than the motives and plans of the taxpayer or his
trust fund doctrine wherein the capital stock, property and other assets of the corporation" 104 that is the fundamental guide in administering Sec. 83(b).
corporation are regarded as equity in trust for the payment of the corporate This tax provision is aimed at the result. 105 It also applies even if at the
creditors. 93 Once capital, it is always capital. 94 That doctrine was intended for time of the issuance of the stock dividend, there was no intention to redeem it
the protection of corporate creditors. 95 as a means of distributing profit or avoiding tax on dividends. 106 The
existence of legitimate business purposes in support of the redemptionof
With respect to the third requisite, ANSCOR redeemed stock dividends issued just stock dividends is immaterial in income taxation. It has no relevance in
2 to 3 years earlier. The time alone that lapsed from the issuance to the determining "dividend equivalence". 107Such purposes may be material only
redemption is not a sufficient indicator to determine taxability. It is a must to upon the issuance of the stock dividends. The test of taxability under the
consider the factual circumstances as to the manner of both the issuance and the exempting clause, when it provides "such time and manner" as would make
redemption. The "time" element is a factor to show a device to evade tax and the the redemption "essentially equivalent to the distribution of a taxable
scheme of cancelling or redeeming the same shares is a method usually adopted dividend", is whether the redemption resulted into a flow of wealth. If no
to accomplish the end sought. 96 Was this transaction used as a "continuing wealth is realized from the redemption, there may not be a dividend
plan," "device" or "artifice" to evade payment of tax? It is necessary to determine equivalence treatment. In the metaphor of Eisner v.Macomber, income is not
the "net effect" of the transaction between the shareholder-income taxpayer and deemed "realize" until the fruit has fallen or been plucked from the tree.
the acquiring (redeeming) corporation. 97 The "net effect" test is not evidence or
testimony to be considered; it is rather an inference to be drawn or a conclusion The three elements in the imposition of income tax are: (1) there must be gain or
to be reached. 98 It is also important to know whether the issuance of stock profit, (2) that the gain or profit is realized or received, actually or
dividends was dictated by legitimate business reasons, the presence of which constructively, 108 and (3) it is not exempted by law or treaty from income tax.
might negate a tax evasion plan. 99 lexlib Any business purpose as to why or how the income was earned by the taxpayer is
not a requirement. Income tax is assessed on income received from any property,
The issuance of stock dividends and its subsequent redemption must be separate, activity or service that produces the income because the Tax Codestands as an
distinct, and not related, for the redemption to be considered a legitimate tax indifferent neutral party on the matter of where income comes from. 109
scheme. 100 Redemption cannot be used as a cloak to distribute corporate
earnings. 101 Otherwise, the apparent intention to avoid tax becomes doubtful as As stated above, the test of taxability under the exempting clause of Section 83(b)
the intention to evade becomes manifest. It has been ruled that: is, whether income was realized through the redemption of stock dividends. The
redemption converts into money the stock dividends which become a realized
"[A]n operation with no business or corporate purpose is a profit or gain and consequently, the stockholder's separate property. 110 Profits
mere devise which put on the form of a corporate derived from the capital invested cannot escape income tax. As realized income,
reorganization as a disguise for concealing its real character, the proceeds of the redeemed stock dividends can be reached by income taxation
and the sole object and accomplishment of which was the regardless of the existence of any business purpose for the redemption.

15
TAXATION II ATTY. ACOSTA -
CAJUSTIN
Otherwise, to rule that the said proceeds are exempt from income tax when the assessments in the early 1970's. Although a corporation under certain exceptions,
redemption is supported by legitimate business reasons would defeat the very has the prerogative when to issue dividends, yet when no cash dividends was
purpose of imposing tax on income. Such argument would open the door for issued for about three decades, this circumstance negates the legitimacy of
income earners not to pay tax so long as the person from whom the income was ANSCOR's alleged purposes. Moreover, to issue stock dividends is to increase the
derived has legitimate business reasons. In other words, the payment of tax under shareholdings of ANSCOR's foreign stockholders contrary to its "filipinization"
the exempting clause of Section 83(b) would be made to depend not on the plan. This would also increase rather than reduce their need for foreign exchange
income of the taxpayer, but on the business purposes of a third party (the remittances in case of cash dividend declaration, considering that ANSCOR is a
corporation herein) from whom the income was earned. This is absurd, illogical family corporation where the majority shares at the time of redemptions were
and impractical considering that the Bureau of Internal Revenue (BIR) would be held by Don Andres' foreign heirs.
pestered with instances in determining the legitimacy of business reasons that
every income earner may interpose. It is not administratively feasible and cannot Secondly, assuming arguendo, that those business purposes are legitimate, the
therefore be allowed. cdasia same cannot be a valid excuse for the imposition of tax. Otherwise, the taxpayer's
liability to pay income tax would be made to depend upon a third person who did
not earn the income being taxed. Furthermore, even if the said purposes support
the redemption and justify the issuance of stock dividends, the same has no
The ruling in the American cases cited and relied upon by ANSCOR that "the bearing whatsoever on the imposition of the tax herein assessed because the
redeemed shares are the equivalent of dividend only if the shares were proceeds of the redemption are deemed taxable dividends since it was shown that
not issued for genuine business purposes", 111 or the "redeemed shares have income was generated therefrom.
beenissued by a corporation bona fide" 112 bears no relevance in determining the
non-taxability of the proceeds of redemption. ANSCOR, relying heavily and Thirdly, ANSCOR argued that to treat as 'taxable dividend' the proceeds of the
applying said cases, argued that so long as the redemption is supported by valid redeemed stock dividends would be to impose on such stock an undisclosed lien
corporate purposes the proceeds are not subject to tax. 113 The adoption by the and would be extremely unfair to intervening purchasers, i.e. those who buys the
courts below 114 of such argument is misleading if not misplaced. A review of the stock dividends after their issuance. 118 Such argument, however, bears no
cited American cases shows that the presence or absence of "genuine business relevance in this case as no intervening buyer is involved. And even if there is an
purposes" may be material with respect to the issuance or declaration of stock intervening buyer, it is necessary to look into the factual milieu of the case if
dividends but not on its subsequent redemption. The issuance and the income was realized from the transaction. Again, we reiterate that the dividend
redemption of stocks are two different transactions. Although the existence of equivalence test depends on such "time and manner" of the transaction and its
legitimate corporate purposes may justify a corporation's acquisition of its own net effect. The undisclosed lien 119 may be unfair to a subsequent stock buyer
shares under Section 41 ofthe Corporation Code, 115 such purposes cannot excuse who has no capital interest in the company. But the unfairness may not be true to
the stockholder from the effects of taxation arising from the redemption. If the an original subscriber like Don Andres, who holds stock dividends as gains from
issuance of stock dividends is part of a tax evasion plan and thus, without his investments. The subsequent buyer who buys stock dividends is investing
legitimate business reasons, the redemption becomes suspicious which may call capital. It just so happen that what he bought is stock dividends. The effect of its
for the application of the exempting clause. The substance of the whole (stock dividends) redemption from that subsequent buyer is merely to return his
transaction, not its form, usually controls the tax consequences. 116 capital subscription, which is income if redeemed from the original
subscriber. cdasia
The two purposes invoked by ANSCOR, under the facts of this case are no excuse
for its tax liability. First, the alleged "filipinization" plan cannot be considered After considering the manner and the circumstances by which the issuance and
legitimate as it was not implemented until the BIR started making assessments on redemption of stock dividends were made, there is no other conclusion but that
the proceeds of the redemption. Such corporate plan was not stated in nor the proceeds thereof are essentially considered equivalent to a distribution of
supported by any Board Resolution but a mere afterthought interposed by the taxable dividends. As "taxable dividend" under Section 83(b), it is part of the
counsel of ANSCOR. Being a separate entity, the corporation can act only through "entire income" subject to tax underSection 22 in relation to Section 21 120 of the
its Board of Directors. 117 The Board Resolutions authorizing the redemptions 1939 Code. Moreover, under Section 29(a) of said Code, dividends are included in
state only one purpose reduction of foreign exchange remittances in case cash "gross income". As income, it is subject to income tax which is required to be
dividends are declared. Not even this purpose can be given credence. Records withheld at source. The 1997 Tax Code may have altered the situation but it does
show that despite the existence of enormous corporate profits no cash dividend not change this disposition.
was ever declared by ANSCOR from 1945 until the BIR started making

16
TAXATION II ATTY. ACOSTA -
CAJUSTIN
EXCHANGE OF COMMON WITH PREFERRED SHARES 121 dividend may arise only once a subscriber disposes of his entire interest and not
when there is still maintenance of proprietary interest. 130
Exchange is an act of taking or giving one thing for another 122 involving
reciprocal transfer 123 and is generally considered as a taxable transaction. The WHEREFORE, premises considered, the decision of the Court of Appeals is
exchange of common stocks with preferred stocks, or preferred for common or a MODIFIED in that ANSCOR's redemption of 82,752.5 stock dividends is herein
combination of either for both, may not produce a recognized gain or loss, so long considered as essentially equivalent to a distribution of taxable dividends for
as the provisions of Section 83(b) is not applicable. This is true in a trade between which it is LIABLE for the withholding tax-at-source. The decision is AFFIRMED
two (2) persons as well as a trade between a stockholder and a corporation. In in all other respects.
general, this trade must be parts of merger, transfer to controlled corporation,
corporate acquisitions or corporate reorganizations. No taxable gain or loss may SO ORDERED.
be recognized on exchange of property, stock or securities related to
reorganizations. 124 Eisner vs Macomber, 252 US 89
Facts:
Both the Tax Court and the Court of Appeals found that ANSCOR reclassified its
shares into common and preferred, and that parts of the common shares of the
Don Andres estate and all of Doa Carmen's shares were exchanged for the whole On January 1, 1916, the Standard Oil Company of California, a
150,000 preferred shares. Thereafter, both the Don Andres estate and Doa corporation of that state, out of an authorized capital stock of
Carmen remained as corporate subscribers except that their subscriptions now $100,000, 000, had shares of stock outstanding, par value $100
include preferred shares. There was no change in their proportional interest after each, amounting in round figures to $50,000,000. In addition, it
the exchange. There was no cash flow. Both stocks had the same par value. Under had surplus and undivided profits invested in plant, property,
the facts herein, any difference in their market value would be immaterial at the and business and required for the purposes of the corporation,
time of exchange because no income is yet realized it was a mere corporate
paper transaction. It would have been different, if the exchange transaction
amounting to about $45,000,000, of which about $20,000,000
resulted into a flow of wealth, in which case income tax may be imposed. 125 had been earned prior to March 1, 1913, the balance thereafter.
In January, 1916, in order to readjust the capitalization, the
Reclassification of shares does not always bring any substantial alteration in the board of directors decided to issue additional shares sufficient to
subscriber's proportional interest. But the exchange is different there would be constitute a stock dividend of 50 per cent. of the outstanding
a shifting of the balance of stock features, like priority in dividend declarations or
stock, and to transfer from surplus account to capital stock
absence of voting rights. Yet neither the reclassification nor exchange per se,
yields realize income for tax purposes. A common stock represents the residual account an amount equivalent to such issue. Appropriate
ownership interest in the corporation. It is a basic class of stock ordinarily and resolutions were adopted, an amount equivalent to the par
usually issued without extraordinary rights or privileges and entitles the value of the proposed new stock was transferred accordingly,
shareholder to a pro ratadivision of profits. 126 Preferred stocks are those which and the new stock duly issued against it and divided among the
entitle the shareholder to some priority on dividends and asset distribution. 127 stockholders.
Both shares are part of the corporation's capital stock. Both stockholders are no
different from ordinary investors who take on the same investment risks. Defendant in error, being the owner of 2,200 shares of the old
Preferred and common shareholders participate in the same venture, willing to stock, received certificates for 1,100 additional [252 U.S. 189,
share in the profits and losses of the enterprise. 128 Moreover, under the doctrine 201] shares, of which 18.07 per cent., or 198.77 shares, par
of equality of shares all stocks issued by the corporation are presumed equal value $19,877, were treated as representing surplus earned
with the same privileges and liabilities, provided that the Articles of Incorporation
between March 1, 1913, and January 1, 1916. She was called
is silent on such differences. 129 cdasia
upon to pay, and did pay under protest, a tax imposed under
In this case, the exchange of shares, without more, produces no realized income to the Revenue Act of 1916, based upon a supposed income of $
the subscriber. There is only a modification of the subscriber's rights and 19,877 because of the new shares; and an appeal to the
privileges which is not a flow of wealth for tax purposes. The issue of taxable Commissioner of Internal Revenue having been disallowed, she

17
TAXATION II ATTY. ACOSTA -
CAJUSTIN
brought action against the Collector to recover the tax. In her use and benefit; on the contrary, every dollar of his original
complaint she alleged the above facts, and contended that in investment, together with whatever accretions and
imposing such a tax the Revenue Act of 1916 violated article 1, accumulations have resulted from employment of his money
2, cl. 3, and article 1, 9, cl. 4, of the Constitution of the United and that of the other stockholders in the business of the
States, requiring direct taxes to be apportioned according to company, still remains the property of the company, and subject
population, and that the stock dividend was not income within to business risks which may result in wiping out the entire
the meaning of the Sixteenth Amendment. A general demurrer investment. Having regard to the very truth of the matter, to
to the complaint was overruled upon the authority of Towne v. substance and not to form, he has recived nothing that answers
Eisner, 245 U.S. 418, 38 Sup. Ct. 158, L. R. A. 1918D, 254; and, the definition of income within the meaning of the Sixteenth
defendant having failed to plead further, final judgment went Amendment.
against him. To review it, the present writ of error is prosecuted.

Issue: Whether in legal or accounting terms the stock dividend


was to be regarded as a taxable event.

Held:

The Supreme Court held that a stock dividend of common on


common was not constitutionally taxable as income, because
the dividend represented a capitalization of surplus, rather than
a distribution of corporate profits to the stockholders. Not only
does a stock dividend really take nothing from the property of
the corporation and add nothing to that of the shareholder, but
that the antecedent accumulation of profits evidenced thereby,
while indicating that the shareholder is richer because of an
increase of his capital, at the same time shows he has not
realized or received any income in the transaction.

The fact that the dividend was charged against profits earned
before the act of 1913 took effect, even before the amendment
was adopted, was neither relied upon nor alluded to in our
consideration of the merits in that case. Not only so, but had we
considered that a stock dividend constituted income in any true
sense, it would have been held taxable under the act of 1913
notwithstanding it was based upon profits earned before the
amendment.

The essential and controlling fact is that the stockholder has


received nothing out of the company's assets for his separate

18
TAXATION II ATTY. ACOSTA -
CAJUSTIN
[G.R. No. 96016. October 17, 1991.]

COMMISSIONER OF INTERNAL
REVENUE, petitioner, vs. THE COURT OF APPEALS
and EFREN P. CASTAEDA, respondents.

SYLLABUS

TAXATION; WITHHOLDING TAX; TERMINAL LEAVE PAY; NOT SUBJECT


THEREOF. The Court has already ruled that the terminal leave pay received
by a government official or employee is not subject to withholding (income)
tax. In the recent case of Jesus N. Borromeo vs. The Hon. Civil Service
Commission, et al., G.R. No. 96032, 31 July 1991, the Court explained
the rationale behind the employee's entitlement to an exemption from
withholding (income) tax on his terminal leave pay as follows: . . . commutation
of leave credits, more commonly known as terminal leave, is applied for by an
officer or employee who retires, resigns or is separated from the service
through no fault of his own. (Manual on Leave Administration Course for
Effectiveness published by the Civil Service Commission, pages 16-17). In the
exercise of sound personnel policy, the Government encourages unused leaves
to be accumulated. The Government recognizes that for most public servants,
retirement pay is always less than generous if not meager and scrimpy. A
modest nest egg which the senior citizen may look forward to is thus avoided.
Terminal leave payments are given not only at the same time but also for the
same policy considerations governing retirement benefits."

TOPIC: GROSS INCOME EXCLUSION


FROM GROSS INCOME
*** SAME AS CASE #2 UNDER FRINGE BENEFITS

19
TAXATION II ATTY. ACOSTA -
CAJUSTIN

PROFESSIONAL EXPENSES
TEST OF DEDUCTIBILITY

[G.R. Nos. L-28508-9. July 7, 1989.]

ESSO STANDARD EASTERN, INC., (formerly,


Standard-Vacuum Oil Company), petitioner, vs. THE
COMMISSIONER OF INTERNAL
REVENUE, respondent.

For an item to be deductible as a business expense, the expense must be


ordinary and necessary; it must be paid or incurred within the taxable year;
and it must be paid or incurred in carrying on a trade or business. In
addition, the taxpayer must substantially prove by evidence or records the
deductions claimed under law, otherwise, the same will be disallowed.

FACTS:
ESSO deducted from its gross income for 1959, as part of its ordinary and
necessary business expenses, the amount it had spent for drilling and
exploration of its petroleum concessions. The Commissioner disallowed the
claim on the ground that the expenses should be capitalized and might be
written off as a loss only when a dry hole should result. Hence, ESSO filed an
amended return where it asked for the refund of P323,270 by reason of its
abandonment, as dry holes, of several of its oil wells. It also claimed as
ordinary and necessary expenses in the same return amount representing
margin fees it had paid to the Central Bank on its profit remittances to its New
York Office.

ISSUE: Whether the margin fees may be considered ordinary and necessary
expenses when paid.
TOPIC: DEDUCTIONS
ORDINARY AND NECESSARY HELD:
TRADE, BUSINESS OR For an item to be deductible as a business expense, the expense must be
ordinary and necessary; it must be paid or incurred within the taxable year;

20
TAXATION II ATTY. ACOSTA -
CAJUSTIN
(2) Whether or not the margin fees are necessary and ordinary business
expenses.
and it must be paid or incurred in carrying on a trade or business. In addition,
the taxpayer must substantially prove by evidence or records the deductions
Held:
claimed under law, otherwise, the same will be disallowed. There has been no
(1) No. A tax is levied to provide revenue for government operations, while the
attempt to define ordinary and necessary with precision. However, as guiding
proceeds of the margin fee are applied to strengthen our country's
principle in the proper adjudication of conflicting claims, an expenses is
international reserves. The margin fee was imposed by the State in the exercise
considered necessary where the expenditure is appropriate and helpful in the
of its police power and not the power of taxation.
development of the taxpayers business. It is ordinary when it connotes a
payment which is normal in relation to the business of the taxpayer and the
(2) No. Ordinarily, an expense will be considered 'necessary' where the
surrounding circumstances. Assuming that the expenditure is ordinary and
expenditure is appropriate and helpful in the development of the taxpayer's
necessary in the operation of the taxpayers business; the expenditure, to be an
business. It is 'ordinary' when it connotes a payment which is normal in
allowable deduction as a business expense, must be determined from the
relation to the business of the taxpayer and the surrounding circumstances.
nature of the expenditure itself, and on the extent and permanency of the work
Since the margin fees in question were incurred for the remittance of funds to
accomplished by the expenditure. Herein, ESSO has not shown that the
Esso's Head Office in New York, which is a separate and distinct income
remittance to the head office of part of its profits was made in furtherance of its
taxpayer from thebranch in the Philippines, for its disposal abroad, it can
own trade or business. The petitioner merely presumed that all corporate
never be said therefore that the margin fees were appropriate and helpful in
expenses are necessary and appropriate in the absence of a showing that they
the development of Esso's business in the Philippines exclusively or were
are illegal or ultra vires; which is erroneous. Claims for deductions are a matter
incurred for purposes proper to the conduct of the affairs of Esso's branch in
of legislative grace and do not turn on mere equitable considerations.
the Philippines exclusively or for the purpose of realizing a profit or of
Facts: In CTA Case No. 1251, Esso Standard Eastern Inc. (Esso)deducted from minimizing a loss in the Philippines exclusively. If at all, the margin fees were
its gross income for 1959, as part of its ordinary and necessary business incurred for purposes proper to the conduct of the corporate affairs of Esso in
expenses, the amount it had spent for drilling and exploration of its petroleum New York, but certainlynot in the Philippines.
concessions. This claim was disallowed by the Commissioner of Internal
Revenue (CIR) on the ground that the expenses should be capitalized and
might be written off as a loss only when a "dry hole" should result. Esso then SYLLABUS
filed an amended return where it asked for the refund of P323,279.00 by
reason of its abandonment as dry holes of several of its oil wells. Also claimed
as ordinary and necessary expenses in the same return was the amount of 1.STATUTORY CONSTRUCTION; LEGISLATIVE HISTORY OF AN ACT
P340,822.04, representing margin fees it had paid to the Central Bank on its RESORTED TO ONLY WHERE THE LANGUAGE OF THE STATUTE IS
profit remittances to its New York head office. AMBIGUOUS. Only in extremely doubtful matters of interpretation does the
legislative history of an act of Congress become important. As a matter of fact,
On August 5, 1964, the CIR granted a tax credit of P221,033.00 only, there may be no resort to the legislative history of the enactment of a statute,
disallowing the claimed deduction for the margin fees paid on the ground that the language of which is plain and unambiguous, since such legislative history
the margin fees paid to the Central Bank could not be considered taxes or may only be resorted to for the purpose of solving doubt, not for the purpose of
allowed as deductible business expenses. creating it. [50 Am. Jur. 328.]
2.TAXATION; REPUBLIC ACT NO. 2009, MARGIN FEE; NOT A TAX BUT
Esso appealed to the Court of Tax Appeals (CTA) for the refund of the margin
AN EXACTION. A margin fee is not a tax but an exaction designed to curb
fees it had earlier paid contending that the margin fees were deductible from
the excessive demands upon our international reserve. (Caltex [Phil.] Inc. v.
gross income either as a tax or as an ordinary and necessary business expense.
Acting Commissioner of Customs, 22 SCRA 779; Chamber of Agriculture and
However, Essos appeal was denied.
Natural Resources of the Philippines v. Central Bank, 14 SCRA 630).
Issues: 3.ID.; ID.; AN EXERCISE OF POLICE POWER. The margin fee
(1) Whether or not the margin fees are taxes. under Republic Act No. 2009 was imposed by the State in the exercise of its

21
TAXATION II ATTY. ACOSTA -
CAJUSTIN
police power and not the power of taxation. same return was the amount of P340,822.04, representing margin fees it had paid
to the Central Bank on its profit remittances to its New York head office.
4.ID.; ID.; NOT A DEDUCTIBLE EXPENSE; REASON. The fees were paid
for the remittance by ESSO as part of the profits to the head office in the On August 5, 1964, the CIR granted a tax credit of P221,033.00 only, disallowing
United States. Such remittance was an expenditure necessary and proper for the claimed deduction for the margin fees paid.
the conduct of its corporate affairs. As stated in the Lopez case, the margin fees
are not expenses in connection with the production or earning of petitioner's In CTA Case No. 1558, the CR assessed ESSO a deficiency income tax for the year
incomes in the Philippines. They were expenses incurred in the disposition of 1960, in the amount of P367,994.00, plus 18% interest thereon of P66,238.92 for
said incomes; expenses for the remittance of funds after they have already been the period from April 18, 1961 to April 18, 1964, for a total of P434,232.92. The
earned by petitioner's branch in the Philippines for the disposal of its Head deficiency arose from the disallowance of the margin fees of P1,226,647.72 paid
Office in New York which is already another distinct and separate income by ESSO to the Central Bank on its profit remittances to its New York head
taxpayer. office. LibLex

5.ID.; NATIONAL INTERNAL REVENUE CODE; INCOME TAX ON ESSO settled this deficiency assessment on August 10, 1964, by applying the tax
BUSINESS; CONDITIONS FOR DEDUCTIBILITY OF EXPENSE. We come, credit of P221,033.00 representing its overpayment on its income tax for 1959
then, to the statutory test of deductibility where it is axiomatic that to be and paying under protest the additional amount of P213,201.92. On August 13,
deductible as a business expense, three conditions are imposed, namely: (1) the 1964, it claimed the refund of P39,787.94 as overpayment on the interest on its
expense must be ordinary and necessary, (2) it must be paid or incurred within deficiency income tax. It argued that the 18% interest should have been imposed
the taxable year, and (3) it must be paid or incurred in carrying on a trade or not on the total deficiency of P367,944.00 but only on the amount of P146,961.00,
business. In addition, not only must the taxpayer meet the business test, he the difference between the total deficiency and its tax credit of P221,033.00.
must substantially prove by evidence or records the deductions claimed under This claim was denied by the CIR, who insisted on charging the 18% interest on
the law, otherwise, the same will be disallowed. The mere allegation of the the entire amount of the deficiency tax. On May 4, 1965, the CIR also denied the
taxpayer that an item of expense is ordinary and necessary does not justify its claims of ESSO for refund of the overpayment of its 1959 and 1960 income taxes,
deduction. (Atlas Consolidated Mining and Development Corporation v. holding that the margin fees paid to the Central Bank could not be considered
Commissioner of Internal Revenue, 102 SCRA 246) taxes or allowed as deductible business expenses.
6.ID.; ID.; CLAIMS FOR DEDUCTIONS, A MATTER OF LEGISLATIVE ESSO appealed to the CTA and sought the refund of P102,246.00 for 1959,
GRACE AND CONSTRUED STRICTLY AGAINST THE TAXPAYER. The contending that the margin fees were deductible from gross income either as a tax
paramount rule is that claims for deductions are a matter of legislative grace or as an ordinary and necessary business expense. It also claimed an overpayment
and do not turn on mere equitable considerations. . . . The taxpayer in every of its tax by P434,232.92 in 1960, for the same reason. Additionally, ESSO argued
instance has the burden of justifying the allowance of any deduction claimed that even if the amount paid as margin fees were not legally deductible, there was
On appeal before us is the decision of the Court of Tax Appeals 1 denying still an overpayment by P39,787.94 for 1960, representing excess interest.
petitioner's claims for refund of overpaid income taxes of P102,246.00 for 1959 After trial, the CTA denied petitioner's claim for refund of P102,246.00 for 1959
and P434,234.93 for 1960 in CTA Cases No. 1251 and 1558 respectively. and P434,234.92 for 1960 but sustained its claim for P39,787.94 as excess
I interest. This portion of the decision was appealed by the CIR but was affirmed by
this Court in Commissioner of Internal Revenue v. ESSO, G.R. No. L-28502-03,
In CTA Case No. 1251, petitioner ESSO deducted from its gross income for 1959, promulgated on April 18, 1989. ESSO for its part appealed the CTA decision
as part of its ordinary and necessary business expenses, the amount it had spent denying its claims for the refund of the margin fees P102,246.00 for 1959 and
for drilling and exploration of its petroleum concessions. This claim was P434,234.92 for 1960. That is the issue now before us.
disallowed by the respondent Commissioner of Internal Revenue on the ground
that the expenses should be capitalized and might be written off as a loss only II
when a "dry hole" should result. ESSO then filed an amended return where it The first question we must settle is whether R.A. 2009, entitled An Act to
asked for the refund of P323,279.00 by reason of its abandonment as dry holes of Authorize the Central Bank of the Philippines to Establish a Margin Over Banks'
several of its oil wells. Also claimed as ordinary and necessary expenses in the Selling Rates of Foreign Exchange, is a police measure or a revenue measure. If it
is a revenue measure, the margin fees paid by the petitioner to the Central Bank

22
TAXATION II ATTY. ACOSTA -
CAJUSTIN
on its profit remittances to its New York head office should be deductible from encourage exports, and ultimately, `curtail any excessive
ESSO's gross income under Sec. 30(c) of the National Internal Revenue Code. demand upon the international reserve' in order to stabilize
This provides that all taxes paid or accrued during or within the taxable year and the currency. Originally adopted to cope with balance of
which are related to the taxpayer's trade, business or profession are deductible payment pressures, exchange restrictions have come to serve
from gross income. various purposes, such as limiting non-essential imports,
protecting domestic industry and when combined with the
The petitioner maintains that margin fees are taxes and cites the background and use of multiple currency rates providing a source of
legislative history of the Margin Fee Law showing that R.A. 2609 was nothing less revenue to the government, and are in many developing
than a revival of the 17% excise tax on foreign exchange imposed by R.A. 601. This countries regarded as a more or less inevitable concomitant
was a revenue measure formally proposed by President Carlos P. Garcia to of their economic development programs. The different
Congress as part of, and in order to balance, the budget for 1959-1960. It was measures of exchange control or restriction cover different
enacted by Congress as such and, significantly, properly originated in the House phases of foreign exchange transactions, i.e., in quantitative
of Representatives. During its two and a half years of existence, the measure was restriction, the control is on the amount of foreign exchange
one of the major sources of revenue used to finance the ordinary operating allowable. In the case of the margin levy, the immediate
expenditures of the government. It was, moreover, payable out of the General impact is on the rate of foreign exchange; in fact, its main
Fund. function is to control the exchange rate without changing the
par value of the peso as fixed in the Bretton Woods
On the claimed legislative intent, the Court of Tax Appeals, quoting established
Agreement Act. For a member nation is not supposed to alter
principles, pointed out that
its exchange rate (at par value) to correct a merely temporary
We are not unmindful of the rule that opinions expressed in disequilibrium in its balance of payments. By its nature, the
debates, actual proceedings of the legislature, steps taken in margin levy is part of the rate of exchange as fixed by the
the enactment of a law, or the history of the passage of the government.
law through the legislature, may be resorted to as an aid in
the interpretation of a statute which is ambiguous or of
doubtful meaning. The courts may take into consideration As to the contention that the margin levy is a tax on the
the facts leading up to, confident with, and in any way purchase of foreign exchange and hence should not form part
connected with, the passage of the act, in order that they may of the exchange rate, suffice it to state that We have already
properly interpret the legislative intent. But it is also well- held the contrary for the reason that a tax is levied to provide
settled jurisprudence that only in extremely doubtful matters revenue for government operations, while the proceeds of the
of interpretation does the legislative history of an act of margin fee are applied to strengthen our country's
Congress become important. As a matter of fact, there may be international reserves.
no resort to the legislative history of the enactment of a
statute, the language of which is plain and unambiguous, Earlier, in Chamber of Agriculture and Natural Resources of the Philippines v.
since such legislative history may only be resorted to for the Central Bank, 3 the same idea was expressed, though in connection with a
purpose of solving doubt, not for the purpose of creating it. different levy, through Justice J.B.L. Reyes:
[50 Am. Jur. 328.]
Neither do we find merit in the argument that the 20%
Apart from the above consideration, there are at least two cases where we have retention of exporter's foreign exchange constitutes an export
held that a margin fee is not a tax but an exaction designed to curb the excessive tax. A tax is a levy for the purpose of providing revenue for
demands upon our international reserve. government operations, while the proceeds of the 20%
retention, as we have seen, are applied to strengthen the
In Caltex (Phil.) Inc. v. Acting Commissioner of Customs, 2 the Court stated Central Bank's international reserve.
through Justice Jose P. Bengzon:
We conclude then that the margin fee was imposed by the State in the exercise of
A margin levy on foreign exchange is a form of exchange its police power and not the power of taxation.
control or restriction designed to discourage imports and

23
TAXATION II ATTY. ACOSTA -
CAJUSTIN
Alternatively, ESSO prays that if margin fees are not taxes, they should in order to be deductible under this section of the statute,
nevertheless be considered necessary and ordinary business expenses and must fall squarely within its language.
therefore still deductible from its gross income. The fees were paid for the
remittance by ESSO as part of the profits to the head office in the United States. We come, then, to the statutory test of deductibility where it
Such remittance was an expenditure necessary and proper for the conduct of its is axiomatic that to be deductible as a business expense, three
corporate affairs. conditions are imposed, namely: (1) the expense must be
ordinary and necessary, (2) it must be paid or incurred within
The applicable provision is Section 30(a) of the National Internal Revenue Code the taxable year, and (3) it must be paid or incurred in
reading as follows: carrying on a trade or business. In addition, not only must
the taxpayer meet the business test, he must substantially
SEC. 30.Deductions from gross income. In computing net prove by evidence or records the deductions claimed under
income there shall be allowed as deductions the law, otherwise, the same will be disallowed. The mere
allegation of the taxpayer that an item of expense is ordinary
(a)Expenses:
and necessary does not justify its deduction.
(1)In general. All the ordinary and necessary expenses
While it is true that there is a number of decisions in the
paid or incurred during the taxable year in carrying on any
United States delving on the interpretation of the terms
trade or business, including a reasonable allowance for
'ordinary and necessary, as used in the federal tax laws, no
salaries or other compensation for personal services actually
adequate or satisfactory definition of those terms is possible.
rendered; traveling expenses while away from home in the
Similarly, this Court has never attempted to define with
pursuit of a trade or business; and rentals or other payments
precision the terms 'ordinary and necessary.' There are
required to be made as a condition to the continued use or
however, certain guiding principles worthy of serious
possession, for the purpose of the trade or business, of
consideration in the proper adjudication of conflicting
property to which the taxpayer has not taken or is not taking
claims. Ordinarily, an expense will be considered `necessary,
title or in which he has no equity.
where the expenditure is appropriate and helpful in the
(2)Expenses allowable to non-resident alien individuals and development of the taxpayer's business. It is 'ordinary' when
foreign corporations. In the case of a non-resident alien it connotes a payment which is normal in relation to the
individual or a foreign corporation, the expenses deductible business of the taxpayer and the surrounding circumstances.
are the necessary expenses paid or incurred in carrying on The term 'ordinary' does not require that the payments be
any business or trade conducted within the Philippines habitual or normal in the sense that the same taxpayer will
exclusively. have to make them often; the payment may be unique or
non-recurring to the particular taxpayer affected.
In the case of Atlas Consolidated Mining and Development Corporation v.
Commissioner of Internal Revenue, 4 the Court laid down the rules on the There is thus no hard and fast rule on the matter. The right to
deductibility of business expenses, thus: a deduction depends in each case on the particular facts and
the relation of the payment to the type of business in which
The principle is recognized that when a taxpayer claims a the taxpayer is engaged. The intention of the taxpayer often
deduction, he must point to some specific provision of the may be the controlling fact in making the determination.
statute in which that deduction is authorized and must be Assuming that the expenditure is ordinary and necessary in
able to prove that he is entitled to the deduction which the the operation of the taxpayer's business, the answer to the
law allows. As previously adverted to, the law allowing question as to whether the expenditure is an allowable
expenses as deduction from gross income for purposes of the deduction as a business expense must be determined from
income tax is Section 30(a) (1) of the National Internal the nature of the expenditure itself, which in turn depends on
Revenue which allows a deduction of 'all the ordinary and the extent and permanency of the work accomplished by the
necessary expenses paid or incurred during the taxable year expenditure.
in carrying on any trade or business.' An item of expenditure,

24
TAXATION II ATTY. ACOSTA -
CAJUSTIN
In the light of the above explanation, we hold that the Court of Tax Appeals did It is clear that ESSO, having assumed an expense properly attributable to its head
not err when it held on this issue as follows: office, cannot now claim this as an ordinary and necessary expense paid or
incurred in carrying on its own trade or business. cdrep
Considering the foregoing test of what constitutes an
ordinary and necessary deductible expense, it may be asked: WHEREFORE, the decision of the Court of Tax Appeals denying the petitioner's
Were the margin fees paid by petitioner on its profit claims for refund of P102,246.00 for 1959 and P434,234.92 for 1960, is
remittances to its Head Office in New York appropriate and AFFIRMED, with costs against the petitioner.
helpful in the taxpayer's business in the Philippines? Were
the margin fees incurred for purposes proper to the conduct SO ORDERED.
of the affairs of petitioner's branch in the Philippines? Or
[G.R. No. L-15290. May 31, 1963.]
were the margin fees incurred for the purpose of realizing a
profit or of minimizing a loss in the Philippines? Obviously
not. As stated in the Lopez case, the margin fees are not MARIANO ZAMORA, petitioner, vs. COLLECTOR OF
expenses in connection with the production or earning of INTERNAL REVENUE and COURT OF TAX
petitioner's incomes in the Philippines. They were expenses APPEALS,respondents.
incurred in the disposition of said incomes; expenses for the
remittance of funds after they have already been earned by
petitioner's branch in the Philippines for the disposal of its
[G.R. No. L-15280. May 31, 1963.]
Head Office in New York which is already another distinct
and separate income taxpayer.
COLLECTOR OF INTERNAL
xxx xxx xxx
REVENUE, petitioner, vs. MARIANO
Since the margin fees in question were incurred for the ZAMORA, respondent.
remittance of finds to petitioner's Head Office in New York,
which is a separate and distinct income taxpayer from the
branch in the Philippines, for its disposal abroad, it can never [G.R. No. L-15289. May 31, 1963.]
be said therefore that the margin fees were appropriate and
helpful in the development of petitioner's business in the
Philippines exclusively or were incurred for purposes proper ESPERANZA A. ZAMORA, as special Administratrix
to the conduct of the affairs of petitioner's branch in the of the Estate of FELICIDAD
Philippines exclusively or for the purpose of realizing a profit ZAMORA, petitioner, vs.COLLECTOR OF INTERNAL
or of minimizing a loss in the Philippines exclusively. If at all, REVENUE and COURT OF TAX
the margin fees were incurred for purposes proper to the APPEALS, respondents.
conduct of the corporate affairs of Standard Vacuum Oil
Company in New York, but certainly not in the Philippines.
[G.R. No. L-15281. May 31, 1963.]
ESSO has not shown that the remittance to the head office of part of its profits
was made in furtherance of its own trade or business. The petitioner merely
presumed that all corporate expenses are necessary and appropriate in the COLLECTOR OF INTERNAL
absence of a showing that they are illegal or ultra vires. This is error. The public REVENUE, petitioner, vs. ESPERANZA A. ZAMORA,
respondent is correct when it asserts that "the paramount rule is that claims for as Special Administratrix, etc., respondent.
deductions are a matter of legislative grace and do not turn on mere equitable
considerations . . . The taxpayer in every instance has the burden of justifying the
allowance of any deduction claimed." 5 There shall be allowed as deductions all the ordinary and necessary expenses
paid or incurred during the taxable year, in carrying on any trade or

25
TAXATION II ATTY. ACOSTA -
CAJUSTIN
business. Since promotion expenses constitute one of the deductions in amount of P20,957.00 was spent by Mrs. Esperanza A. Zamora (wife of
conducting a business, same must testify these requirements. Claim for the Mariano), during her travel to Japan and the United States to purchase
deduction of promotion expenses or entertainment expenses must also be machinery for a new Tiki-Tiki plant, and to observe hotel management in
substantiated or supported by record showing in detail the amount and modern hotels. The CTA, however, found that for said trip Mrs. Zamora
nature of the expenses incurred. obtained only the sum of P5,000.00 from the Central Bank and that in her
application for dollar allocation, she stated that she was going abroad on a
combined medical and business trip, which facts were not denied by Mariano
That to be deductible, said business expenses must be ordinary and necessary Zamora. No evidence had been submitted as to where Mariano had obtained
expenses paid or incurred in carrying on any trade or business; that those the amount in excess of P5,000.00 given to his wife which she spent abroad.
expenses must also meet the further test of reasonableness in amount; that No explanation had been made either that the statement contained in Mrs.
when some of the representation expenses claimed by the taxpayer were Zamora's application for dollar allocation that she was going abroad on a
evidenced by vouchers or chits, but others were without vouchers or chits, the combined medical and business trip, was not correct. The alleged expenses
court should determine from all available data, the amount properly were not supported by receipts. Mrs. Zamora could not even remember how
deductible as representation expenses. much money she had when she left abroad in 1951, and how the alleged
amount of P20,957.00 was spent.

FACTS:

ISSUE:
Mariano Zamora, owner of the Bay View Hotel and Farmacia Zamora,
Manila, filed his income tax returns the years 1951 and 1952. The Collector of Whether or not the CTA erred in disallowing P10,478.50 as promotion
Internal Revenue found that he failed to file his return of the capital gains expenses incurred by his wife for the promotion of the Bay View Hotel and
derived from the sale of certain real properties and claimed deductions which Farmacia Zamora in the absence of receipts proving the same.
were not allowable. The collector required him to pay the sums of P43,758.50
and P7,625.00, as deficiency income tax for the years 1951 and 1952.
HELD: NO
On appeal by Zamora, the Court of Tax Appeals modified the decision
appealed from and ordered him to pay the reduced total sum of P30,258.00
Section 30, of the Tax Code, provides that in computing net income, there
(P22,980.00 and P7,278.00, as deficiency income tax for the years 1951 and
shall be allowed as deductions all the ordinary and necessary expenses paid or
1952.
incurred during the taxable year, in carrying on any trade or business. Since
promotion expenses constitute one of the deductions in conducting a business,
Having failed to obtain a reconsideration of the decision, Mariano Zamora same must testify these requirements. Claim for the deduction of promotion
appealed alleging that the Court of Tax Appeals erred (amongst other things, expenses or entertainment expenses must also be substantiated or supported
this being the only relevant to the topic) in disallowing P10,478.50, as by record showing in detail the amount and nature of the expenses incurred
promotion expenses incurred by his wife for the promotion of the Bay View (N.H. Van Socklan, Jr. v. Comm. of Int. Rev.; 33 BTA 544). Considering, as
Hotel and Farmacia Zamora (which is of P20,957.00, supposed business heretofore stated, that the application of Mrs. Zamora for dollar allocation
expenses). shows that she went abroad on a combined medical and business trip, not all of
her expenses came under the category of ordinary and necessary expenses;
part thereof constituted her personal expenses. There having been no means by
Note: He contends that the whole amount of P20,957.00 as promotion which to ascertain which expense was incurred by her in connection with the
expenses in his 1951 income tax returns, should be allowed and not merely business of Mariano Zamora and which was incurred for her personal benefit,
one-half of it or P10,478.50, on the ground that, while not all the itemized the Collector and the CTA in their decisions, considered 50% of the said
expenses are supported by receipts, the absence of some supporting receipts amount of P20,957.00 as business expenses and the other 50%, as her
has been sufficiently and satisfactorily established. For, as alleged, the said personal expenses. We hold that said allocation is very fair to Mariano Zamora,

26
TAXATION II ATTY. ACOSTA -
CAJUSTIN
there having been no receipt whatsoever, submitted to explain the alleged Terminal Co., Inc., vs. Collector of Int. Rev., 108 Phil., 320).
business expenses, or proof of the connection which said expenses had to the
business or the reasonableness of the said amount of P20,957.00. While in 3. ID.; CAPITAL GAINS TAXES; COST BASIS OF PROPERTY ACQUIRED IN
situations like the present, absolute certainty is usually not possible, the CTA JAPANESE WAR NOTES. The cost basis of property acquired in Japanese
should make as close an approximation as it can, bearing heavily, if it chooses, war notes is the equivalent of the war notes in genuine Philippine currency in
upon the taxpayer whose inexactness is of his own making. accordance with the Ballantyne Scale of values, and the determination of the
gain derived or loss sustained in the sale of such property is not affected by the
decline at the time of sale, in the purchasing power of the Philippine currency.
In the case of Visayan Cebu Terminal Co., Inc. v. Collector of Int. Rev, it was
declared that representation expenses fall under the category of business 4. STATUTORY CONSTRUCTION; ANTECEDENTS OR LEGISLATIVE
expenses which are allowable deductions from gross income, if they meet the HISTORY OF STATUTE TO BE CONSIDERED IN ITS INTERPRETATION.
conditions prescribed by law, particularly section 30 (a) [1], of the Tax Code; Courts are permitted to look into and investigate the antecedents or the
that to be deductible, said business expenses must be ordinary and necessary legislative history of the statutes involved (Director of Lands vs. Abaya, et al.,
expenses paid or incurred in carrying on any trade or business; that those 63 Phil. 559).
expenses must also meet the further test of reasonableness in amount; that
when some of the representation expenses claimed by the taxpayer were
evidenced by vouchers or chits, but others were without vouchers or chits,
documents or supporting papers; that there is no more than oral proof to the In the above-entitled cases, a joint decision was rendered by the lower court
effect that payments have been made for representation expenses allegedly because they involved practically the same issues. We do so, likewise, for the same
made by the taxpayer and about the general nature of such alleged expenses; reason.
that accordingly, it is not possible to determine the actual amount covered by Cases Nos. L-15290 and L-15280
supporting papers and the amount without supporting papers, the court
should determine from all available data, the amount properly deductible as Mariano Zamora, owner of the Bay View Hotel and Farmacia Zamora, Manila,
representation expenses. filed his income tax returns for the years 1951 and 1952. The Collector of Internal
Revenue found that he failed to file his return of the capital gains derived from the
sale of certain real properties and claimed deductions which were not allowable.
SYLLABUS The Collector required him to pay the sums of P43,758.50 and P7,625.00, as
deficiency income tax for the years 1951 and 1952, respectively (C.T.A. Case No.
234, now L-15290). On appeal by Zamora, the Court of Tax Appeals on December
1. TAXATION; INCOME TAXES; BUSINESS EXPENSES AS DEDUCTION. 29, 1958, modified the decision appealed from and ordered him to pay the
Promotion expenses constitute one of the deductions in conducting a business, reduced total sum of P30,258.00 (P22,980.00 and P7,278.00, as deficiency
and should satisfy the requirements of Section 30 of the Tax Code, which income tax for the years 1951 and 1952, respectively), within thirty (30) days from
provides that in computing net income, there shall be allowed as deductions all the date the decision becomes final, plus the corresponding surcharges and
the ordinary and necessary expenses paid or incurred during the taxable year, interest in case of delinquency, pursuant to section 51 (e), Int. Revenue Code.
in carrying on any trade or business (Vol. 4, Martens, Law of Federal Income With costs against petitioner.
Taxation, sec. 25.03, p. 307).
Having failed to obtain a reconsideration of the decision, Mariano Zamora
2. ID.; ID.; ID.; REQUISITES FOR DEDUCTION OF BUSINESS EXPENSES. appealed (L-15290), alleging that the Court of Tax Appeals erred
Representation expenses fall under the category of business expenses which
are allowable deductions from gross income, if they meet the conditions (1) In disallowing P10,478.50, as promotion expenses
prescribed by law, particularly section 30 (a) (1), of the Tax Code. To be incurred by his wife for the promotion of the Bay View Hotel
deductible, they must be ordinary and necessary expenses paid or incurred in and Farmacia Zamora (which is 1/2 of P20,957.00, supposed
carrying on any trade or business, and should meet the further test of business expenses):
reasonableness in amount. They should, moreover, be covered by supporting (2) In disallowing 3 1/2% per annum, at the rate of
paper; in the absence thereof the amount properly deductible as representation depreciation of the Bay View Hotel Building;
expenses should be determined from all available data. (Visayan Cebu

27
TAXATION II ATTY. ACOSTA -
CAJUSTIN
(3) In disregarding the price stated in the deed of sale, as the (2) In not holding that Esperanza A. Zamora, as
cost of a Manila property, for the purpose of determining administratrix, is liable for the payment of the sum of
alleged capital gains; and P613.00 as deficiency income tax and 50% surcharge for
1951, plus 50% surcharge and 1% monthly interest from the
(4) In applying the Ballantyne scale of values in determining date said amount became due, to the date of actual payment.
the cost of said property. cdasia
It is alleged by Mariano Zamora that the CTA erred in disallowing P10,478.50 as
The Collector of Internal Revenue (L-15280) also appealed, claiming that the promotion expenses incurred by his wife for the promotion of the Bay View Hotel
Court of Tax Appeals erred and Farmacia Zamora. He contends that the whole amount of P20,957.00 as
promotion expenses in his 1951 income tax returns, should be allowed and not
(1) In giving credence to the uncorroborated testimony of
merely one-half of it or P10,478.50, on the ground that, while not all the itemized
Mariano Zamora that he bought the said real property in
expenses are supported by receipts, the absence of some supporting receipts has
question during the Japanese occupation, partly in Philippine
been sufficiently and satisfactorily established. For, as alleged, the said amount of
currency and partly in Japanese war notes, and
P20,957.00 was spent by Mrs. Esperanza A. Zamora (wife of Mariano), during her
(2) In not holding that Mariano Zamora is liable for the travel to Japan and the United States to purchase machinery for a new Tiki-Tiki
payment of the sums of P43,758.00 and P7,625.00, as plant, and to observe hotel management in modern hotels. The CTA, however,
deficiency income taxes, for the years 1951 and 1952, plus the found that for said trip Mrs. Zamora obtained only the sum of P5,000.00 from
5% surcharge and 1% monthly interest, from the date said the Central Bank and that in her application for dollar allocation, she stated that
amounts became due to the date of actual payment. she was going abroad on a combined medical and business trip, which facts were
not denied by Mariano Zamora. No evidence had been submitted as to where
Cases Nos. L-15289 and L-15281 Mariano had obtained the amount in excess of P5,000.00 given to his wife which
Mariano Zamora and his deceased sister Felicidad Zamora, bought a piece of land she spent abroad. No explanation had been made either that the statement
located in Manila on May 16, 1944, for P132,000.00 and sold it for P75,000.00 on contained in Mrs. Zamora's application for dollar allocation that she was going
March 5, 1951. They also purchased a lot located in Quezon City for P68,959.00 abroad on a combined medical and business trip, was not correct. The alleged
on January 19, 1944, which they sold for P94,000 on February 9, 1951. The CTA expenses were not supported by receipts. Mrs. Zamora could not even remember
ordered the estate of the late Felicidad Zamora (represented by Esperanza A. how much money she had when she left abroad in 1951, and how the alleged
Zamora, as special administratrix of her estate), to pay the sum of P235.50, amount of P20,957.00 was spent.
representing alleged deficiency income tax and surcharge due from said estate. Section 30, of the Tax Code, provides that in computing net income, there shall be
Esperanza A. Zamora appealed and alleged that the CTA erred: allowed as deductions all the ordinary and necessary expenses paid or incurred
(1) In disregarding the price stated in the deed of sale, as the during the taxable year, in carrying on any trade or business (Vol. 4, Mertens, Law
cost of the Manila Property for the purpose of determining of Federal Income Taxation, sec. 25.03, p. 307). Since promotion expenses
alleged capital gains; and constitute one of the deductions in conducting a business, same must satisfy these
requirements. Claims for the deduction of promotion expenses or entertainment
(2) In applying the Ballantyne Scale of values in determining expenses must also be substantiated or supported by record showing in detail the
the cost thereof. amount and nature of the expense incurred (N.H. Van Sicklen, Jr. vs. Comm. of
Int. Rev., 33 BTA 544). Considering, as heretofore stated, that the application of
The Commissioner of Internal Revenue likewise appealed from the decision, Mrs. Zamora for dollar allocation shows that she went abroad on a combined
claiming that the lower court erred: medical and business trip, not all of her expenses came under the category of
ordinary and necessary expenses; part thereof constituted her personal expenses.
(1) In giving credence to the uncorroborated testimony of There having been no means by which to ascertain which expense was incurred by
Mariano Zamora that he bought the real property involved her in connection with the business of Mariano Zamora and which was incurred
during the Japanese occupation, partly in genuine Philippine for her personal benefit, the Collector and the CTA in their decisions, considered
currency and partly in Japanese war notes; and 50% of the said amount of P20,957.00 as business expense and the other 50%, as
her personal expenses. We hold that said allocation is very fair to Mariano
Zamora, there having been no receipt whatsoever, submitted to explain the

28
TAXATION II ATTY. ACOSTA -
CAJUSTIN
alleged business expenses, or proof of the connection which said expenses had to corresponding to a normal useful life of only 40 years (1955
the business or the reasonableness of the said amount of P20,957.00. While in PH Federal Taxes, Par 14 160-K). Consequently, the stand of
situations like the present, absolute certainty is usually not possible, the CTA the petitioners can not be sustained".
should make as close an approximate as it can, bearing heavily, if it chooses, upon
the taxpayer whose inexactness is of his own making. As the lower court based its findings on Bulletin F, petitioner Zamora, argues
that the same should have been first proved as a law, to be subject to judicial
notice. Bulletin F, is a publication of the US Federal Internal Revenue
Service, which was made after a study of the lives of the properties. In the
In the case of Visayan Cebu Terminal Co., Inc., v. Collector of Int. Rev., G.R. No. words of the lower court: "It contains the list of depreciable assets, the
L-12798, May 30, 1960, it was declared that representation expenses fall under estimated average useful lives thereof and the rates of depreciation allowable
the category of business expenses which are allowable deductions from gross for each kind of property. (See 1955 PH Federal Taxes, Par. 14, 160 to Par. 14,
income, if they meet the conditions prescribed by law, particularly section 30 (a) 163-0). It is true that Bulletin F has no binding force, but it has a strong
[1], of the Tax Code; that to be deductible, said business expenses must be persuasive effect considering that the same has been the result of scientific
ordinary and necessary expenses paid or incurred in carrying on any trade or studies and observation for a long period in the United State after whose
business; that those expenses must also meet the further test of reasonableness in Income Tax Law ours is patterned." Verily, courts are permitted to look into
amount; that when some of the representation expenses claimed by the taxpayer and investigate the antecedents or the legislative history of the statutes
were evidenced by vouchers or chits, but others were without vouchers or chits, involved (Director of Lands v. Abaya, et al., 63 Phil. 559). Zamora also
documents or supporting papers; that there is no more than oral proof to the contends that his basis for applying the 3-1/2% rate is the testimony of its
effect that payments have been made for representation expenses allegedly made witness Mariano Katipunan, who cited a book entitled "Hotel Management
by the taxpayer and about the general nature of such alleged expenses; that Principles and Practice" by Lucius Boomer, President, Hotel Waldorf Astoria
accordingly, it is not possible to determine the actual amount covered by Corporation. As well commented by the Solicitor General, "while the
supporting papers and the amount without supporting papers, the court should petitioner would deny us the right to use Bulletin F, he would insist on using
determine from all available data, the amount properly deductible as as authority, a book in Hotel management written by a man who knew more
representation expenses. about hotels than about taxation. All that the witness did (Katipunan) . . . is
to read excerpts from the said book (t.s.n. pp. 99-101), which admittedly were
In view hereof, We are of the opinion that the CTA, did not commit error in
based on the decision of the U.S. Tax Courts, made in 1928 (t.s.n. p. 106)". In
allowing as promotion expenses of Mrs. Zamora claimed in Mariano Zamora's
view hereof, We hold that the 2-1/2% rate of depreciation of the Bay View
1951 income tax returns, merely one-half or P10,478.50.
Hotel building, is approximately correct.
Petitioner Mariano Zamora alleges that the CTA erred in disallowing 3-1/2% per The next items in dispute are the undeclared capital gains derived from the sales
annum as the rate of depreciation of the Bay View Hotel Building but only 2- in 1951 of certain real properties in Malate, Manila and in Quezon City, acquired
1/2%. In justifying depreciation deduction of 3-1/2%, Mariano Zamora contends during the Japanese occupation.
that (1) the Ermita Districts, where the Bay View Hotel is located, is now
becoming a commercial districts; (2) the hotel has no room for improvement; and The Manila property (Esperanza Zamora v. Coll. of Int. Rev., Case No. L-15289).
(3) the changing modes in architecture, styles of furniture and decorative designs, The CTA held in this case, that the cost basis of property acquired in Japanese war
"must meet the taste of a fickle public". It is a fact, however, that the CTA, in notes is the equivalent of the war notes in genuine Philippine currency in
estimating the reasonable rate of depreciation allowance for hotels made of accordance with the Ballantyne Scale of values, and that the determination of the
concrete and steel at 2-1/2%, the three factors just mentioned had been taken into gain derived or loss sustained in the sale of such property is not affected by the
account already. Said the CTA decline at the time of sale, in the purchasing power of the Philippine currency. It
was found by the CTA that the purchase price of P132,000.00 was not entirely
"Normally, an average hotel building is estimated to have a paid in Japanese War notes but 1/2 thereof or P66,000.00 was in Philippine
useful life of 50 years, but inasmuch as the useful life of the currency, and that during certain periods of the enemy occupation, the value of
building for business purposes depends to a large extent on the Japanese war notes was very much less than the value of the genuine
the suitability of the structure to its use and location, its Philippine currency. On this point, the CTA declared
architectural quality, the rate of change in population, the
shifting of land values, as well as the extent and maintenance
and rehabilitation. It is allowed a depreciation rate of 2-1/2%

29
TAXATION II ATTY. ACOSTA -
CAJUSTIN
"Finally, it is alleged that the purchase price of P132,000.00 The Quezon City Property (Mariano Zamora vs. Coll. of Customs, Case
was not entirely paid in Japanese war notes, Mariano No. 15290). The Zamoras alleged that the entire purchase price of P68,959.00
Zamora, co-owner of the property in question, testified that was paid in Philippine currency. The collector, on the other hand, contends that
P66,000.00 was paid in Philippine currency and the other the purchase price of P68,959.00 was paid in Japanese war notes. The CTA,
P66,000.00 was paid in Japanese war notes. No evidence however, giving credence to Zamora's version, said
was presented by respondent to rebut the testimony of
Mariano Zamora; it is assailed merely as being improbable. ". . . If, as contended by respondent, the purchase price of
We have examined this question thoroughly and we are P68,959.00 was paid in Japanese war notes, the purchase
inclined to give credence to the allegation that a portion of price in Philippine currency would be only P17,239.70
the purchase price of the property was paid in Philippine (P68,959.00 divided by 4, 34.00 in war notes being
money. In the first place, it appears that the Zamoras owned equivalent to P1.00 in Philippine currency). The assessed
the Farmacia Zamora which continued to engage in business value of said property in Philippine currency at the time of
during the war years and that a considerable portion of its acquisition was P46,910.00. It is quite incredible that real
sales was paid for in genuine Philippine currency. This property with an assessed value of P46,910.00 should have
circumstances enabled the Zamoras to accumulate Philippine been sold by the owner thereof in Japanese war notes with an
money which they used in acquiring the property in question equivalent value in Philippine currency of only P17,239.75.
and another property in Quezon City. In the second place, We are more inclined to believe the allegation that it was
P132,000.00 in Japanese war notes in May, 1944 is purchased for P68,959.00 in genuine Philippine currency.
equivalent to only P11,000.00. The property in question had Since the property was sold for P94,000.00 on February 9,
at the time an assessed value of P27,031.00 (in Philippine 1951, the gain derived from the sale is P15,361.75, after
currency). Considering the well known fact that theassessed deducting from the selling price the cost of acquisition in the
value of real property is very much below the fair market sum of P68,959.00 and the expense of sale in the sum of
value, it is incredible that said property should have been P9,679.25."
sold by the owner thereof for less than one-half of its
The above appraisal is correct, and We have no plausible reason to disturb
assessed value. These facts have convinced us of the veracity
the same. aisa dc
of the allegation that of the purchase price of P132,000.00
the sum of P66,000.00 was paid in Philippine currency, so Consequently, the total undeclared income of petitioners derived from the sales of
that only the sum of P66,000.00 was paid in Japanese War the Manila and Quezon City properties in 1951 is P17,111.75 (P1,750.00 plus
notes." P15,361.75), 50% of which is the sum of P8,555.88 is taxable, the said properties
being capital assets held for more than one year.
This being the case, the Ballantyne Scale of values, which was the result of an
impartial scientific study, adopted and given judicial recognition, should be IN VIEW HEREOF, the petition in each of the above-entitled cases is dismissed,
applied. As the value of the Japanese war notes in May, 1944 when the and the decision appealed from is affirmed, without special pronouncement as to
Manila property was bought, was 1/12 of the genuine Philippine Peso costs.
(Ballantyne Scale), and since the gain derived or loss sustained in the
disposition of this property is to be reckoned in terms of Philippine Peso, the
value of the Japanese war notes used in the purchase of the property, must be
reduced in terms of the genuine Philippine Peso to determine the cost of
acquisition. It, therefore, results that since the sum of P66,000.00 in
Japanese war notes in May, 1944 is equivalent to P5,500.00 in Philippine
currency (P66,000.00 divided by 12), the acquisition cost of the property in
question is P66,000.00 plus P5,500.00 or P71,500.00 and that as the
property was sold for P75,000.00 in 1951, the owners thereof Mariano and
Felicidad Zamora derived a capital gain of P3,500.00 or P1,750.00 each.

30
TAXATION II ATTY. ACOSTA -
CAJUSTIN

TOPIC: DEDUCTIONS
ORDINARY AND NECESSARY
TRADE, BUSINESS OR
PROFESSIONAL EXPENSES
TEST OF REASONABLENESS

[G.R. No. L-24059. November 28, 1969.]

C. M. HOSKINS & CO., INC., petitioner, vs.


COMMISSIONER OF INTERNAL
REVENUE, respondent.

SYLLABUS

1. TAXATION; INCOME TAX; DEDUCTIONS FROM NET INCOME;


ORDINARY AND NECESSARY EXPENSES; INORDINATELY LARGE
COMMISSIONS AND FEES PAID TO CONTROLLING STOCKHOLDER ARE
DISALLOWED AS DEDUCTIBLE EXPENSES; CASE AT BAR. Considering
that in addition to being Chairman of the board of directors of petitioner
corporation, which bears his name, Hoskins, who owned 99.6% of its total
authorized capital stock while the four other officers-stockholders of the firm
owned a total of four-tenths of 1%, or one-tenth of 1% each, with their
respective nominal shareholdings of one share each, was also salesman/broker
for his company, receiving a 50% share of the sales commissions earned by
petitioner, besides his monthly salary of P3,750.00 amounting to an annual
compensation of P45,000.00 and an annual salary bonus of P40,000.00, plus
free use of the company car and receipt of other similar allowances and
benefits, the Tax Court correctly ruled that the payment by petitioner to
Hoskins of the additional sum of P99,977.91 as his equal or 50% share of the
8% supervision fees received by petitioner as managing agents of the real
estate, subdivision projects of Paradise Farms, Inc. and Realty Investments,
Inc. was inordinately large and could not be accorded the treatment of
ordinary and necessary expenses allowed as deductible items within the
purview of Section 30(a)(i) of the Tax Code.

31
TAXATION II ATTY. ACOSTA -
CAJUSTIN
2. ID.; ID.; ID.; BONUSES AS DEDUCTIBLE EXPENSE. It is a general rule representing 50% of supervision fees earned by it and set aside respondent's
that bonuses to employees made in good faith and as additional compensation disallowance of three other minor items. The Tax Court therefore determined
for the services actually rendered by the employees are deductible, provided petitioner's tax deficiency to be in the amount of P27,145.00 and on November 8,
such payments, when added to the stipulated salaries, do not exceed a 1964 rendered judgment against it, as follows:
reasonable compensation for the services rendered.
"WHEREFORE, premises considered, the decision of the
3. ID.; ID.; ID.; ID.; NECESSARY CONDITIONS FOR SUCH DEDUCTION. respondent is hereby modified. Petitioner is ordered to pay to
The condition precedents to the deduction of bonuses to employees are: (1) the the latter or his representative the sum of P27,145.00,
payment of the bonuses is in fact compensation; (2) it must be for personal representing deficiency income tax for the year 1957, plus
services actually rendered; and (3) the bonuses, when added to the salaries, are interest at 1/2% per month from June 20, 1959 to be
reasonable . . . when measured by the amount and quality of the services computed in accordance with the provisions of Section 51(d)
performed with relation to the business of the particular taxpayer. of the National Internal Revenue Code. If the deficiency tax is
not paid within thirty (30) days from the date this decision
4. ID.; ID.; ID.; ID.; REASONABLENESS THEREOF TO BE SHOWN. As far becomes final, petitioner is also ordered to pay surcharge and
as petitioner's contention that as employer it has the right to fix the interest as provided for in Section 51(e) of the Tax Code,
compensation of its officers and employees and that it was in the exercise of without costs."
such right that it deemed proper to pay the bonuses in question, all that We
need say is this: that right may be conceded, but for income tax purposes the Petitioner questions in this appeal the Tax Court's findings that the disallowed
employer cannot legally claim such bonuses as deductible expenses unless they payment to Hoskins was an inordinately large one, which bore a close relationship
are shown to be reasonable. To hold otherwise would open the gate of rampant to the recipient's dominant stockholdings and therefore amounted in law to a
tax evasion. distribution of its earnings and profits.

5. ID.; ID.; CORPORATE TAX LIABILITY OF CORPORATION OF SOLE We find no merit in petitioner's appeal.
PROPRIETORSHIP. Petitioner corporation, a sole proprietorship of C.M.
Hoskins who virtually holds 99.6% of the stocks of said corporation is duty As found by the Tax Court, "petitioner was founded by Mr. C. M. Hoskins in 1937,
bound to pay the income tax imposed on corporations and may not legally be with a capital stock of 1,000 shares at a par value of P1.00 each share; that of
permitted, by way of corporate resolution authorizing payment of inordinately these 1,000 shares, Mr. C. M. Hoskins owns 996 shares (the other 4 shares being
large commissions and fees to its controlling stockholder, to dilute and held by the other four officers of the corporation), which constitute exactly 99.6%
diminish its corresponding corporate tax liability. of the total authorized capital stock (p. 92, t.s.n.); that during the first four years
of its existence, Mr. C. M. Hoskins was the President, but during the taxable
period in question, that is, from October 1, 1956 to September 30, 1957, he was
We uphold in this taxpayer's appeal the Tax Court's ruling that payment by the the chairman of the Board of Directors and salesman-broker for the company (p.
taxpayer to its controlling stockholder of 50% of its supervision fees or the 93, t.s.n.); that as chairman of the Board of Directors, he received a salary of
amount of P99,977.91 is not a deductible ordinary and necessary expense and P3,750.00 a month, plus a salary bonus of about P40,000 00 a year (p. 94, t.s.n.);
should be treated as a distribution of earnings and profits of the taxpayer. that he was also a stockholder and officer of the Paradise Farms, Inc. and Realty
Investments, Inc., from which petitioner derived a large portion of its income in
Petitioner, a domestic corporation engaged in the real estate business as brokers, the form of supervision fees and commissions earned on sales of lots (pp. 97-99,
managing agents and administrators, filed its income tax return for its fiscal year t.s.n.; Financial Statements, attached to Exhibit '1', p. 11, BIR rec.); that as
ending September 30, 1957 showing a net income of P92,540.25 and a tax liability chairman of the Board of Directors of petitioner, his duties were: "To act as a
due thereon of P18,508.00, which it paid in due course. Upon verification of its salesman; as a director, preside over meetings and to get all of the real estate
return, respondent Commissioner of Internal Revenue, disallowed four items of business I could for the company by negotiating sales, purchases, making
deduction in petitioner's tax returns and assessed against it an income tax appraisals, raising funds to finance real estate operations where that was
deficiency in the amount of P28,054.00 plus interests. The Court of Tax Appeals necessary' (p. 96, t.s.n.); that he was familiar with the contract entered into by the
upon reviewing the assessment at the taxpayer's petition, upheld respondent's petitioner with the Paradise Farms, Inc. and the Realty Investments. Inc. by the
disallowance of the principal item of petitioner's having paid to Mr. C. M. terms of which petitioner was 'to program the development, arrange financing,
Hoskins, its founder and controlling stockholder the amount of P99,977.91 plan the proposed subdivision as outlined in the prospectus of Paradise Farms,

32
TAXATION II ATTY. ACOSTA -
CAJUSTIN
Inc., arrange contract for road constructions, with the provision of water supply to Petitioner's invoking of its policy since its incorporation of sharing equally sales
all of the lots and in general to serve as managing agents for the Paradise Farms, commissions with its salesmen, in accordance with its board resolution of June
Inc. and subsequently for the Realty Investment, Inc." (pp. 96-97, t.s.n.). 18, 1946, is equally untenable. Petitioner's Sales Regulations provide:
Considering that in addition to being Chairman of the board of directors of "Compensation of Salesmen
petitioner corporation, which bears his name, Hoskins, who owned 99.6% of its
total authorized capital stock while the four other officers-stockholders of the firm "8. Schedule I In the case of sales to prospects discovered
owned a total of four-tenths of 1%, or one tenth of 1% each, with their respective and worked by a salesman, even though the closing is done
nominal shareholdings of one share each, was also salesman-broker for his by or with the help of the Sales Manager or other members of
company, receiving a 50% share of the sales commissions earned by petitioner, the staff, the salesmen get one-half (1/2) of the total
besides his monthly salary of P3,750.00 amounting to an annual compensation of commission received by the Company, but not exceeding five
P45,000.00 and an annual salary bonus of P40,000.00, plus free use of the percent (5%). In the case of subdivisions, when the office
company car and receipt of other similar allowances and benefits, the Tax Court commission covers general supervision, the 1/2-rule does
correctly ruled that the payment by petitioner to Hoskins of the additional sum of not apply, the salesman's share being stipulated in the case
P99,977.91 as his equal or 50% share of the 8% supervision fees received by of each subdivision. In most cases the salesman's share is
petitioner as managing agents of the real estate, subdivision projects of Paradise 4%. (Exh. 'N-1')." 2
Farms, Inc. and Realty Investments, Inc. was inordinately large and could not be
It will be readily seen therefrom that when the petitioner's commission
accorded the treatment of ordinary and necessary expenses allowed as deductible
covers general supervision, it is provided that the 1/2 rule of equal sharing of
items within the purview of Section 30 (a) (i) of the Tax Code.
the sales commissions does not apply and that the salesman's share is
If such payment of P99,977.91 were to be allowed as a deductible item, then stipulated in the case of each subdivision. Furthermore, what is involved here
Hoskins would receive on these three items alone (salary, bonus and supervision is not Hoskins' salesman's share in the petitioner's 12% sales commission,
fee) a total of P184,977.91, which would be double the petitioner's reported net which he presumably collected also from petitioner without respondent's
income for the year of P92,540.25. As correctly observed by respondent, if questioning it, but a 50% share besides in petitioner's planning and
independently, a one-time P100,000.00-fee to plan and lay down the rules for supervision fee of 8% of the gross sales, as mentioned above. This is evident
supervision of a subdivision project were to be paid to an experienced realtor such from petitioner's board's resolution of July 14, 1953 (Exhibit 7), wherein it is
as Hoskins, its fairness and deductibility by the taxpayer could be conceded; but recited that in addition to petitioner's sales commission of 12% of gross sales,
here 50% of the supervision fee of petitioner was being paid by it to Hoskins every the subdivision owners were paying to petitioner 8% of gross sales as
year since 1955 up to 1963 and for as long as its contract with the subdivision supervision fee, and a collection fee of 5% of gross collections, or total fees of
owner subsisted, regardless of whether services were actually rendered by 25% of gross sales.
Hoskins, since his services to petitioner included such planning and supervision The case before us is similar to previous cases of disallowances as deductible
and were already handsomely paid for by petitioner. items of officers' extra fees, bonuses and commissions, upheld by this Court as not
being within the purview of ordinary and necessary expenses and not passing the
test of reasonable compensation. 3 In Kuenzle & Streiff, Inc. vs. Commissioner of
The fact that such payment was authorized by a standing resolution of petitioner's Internal Revenue decided by this Court on May 29, 1969, 4 we reaffirmed the test
board of directors, since "Hoskins had personally conceived and planned the of reasonableness, enunciated in the earlier 1967 case involving the same parties,
project" cannot change the picture. There could be no question that as Chairman that: "It is a general rule that 'Bonuses to employees made in good faith and as
of the board and practically an absolutely controlling stockholder of petitioner, additional compensation for the services actually rendered by the employees are
holding 99.6% of its stock, Hoskins wielded tremendous power and influence in deductible, provided such payments, when added to the stipulated salaries, do not
the formulation and making of the company's policies and decisions. Even just as exceed a reasonable compensation for the services rendered' (4 Mertens, Law of
board chairman, going by petitioner's own enumeration of the powers of the Federal Income Taxation, Sec. 25.50, p. 410). The conditions precedent to the
office, Hoskins could exercise great power and influence within the corporation, deduction of bonuses to employees are: (1) the payment of the bonuses is in fact
such as directing the policy of the corporation, delegating powers to the president compensation; (2) it must be for personal services actually rendered; and (3) the
and advising the corporation in determining executive salaries, bonus plans and bonuses, when added to the salaries, are 'reasonable . . . when measured by the
pensions, dividend policies, etc. 1

33
TAXATION II ATTY. ACOSTA -
CAJUSTIN
amount and quality of the services performed with relation to the business of the exercise it for the purpose of evading payment of taxes
particular taxpayer' (Idem., Sec. 25, 44, p. 395). legitimately due to the State."
"There is no fixed test for determining the reasonableness of Finally, it should be noted that we have here a case practically of a sole
a given bonus as compensation. This depends upon many proprietorship of C. M. Hoskins, who however chose to incorporate his business
factors, one of them being 'the amount and quality of the with himself holding virtually absolute control thereof with 99.6% of its stock with
services performed with relation to the business.' Other tests four other nominal shareholders holding one share each. Having chosen to use
suggested are: payment must be 'made in good faith'; 'the the corporate form with its legal advantages of a separate corporate personality as
character of the taxpayer's business, the volume and amount distinguished from his individual personality, the corporation so created, i.e.,
of its net earnings, its locality, the type and extent of the petitioner, is bound to comport itself in accordance with corporate norms and
services rendered, the salary policy of the corporation'; 'the comply with its corporate obligations. Specifically, it is bound to pay the income
size of the particular business'; 'the employees' qualifications tax imposed by law on corporations and may not legally be permitted, by way of
and contributions to the business venture'; and 'general corporate resolutions authorizing payment of inordinately large commissions and
economic conditions' (4 Mertens, Law of Federal Income fees to its controlling stockholder, to dilute and diminish its corresponding
Taxation, Secs. 25.44, 25.49, 25.50, 25.51, pp. 407-412). corporate tax liability.
However, 'in determining whether the particular salary or
compensation payment is reasonable, the situation must be ACCORDINGLY, the decision appealed from is hereby affirmed, with costs in
considered as a whole. Ordinarily, no single factor is decisive. both instances against petitioner.
. . . it is important to keep in mind that it seldom happens
that the application of one test can give satisfactory answer, |||

and that ordinarily it is the interplay of several factors,


properly weighted for the particular case, which must furnish
the final answer."
Petitioner's case fails to pass the test. On the right of the employer as against
respondent Commissioner to fix the compensation of its officers and employees,
we there held further that while the employer's right may be conceded, the
question of the allowance or disallowance thereof as deductible expenses for
income tax purposes is subject to determination by respondent Commissioner of
Internal Revenue. Thus: "As far as petitioner's contention that as employer it has
the right to fix the compensation of its officers and employees and that it was in
the exercise of such right that it deemed proper to pay the bonuses in question, all
that We need say is this: that right may be conceded, but for income tax purposes
the employer cannot legally claim such bonuses as deductible expenses unless
they are shown to be reasonable. To hold otherwise would open the gate of
rampant tax evasion.
"Lastly, We must not lose sight of the fact that the question of
allowing or disallowing as deductible expenses the amounts
paid to corporate officers by way of bonus is determined by
respondent exclusively for income tax purposes. Concededly,
he has no authority to fix the amounts to be paid to corporate
officers by way of basic salary, bonus or additional
remuneration a matter that lies more or less exclusively
within the sound discretion of the corporation itself. But this
right of the corporation is, of course, not absolute. It cannot

34
TAXATION II ATTY. ACOSTA -
CAJUSTIN

TOPIC: DEDUCTIONS
ORDINARY AND NECESSARY
TRADE, BUSINESS OR
PROFESSIONAL EXPENSES
OTHER CASES

[G.R. No. L-54108. January 17, 1984.]

COMMISSIONER OF INTERNAL
REVENUE, petitioner, vs. COURT OF TAX APPEALS
and SMITH KLINE & FRENCH OVERSEAS CO.
(PHILIPPINE BRANCH), respondents.

SYLLABUS

1. TAXATION; NATIONAL INTERNAL REVENUE CODE; INCOME TAX OF


CORPORATIONS; DEDUCTIONS; EXPENSES RELATED TO PRODUCTION
OF PHILIPPINE DERIVED INCOME AND TO PHILIPPINE OPERATIONS
DEDUCTIBLE. From the provisions, Section 37(b) of the old National
Internal Revenue Code,Commonwealth Act No. 466, which is reproduced
in Presidential Decree No. 1158, the National Internal Revenue Code of 1977,
and Sec. 160 of Revenue Regulations No. 2 it is manifest that where an expense
is clearly related to the production of Philippine-derived income or to
Philippine operations (e.g. salaries of Philippine personnel, rental of office
building in the Philippines), that expense can be deducted from the gross
income acquired in the Philippines without resorting to apportionment. Under
the same provisions also, where there are items included in the overhead
expenses incurred by the parent company, all of which directly benefit its
branches, including the Philippines, which cannot be definitely allocated or
identified with the operations of the Philippine branch, the company may
claim as its deductible share a ratable part of such expenses based upon the

35
TAXATION II ATTY. ACOSTA -
CAJUSTIN
ratio of the local branch's gross income to the total gross income, worldwide, of It appears that sometime in October, 1972, Smith Kline received from its
the multinational corporation. international independent auditors, Peat, Marwick, Mitchell and Company, an
authenticated certification to the effect that the Philippine share in the
2. ID.; ID.; ID.; AMENDED RETURN ALLOWED WHERE OVERHEAD unallocated overhead expenses of the main office for the year ended December 31,
EXPENSES WERE ESTIMATED. Smith Kline had to amend its return 1971 was actually $219,547 (1,427,484). It further stated in the certification that
because it is of common knowledge that audited financial statements are the allocation was made on the basis of the percentage of gross income in the
generally completed three or four months after the close of the accounting Philippines to gross income of the corporation as a whole. By reason of the new
period. There being no financial statements yet when the certification of adjustment, Smith Kline's tax liability was greatly reduced from P511,247 to
January 11, 1972 was made, the treasurer could not have correctly computed P186,992 resulting in an overpayment of P324,255.
Smith Kline's share in the home office overhead expenses in accordance with
the gross income formula prescribed in section 160 of the Revenue On April 2, 1974, without awaiting the action of the Commissioner of Internal
Regulations. What the treasurer certified was a mere estimate. Smith Kline Revenue on its claim, Smith Kline filed a petition for review with the Court of Tax
likewise submits that it has presented ample evidence to support its claim for Appeals.
refund. To this end, it has presented before the Tax Court the authenticated
statement of Peat, Marwick, Mitchell and Company to show that since the In its decision of March 21, 1980, the Tax Court ordered the Commissioner to
gross income of the Philippine branch was P7,143,155 ($1,098,617) for 1971 as refund the overpayment or grant a tax credit to Smith Kline. The Commissioner
per audit report prepared by Sycip, Gorres, Velayo and Company, and the gross appealed to this Court.
income of the corporation as a whole was $6,891,052, Smith Kline's share at The governing law is found in section 37 of the old National Internal Revenue
15.94% of the home office overhead expenses was P1,427,484 ($219,547). Code,Commonwealth Act No. 466, which is reproduced in Presidential Decree
Clearly, the weight of evidence bolsters its position that the amount of No. 1158, the National Internal Revenue Code of 1977 and which reads:
P1,427.484 represents the correct ratable share, the same having been
computed pursuant to Section 37(b) and Section 160. "SEC. 37. Income from sources within the Philippines.
3. ID.; ID.; ID.; ID.; REFUND, PROPER. In a manifestation dated July 19, xxx xxx xxx
1983, Smith Kline declared that with respect to its share of the head office
overhead expenses in its income tax returns for the years 1973 to 1981, it "(b) Net income from sources in the Philippines. From the
deducted its ratable share of the total overhead expenses of its head office for items of gross income specified in subsection (a) of this
those years as computed by the independent auditors hired by the parent section there shall be deducted the expenses, losses, and
company in Philadelphia, Pennsylvania, U.S.A., as soon as said computations other deductions properly apportioned or allocated
were made available to it. We hold that Smith Kline's amended 1971 return is thereto and a ratable part of any expenses, losses, or other
in conformity with the law and regulations. The Tax Court correctly held that deductions which cannot definitely be allocated to some item
the refund or credit of the resulting overpayment is in order. or class of gross income. The remainder, if any, shall be
included in full as net income from sources within the
This case is about the refund of a 1971 income tax amounting to P324,255. Smith Philippines.
Kline and French Overseas Company, a multinational firm domiciled in
Philadelphia, Pennsylvania, is licensed to do business in the Philippines. It is xxx xxx xxx"
engaged in the importation, manufacture and sale of pharmaceuticals, drugs and
Revenue Regulations No. 2 of the Department of Finance contains the following
chemicals. LLjur
provisions on the deductions to be made to determine the net income from
In its 1971 original income tax return, Smith Kline declared a net taxable income Philippine sources:
of P1,489,277 (Exh. A) and paid P511,247 as tax due. Among the deductions
"SEC. 160. Apportionment of deductions. From the items
claimed from gross income was P501,040 ($77,060) as its share of the head office
specified in section 37(a), as being derived specifically from
overhead expenses. However, in its amended return filed on March 1, 1973, there
sources within the Philippines there shall be deducted the
was an overpayment of P324,255 "arising from underdeduction of home office
expenses, losses, and other deductions properly apportioned
overhead" (Exh, E). It made a formal claim for the refund of the alleged
or allocated thereto and a ratable part of any other expenses,
overpayment.
losses or deductions which can not definitely be allocated to

36
TAXATION II ATTY. ACOSTA -
CAJUSTIN
some item or class of gross income. The remainder shall be from sources within the Philippines expenses amounting to
included in full as net income from sources within the P14,000 [representing P8,000 properly apportioned to the
Philippines. The ratable part is based upon the ratio of gross income from sources within the Philippines and P6,000, a
income from sources within the Philippines to the total gross ratable part (one fifth) of the expenses which could not be
income. allocated to any item or class of gross income]. The
remainder, P22,000, is the net income from sources within
"Example: A non-resident alien individual whose taxable year the Philippines."
is the calendar year, derived gross income from all sources
for 1939 of P180,000, including therein: From the foregoing provisions, it is manifest that where an expense is clearly
related to the production of Philippine-derived income or to Philippine operations
Interest on bonds of a domestic (e.g. salaries of Philippine personnel, rental of office building in the Philippines),
that expense can be deducted from the gross income acquired in the Philippines
corporation P9,000
without resorting to apportionment.
Dividends on stock of a domestic
The overhead expenses incurred by the parent company in connection with
corporation 4,000 finance, administration, and research and development, all of which directly
benefit its branches all over the world, including the Philippines, fall under a
Royalty for the use of patents different category however. These are items which cannot be definitely allocated
or identified with the operations of the Philippine branch. For 1971, the parent
within the Philippines 12,000 company of Smith Kline spent $1,077,739. Under section 37(b) of the Revenue
Gain from sale of real property located Code and section 160 of the regulations, Smith Kline can claim as its deductible
share a ratable part of such expenses based upon the ratio of the local branch's
within the Philippines 11,000 gross income to the total gross income, worldwide, of the multinational
corporation. LexLib

In his petition for review, the Commissioner does not dispute the right of Smith
Total P36,000 Kline to avail itself of section 37(b) of the Tax Code and section 160 of the
regulations. But the Commissioner maintains that such right is not absolute and
======= that as there exists a contract (in this case a service agreement) which Smith Kline
that is, one-fifth of the total gross income was from sources has entered into with its home office, prescribing the amount that a branch can
within the Philippines. The remainder of the gross income deduct as its share of the main office's overhead expenses, that contract is
was from sources without the Philippines, determined under binding.
section 37(c).
"The expenses of the taxpayer for the year amounted to
P78,000. Of these expenses the amount of P8,000 is properly The Commissioner contends that since the share of the Philippine branch has
allocated to income from sources within the Philippines and been fixed at $77,060, Smith Kline itself cannot claim more than the said amount.
the amount of P40,000 is properly allocated to income from To allow Smith Kline to deduct more than what was expressly provided in the
sources without the Philippines. agreement would be to ignore its existence. It is a cardinal rule that a contract is
the law between the contracting parties and the stipulations therein must be
"The remainder of the expense, P30,000, cannot be definitely respected unless these are proved to be contrary to law, morals, good customs and
allocated to any class of income. A ratable part thereof, based public policy. There being allegedly no showing to the contrary, the provisions
upon the relation of gross income from sources within the thereof must be followed.
Philippines to the total gross income, shall be deducted in
computing net income from sources within the Philippines. The Commissioner also argues that the Tax Court erred in relying on the
Thus, there are deducted from the P36,000 of gross income certification of Peat, Marwick, Mitchell and Company that Smith Kline is entitled
to deduct P1,427,484 ($219,547) as its allotted share and that Smith Kline has not

37
TAXATION II ATTY. ACOSTA -
CAJUSTIN
presented any evidence to show that the home office expenses chargeable to SANTIAGO GANCAYCO, petitioner, vs. THE
Philippine operations exceeded $77,060. COLLECTOR OF INTERNAL REVENUE, respondent.
On the other hand, Smith Kline submits that the contract between itself and its
home office cannot amend tax laws and regulations. The matter of allocated
Gancyaco files his income tax return for the year 1949. Respondent issued a
expenses which are deductible under the law cannot be the subject of an
warrant of distraint and levy against the properties of Gancayco for the
agreement between private parties nor can the Commissioner acquiesce in such
satisfaction of his deficiency income tax liability, and accordingly, the
an agreement.
municipal treasurer issued a notice of sale of said property at public auction.
Smith Kline had to amend its return because it is of common knowledge that Gancayco filed a petition to cancel the sale and direct that the same be re-
audited financial statements are generally completed three or four months after advertised at a future date
the close of the accounting period. There being no financial statements yet when
the certification of January 11, 1972 was made, the treasurer could not have ISSUE: Whether the sum of PhP 16,860.31 is due from Gancayco as deficiency
correctly computed Smith Kline's share in the home office overhead expenses in income tax for 1949 hinges on the validity of his claim for deduction:
accordance with the gross income formula prescribed in section 160 of the a) farming expense PhP 27,459
Revenue Regulations. What the treasurer certified was a mere estimate. b) representation expenses PhP 8,933.45

Smith Kline likewise submits that it has presented ample evidence to support its HELD:
claim for refund. To this end, it has presented before the Tax Court the a)Farming Expenses - no evidence has been presnted as to the nature of the
authenticated statement of Peat, Marwick, Mitchell and Company to show that said farming expenses other than the care statement of petitioner that they
since the gross income of the Philippine branch was P7,143,155 ($1,098,617) for were spent for the development and cultivation of his property.
1971 as per audit report prepared by Sycip, Gorres, Velayo and Company, and the
gross income of the corporation as a whole was $6,891,052, Smith Kline's share at No specification has been made as to the actual amount spent for purchase of
15.94% of the home office overhead expenses was P1,427,484 ($219,547) (Exh. G tools, equipment or materials or the amount spent for improvement.
to G-2, BIR Records, 4-5). LLpr
b) Representation expense
Clearly, the weight of evidence bolsters its position that the amount of P1,427,484
represents the correct ratable share, the same having been computed pursuant to PhP 22, 820 is allowed
section 37(b) and section 160. PhP 8,993.45 is disallowed because of the absence of recipt, invoices or
In a manifestation dated July 19, 1983, Smith Kline declared that with respect to vouchers of the expenditures in question, petitioner could not sspecify the
its share of the head office overhead expenses in its income tax returns for the items constituting the same when or on whom or on what they were incurred.
years 1973 to 1981, it deducted its ratable share of the total overhead expenses of
its head office for those years as computed by the independent auditors hired by
the parent company in Philadelphia, Pennsylvania, U.S.A., as soon as said SYLLABUS
computations were made available to it.
We hold that Smith Kline's amended 1971 return is in conformity with the law and 1. TAXATION; INTERNAL REVENUE TAXES; TWO CIVIL REMEDIES FOR
regulations. The Tax Court correctly held that the refund or credit of the resulting COLLECTION. There are two (2) civil remedies for the collection of internal
overpayment is in order. cdrep revenue taxes, namely; (a) by distraint of personal property; and (b) by
"judicial action" (Commonwealth Act 456, Section 316). The first may not be
WHEREFORE, the decision of the Tax Court is hereby affirmed. No costs. availed of except within three (3) years after the "return is due or has been
made . . ." (Tax Code, Section 51 [d]). After the expiration of said period,
SO ORDERED. income taxes may not be legally and validly collected by distraint and/or levy
[G.R. No. L-13325. April 20, 1961.] (Internal Revenue vs. Avelino, 100 Phil., 327 53 Off. Gaz 546; Collector of
Internal Revenue vs. Zulueta, 100 Phil., 872 53 Off. Gaz., 6532; Sambrano vs.

38
TAXATION II ATTY. ACOSTA -
CAJUSTIN
Court of Tax Appeals 101 Phil., 1; 53 Off. Gaz., [15] 4839). which was partly granted by respondent, who in a letter dated April 8, 1953
(Exhibit D), informed petitioner that his income tax deficiency for 1949 amounted
2. ID.; ID.; WHEN JUDICIAL ACTION MAY BE RESORTED TO. The to P16,860.31. Gancayco urged another reconsideration (Exhibit O), but no action
"judicial action" mentioned in the Tax Code may be resorted to within five (5) was taken on this request, although he had sent several communications calling
years from the date return has been filed, if there has been no assessment, or respondent's attention thereto.
within five (5) years from the date of the assessment made within the statutory
period, or within the period agreed upon, in writing, by the Collector of On April 15, 1956, respondent issued a warrant of distraint and levy against the
Internal Revenue, and the taxpayer, before the expiration of said five-year properties of Gancayco for the satisfaction of his deficiency income tax liability,
period, or within such extension of said stipulated period as may have been and, accordingly, the municipal treasurer of Catanauan, Quezon, issued on May
agreed upon, in writing, made before the expiration of the period previously 29, 1956, a notice of sale of said properties at public auction on June 19, 1956.
stipulated, except that in the case of a false or fraudulent return with intent to Upon petition of Gancayco, filed on June 16, 1956, the Court of Tax Appeals
evade tax or of a failure to file a return, the judicial action may be begun at any issued a resolution ordering the cancellation of the sale and directing that the
time within ten (10) years after the discovery of the falsity, fraud or omission same be readvertised at a future date, in accordance with the procedure
(Section 331 and 332 of the Tax Code). established by the National Internal Revenue Code. Subsequently, or on June 22,
1956, Gancayco filed an amended petition praying that said Court:
3. COURT OF TAX APPEALS; JURISDICTION;. Republic Act No. 1125 has
vested the Court of Tax Appeals, not only with exclusive appellate jurisdiction "(a) Issue a writ of preliminary injunction, enjoining the
to review decisions of the Commissioner of Internal Revenue in cases involving respondents from enforcing the collection of the alleged tax
disputed assessments, but, also, with the authority to decide "all cases liability due from the petitioner through summary
involving disputed assessment of internal revenue taxes or customs duties proceedings pending the determination of the present case;
pending determination before the Court of First Instance" at the time of the
approval of said Act, on June 16, 1954 (Section 22, Republic Act No. 1125). "(b) After a review of the present case adjudge that the right
of the government to enforce collection of any liability due on
4. ID.; ID.; WHAT IT IMPLIES. The jurisdiction of the Court of Tax Appeals this account had already prescribed;
to decide all cases involving disputed assessments of internal revenue and
customs duties necessarily implies the power to authorize and sanction the "(c) That even assuming that prescription had not yet set in
collection of the taxes and duties involved in such assessments as may be the objections of petitioner to the disallowance of the
upheld by the Court of Tax Appeals. The same now has the authority formerly entertainment, representation and farming expenses be
vested in Courts of First Instance to hear and decide cases involving disputed allowed;
assessments of internal revenue taxes and customs duties. Inasmuch as those "xxx xxx xxx."
cases filed with Courts of First Instance constituted judicial actions, such is,
likewise, the nature of the proceedings before the Court of Tax Appeals, insofar In his answer respondent admitted some allegations of the amended petition,
as Sections 316 and 332 of the Tax Code are concerned. denied other allegations thereof and set up some special defenses. Thereafter
Gancayco received from the municipal treasurer of Catanauan, Quezon another
notice of auction sale of his properties, to take place on August 29, 1956. On
Petitioner Santiago Gancayco seeks the review of a decision of the Court of Tax motion of Gancayco, the Court of Tax Appeals, by resolution dated August 27,
Appeals, requiring him to pay P16,860.31, plus surcharge and interest, by way of 1956, "canceled" the aforementioned sale and enjoined respondent and the
deficiency income tax for the year 1949. municipal treasurer of Catanauan, Quezon, from proceeding with the same. After
appropriate proceedings, the Court of Tax Appeals rendered, on November 14,
On May 10, 1950, Gancayco filed his income tax return for the year 1949. Two (2) 1957, the decision adverted to above.
days later, respondent Collector of Internal Revenue issued the corresponding
notice advising him that his income tax liability for that year amounted Gancayco maintains that the right to collect the deficiency income tax in question
P9,793.62, which he paid on May 15, 1950. A year later, or on May 14, 1951, is barred by the statute of limitations. In this connection, it should be noted,
respondent wrote the communication Exhibit C, notifying Gancayco, inter however, that there are two (2) civil remedies for the collection of internal
alia, that, upon investigation, there was still due from him, as deficiency income revenue taxes, namely: (a) by distraint of personal property and levy upon real
tax for the year 1949, the sum of P29,554.05. Gancayco sought a reconsideration, property; and (b) by "judicial action" (Commonwealth Act 456, Section 316). The

39
TAXATION II ATTY. ACOSTA -
CAJUSTIN
first may not be availed of except within three (3) years after the "return is due or a. "The Court of Tax Appeals does not have original
has been made . . ." (Tax Code, Section 51[d]). After the expiration of said period, jurisdiction to entertain an action for the collection of the tax
income taxes may not be legally and validly collected by distraint and/or levy due;
(Collector of Internal Revenue v. Avelino, L-9202, November 19, 1956; Collector
of Internal Revenue v. Reyes, L-8685, January 31, 1957; Collector of Internal
Revenue v. Zulueta, L-8840, February 8, 1957; Sambrano v. Court of Tax
b. "The proper party to commence the judicial action to
Appeals, L-8652, March 30, 1957). Gancayco's income tax return for 1949 was
collect the tax due is the government, and
filed on May 10, 1950; so that the warrant of distraint and levy issued on May 15,
1956, long after the expiration of said three-year period, was illegal and void, and c. "The remedies provided by law for the collection of the tax
so was the attempt to sell his properties in pursuance of said warrant. are exclusive."
The "judicial action" mentioned in the Tax Code may be resorted to within five (5) Said Section 316 provides:
years from the date the return has been filed, if there has been no assessment, or
within five (5) years from the date of the assessment made within the statutory "The civil remedies for the collection of internal revenue
period, or within the period agreed upon, in writing, by the Collector of Internal taxes, fees, or charges, and any increment thereto, resulting
Revenue and the taxpayer, before the expiration of said five-year period, or within from delinquency shall be (a) by distraint of goods, chattels,
such extension of said stipulated period as may have been agreed upon, in or effects, and other personal property of whatever character,
writing, made before the expiration of the period previously stipulated, except including stocks and other securities, debts, credits, bank
that in the case of a false or fraudulent return with intent to evade tax or of a accounts, and interest in and rights to personal property, and
failure to file a return, the judicial action may be begun at anytime within ten (10) by levy upon real property; and (b) by judicial action. Either
years after the discovery of the falsity, fraud or omission (Sections 331 and 332 of of these remedies or both simultaneously may be pursued in
the Tax Code). In the case at bar, respondent made three (3) assessments: (a) the the discretion of the authorities charged with the collection of
original assessment of P9,793.62, made on May 12, 1950; (b) the first deficiency such taxes.
income tax assessment of May 14, 1951, for P29,554.05; and (c) the amended
deficiency income tax assessment of April 8, 1953, for P16,860.31. "No exemption shall be allowed against the internal revenue
taxes in any case."
Gancayco argues that the five-year period for the judicial action should be
counted from May 12, 1950, the date of the original assessment, because the Petitioner contends that the judicial action referred to in this provision is
income tax for 1949, he says, could have been collected from him since then. Said commenced by filing, with a court of first instance, of a complaint for the
assessment was, however, not for the deficiency income tax involved in this collection of taxes. This was true at the time of the approval of Commonwealth
proceedings, but for P9,793.62, which he paid forthwith. Hence, there never had Act No. 456, on June 15, 1939. However, Republic Act No. 1125 has vested the
been any cause for judicial action against him, and, per force, no statute of Court of Tax Appeals, not only with exclusive appellate jurisdiction to review
limitations to speak of, in connection with said sum of P9,793.62. decisions of the Collector (now Commissioner) of Internal Revenue in cases
involving disputed assessments, like the one at bar, but, also, with authority to
Neither could said statute have begun to run from May 14, 1951, the date of the decide "all cases involving disputed assessments of Internal Revenue taxes or
first deficiency income tax assessment for P29,554.05, because the same was, customs duties pending determination before the court of first instance" at the
upon Gancayco's request, reconsidered or modified by the assessment made on time of the approval of said Act, on June 16, 1954 (Section 22, Republic Act No.
April 8, 1953, for P16,860.31. Indeed, this last assessment is what Gancayco 1125). Moreover, this jurisdiction to decide all cases involving disputed
contested in the amended petition filed by him with the Court of Tax Appeals. The assessments of internal revenue taxes and customs duties necessarily implies the
amount involved in such assessment which Gancayco refused to pay and power to authorize and sanction the collection of the taxes and duties involved in
respondent tried to collect by warrant of distraint and/or levy, is the one in issue such assessments as may be upheld by the Court of Tax Appeals. At any rate, the
between the parties. Hence, the five-year period aforementioned should be same now has the authority formerly vested in courts of first instance to hear and
counted from April 8, 1953, so that the statute of limitations does not bar the decide cases involving disputed assessments of internal revenue taxes and
present proceedings, instituted on April 12, 1956, if the same is a judicial action, customs duties. Inasmuch as those cases filed with courts of first instance
as contemplated in Section 316 of the Tax Code, which petitioner denies, upon the constituted judicial actions, such is, likewise, the nature of the proceedings before
ground that:

40
TAXATION II ATTY. ACOSTA -
CAJUSTIN
the Court of Tax Appeals, insofar as Sections 316 and 332 of the Tax Code are We concur in this view, which is a necessary consequence of Section 31 of the Tax
concerned. Code, pursuant to which:
The question whether the sum of P16,860.31 is due from Gancayco as deficiency "(a) General Rule In computing net income no deduction
income tax for 1949 hinges on the validity of his claim for deduction of two (2) shall in any case be allowed in respect of
items, namely: (a) for farming expenses, P27,459.00; and (b) for representation
expenses, P8,933.45. "(1) Personal, living, or family expenses;

Section 30 of the Tax Code partly reads: "(2) Any amount paid out for new buildings or
for permanent improvements, or betterments made
"(a) Expenses: to increase the value of any property or estate;
(1) In General All the ordinary and necessary expenses "(3) Any amount expended in restoring property or
paid or incurred during the taxable year in carrying on any in making good the exhaustion thereof for which an
trade or business, including a reasonable allowance for allowance is or has been made; or
salaries or other compensation for personal services actually
rendered; traveling expenses while away from home in the "(4) Premiums paid on any life insurance policy covering the
pursuit of a trade or business; and rentals or other payments life of any officer or employee, or any person financially
required to be made as a condition to the continued use or interested in any trade or business carried on by the taxpayer,
possession, for the purposes of the trade or business, of individual or corporate, when the taxpayer is directly or
property to which the taxpayer has not taken or is not indirectly a beneficiary under such policy." (Emphasis
taking title or in which he has no equity." (Emphasis supplied.)
supplied.)
Said view is, likewise, in accord with the consensus of the authorities on the
Referring to the item of P27,459, for farming expenses allegedly incurred by subject.
Gancayco, the decision appealed from has the following to say:
"Expenses incident to the acquisition of property follow the
"No evidence has been presented as to the nature of the said same rule as applied to payments made as direct
'farming expenses' other than the bare statement of consideration for the property. For example, commission
petitioner that they were spent for the 'development and paid in acquiring property are considered as representing
cultivation of (his) property'. No specification has beenmade part of the cost of the property acquired. The same treatment
as to the actual amount spent for purchase of tools, is to be accorded to amounts expended for maps, abstracts,
equipment or materials, or the amount spent for legal opinions on titles, recording fees and surveys. Other
improvement. Respondent claims that the entire amount was non-deductible expenses include amounts paid in connection
spent exclusively for clearing and developing the farm which with geological explorations, development and subdividing of
were necessary to place it in a productive state. It is not, real estate;clearing and grading; restoration of soil, drilling
therefore, an ordinary expense but a capital expenditure. wells, architect's fees and similar types of expenditures." (4
Accordingly, it is not deductible but it may be amortized, in Merten's Law of Federal Income Taxation Sec. 25.20, pp.
accordance with Section 75 of Revenue Regulations No. 2, 348-349; see also sec. 75 of the Income Regulation of the
cited above. See also, Section 31 of the Revenue Code which B.I.R.; (Emphasis supplied.)
provides that in computing net income, no deduction shall in
"The cost of farm machinery, equipment and farm building
any case be allowed in respect of any amount paid out
represents a capital investment and is not an allowable
for new buildings or for permanent improvements or
deduction as an item of expense. Amounts expended in
betterments made to increase the value of any property or
the development of farms, orchards, and ranches prior to the
estate." (Emphasis supplied.)
time when the productive state is reached may be regarded

41
TAXATION II ATTY. ACOSTA -
CAJUSTIN
as investments of capital." (Merten's Law of Federal Income
Taxation, supra, Sec. 25.108, p. 525.)
"Expenses for clearing off and grading lots acquired is
a capital expenditure, representing part of the cost of the
land and was not deductible as an expense." (Liberty Baking
Co. v. Heiner, 37 F [2d] 703 [8 AFTR 10011] [CCA 3rd]; The
B.L. Marble Chair Company v. U.S., 15 AFTR 746)
"An item of expenditure, in order to be deductible under this
section of the statute providing for the deduction
ofordinary and necessary business expenses, must
fall squarely within the language of the statutory provision.
This section is intended primarily although not always TOPIC: DEDUCTIONS
necessarily, to cover expenditures of a recurring nature
where the benefit derived from the payment is realized and INTEREST
exhausted within the taxable year. Accordingly, if the result
of the expenditure is the acquisition of an asset which has an
economically useful life beyond the taxable year, [G.R. No. L-16626. October 29, 1966.]
nodeduction of such payment may be obtained under the
provisions of the statute. In such cases, to the extent that a
deduction is allowable, it must be obtained under the COMMISSIONER OF INTERNAL
provisions of the statute which permit deductions for REVENUE, petitioner, vs. CARLOS PALANCA,
amortization, depreciation, depletion or loss." (W. B. JR., respondent.
Harbeson Co. 24 BTA, 542; Clark Thread Co., 28 BTA 1128
aff'd 100 F [2d] 257 CCA 3rd, 1938) 4 Merten's Law of
Federal Income Taxation, Sec. 25.17, pp. 337-338.)" FACTS:
Gancayco's claim for representation expenses aggregated P31,753.97, of which The late Don Carlos Palanca, Sr. donated in favor of his son, Carlos Palanca, Jr.
P22,820.52 was allowed, and P8,933.45 disallowed. Such disallowance is justified shares of stock in La Tondea Inc. amounting to 12,500 shares. Later, the BIR
by the record, for, apart from the absence of receipts, invoices or vouchers of the considered the donation as transfer in contemplation of death; consequently,
expenditures in question, petitioner could not specify the items constituting the the BIR assessed against the respondent, Palanca Jr., the sum of P191,591.62
same, or when or on whom or on what they were incurred. The case of Cohan v. as estate and inheritance taxes on the transfer of said 12,500 shares of stock,
Commissioner, 39 F (2d) 540, cited by petitioner is not in point, because in that including therein interest for delinquency of P60,581.80. The respondent then
case there was evidence on the amounts spent and the persons entertained and filed an amended income tax return, claiming an additional deduction in the
the necessity of entertaining them, although there were not receipts and vouchers amount P60,581.80; hence, his new income tax due is only P428. He attached
of the expenditures involved therein. Such is not the case of petitioner herein. a letter requesting the refund of P20,624.01. However, the said request for
refund was denied by the BIR. Court of tax appeals ordered the refund. Hence,
Being in accordance with the facts and law, the decision of the Court of Tax this petition.
Appeals is hereby affirmed therefore, with costs against petitioner Santiago
Gancayco. It is so ordered. ISSUES:

1. Whether the interest on the delinquent estate and inheritance tax is


deductible from the gross income

42
TAXATION II ATTY. ACOSTA -
CAJUSTIN
a broad sense, a debt." (Idem)
2. Whether the respondents claim for refund has prescribed "Some American authorities hold that, especially for remedial purposes,
Federal taxes are debts." (Tax Commission vs. National Malleable Castings Co.,
RULING: 35 ALR 1448.)
2. ID.; ID.; ID.; RULE IN OUR JURISDICTION. In our jurisdiction, the rule
1. Yes, the interest is deductible. The rule is settled that although taxes is settled that although taxes already due have not, strictly speaking, the same
already due have not, strictly speaking, the same concept as debts, they are however, obligations that may be considered as such.
concept as debts, they are, however, obligations that may be (Sambrano vs. Court of Tax Appeals, G. R. No. L-8652, March 30, 1957). In a
considered as such. In CIR v Prieto, the Court explicitly announced more recent case, Commissioner of Internal Revenue vs. Prieto, G. R. No. L-
that while the distinction between taxes and debts was recognized 13192, September 30, 1960, we explicitly announced that while the distinction
in this jurisdiction, the variance in their legal conception does not between "taxes" and "debts" was recognized in this jurisdiction, the variance in
extend to the interests paid on them. their legal conception does not extend to the interests paid on them, at least
insofar as section 30(b) (1) of the National Internal Revenue Code is
2. No, respondents claim has not yet prescribed. Considering that it is concerned.
the interest paid on this latter-assessed estate and inheritance tax that
respondent is claiming for refund, then the 30-day period for 3. ID.; INTERESTS ON TAXES TO BE CONSIDERED AS INTERESTS ON
prescription under RA 1125 should be computed from the receipt of INDEBTEDNESS. The taxpayer sought the allowance as deductible items
the final denial by the BIR of the said claim. from the gross income of the amounts paid by them as interests on delinquent
tax liabilities. Of course, what was involved in the cited case (Prieto case) was
the donor's tax while the present suit pertains to interest paid on the estafa and
Inasmuch as the said account was paid by him by installment, then inheritance tax. This difference, however, submits no appreciable consequence
the computation of the two-year prescriptive period, under Section to the rationale of this Court's previous determination that interests on taxes
306 of the National Internal Revenue Code, should be from the date should be considered as interests on indebtedness within the meaning of
of the last installment. section 30(b) (1) of the Tax Code. The interpretation we have placed upon the
said section was predicated on the congressional intent, not on the nature of
Respondent Palanca paid the last installment on his 1955 income tax the tax for which the interest was paid.
account on August 14, 1956. His claim for refund was filed on August
13, 1958. It was, therefore, still timely instituted. 4. ID.; PRESCRIPTION OF CLAIM FOR REFUND OF TAX UNDER SEC. 11
OF REP. ACT 1125 AND UNDER SEC. 306 OF THE INTERNAL REVENUE
CODE. The 30-day period under Section 11 of Republic Act 1125 should be
computed from the receipt of the final denial by the Bureau of Internal
SYLLABUS
Revenue of the said claim. Palanca's claim in this incident was filed with the
Court of Tax Appeals even before it had been denied by the herein petitioner or
1. TAXATION; TAXES AND DEBTS AS LEGAL CONCEPTS DISTINGUISHED. the Bureau of Internal Revenue, i.e., the claim was filed with the former Court
While taxes and debts are distinguishable legal concepts, in certain cases as on Aug. 13, 1958, while the petitioner denied it on July 24, 1959. The claim at
in the suit at bar, on account of their nature, the distinction becomes bar refers to the alleged overpayment by respondents of the 1955 income tax.
inconsequential. This qualification is recognized even in the United States. Inasmuch as the said account was paid by him by installment, then the
Thus, "The term debt is properly used in a comprehensive sense as embracing computation of the two-year prescriptive period, under Section 306 of the
not merely money due by contract, but whatever one is bound to render to National Internal Revenue Code, should be from the date of the last
another, either for contract or the requirement of the law. (Camden vs. Fink installment, (Antonio Prieto, et al. vs. Collector of Internal Revenue, G. R.
Coule and Coke Co., 61 ALR 584). No. L-11976, August 29, 1961). Palanca paid the last installment on his 1955
income tax on August 14, 1956. His claim for refund of the alleged
"Where statutes impose a personal liability for a tax, the tax becomes at least in overpayment was filed with the Court on August 13, 1958. It was, therefore,

43
TAXATION II ATTY. ACOSTA -
CAJUSTIN
still timely instituted. original return and of P3,167.00, was filed together with this
amended return. In a communication dated June 20, 1957,
the respondent (BIR) denied the claim for refund.
This is an appeal by the Government from the decision of the Court of Tax "On August 27, 1957, the petitioner reiterated his claim for
Appeals in CTA Case No. 571 ordering the petitioner to refund to the respondent refund, and at the same time requested that the case be
the amount of P20,624.01 representing alleged overpayment of income taxes for elevated to the Appellate Division of the Bureau of Internal
the calendar year 1955. The facts are: Revenue for decision. The reiterated claim was denied on
October 14, 1957.
"Sometime in July, 1950, the late Don Carlos Palanca, Sr.
donated in favor of his son, the petitioner herein, shares of "On November 2, 1957, the petitioner requested that the case
stock in La Tondea, Inc. amounting to 12,500 shares. For be referred to the Conference Staff of the Bureau of Internal
failure to file a return on the donation within the statutory Revenue for review. Later, on November 6, 1957, he
period, the petitioner was assessed the sums of P97,691.23, requested the respondent to hold his action on the case in
P24,442.81 and P47,868.70 as gift tax, 25% surcharge and abeyance until after the Court of Tax Appeals renders its
interest, respectively, which he paid on June 22, 1955. decision on a similar case. And on November 7, 1957, the
respondent denied the claim for the refund of the sum of
"On March 1, 1956, the petitioner filed with the Bureau of P17,885.01.
Internal Revenue his income tax return for the calendar year
1955, claiming, among others, a deduction for interest "Meanwhile, the Bureau of Internal Revenue considered the
amounting to P9,706.45 and reporting a taxable income of transfer of 12,500 shares of stock of La Tondea, Inc. to be a
P65,982.12. On the basis of this return, he was assessed the transfer in contemplation of death pursuant to section 88(b)
sum of P21,052.91*, as income tax, which he paid, as follows: of the National Internal Revenue Code. Consequently the
respondent assessed against the petitioner the sum of
Taxes withheld by the La Tondea Inc. P191,591.62 as estate and inheritance taxes on the transfer of
from Mr.Palanca's wage P13,172.41 said 12,500 shares of stock. The amount of P170,002.74 paid
on June 22, 1955 by the petitioner as gift tax, including
Payment under Income Tax Receipt
interest and surcharge, under Official Receipt No. 2855 was
No. 677359 dated May 11, 1956 3,939.80 applied to his estate and inheritance tax liability. On the tax
liability of P191,591.62, the petitioner paid the amount of
Payment under Income Tax Receipt
P60,581.80 as interest for delinquency as follows:
No. 742334 dated August 14, 1956 3,939.80
1 % monthly interest on P76,724.38
Total P21,052.01
September 2, 1952 to February 16, 1955
"Subsequently, on November 10, 1956, the petitioner filed an P22,633.69
amended return for the calendar year 1955, claiming therein
an additional deduction in the amount of P47,868.70 1 % monthly interest on P71,264.77
representing interest paid on the donee's gift tax, thereby February 16, 1955 to March 31, 1955 1,068.97
reporting a taxable net income of P18,113.42 and a tax due
thereon in the sum of P3,167.00. The claim for deduction was 1 % monthly interest on P114,867.24
based on the provisions of Section 30(b)(1) of the Tax Code, September 2, 1952 to April 16, 1953 4,287.99
which authorizes the deduction from gross income of interest
paid within the taxable year on indebtedness. A claim for the 1 % monthly interest on P50,832.77
refund of alleged overpaid income taxes for the year 1955 March 31, 1955 to June 22, 1955 1,372.48
amounting to P17,885.01, which is the difference between the
amount of P21,052.01 he paid as income taxes under his 1 % monthly interest on P119,155.23

44
TAXATION II ATTY. ACOSTA -
CAJUSTIN
April 16, 1953 to June 22, 1955 31,218.67 We find for the respondents. While "taxes" and "debt" are distinguishable legal
concepts, in certain cases as in the suit at bar, on account of their nature, the

distinction becomes inconsequential. This qualification is recognized even in the
Total P60,581.80 United States. Thus,
"On August 12, 1958, the petitioner once more filed an "The term 'debt' is properly used in a comprehensive sense as
amended income tax return for the calendar year 1955, embracing not merely money due by contract, but whatever
claiming, in addition to the interest deduction of P9,076.45 one is bound to render to another, either for contract or the
appearing in his original return, a deduction in the amount of requirements of the law. (Camden vs. Fink Coule and Coke
P60,581.80, representing interest on the estate and Co., 61 ALR 584).
inheritance taxes on the 12,500 shares of stock, thereby
reporting a net taxable income for 1955 in the amount of "Where statutes impose a personal liability for a tax, the tax
P5,400.32 and an income tax due thereon in the sum of becomes at least in a broad sense, a debt." (Idem.)
P428.00. Attached to this amended return was a letter of the
petitioner, dated August 11, 1958, wherein he requested the "Some American authorities hold that, especially for remedial
refund of P20,624.01 which is the difference between the purposes, Federal taxes are debts." (Tax Commissionvs.
amounts of P21,052.01 he paid as income tax under his National Malleable Castings Co., 35 ALR 1448)
original return and of P428.00.
In our jurisdiction, the rule is settled that although taxes already due have not,
"Without waiting for the respondent's decision on this claim strictly speaking, the same concept as debts, they are, however, obligations that
for refund, the petitioner filed his petition for review before may be considered as such. (Sambrano vs. Court of Tax Appeals, 101 Phil. 1). In a
this Court on August 13, 1958. On July 24, 1959, the more recent case, Commissioner of Internal Revenue vs. Prieto, 109 Phil. 592, we
respondent denied the petitioner's request for the refund of explicitly announced that while the distinction between "taxes" and "debts" was
the sum of P20,624.01." recognized in this jurisdiction, the variance in their legal conception does not
extend to the interests paid on them, at least insofar as Section 30(b) (1) of the
The Commissioner of Internal Revenue now seeks the reversal of the Court of Tax National Internal Revenue Code is concerned. Thus,
Appeal's ruling on the aforementioned petition for review. Specifically, he takes
issue with the said court's determination that the amount paid by respondent "Under the law, for interest to be deductible, it must be
Palanca for interest on his delinquent estate and inheritance tax is deductible shown that there be an indebtedness, that there should be
from the gross income for that year under section 30(b)(1) of the Revenue Code, interest upon it, and that what is claimed as an interest
and, that said respondent's claim for refund therefor has not prescribed. deduction should have been paid or accrued within the year.
It is here conceded that the interest paid by respondent was
On the first point, the Commissioner urges that a tax is not an indebtedness. in consequence of the late payment of her donor's tax, and
Citing American cases, he argues that there is a material and fundamental the same was paid within the year it is sought to be deducted.
distinction between a "tax" and a "debt." (Meriwather vs. Garrett, 102 U.S. 427; The only question to be determined, as stated by the parties,
Liberty Mutual Ins. Co. vs. Johnson Shipyards Corporation, 5 AFTR pp. 5504, is whether or not such interest was paid upon an
5507; City of Camden vs. Allen, 26, N. J. Law, p. 398). He adopts the view that indebtedness within the contemplation of Section (30) (b) (1)
"debts are due to the government in its corporate capacity, while taxes are due to of the Tax Code, the pertinent part of which reads:
the government in its sovereign capacity. A debt is a sum of money due upon
contract express or implied or one which is evidenced by a judgment. Taxes are "Sec. 30. Deductions from gross income.
imposts levied by government for its support or some special purpose which the In computing net income there shall be allowed as
government has recognized." In view of the distinction, then, the Commissioner deductions
submits that the deductibility of "interest on indebtedness" from a person's
xxx xxx xxx
income tax under section 30(b)(1) cannot extend to "interest on taxes."
'Interest:

45
TAXATION II ATTY. ACOSTA -
CAJUSTIN
'(1) In general. The amount of interest denied by the petitioner for the first time on June 20, 1957. Thereafter, the said
paid within the taxable year on indebtedness, except denial was twice reiterated: on October 14, 1957 and November 7, 1957, upon
on indebtedness incurred or continued to purchase respondent Palanca's plea for the reconsideration of the ruling of June 20, 1957.
or carry obligations the interest upon which is The second claim was filed with the Court of Tax Appeals on August 13, 1958, or
exempt from taxation as income under this Title. even before the same had been denied by the petitioner. Respondent Palanca's
second claim was denied by the latter on July 24, 1959.
'The term "indebtedness" as used in the
Tax Code of the United States containing similar The petitioner contends that under Section 11 of Republic Act 1124, 1 the herein
provisions as in the above-quoted section has been claimant's claim for refund has prescribed since the same was filed outside the
defined as the unconditional and legally enforceable 30-day period provided for therein. According to the petitioner, the said
obligation for the payment of money. (Federal Taxes prescriptive period commenced to run on October 14, 1947 when the denial by the
Vol. 2, p. 13,019, Prentice Hall, Inc.; Merterns' Law Bureau of Internal Revenue of the respondent Palanca's claim for refund, under
of Federal Income Taxation, Vol. 4 p. 542.) Within his letter of November 10, 1956, became final. Considering that the case was filed
the meaning of that definition it is apparent that a with the Court of Tax Appeals only on August 13, 1958, then it is urged that the
tax may be considered an indebtedness . . . same had prescribed.
(Emphasis supplied)
The petitioner also invokes prescription, at least with respect to the sum of
'It follows that the interest paid by herein P17,112.21, under Section 306 of the Tax Code. 2 He claims that for the calendar
respondent for the late payment of her donor's tax is year 1955, respondent Palanca paid his income tax as follows:
deductible from her gross income under Section
30(b) of the Tax Code above-quoted.'" Taxes withheld by La Tondea, Inc. from
Mr. Palanca's wages P13,172.41
We do not see any element in this case which can justify a departure from or
abandonment of the doctrine in the Prieto case above. In both this and the said Payment under Income Tax Receipt No.
case, the taxpayer sought the allowance as deductible items from the gross income
677395 dated May 11, 1956 3,939.89 *
of the amounts paid by them as interests on delinquent tax liabilities. Of course,
what was involved in the cited case was the donor's tax while the present suit Payment under Income Tax Receipt No.
pertains to interest paid on the estate and inheritance tax. This difference, 742334 dated August 14, 1956 3,939.89
however, submits no appreciable consequence to the rationale of this Court's
previous determination that interests on taxes should be considered as interest on
indebtedness within the meaning of Section 30(b)(1) of the Tax Code. The P21,952.01**
interpretation we have placed upon the said section was predicated on the
congressional intent, not on the nature of the tax for which the interest was paid. Therefore, the petitioner contends, the amounts paid by claimant Palanca under
his withheld tax and under Receipt No. 677395 dated May 11, 1956 may no longer
On the issue of prescription: There were actually two claims for refund filed by the be refunded since the claim therefor was filed in court only on August 13, 1958, or
herein respondent, Carlos Palanca, Jr., anent the case at bar. The first one was on beyond two years of their payment.
November 10, 1956, when he filed a claim for refund on the interest paid by him
on the donee's gift tax of P17,885.01, as originally demanded by the Bureau of We find the petitioner's contention on prescription untenable.
Internal Revenue. The second one was the one filed by him on August 12, 1958,
which was a claim for refund on the interest paid by him on the estate and In the first place, the 30-day period under Section 11 of Republic Act 1125 did not
inheritance tax assessed by the same Bureau in the amount of P20,624.01. even commence to run in this incident. It should be recalled that while the herein
Actually, this second assessment by the Bureau was for the same transaction as petitioner originally assessed the respondent-claimant for alleged gift tax
that for which they assessed respondent Palanca the above donee's gift tax. The liabilities, the said assessment was subsequently abandoned and in its lieu, a new
Bureau, however, on further consideration, decided that the donation of the one was prepared and served on the respondent-taxpayer. In this new
stocks in question was made in contemplation of death, and hence, should be assessment, the petitioner charged the said respondent with an entirely new
assessed as an inheritance. Thus the second assessment. The first claim was liability and for a substantially different amount from the first. While initially the
petitioner assessed the respondent for donee's gift tax in the amount of

46
TAXATION II ATTY. ACOSTA -
CAJUSTIN
P170,002.74, in the subsequent assessment the latter was asked to pay Paper Industries Corporation of the Philippines (PICOP) is a Philippine
P191,591.62 for delinquent estate and inheritance tax. Considering that it is the Corporation registered with Board of Investment (BOI) as preferred pioneer
interest paid on this latter-assessed estate and inheritance tax that respondent enterprise with respect to its integrated pulp and paper mill, and as preferred
Palanca is claiming refund for, then the 30-day period under the above- non-pioneer enterprise with respect to its integrated plywood and veneer mills.
mentioned section of Republic Act 1125 should be computed from the receipt of
the final denial by the Bureau of Internal Revenue of the said claim. As has earlier PICOP received from CIR two (2) letters of assessment (a) for deficiency
been recited, respondent Palanca's claim in this incident was filed with the Court transaction tax and for documentary and science stamp tax (b) deficiency
of Tax Appeals even before it had been denied by the herein petitioner or the income tax for 1977.
Bureau of Internal Revenue. The case was filed with the said court on August 13,
1958 while the petitioner denied the claim subject of the said case only on July 24, PICOP maintains that it is not liable at all to pay any of the assessments or any
1959. part thereof. PICOP questions the imposition by the CA of the deficiency
income tax resulting from disallowance of certain claimed financial guarantee
In the second place, the claim at bar refers to the alleged overpayment by expense and claimed year-end adjustment of sales and cost of sales.
respondent Palanca of his 1955 income tax. Inasmuch as the said account was
paid by him by installment, then the computation of the 2-year prescriptive ISSUE: Whether PICOP is liable for: (1) the 35% transaction; (2) interest and
period, under Section 306 of the National Internal Revenue Code, should be from surcharge on unpaid transaction tax.
the date of the last installment. (Antonio Prieto et al vs. Collector of Internal
Revenue, G. R. No. L-11976, August 29, 1961). Respondent Palanca paid the last HELD:
installment on his 1955 income tax account on August 14, 1956. His claim for
refund of the alleged overpayment on it was filed with the court on August 13, PICOP is liable for the 35% transaction tax due in respect of interest payments
1958. It was, therefore, still timely instituted. on its money market borrowings.
WHEREFORE, the decision appealed from is affirmed in full, without
pronouncement on costs. The transaction tax maybe levied only in respect of the interest earnings of
PICOP money market lenders accruing after PD No. 1154 went into effect, and
\
not in respect of all the 1977 interest earning of such lenders.
[G.R. Nos. 106949-50. December 1, 1995.] Facts:
On various years (1969, 1972 and 1977), Picop obtained loans from
PAPER INDUSTRIES CORPORATION OF THE foreign creditors in order to finance the purchase of machinery and equipment
PHILIPPINES (PICOP), petitioner, vs. COURT OF needed for its operations. In its 1977 Income Tax Return, Picop claimed
APPEALS, COMMISSIONER OF INTERNAL interest payments made in 1977, amounting to P42,840,131.00, on these loans
REVENUE and COURT OF TAX as a deduction from its 1977 gross income.
APPEALS, respondents. The CIR disallowed this deduction upon the ground that, because the
loans had been incurred for the purchase of machinery and equipment, the
interest payments on those loans should have been capitalized instead and
claimed as a depreciation deduction taking into account the adjusted basis of
[G.R. Nos. 106984-85. December 1, 1995.] the machinery and equipment (original acquisition cost plus interest charges)
over the useful life of such assets.
COMMISSIONER OF INTERNAL Both the CTA and the Court of Appeals sustained the position of Picop
REVENUE, petitioner, vs. PAPER INDUSTRIES and held that the interest deduction claimed by Picop was proper and
CORPORATION OF THE PHILIPPINES (PICOP), allowable. In the instant Petition, the CIR insists on its original position.
THE COURT OF APPEALS, and THE COURT OF TAX ISSUE:
APPEALS, respondents.

47
TAXATION II ATTY. ACOSTA -
CAJUSTIN
Whether Picop is entitled to deductions against income of interest Only if sir asks: (For further discussion of CIRs contention)
payments on loans for the purchase of machinery and equipment. It is claimed by the CIR that Section 79 of Revenue Regulations No. 2 was
HELD: "patterned after" paragraph 1.266-1 (b), entitled "Taxes and Carrying Charges
YES. Interest payments on loans incurred by a taxpayer (whether Chargeable to Capital Account and Treated as Capital Items" of the U.S.
BOI-registered or not) are allowed by the NIRC as deductions against the Income Tax Regulations, which paragraph reads as follows:
taxpayer's gross income. The basis is 1977 Tax Code Sec. 30 (b). 1 Thus, the (B) Taxes and Carrying Charges. The items thus
general rule is that interest expenses are deductible against gross income and chargeable to capital accounts are
this certainly includes interest paid under loans incurred in connection with
the carrying on of the business of the taxpayer. In the instant case, the CIR (11) In the case of real property, whether improved or
does not dispute that the interest payments were made by Picop on unimproved and whether productive or nonproductive.
loans incurred in connection with the carrying on of the registered operations
of Picop, i.e., the financing of the purchase of machinery and equipment
actually used in the registered operations of Picop. Neither does the CIR deny (a) Interest on a loan (but not theoretical interest of a
that such interest payments were legally due and demandable under the terms taxpayer using his own funds).
of such loans, and in fact paid by Picop during the tax year 1977.
The contention of CIR does not spring of the 1977 Tax Code but from Revenue The truncated excerpt of the U.S. Income Tax Regulations quoted by the CIR
Regulations 2 Sec. 79.2 However, the Court said that the term interest here needs to be related to the relevant provisions of the U.S. Internal Revenue
should be construed as the so-called "theoretical interest," that is to say, Code, which provisions deal with the general topic of adjusted basis for
interest "calculated" or computed (and not incurred or paid) for the determining allowable gain or loss on sales or exchanges of property and
purpose of determining the "opportunity cost" of investing funds in allowable depreciation and depletion of capital assets of the taxpayer:
a given business. Such "theoretical" or imputed interest does not
arise from a legally demandable interest-bearing obligation
Present Rule. The Internal Revenue Code, and the
incurred by the taxpayer who however wishes to find out, e.g., whether he
Regulations promulgated thereunder provide that "No
would have been better off by lending out his funds and earning interest rather
deduction shall be allowed for amounts paid or
than investing such funds in his business. One thing that Section 79 quoted
accrued for such taxes and carrying charges as, under
above makes clear is that interest which does constitute a charge arising under
regulations prescribed by the Secretary or his delegate, are
an interest-bearing obligation is an allowable deduction from gross income.
chargeable to capital account with respect to property, if the
taxpayer elects, in accordance with such regulations, to
1 Sec. 30. Deduction from Gross Income. The following treat such taxes orcharges as so chargeable."
may be deducted from gross income:xxx xxx xxx
(b) Interest: At the same time, under the adjustment of basis provisions
(1) In general. The amount of interest paid within which have just been discussed, it is provided that
the taxable year on indebtedness, except on indebtedness adjustment shall be made for all "expenditures, receipts,
losses, or other items" properly chargeable to a capital
incurred or continued to purchase or carry obligations the account, thus including taxes and carrying charges;
interest upon which is exempt from taxation as income however, an exception exists, in which event such
under this Title: . . . (Emphasis supplied) adjustment to the capital account is not made, with respect
to taxes and carrying charges which the taxpayer has not
elected to capitalize but for which a deduction instead has
been taken. 22 (Emphasis supplied)

2 Sec. 79. Interest on Capital. Interest calculated for cost-keeping or other purposes The "carrying charges" which may be capitalized under the above
on account of capital or surplus invested in the business, which does not represent a quoted provisions of the U.S. Internal Revenue Code include, as the
charge arising under an interest-bearing obligation, is not allowable deduction from
CIR has pointed out, interest on a loan "(but not theoretical interest
gross income. (Emphases supplied)

48
TAXATION II ATTY. ACOSTA -
CAJUSTIN
of a taxpayer using his own funds)." What the CIR failed to point out Internal Revenue ("BIR"). As a withholding agent, Picop is madepersonally
is that such "carrying charges" may, at the election of the liable for the thirty-five percent (35%) transaction tax and if it did not actually
taxpayer, either be (a) capitalized in which case the cost basis of the withhold thirty-five percent (35%) of the interest monies it had paid to its
capital assets, e.g., machinery and equipment, will be adjusted by lenders, Picop had only itself to blame. We conclude that Picop was properly
adding the amount of such interest payments or alternatively, be (b) held liable for the thirty-five percent (35%) transaction tax due in respect of
deducted from gross income of the taxpayer. Should the taxpayer interest payments on its money market borrowings. AHacIS
elect to deduct the interest payments against its gross income, the
taxpayer cannot at the same time capitalize the interest payments. In 2. ID.; PRESIDENTIAL DECREE NO. 1154; 35% TRANSACTION TAX ON
other words, the taxpayer is not entitled to both the deduction from COMMERCIAL PAPER; WITH NO RETROACTIVE APPLICATION. The
gross income and the adjusted (increased) basis for determining gain transaction tax may be levied only in respect of the interest earnings of Picop's
or loss and the allowable depreciation charge. The U.S. Internal money market lenders accruing after P.D. No. 1154 went into effect, and not in
Revenue Code does not prohibit the deduction of interest on a respect of all the 1977 interest earnings of such lenders. P.D. No. 1154 is not, in
loan obtained for purchasing machinery and equipment against gross other words, to be given retroactive effect by imposing the thirty-five percent
income, unless the taxpayer has also or previously capitalized the (35%) transaction tax in respect of interest earnings which accrued before the
same interest payments and thereby adjusted the cost basis of such effectivity date of P.D. No. 1154, there being nothing in the statute to suggest
assets. that the legislative authority intended to bring about such retroactive
imposition of the tax.
3. ID.; NATIONAL INTERNAL REVENUE CODE; AUTHORITY OF THE
SYLLABUS SECRETARY OF FINANCE TO PROMULGATE RULES AND REGULATIONS;
AUTHORITY TO IMPOSE CIVIL PENALTIES MUST BE EXPRESSLY GIVEN
BY THE ENABLING STATUTE; IMPOSITION OF 25% SURCHARGE AND
1. TAXATION; RA 5186 (INVESTMENT INCENTIVES ACT); EXEMPTION OF
14% INTEREST PER ANNUM FOR NON-PAYMENT OF TRANSACTION,
PIONEER ENTERPRISES FROM ALL TAXES UNDER THE NIRC EXCEPT
WITHOUT LEGAL BASIS. With respect to the transaction tax due, the CIR
INCOME TAX; NOT EXEMPT FROM PAYMENT OF TRANSACTION TAX
prays that Picop be held liable for a twenty-five percent (25%) surcharge and
WHICH IS INCOME TAX. We agree with the CTA and the Court of Appeals
for interest at the rate of fourteen percent (14%) per annum from the date
that Picop's tax exemption under RA. No. 5186, as amended, does notinclude
prescribed for its payment. In so praying, the CIR relies upon Section 10 of
exemption from the thirty-five percent (35%) transaction tax. In the first place,
Revenue Regulation 7-77 dated 3 June 1977, issued by the Secretary of
the thirty-five percent (35%) transaction tax is an income tax, that is, it is a tax
Finance. The 1977 Tax Code itself, in Section 326 in relation to Section 4 of the
on the interest income of the lenders or creditors. The 35% transaction tax is
same Code, invoked by the Secretary of Finance in issuing Revenue Regulation
imposed on interest income from commercial papers issued in the primary
7-77, set out, in comprehensive terms, the rule-making authority of the
money market. Being a tax on interest, it is a tax on income. The 35%
Secretary of Finance. Section 4 of the same Code contains a list of subjects or
transaction tax is an income tax on interest earnings to the lenders or
areas to be dealt with by the Secretary of Finance through the medium of an
placers. The latter are actually the taxpayers. Therefore, the tax cannot be a
exercise of his quasi-legislative or rule-making authority. This list, however,
tax imposed upon the petitioner. In other words, the petitioner who borrowed
while it purports to be open-ended, does not include the imposition of
funds from several financial institutions by issuing commercial papers
administrative or civil penalties such as the payment of amounts additional to
merely withheld the 35% transaction tax before paying to the financial
the tax due. Thus, in order that it may be held to be legally effective in respect
institutions the interests earned by them and later remitted the same to the
of Picop in the present case, Section 10 of Revenue Regulation 7-77 must
respondent Commissioner of Internal Revenue. The tax could have been
embody or rest upon some provision in the Tax Code itself which imposes
collected by a different procedure but the statute chose this method. Whatever
surcharge and penalty interest for failure to make a transaction tax payment
collecting procedure is adopted does not change the nature of the tax. It is thus
when due. P.D. No. 1154 did not itself impose, nor did it expressly authorize
clear that the transaction tax is an income tax and as such, in any event, falls
the imposition of, a surcharge and penalty interest in case of failure to pay the
outside the scope of the tax exemption granted to registered pioneer
thirty-five percent (35%) transaction tax when due. Neither did Section 210(b)
enterprises by Section 8 of RA. No. 5186, as amended. Picop was the
of the 1977 Tax Codewhich re-enacted Section 195-C inserted into the Tax Code
withholding agent, obliged to withhold thirty-five percent (35%) of the interest
by P.D. No. 1154. It will be seen that Section 51(c)(1) and (e)(1) and (3), of
payable to its lenders and to remit the amounts so withheld to the Bureau of
the 1977 Tax Code, authorize the imposition of surcharge and interest only in

49
TAXATION II ATTY. ACOSTA -
CAJUSTIN
respect of a "tax imposed by this Title," that is to say, Title II on "Income raising money by issuing bonds or other instruments to the general public. We
Tax." It will also be seen that Section 72 of the 1977 Tax Code imposes a consider that the actual dedication of the proceeds of the bonds to the carrying
surcharge only in case of failure to file a return or list "required by this Title," out of Picop's registered operations constituted a sufficient nexus with such
that is, Title II on "Income Tax." The thirty-five percent (35%) transaction tax registered operations so as to exempt Picop from stamp taxes ordinarily
is, however, imposed in the 1977 Tax Code by Section 210(b) thereof which imposed upon or in connection with issuance of such bonds. We agree,
Section is embraced in Title V on "Taxes on Business" of that Code. Thus, therefore, with the Court of Appeals on this matter that the CTA and the CIR
while the thirty-five percent (35%) transaction tax is in truth a tax imposed had erred in rejecting Picop's claim for exemption from stamp taxes. It remains
on interest income earned by lenders or creditors purchasing commercial only to note that after commencement of the present litigation before the CTA,
paper on the money market, the relevant provisions, i.e., Section 210(b), the BIR took the position that the tax exemption granted by RA No. 5186, as
were not inserted in Title II of the 1977 Tax Code. The end result is that the amended, does include exemption from documentary stamp taxes on
thirty-five percent (35%) transaction tax is not one of the taxes in respect of transactions entered into by BOI-registered enterprises. BIR Ruling No. 088,
which Section 51(e) authorized the imposition of surcharge and interest and dated 28 April 1989, for instance, held that a registered preferred pioneer
Section 72 the imposition of a fraud surcharge. It is not without reluctance that enterprise engaged in the manufacture of integrated circuits, magnetic heads,
we reach the above conclusion on the basis of what may well have been an printed circuit boards, etc., is exempt from the payment of documentary stamp
inadvertent error in legislative draftsmanship, a type of error common enough taxes. Similarly, in BIR Ruling No. 013, dated 6 February 1989, the
during the period of Martial Law in our country. Nevertheless, we are Commissioner held that a registered pioneer enterprise producing polyester
compelled to adopt this conclusion. We consider that the authority to impose filament yarn was entitled to exemption "from the documentary stamp tax on
what the present Tax Code calls (in Section 248) civil penalties consisting of [its] sale of real property in Makati up to December 31, 1989." It appears clear
additions to the tax due, must be expressly given in the enabling statute, in to the Court that the CIR, administratively at least, no longer insists on the
language too clear to be mistaken. The grant of that authority is not lightly to position it originally took in the instant case before the CTA. TSEcAD
be assumed to have been made to administrative officials, even to one as highly
placed as the Secretary of Finance. aIcHSC
4. ID.; ID.; SECTION 247 (A) IMPOSING CIVIL PENALTIES AS SURCHARGE 6. ID.; TAX EXEMPTIONS; STRICTLY CONSTRUED. Tax exemptions are,
AND INTEREST WITHOUT RETROACTIVE APPLICATION. The state of to be sure, to be "strictly construed," that is, they are not to be extended
the present law tends to reinforce our conclusion that Section 51 (c) and (e) of beyond the ordinary and reasonable intendment of the language actually used
the 1977 Tax Code did not authorize the imposition of a surcharge and penalty by the legislative authority in granting the exemption.
interest for failure to pay the thirty-five percent (35%) transaction tax imposed
7. ID.; NATIONAL INTERNAL REVENUE CODE; GROSS INCOME;
under Section 210 (b) of the same Code. The corresponding provision in
INTEREST PAYMENTS ON LOANS, DEDUCTIBLE. Interest payments on
the current Tax Code very clearly embraces failure to pay all taxes imposed in
loans incurred by a taxpayer (whether BOI-registered or not) are allowed by
the Tax Code, without any regard to the Title of the Code where provisions
the NIRC as deductions against the taxpayer's gross income. (Section 30 of
imposing particular taxes are textually located. In other words, Section 247 (a)
the 1977 Tax Code) Thus, the general rule is that interest expenses are
of the current NIRC supplies what did not exist back in 1977 when Picop's
deductible against gross income and this certainly includes interest paid under
liability for the thirty-five percent (35%) transaction tax became fixed. We do
loans incurred in connection with the carrying on of the business of the
not believe we can fill that legislative lacuna by judicial fiat. There is nothing to
taxpayer. Our 1977 NIRC does not prohibit the deduction of interest on a loan
suggest that Section 247 (a) of the present Tax Code, which was inserted in
incurred for acquiring machinery and equipment. Neither does our 1977
1985, was intended to be given retroactive application by the legislative
NIRC compel the capitalization of interest payments on such a loan. The 1977
authority.
Tax Code is simply silent on a taxpayer's right to elect one or the other tax
5. ID.; RA 5186 (INVESTMENT INCENTIVES ACT); EXEMPTION OF treatment of such interest payments. Accordingly, the general rule that interest
PIONEER ENTERPRISES FROM ALL TAXES UNDER THE NIRC EXCEPT payments on a legally demandable loan are deductible from gross income must
INCOME TAX; EXEMPTION INCLUDES PAYMENT FROM DOCUMENTARY be applied.
STAMP TAXES. The issuance of debenture bonds is certainly conceptually
8. ID.; ID.; ID.; LOSSES ACTUALLY SUSTAINED CAN BE CHARGED OFF
distinct from pulping and paper manufacturing operations. But no one
ONLY AGAINST INCOME EARNED DURING THE SAME YEAR. The rule
contends that issuance of bonds was a principal or regular business activity of
applicable in respect of corporations not registered with the BOI as a preferred
Picop; only banks or other financial institutions are in the regular business of

50
TAXATION II ATTY. ACOSTA -
CAJUSTIN
pioneer enterprise is that net operating losses cannot be carried over. a corporation which had run no risks and suffered no losses, but had merely
Under our Tax Code, both in 1977 and at present, losses may be deducted from purchased another's losses. We conclude that the deduction claimed by Picop
gross income only if such losses were actually sustained in the same year that in the amount of P44,196,106.00 in its 1977 Income Tax Return must be
they are deducted or charged off. (Section 30 of the 1977 Tax Code, Section 76 disallowed.
of the Philippine Income Tax Regulations [Revenue Regulation No. 2, as
amended]) It is thus clear that under our law, and outside the special realm of 11. REMEDIAL LAW; EVIDENCE; BURDEN OF PROOF; TAXPAYER HAS
BOI-registered enterprises, there is no such thing as a carry-over of net THE BURDEN OF PROVING ENTITLEMENT TO CLAIMED DEDUCTION.
operating loss. To the contrary, losses must be deducted against current A taxpayer has the burden of proving entitlement to a claimed deduction. In
income in the taxable year when such losses were incurred. Moreover, such the instant case, even Picop's own vouchers were not submitted in evidence
losses may be charged off only against income earned in the same taxable and the BIR Examiners denied that such vouchers and other documents had
year when the losses were incurred. been exhibited to them. Moreover, cash vouchers can only confirm the fact of
disbursement but not necessarily the purpose thereof. The best evidence that
9. ID.; RA 5186 (INVESTMENT INCENTIVES ACT); CARRY-OVER OF NET Picop should have presented to support its claimed deduction were the
OPERATING LOSSES WITH RESPECT TO THEIR REGISTERED invoices and official receipts issued by the Register of Deeds. Picop not only
OPERATION; PURPOSE. Thus it is that RA. No. 5185 introduced the carry- failed to present such documents; it also failed to explain the loss thereof,
over of net operating losses as a very special incentive to be granted only to assuming they had existed before. Under the best evidence rule, therefore, the
registered pioneer enterprises and only with respect to their registered testimony of Picop's employee was inadmissible and was in any case entitled to
operations. The statutory purpose here may be seen to be the encouragement very little, if any, credence. We consider that entitlement to Picop's claimed
of the establishment and continued operation of pioneer industries by allowing deduction of P1,237,421.00 was not adequately shown and that such deduction
the registered enterprise to accumulate its operating losses which may be must be disallowed. ScAIaT
expected during the early years of the enterprise and to permit the enterprise
to offset such losses against income earned by it in later years after successful 12. ID.; ID.; ADMISSION; HIGHER SALES REFLECTED IN PICOP'S BOOK
establishment and regular operations. To promote its economic development OF ACCOUNTS, AN ADMISSION AGAINST ITS OWN INTEREST. In its
goals, the Republic foregoes or defers taxing the income of the pioneer assessment for deficiency income tax for 1977, the CIR claimed that Picop had
enterprise until after that enterprise has recovered or offset its earlier losses. understated its sales by P2,391,644.00 and, upon the other hand, overstated its
We consider that the statutory purpose can be served only if the accumulated cost of sales by P604,018.00. Thereupon, the CIR added back both sums to
operating losses are carried over and charged off against income subsequently Picop's net income figure per its own return. The 1977 Income Tax Return of
earned and accumulatedby the same enterprise engaged in the same Picop set forth its total sales as P800,814,851.00. Upon the other hand, Picop's
registered operations. IDASHa Books of Accounts reflected higher sales figures of P803,206,495.00. The
above figures thus show a discrepancy between the sales figures reflected in
10. ID.; ID.; ID.; NET OPERATING LOSS OF ONE ENTERPRISE NOT Picop's Books of Accounts and the sales figures reported in its 1977 Income Tax
DEDUCTIBLE FROM GROSS INCOME OF ANOTHER DESPITE MERGE. Return, amounting to: P2,391,644.00. The CIR has made out at least a prima
The CTA and the Court of Appeals allowed the offsetting of RPPM's facie case that Picop had understated its sales and overstated its cost of sales as
accumulated operating losses against Picop's 1977 gross income, basically set out in its Income Tax Return. For the CIR has a right to assume that Picop's
because towards the end of the taxable year 1977, upon the arrival of the Books of Accounts speak the truth in this case since, as already noted, they
effective date of merger, only one (1) corporation, Picop, remained. The losses embody what must appear to be admissions against Picop's own interest.
suffered by RPPM's registered operations and the gross income generated by Accordingly, we must affirm the findings of the Court of Appeals and the CTA.
Picop's own registered operations now came under one and the same corporate
roof. We consider that this circumstance relates much more to form than to 13. TAXATION; R.A. 5186 (INVESTMENT INCENTIVES ACT); NON-
substance. We do not believe that the single purely technical factor is enough EXEMPTION FROM PAYMENT OF INCOME TAX; CORPORATE
to authorize and justify the deduction claimed by Picop. Picop's claim for DEVELOPMENT TAX, AN INCOME TAX; REQUISITE FOR PAYMENT.
deduction is not only bereft of statutory basis; it does violence to the legislative The five percent (5%) corporate development tax is an additional corporate
intent which animates the tax incentive granted by Section 7 (c) of R.A. No. income tax imposed in Section 24 (e) of the 1977 Tax Code. This additional tax
5186. In granting the extraordinary privilege and incentive of a net operating shall be imposed only if the net income exceeds 10 per cent of the net worth, in
loss carry-over to BOI-registered pioneer enterprises, the legislature could not case of a domestic corporation, or net assets in the Philippines in case of a
have intended to require the Republic to forego tax revenues in order to benefit resident foreign corporation. Since this five percent (5%) corporate

51
TAXATION II ATTY. ACOSTA -
CAJUSTIN
development tax is an income tax, Picop is not exempted from it under the and as a preferred non-pioneerenterprise with respect to its integrated plywood
provisions of Section 8 (a) of R.A. No. 5186. The adjusted net income of Picop and veneer mills. cdlex
for 1977, as will be seen below, is P48,687,355.00. Its net worth figure or total
stockholders' equity as reflected in its Audited Financial Statements for 1977 is On 21 April 1983, Picop received from the Commissioner of Internal Revenue
P464,749,528.00. Since its adjusted net income for 1977 thus exceeded ten ("CIR") two (2) letters of assessment and demand both dated 31 March 1983: (a)
percent (10%) of its net worth, Picop must be held liable for the five percent one for deficiency transaction tax and for documentary and science stamp tax;
(5%) corporate development tax in the amount of P2,434,367.75. and (b) the other for deficiency income tax for 1977, for an aggregate amount
of P88,763,255.00. These assessments were computed as follows
VITUG, J., concurring and dissenting opinion: "Transaction Tax

1. TAXATION; REPUBLIC ACT NO. 5186 (INVESTMENT INCENTIVES ACT); Interest payments on
EXEMPTION FROM PAYMENT OF INCOME TAX DOES NOT INCLUDE
money market borrowings P45,771,849.00
EXEMPTION FROM TRANSACTION TAX. R.A. No. 5186, also known as
the Investment Incentives Act, has provided for incentives by, among other
things, granting to registered pioneer enterprises an exemption from all taxes,
except income tax, under the National Internal Revenue Code. The income tax, 35% Transaction tax due thereon 16,020,147.00
referred to, in my view, is that imposed in Title II, entitled "Income Tax," of the
Revenue Code. Nowhere under that title is there a 35% transaction tax. Add: 25% surcharge 4,005,036.75

2. ID.; NATIONAL INTERNAL REVENUE CODE; 35% TRANSACTION TAX;


LEVIED ON BORROWER-ISSUER OF COMMERCIAL PAPERS NOT ON
Total P20,025,183.75
INVESTOR-LENDER There was, to be sure, a 35% transaction tax still in
effect in 1977 but it was a tax, not on the investor-lender in whose favor the Add:
interest income on the commercial paper accrues. The tax was, instead, levied
on the borrower-issuer of commercial papers transacted in the primary 14% int. fr.
market. Being the principal taxpayer, the borrower-issuer could not have been
likewise contemplated to be a mere tax withholding agent. The tax was 1-20-78 to
conceived as a tax on business transaction, and so it was rightly incorporated 7-31-80 P7,093,302.57
in Title V, entitled "Privilege Taxes on Business and Occupation" of the Tax
Code. HAECID 20% int. fr.
3. ID.; TAXES; FACT THAT TAXPAYER CAN SHIFT PAYMENT OF 8-1-80 to
INDIRECT TAXES TO ANOTHER DOES NOT MAKE THE LATTER THE
TAXPAYER AND THE FORMER THE WITHHOLDING AGENT. The fact 3-31-83 10,675,532.58
that a taxpayer on whom the tax is imposed can shift, characteristic of indirect
taxes, the burden thereof to another does not make the latter the taxpayer and
the former the withholding agent. Indeed, the facility of shifting the burden of 17,768,826.15
the tax is opposed to the idea of adirect tax to which class the income tax
actually belongs.
P37,794,009.90
The Paper Industries Corporation of the Philippines ("Picop"), which is petitioner
in G.R. Nos. 106949-50 and private respondent in G.R. Nos. 106984-85, is a
Philippine corporation registered with the Board of Investments ("BOI") as a Documentary and Science Stamps Tax
preferred pioneer enterprise with respect to its integrated pulp and paper mill,

52
TAXATION II ATTY. ACOSTA -
CAJUSTIN
Total face value of debentures P100,000,000.00 4) Understatement of sales 2,391,644.00
Documentary Stamps 5) Overstatement of cost of sales 604,018.00
Tax Due
(P0.30 x P100,000.00) P91,406,194.00
( P200 ) P 150,000.00 Net income per investigation P91,644,360.00
Science Stamps Tax Due Income tax due thereon 34,734,559.00
(P0.30 x P100,000.00) Less: Tax already assessed per return 80,358.00
( P200 ) 150,000.00
Deficiency P34,654,201.00
Total P300,000.00 Add:
Add: Compromise for 14% int. fr.
non-affixture 300.00 4-15-78 to
7-31-81 P11,128,503.56

20% int. fr.
300,300.00 8-1-80 to
4-15-81 4,886,242.34

P16,014,745.90
TOTAL AMOUNT DUE AND
COLLECTIBLE P38,094,309.90
============ TOTAL AMOUNT DUE AND
COLLECTIBLE P50,668,946.90" 1
Deficiency Income Tax for 1977
=============
Net income per return P258,166.00
Add: Unallowable deductions On 26 April 1983, Picop protested the assessment of deficiency
transaction tax and documentary and science stamp taxes. Picop also
1) Disallowed deductions protested on 21 May 1983 the deficiency income tax assessment for 1977.
availed of under R.A. These protests were not formally acted upon by respondent CIR. On 26
No. 5186 P44,332,980.00 September 1984, the CIR issued a warrant of distraint on personal property
and a warrant of levy on real property against Picop, to enforce collection of
2) Capitalized interest the contested assessments; in effect, the CIR denied Picop's protests.
expenses on funds used
for acquisition of machinery Thereupon, Picop went before the Court of Tax Appeals ("CTA") appealing the
& other equipment 42,840,131.00 assessments. After trial, the CTA rendered a decision dated 15 August 1989,
modifying the findings of the CIR and holding Picop liable for the reduced
3) Unexplained financial
guarantee expense 1,237,421.00

53
TAXATION II ATTY. ACOSTA -
CAJUSTIN
aggregate amount of P20,133,762.33, which was itemized in the dispositive Picop now maintains that it is not liable at all to pay any of the assessments or any
portion of the decision as follows: part thereof. It assails the propriety of the thirty-five percent (35%) deficiency
transaction tax which the Court of Appeals held due from it in the amount of
"35% Transaction Tax P16,020,113.20 P3,578,543.51. Picop also questions the imposition by the Court of Appeals of the
deficiency income tax of P1,481,579.15, resulting from disallowance of certain
Documentary & Science Stamp Tax 300,300.00
claimed financial guarantee expenses and claimed year-end adjustments of sales
Deficiency Income Tax Due 3,813,349.33 and cost of sale figures by Picop's external auditors. 3

The CIR, upon the other hand, insists that the Court of Appeals erred in finding
Picop not liable for surcharge and interest on unpaid transaction tax and for
TOTAL AMOUNT DUE AND PAYABLE P20,133,762.53" 2 documentary and science stamp taxes and in allowing Picop to claim as
deductible expenses:
============
(a) the net operating losses of another corporation (i.e.,
Picop and the CIR both went to the Supreme Court on separate Petitions for Rustan Pulp and Mills, Inc.); and
Review of the above decision of the CTA. In two (2) Resolutions dated 7 February
1990 and 19 February 1990, respectively, the Court referred the two (2) Petitions (b) interest payments on loans for the of machinery and
to the Court of Appeals. The Court of Appeals consolidated the two (2) cases and equipment.
rendered a decision, dated 31 August 1992, which further reduced the liability of
Picop to P6,338,354.70. The dispositive portion of the Court of Appeals decision The CIR also claims that Picop should be held liable for interest at fourteen
reads as follows: percent (14%) per annum from 15 April 1978 for three (3) years, and interest at
twenty percent (20%) per annum for a maximum of three (3) years; and for a
"WHEREFORE, the appeal of the Commissioner of Internal surcharge of ten percent (10%), on Picop's deficiency income tax. Finally, the CIR
Revenue is denied for lack of merit. The judgment against contends that Picop is liable for the corporate development tax equivalent to five
PICOP is modified, as follows: percent (5%) of its correct 1977 net income.
1. PICOP is declared liable for the 35% transaction tax in the The issues which we must here address may be sorted out and grouped in the
amount of P3,578,543.51; following manner:

2. PICOP is absolved from the payment of documentary and I. Whether Picop is liable for:
science stamp tax of P300,000.00 and the compromise
(1) the thirty-five percent (35%) transaction tax;
penalty of P300.00;
(2) interest and surcharge on unpaid transaction
3. PICOP shall pay 20% interest per annum on the deficiency
tax; and
income tax of P1,481,579.15, for a period of three (3) years
from 21 May 1983, or in the total amount of P888,947.49, (3) documentary and science stamp taxes:
and a surcharge of 10% on the latter amount, or P88,984.75.
II. Whether Picop is entitled to deductions against income of:
No pronouncement as to costs.
(1) interest payments of loans for the purchase of
SO ORDERED." machinery and equipment;
Picop and the CIR once more filed separate Petitions for Review before the (2) net operating losses incurred by the Rustan Pulp
Supreme Court. These cases were consolidated and, on 23 August 1993, the Court and Paper Mills, Inc.; and
resolved to give due course to both Petitions in G.R. Nos. 106949-50and 106984-
85 and required the parties to file their Memoranda. (3) certain claimed financial guarantee expenses;
and

54
TAXATION II ATTY. ACOSTA -
CAJUSTIN
III. (1) Whether Picop had understated its sales and The transaction tax imposed in this section shall
overstated its cost of sales for 1977; and be a final tax to be paid by the borrower and shall
be allowed as a deductible item for purposes of
(2) Whether Picop is liable for the corporate development computing the borrower's taxable income.
tax of
For purposes of this tax
five percent (5%) of its net income for 1977.
(a) "Commercial paper" shall be
We will consider these issues in the foregoing sequence. defined as an instrument evidencing
indebtedness of any person or entity,
I.
including banks and non-banks performing
(1) Whether Picop is liable for the thirty-five percent (35%) quasi-banking functions, which is issued,
transaction tax. endorsed, sold, transferred or in any
manner conveyed to another person or
With the authorization of the Securities and Exchange Commission, Picop issued entity, either with or without recourse and
commercial paper consisting of serially numbered promissory notes with the total irrespective of maturity. Principally,
face value of P229,864,000.00 and a maturity period of one (1) year, i.e., from 24 commercial papers are promissory
December 1977 to 23 December 1978. These promissory notes were purchased by notesand/or similar instruments issued in
various commercial banks and financial institutions. On these promissory notes, the primary market and shall not include
Picop paid interest in the aggregate amount of P45,771,849.00. In respect of these repurchase agreements, certificates of
interest payments, the CIR required Picop to pay the thirty-five percent (35%) assignments, certificates of participation,
transaction tax. llcd and such other debt instruments issued in
The CIR based this basement on Presidential Decree No. 1154 dated 3 June 1977, the secondary market.
which reads in part as follows: (b) The term "interest" shall mean
"SECTION 1. The National Internal Revenue Code,as the difference between what the principal
amended, is hereby further amended by adding a new section borrower received and the amount it paid
thereto to read as follows: upon maturity of the commercial paper
which shall, in no case, be lower than the
'SECTION 195-C. Tax on certain interest. There interest rate prevailing at the time of the
shall be levied, assessed, collected and paid on issuance or renewal of the commercial
every commercial paper issued in the primary paper. Interest shall be deemed
market as principal instrument, a transaction tax synonymous with discount and shall
equivalent to thirty-five percent (35%) based on the include all fees, commissions, premiums
gross amount of interest thereto as defined and other payments which form integral
hereunder, which shall be paid by the parts of the charges imposed as a
borrower/issuer: Provided, however, that in the consequence of the use of money.
case of a long-term commercial paper whose
maturity exceeds more than one year, the borrower
shall pay the tax based on the amount of interest In all cases, where no interest rate
corresponding to one year, and thereafter shall pay is stated or if the rate stated is lower than
the tax upon accrual or actual payment (whichever the prevailing interest rate at the time of
is earlier) of the untaxed portion of the interest the issuance or renewal of commercial
which corresponds to a period not exceeding one paper, the Commissioner of Internal
year. Revenue, upon consultation with the
Monetary Board of the Central Bank of the

55
TAXATION II ATTY. ACOSTA -
CAJUSTIN
Philippines, shall adjust the interest rate in (5) Ten per cent (10%) for the thirteenth through the
accordance herewith, and assess the tax on fifteenth year.
the basis thereof.
xxx xxx xxx" 4
The tax herein imposed shall be
remitted by the borrower to the We agree with the CTA and the Court of Appeals that Picop's tax exemption
Commissioner of Internal Revenue or his under R.A. No. 5186, as amended, does not include exemption from the thirty-five
Collection Agent in the municipality where percent (35%) transaction tax. In the first place, the thirty-five percent (35%)
such borrower has its principal place of transaction tax 5 is an income tax, that is, it is a tax on the interest income of the
business within five (5) working days from lenders or creditors. In Western Minolco Corporation v. Commissioner of
the issuance of the commercial paper. In Internal Revenue, 6 the petitioner corporation borrowed funds from several
the case of long term commercial paper, financial institutions from June 1977 to October 1977 and paid the corresponding
the tax upon the untaxed portion of the thirty-five (35%) transaction tax thereon in the amount of P1,317,801.03,
interest which corresponds to a period not pursuant to Section 210 (b) of the 1977 Tax Code. Western Minolco applied for
exceeding one year shall be paid upon refund of that amount alleging it was exempt from the thirty-five (35%)
accrual payment, whichever is earlier.'" transaction tax by reason of Section 79-A of C.A. No. 137, as amended, which
(Italics supplied) granted new mines and old mines resuming operation "five (5) years complete tax
exemptions, except income tax, from the time of its actual bona-fide orders for
Both the CTA and the Court of Appeals sustained the assessment of equipment for commercial production." In denying the claim for refund, this
transaction tax. Court held:
In the instant Petition, Picop reiterates its claim that it is exempt from the "The petitioner's contentions deserve scant
payment of the transaction tax by virtue of its tax exemption under R.A. No. 5186, consideration. The 35% transaction tax is imposed on
as amended, known as the Investment Incentives Act, which in the form it existed interest income from commercial papers issued in the
in 19771978, read in relevant part as follows: primary money market. Being a tax on interest, it is a tax
on income.
"SECTION 8. Incentives to a Pioneer Enterprise. In
addition to the incentives provided in the preceding section, As correctly ruled by the respondent Court of Tax Appeals:
pioneer enterprises shall be granted the following incentive
benefits: 'Accordingly, we need not and do not think it
necessary to discuss further the nature of the
(a) Tax Exemption. Exemption from all taxes under transaction tax more than to say that the incipient
the National Internal Revenue Code,except income tax, from scheme in the issuance of Letter of Instructions No.
the date the area of investment is included in the Investment 340 on November 24, 1975 (O.G. Dec. 15, 1975), i.e.,
Priorities Plan to the following extent: to achieve operational simplicity and effective
administration in capturing the interest-income
(1) One hundred per cent (100%) for the first five
"windfall" from money market operations as a new
years;
source of revenue, has lost none of its animating
(2) Seventy-five per cent (75%) for the sixth through principle in parturition of amendatory Presidential
the eighth years; Decree No. 1154, now Section 210 (b) of theTax
Code. The tax thus imposed is actually a tax on
(3) Fifty per cent (50%) for the ninth and tenth interest earnings of the lenders or placers who are
years; actually the taxpayers in whose income is imposed.
Thus "the borrower withholds the tax of 35% from
(4) Twenty per cent (20%) for the eleventh and the interest he would have to pay the lender so that
twelfth years; and he (borrower) can pay the 35% of the interest to the
Government." (Citation omitted). . . .. Suffice it to

56
TAXATION II ATTY. ACOSTA -
CAJUSTIN
state that the broad consensus of fiscal and enterprises by Section 8 of R.A. No. 5186, as amended. Picop was the withholding
monetary authorities is that "even if nominally, the agent, obliged to withhold thirty-five percent (35%) of the interest payable to its
borrower is made to pay the tax, actually, the tax is lenders and to remit the amounts so withheld to the Bureau of Internal Revenue
on the interest earning of the immediate and all ("BIR"). As a withholding, agent, Picop is made personally liablefor the thirty-five
prior lenders/placers of the money. . . .'" (Rollo, pp. percent (35%) transaction tax 10 and if it did not actually withhold thirty-five
3637) percent (35%) of the interest monies it had paid to its lenders, Picop had only
itself to blame.
The 35% transaction tax is an income tax on
interest earnings to the lenders or placers. The Picop claims that it had relied on a ruling, dated 6 October 1977, issued by the
latter are actually the taxpayers. Therefore, the tax CIR, which held that Picop was not liable for the thirty-five (35%) transaction tax
cannot be a tax imposed upon the petitioner. In in respect of debenture bonds issued by Picop. Prior to the issuance of the
other words, the petitioner who borrowed funds promissory notes involved in the instant case, Picop had also issued debenture
from several financial institutions by issuing bonds of P100,000,000.00 in aggregate face value. The managing underwriter of
commercial papers merely withheld the 35% this debenture bond issue, Bancom Development Corporation, requested a formal
transaction tax before paying to the financial ruling from the Bureau of Internal Revenue on the liability of Picop for the thirty-
institutions the interest earned by them and later five percent (35%) transaction tax in respect of such bonds. The ruling rendered
remitted the same to the respondent Commissioner by the then Acting Commissioner of Internal Revenue, Efren I. Plana, stated in
of Internal Revenue. The tax could have been relevant part:
collected by a different procedure but the statute
chose this method. Whatever collecting procedure is "It is represented that PICOP will be offering to the public
adopted does not change the nature of the tax. primary bonds in the aggregate principal sum of one hundred
million pesos (P100,000,000.00); that the bonds will be
xxx xxx xxx." 7 (Italics supplied) issued as debentures in denominations of one thousand
pesos (P1,000.00) or multiples, to mature in ten (10)
Much the same issue was passed upon in Marinduque Mining and Industrial years at 14% interest per annum payable semi-annually;
Corporation v. Commissioner of Internal Revenue 8 and resolved in the same that the bonds are convertible into common stock of the
way: issuer at the option of the bond holder at an agreed
"It is very obvious that the transaction tax, which is tax on conversion price; that the issue will be covered by a 'Trust
interest derived from commercial paper issued in the money Indenture' with a duly authorized trust corporation as
market, is not a tax contemplated in the above-quoted legal required by the Securities and Exchange Commission, which
provisions. The petitioner admits that it is subject to income trustee will act for and in behalf of the debenture bond
tax. Its tax exemption should be strictly construed. holders as beneficiaries; that once issued, the bonds cannot
be preterminated by the holder and cannot be redeemed by
We hold that petitioner's claim for refund was justifiably the issuer until after eight (8) years from date of issue; that
denied. The transaction tax, although nominally categorized the debenture bonds will besubordinated to present and
as a business tax, is in reality a withholding tax as positively future debts of PICOP; and that said bonds are intended to be
stated in LOI No. 340. The petitioner could have shifted the listed in the stock exchanges, which will place them alongside
tax to the lenders or recipients of the interest. It did not listed equity issues.
choose to do so. It cannot be heard now to complain about
the tax. LOI No. 340 is an extraneous or extrinsic aid to the In reply, I have the honor to inform you that although the
construction of section 210 (b). bonds hereinabove described are commercial papers which
will be issued in the primary market, however, it is clear
xxx xxx xxx" 9 (Italics supplied) from the abovestated facts that said bonds will not be issued
as money market instruments. Such being the case, and
It is thus clear that the transaction tax is an income tax and as such, in any event, considering that the purposes of Presidential Decree No.
falls outside the scope of the tax exemption granted to registered pioneer 1154, as can be gleaned from Letter of Instruction No. 340,

57
TAXATION II ATTY. ACOSTA -
CAJUSTIN
dated November 21, 1975, are (a) to regulate money market accruing after P.D. No. 1154 went into effect, and not in respect of all the 1977
transactions and (b) to ensure the collection of the tax on interest earnings of such lenders. The Court of Appeals pointed out that:
interest derived from money market transactions by
imposing a withholding tax thereon, said bonds do not come "PICOP, however, contends that even if the tax has to be
within the purview of the 'commercial papers' intended to be paid, it should be imposed only for the interests earned after
subjected to the 35% transaction tax prescribed 20 September 1977 when PD 1154 creating the tax became
in Presidential Decree No. 1154, as implemented by Revenue effective. We find merit in this contention. It appears that
Regulations No. 7-77. (See Section 2 of said Regulation). the tax was levied on interest earnings from January to
Accordingly, PICOP is not subject to 35% transaction tax on October, 1977. However, as found by the lower court, PD
its issues of the aforesaid bonds. However, those investing in 1154 was published in the Official Gazette only on 5
said bonds should be made aware of the fact that the September 1977, and became effective only fifteen (15) days
transaction tax is not being imposed on the issuer of said after the publication, or on 20 September 1977, no other
bonds by printing or stamping thereon, in bold letters, the effectivity date having been provided by the PD. Based on the
following statement: 'ISSUER NOT SUBJECT TO Worksheet prepared by the Commissioner's office, the
TRANSACTION TAX UNDER P.D. 1154. BONDHOLDER interests earned from 20 September to October 1977 was
SHOULD DECLARE INTEREST EARNING FOR INCOME P10,224,410.03. Thirty-five (35%) per cent of this is
TAX.'" 11 (Emphases supplied) P3,578,543.51 which is all PICOP should pay as transaction
tax." 13 (Emphasis supplied)
P.D. No. 1154 is not, in other words, to be given retroactive effect by imposing the
In the above quoted ruling, the CIR basically held that Picop's debenture bonds thirty-five percent (35%) transaction tax in respect of interest earnings which
did not constitute "commercial papers" within the meaning of P.D. No. 1154, and accrued before the effectivity date of P.D. No. 1154, there being nothing in the
that, as such, those bonds were not subject to the thirty-five percent (35%) statute to suggest that the legislative authority intended to bring about such
transaction tax imposed by P.D. No. 1154. retroactive imposition of the tax.
The above ruling, however, is not applicable in respect of the promissory notes (2) Whether Picop is liable for interest and surcharge on unpaid
which are the subject matter of the instant case. It must be noted that the transaction tax.
debenture bonds which are the subject matter of Commissioner Plana's ruling
were long-term bonds maturing in ten (10) years and which could not be pre- With respect to the transaction tax due, the CIR prays that Picop be held liable for
terminated and could not be redeemed by Picop until after eight (8) years from a twenty-five percent (25%) surcharge and for interest at the rate of fourteen
date of issue; the bonds were moreover subordinated to present and future debts percent (14%) per annum from the date prescribed for its payment. In so praying,
of Picop and convertible into common stock of Picop at the option of the the CIR relies upon Section 10 of Revenue Regulation 7-77 dated 3 June
bondholder. In contrast, the promissory notes involved in the instant case are 1977, 14 issued by the Secretary of Finance. This Section reads:
short-term instruments bearing a one-year maturity period. These promissory
"SEC. 10. Penalties. Where the amount shown by the
notes constitute the very archetype of money market instruments. For money
taxpayer to be due on its return or part of such payment is
market instruments are precisely, by custom and usage of the financial
not paid on or before the date prescribed for its payment, the
markets, short-term instruments with a tenor of one (1) year or less. 12 Assuming,
amount of the tax shall be increased by twenty-five (25%)
therefore, (without passing upon) the correctness of the 6 October 1977 BIR
per centum, the increment to be a part of the tax and
ruling, Picop's short-term promissory notes must be distinguished, and treated
the entire amount shall be subject to interest at the rate of
differently, from Picop's long-term debenture bonds. cda
fourteen (14%) per centum per annum from the date
We conclude that Picop was properly held liable for the thirty-five percent (35%) prescribed for its payment.
transaction tax due in respect of interest payments on its money market
In the case of wilful neglect to file the return within the
borrowings.
period prescribed herein or in case a false or fraudulent
At the same time, we agree with the Court of Appeals that the transaction tax may return is wilfully made, there shall be added to the tax or to
be levied only in respect of the interest earnings of Picop's money market lenders the deficiency tax in case any payment has been made on the

58
TAXATION II ATTY. ACOSTA -
CAJUSTIN
basis of such return before the discovery of the falsity or (c) Definition of deficiency. As used in this Chapter in
fraud, a surcharge of fifty (50%) per centum of its amount. respect of a tax imposed by this Title, the term 'deficiency'
The amount so added to any tax shall be collected at the same means:
time and in the same manner and as part of the tax unless the
tax has been paid before the discovery of the falsity or fraud, (1) The amount by which the tax imposed by this
in which case the amount so added shall be collected in the Title exceeds the amount shown as the tax by the taxpayer
same manner as the tax. upon his return; but the amount so shown on the return shall
first be increased by the amounts previously assessed (or
In addition to the above administrative penalties, collected without assessment) as a deficiency, and decreased
the criminal and civil penalties as provided for under Section by the amount previously abated, credited, returned, or
337 of the Tax Code of 1977 shall be imposed for violation of otherwise in respect of such tax; . . .
any provision of Presidential Decree No. 1154." 15 (Emphases
supplied) xxx xxx xxx

The 1977 Tax Code itself, in Section 326 in relation to Section 4 of the same (e) Additions to the tax in case of non-payment.
Code, invoked by the Secretary of Finance in issuing Revenue Regulation 7-
(1) Tax shown on the return. Where the amount
77, set out, in comprehensive terms, the rule-making authority of the
determined by the taxpayer as the tax imposed by this Titleor
Secretary of Finance:
any installment thereof, or any part of such amount or
"SEC. 326. Authority of Secretary of Finance to Promulgate installment is not paid on or before the date prescribed for its
Rules and Regulations. The Secretary of Finance, upon payment, there shall be collected as a part of the tax, interest
recommendation of the Commissioner of Internal Revenue, upon such unpaid amount at the rate of fourteen per centum
shall promulgate all needful rules and regulations for the per annum from the date prescribed for its payment until it is
effective enforcement of the provisions of this Code." paid: Provided, That the maximum amount that may be
(Emphasis supplied) collected as interest on deficiency shall in no case exceed the
amount corresponding to a period of three years, the present
Section 4 of the same Code contains a list of subjects or areas to be dealt with provisions regarding prescription to the contrary
by the Secretary of Finance through the medium of an exercise of his quasi- notwithstanding.
legislative or rule-making authority. This list, however, while it purports to be
open-ended, does not include the imposition of administrative or civil (2) Deficiency. Where a deficiency, or any interest assessed
penalties such as the payment of amounts additional to the tax due. Thus, in in connection therewith under paragraph (d) of this
order that it may be held to be legally effective in respect of Picop in the section, or any addition to the taxes provided for in Section
present case, Section 10 of Revenue Regulation 7-77 must embody or rest seventy-two of this Code is not paid in full within thirty days
upon some provision in the Tax Code itself which imposes surcharge and from the date of notice and demand from the Commissioner
penalty interest for failure to make a transaction tax payment when due. of Internal Revenue, there shall be collected upon the unpaid
amount as part of the tax, interest at the rate of fourteen per
P.D. No. 1154 did not itself impose, nor did it expressly authorize the imposition
centum per annum from the date of such notice and demand
of, a surcharge and penalty interest in case of failure to pay the thirty-five percent
until it is paid: Provided, That the maximum amount that
(35%) transaction tax when due. Neither did Section 210 (b) of the 1977 Tax
may be collected as interest on deficiency shall in no case
Code which re-enacted Section 195-C inserted into the Tax Code by P.D. No. 1154.
exceed the amount corresponding to a period of three years,
The CIR, both in its petition before the Court of Appeals and its Petition in the the present provisions regarding prescription to the contrary
instant case, points to Section 51 (e) of the 1977 Tax Code as its source of notwithstanding.
authority for assessing a surcharge and penalty interest in respect of the thirty-
(3) Surcharge. If any amount of tax included in the notice
five percent (35%) transaction tax due from Picop. This Section needs to be
and demand from the Commissioner of Internal Revenue is
quoted in extenso:
not paid in full within thirty days after such notice and
"SEC. 51. Payment and Assessment of Income Tax. demand, there shall be collected in addition to the interest

59
TAXATION II ATTY. ACOSTA -
CAJUSTIN
prescribed herein and in paragraph (d) above and as part of respect of which Section 51 (e) authorized the imposition of surcharge and
the tax a surcharge of five per centum of the amount of tax interest and Section 72 the imposition of a fraud surcharge.
unpaid." (Emphases supplied)
It is not without reluctance that we reach the above conclusion on the basis of
Section 72 of the 1977 Tax Code referred to in Section 51(e)(2) above, what may well have been an inadvertent error in legislative draftsmanship, a type
provides: of error common enough during the period of Martial Law in our country.
Nevertheless, we are compelled to adopt this conclusion. We consider that the
"SEC. 72. Surcharges for failure to render returns and for
authority to impose what the presentTax Code calls (in Section 248) civil
rendering false and fraudulent returns. In case of willful
penalties consisting of additions to the tax due, must be expressly given in the
neglect to file the return or list required by this Title within
enabling statute, in language too clear to be mistaken. The grant of that authority
the time prescribed by law, or in case a false or fraudulent
is not lightly to be assumed to have been made to administrative officials, even to
return or list is wilfully made, the Commissioner of Internal
one as highly placed as the Secretary of Finance.
Revenue shall add to the tax or to the deficiency tax, in case
any payment has been made on the basis of such return The state of the present law tends to reinforce our conclusion that Section 51 (c)
before the discovery of the falsity or fraud, as surcharge of and (e) of the 1977 Tax Code did not authorize the imposition of a surcharge and
fifty per centum of the amount of such tax or deficiency tax. penalty interest for failure to pay the thirty-five percent (35%) transaction tax
In case of any failure to make and file a return or list within imposed under Section 210 (b) of the same Code. The corresponding provision in
the time prescribed by law or by the Commissioner or other the current Tax Code very clearly embraces failure to pay all taxes imposed in
Internal Revenue Officer,not due to willful neglect, the the Tax Code, without any regard to the Title of the Code where provisions
Commissioner of Internal Revenue shall add to the imposing particular taxes are textually located. Section 247 (a) of the NIRC, as
tax twenty-five per centum of its amount, except that, when amended, reads:
a return is voluntarily and without notice from the
Commissioner or other officer filed after such time, and it is "Title X
shown that the failure to file it was due to a reasonable cause,
no such addition shall be made to the tax. The amount so Statutory Offenses and Penalties
added to any tax shall be collected at the same time, in the Chapter I
same manner and as part of the tax unless the tax has been
paid before the discovery of the neglect, falsity, or fraud, in Additions to the Tax
which case the amount so added shall be collected in the
same manner as the tax." (Emphases supplied) SECTION 247. General Provisions. (a) The additions to
the tax or deficiency tax prescribed in this Chapter
shall apply to all taxes, fees and charges imposed in this
Code. The amount so added to the tax shall be collected at the
It will be seen that Section 51 (c)(1) and (e)(1) and (3), of the 1977 Tax Code, same time, in the same manner and as part of the tax. . . .
authorize the imposition of surcharge and interest only in respect of a "tax
imposed by this Title," that is to say, Title II on "Income Tax." It will also be seen SECTION 248. Civil Penalties. (a) There shall be
that Section 72 of the 1977 Tax Code imposes a surcharge only in case of failure to imposed, in addition to the tax required to be paid, penalty
file a return or list "required by this Title," that is, Title II on "Income Tax." The equivalent to twenty-five percent (25%) of the amount due,
thirty-five percent (35%) transaction tax is, however, imposed in the 1977 Tax in the following cases:
Codeby Section 210 (b) thereof which Section is embraced in Title V on "Taxes on
Business" of that Code. Thus, while the thirty-five percent (35%) transaction tax is xxx xxx xxx
in truth a tax imposed on interest income earned by lenders or creditors
purchasing commercial paper on the money market, the relevant provisions, i.e., (3) failure to pay the tax within the time prescribed for its
Section 210 (b), were not inserted in Title II of the 1977 Tax Code. The end result payment; or
is that the thirty-five percent (35%) transaction tax is not one of the taxes in xxx xxx xxx

60
TAXATION II ATTY. ACOSTA -
CAJUSTIN
(c) the penalties imposed hereunder shall form part of the tax applied strictly against the beneficiary in order to deter their
and the entire amount shall be subject to the interest abuse. It would indeed be altogether a different matter if
prescribed in Section 249. there is a showing that the issuance of the debenture bonds
had no bearing whatsoever on the registered operations of
SECTION 249. Interest. (a) In General. There shall be PICOP and that they were issued in connection with a
assessed and collected on any unpaid amount of tax, totally different business undertaking of PICOP other than
interest at the rate of twenty percent (20%) per its registered operation. There is, however, a dearth of
annum or such higher rate as may be prescribed by evidence in this regard. It cannot be denied that PICOP
regulations, from the date prescribed for payment until the needed funds for its operations. One of the means it used to
amount is fully paid. . . .." (Emphases supplied) raise said funds was to issue debenture bonds. Since the
money raised thereby was to be used in its registered
In other words, Section 247 (a) of the current NIRC supplies what did not
operation, PICOP should enjoy the incentives granted to
exist back in 1977 when Picop's liability for the thirty-five percent (35%)
it by R.A. 5186, one of which is the exemption from payment
transaction tax became fixed. We do not believe we can fill that
of all taxes under the National Internal Revenue Code,except
legislative lacuna by judicial fiat. There is nothing to suggest that Section 247
income taxes, otherwise the Purpose of the incentives would
(a) of the present Tax Code, which was inserted in 1985, was intended to be
be defeated. Documentary and science stamp taxes on
given retroactive application by the legislative authority. 16
debenture bonds are certainly not income
(3) Whether Picop is Liable for Documentary and Science Stamp taxes." 19 (Emphasis supplied)
Taxes.
Tax exemptions are, to be sure, to be "strictly construed," that is, they are not to
As noted earlier, Picop issued sometime in 1977 long-term subordinated be extended beyond the ordinary and reasonable intendment of the language
convertible debenture bonds with an aggregate face value of P100,000,000.00. actually used by the legislative authority in granting the exemption. The issuance
Picop stated, and this was not disputed by the CIR, that the proceeds of the of debenture bonds is certainly conceptually distinct from pulping and paper
debenture bonds were in fact utilized to finance the BOI-registered operations of manufacturing operations. But no one contends that issuance of bonds was a
Picop. The CIR assessed documentary and science stamp taxes, amounting to principal or regular business activity of Picop; only banks or other financial
P300,000.00, on the issuance of Picop's debenture bonds. It is claimed by Picop institutions are in the regular business of raising money by issuing bonds or other
that its tax exemption "exemption from all taxes under the National Internal instruments to the general public. We consider that the actual dedication of the
Revenue Code,except income tax" on a declining basis over a certain period of proceeds of the bonds to the carrying out of Picop's registered operations
time includes exemption from the documentary and science stamp taxes imposed constituted a sufficient nexus with such registered operations so as to exempt
under the NIRC. Picop from taxes ordinarily imposed upon or in connection with issuance of such
bonds. We agree, therefore, with the Court of Appeals on this matter that the CTA
The CIR, upon the other hand, stresses that the tax exemption under and the CIR had erred in rejecting Picop's claim for exemption from stamp taxes.
the Investment Incentives Act may be granted or recognized only to the extent
that the claimant Picop was engaged in registered operations, i.e., operations It remains only to note that after commencement of the present litigation before
forming part of its integrated pulp and paper project. 17 The borrowing of funds the CTA, the BIR took the position that the tax exemption granted by R.A. No.
from the public, in the submission of the CIR, was not an activity included in 5186, as amended, does include exemption from documentary stamp taxes on
Picop's registered operations. The CTA adopted the view of the CIR and held that transactions entered into by BOI-registered enterprises. BIR Ruling No. 088,
"the issuance of convertible debenture bonds [was] not synonymous [with] the dated 28 April 1989, for instance, held that a registered preferred pioneer
manufactur[ing] operations of an integrated pulp and paper mill." 18 enterprise engaged in the manufacture of integrated circuits, magnetic heads,
printed circuit boards, etc., is exempt from the payment of documentary stamp
The Court of Appeals took a less rigid view of the ambit of the tax exemption taxes. The Commissioner said:
granted to registered pioneer enterprises. Said the Court of Appeals:
"You now request a ruling that as a preferred pioneer
". . . PICOP's explanation that the debenture bonds were enterprise, you are exempt from the payment of
issued to finance its registered operation is logical and is Documentary Stamp Tax (DST).
unrebutted. We are aware that tax exemptions must be

61
TAXATION II ATTY. ACOSTA -
CAJUSTIN
In reply, please be informed that your request is hereby depreciation deduction taking into account the adjusted basis of the machinery
granted. Pursuant to Section 46 (a) of Presidential Decree and equipment (original acquisition cost plus interest charges) over the useful life
No. 1789, pioneer enterprises registered with the BOI are of such assets.
exempt from all taxes under the National Internal Revenue
Code,except from all taxes under the National Internal Both the CTA and the Court of Appeals sustained the position of Picop and held
Revenue Code,except income tax, from the date the area of that the interest deduction claimed by Picop was proper and allowable. In the
investment is included in the Investment Priorities Plan to instant Petition, the CIR insists on its original position.
the following extent:
We begin by noting that interest payments on loans incurred by a taxpayer
xxx xxx xxx (whether BOI-registered or not) are allowed by the NIRC as deductions against
the taxpayer's gross income. Section 30 of the 1977 Tax Code provided as follows:
Accordingly, your company is exempt from the payment of
documentary stamp tax to the extent of the percentage "Section 30. Deduction from Gross Income. The following
aforestated on transactions connected with the registered may be deducted from gross income:
business activity. (BIR Ruling No. 111-81) However, if said
(a) Expenses:
transactions conducted by you require the execution of a
taxable document with other parties, said parties who are not xxx xxx xxx
exempt shall be the one directly liable for the tax. (Sec.
173, Tax Code, as amended; BIR Ruling No. 236-87) (b) Interest:

In other words, said parties shall be liable to the same (1) In general. The amount of interest paid within
percentage corresponding to your tax exemption." (Emphasis the taxable year on indebtedness, except on
supplied) indebtedness incurred or continued to purchase or
carry obligations the interest upon which is exempt
Similarly, in BIR Ruling No. 013, dated 6 February 1989, the Commissioner held from taxation as income under this Title: . . ."
that a registered pioneer enterprise producing polyester filament yarn was (Emphasis supplied)
entitled to exemption "from the documentary stamp tax on [its] sale of real
property in Makati up to December 31, 1989." It appears clear to the Court that Thus, the general rule is that interest expenses are deductible against gross
the CIR, administratively at least, no longer insists on the position it originally income and this certainly includes interest paid under loans incurred in
took in the instant case before the CTA. connection with the carrying on of the business of the taxpayer. 20 In the
instant case, the CIR does not dispute that the interest payments were made
II by Picop on loans incurred in connection with the carrying on of the
(1) Whether Picop is entitled to deduct against current income registered operations of Picop, i.e., the financing of the purchase of
interest payments on loans for the purchase of machinery machinery and equipment actually used in the registered operations of Picop.
and equipment. Neither does the CIR deny that such interest payments were legally due and
demandable under the terms of such loans, and in fact paid by Picop during
the tax year 1977.

In 1969, 1972 and 1977, Picop obtained loans from foreign creditors in order to The CIR has been unable to point to any provision of the 1977 Tax Code or any
finance the purchase of machinery and equipment needed for its operations. In its other statute that requires the disallowance of the interest payments made by
1977 Income Tax Return, Picop claimed interest payments made in 1977, Picop . The CIR invokes Section 79 of Revenue Regulations No. 2 as amended
amounting to P42,840,131.00, on these loans as a deduction from its 1977 gross which reads as follows:
income.
"Section 79. Interest on Capital. Interest calculated for
The CIR disallowed this deduction upon the ground that, because the loans had cost-keeping or other purposes on account of capital or
been incurred for the purchase of machinery and equipment, the interest surplus invested in the business, which does not represent a
payments on those loans should have been capitalized instead and claimed as a charge arising under an interest-bearing obligation, is not

62
TAXATION II ATTY. ACOSTA -
CAJUSTIN
allowable deduction from gross income." (Emphases account, thus including taxes and carrying charges;
supplied) however, an exception exists, in which event such
adjustment to the capital account is not made, with respect
We read the above provision of Revenue Regulations No. 2 as referring to so to taxes and carrying charges which the taxpayer has not
called "theoretical interest," that is to say, interest "calculated" or computed (and elected to capitalize but for which a deduction instead has
not incurred or paid) for the purpose of determining the "opportunity cost" of been taken." 22 (Emphasis supplied)
investing funds in a given business. Such "theoretical" or imputed interest
does not arise from a legally demandable interest-bearing obligation incurred by The "carrying charges" which may be capitalized under the above quoted
the taxpayer who however wishes to find out, e.g., whether he would have been provisions of the U.S. Internal Revenue Codeinclude, as the CIR has pointed out,
better off by lending out his funds and earning interest rather than investing such interest on a loan "(but not theoretical interest of a taxpayer using his own
funds in his business. One thing that Section 79 quoted above makes clear is that funds)." What the CIR failed to point out is that such "carrying charges" may, at
interest which does constitute a charge arising under an interest-bearing the election of the taxpayer, either be (a) capitalized in which case the cost basis
obligation is an allowable deduction from gross income. of the capital assets, e.g., machinery and equipment, will be adjusted by adding
the amount of such interest payments or, alternatively, be (b) deducted from
It is claimed by the CIR that Section 79 of Revenue Regulations No. 2 was gross income of the taxpayer. Should the taxpayer elect to deduct the interest
"patterned after" paragraph 1.266-1 (b), entitled "Taxes and Carrying Charges payments against its gross income, the taxpayer cannot at the same timecapitalize
Chargeable to Capital Account and Treated as Capital Items" of the U.S. Income the interest payments. In other words, the taxpayer is not entitled to both the
Tax Regulations, which paragraph reads as follows: deduction from gross income and the adjusted (increased) basis for determining
gain or loss and the allowable depreciation charge. The U.S. Internal Revenue
"(B) Taxes and Carrying Charges. The items thus
Code does not prohibit the deduction of interest on a loan obtained for
chargeable to capital accounts are
purchasing machinery and equipment against gross income, unless the taxpayer
(11) In the case of real property, whether improved or has also or previously capitalized the same interest payments and thereby
unimproved and whether productive or nonproductive. adjusted the cost basis of such assets.

(a) Interest on a loan (but not theoretical interest of a We have already noted that our 1977 NIRC does not prohibit the deduction of
taxpayer using his own funds)." 21 interest on a loan incurred for acquiring machinery and equipment. Neither does
our 1977 NIRC compel the capitalization of interest payments on such a loan.
The truncated excerpt of the U.S. Income Tax Regulations quoted by the CIR The 1977 Tax Code is simply silent on a taxpayer's right to elect one or the other
needs to be related to the relevant provisions of the U.S. Internal Revenue Code, tax treatment of such interest payments. Accordingly, the general rule that
which provisions deal with the general topic of adjusted basis for determining interest payments on a legally demandable loan are deductible from gross income
allowable gain or loss on sales or exchanges of property and allowable must be applied. prLL
depreciation and depletion of capital assets of the taxpayer:
The CIR argues finally that to allow Picop to deduct its interest payments against
"Present Rule. The Internal Revenue Code, and the its gross income would be to encourage fraudulent claims to double deductions
Regulations promulgated thereunder provide that 'No from gross income:
deduction shall be allowed for amounts paid or
accrued for such taxes and carrying charges as, under "[t]o allow a deduction of incidental expense/cost incurred in
regulations prescribed by the Secretary or his delegate, are the purchase of fixed asset in the year it was incurred
chargeable to capital account with respect to property, if the would invite tax evasion through fraudulent application of
taxpayer elects, in accordance with such regulations, to treat double deductions from a gross income." 23 (Emphases
such taxes or charges as so chargeable.' supplied)

At the same time, under the adjustment of basis provisions The Court is not persuaded. So far as the records of the instant cases show,
which have just been discussed, it is provided that Picop has not claimed to be entitled to double deduction of its 1977 interest
adjustment shall be made for all 'expenditures, receipts, payments. The CIR has neither alleged nor proved that Picop had previously
losses, or other items' properly chargeable to a capital adjusted its cost basis for the machinery and equipment purchased with the
loan proceeds by capitalizing the interest payments here involved. The Court

63
TAXATION II ATTY. ACOSTA -
CAJUSTIN
will not assume that the CIR would be unable or unwilling to disallow "a In claiming such deduction, Picop relies on Section 7 (c) of R.A. No. 5186 which
double deduction" should Picop, having deducted its interest cost from its provides as follows:
gross income, also attempt subsequently to adjust upward the cost basis of
the machinery and equipment purchased and claim, e.g., increased "Section 7. Incentives to Registered Enterprise. A
deductions for depreciation. registered enterprise, to the extent engaged in a preferred
area of investment, shall be granted the following incentive
We conclude that the CTA and the Court of Appeals did not err in allowing the benefits:
deductions of Picop's 1977 interest payments on its loans for capital equipment
against its gross income for 1977. xxx xxx xxx

(2) Whether Picop is entitled to deduct against current income net (c) Net Operating Loss Carry-over. A net operating loss
operating losses incurred by Rustan Pulp and Paper Mills, incurred in any of the first ten years of operations may be
Inc. carried over as a deduction from taxable income for the six
years immediately following the year of such loss. The entire
On 18 January 1977, Picop entered into a merger agreement with the Rustan Pulp amount of the loss shall be carried over to the first of the six
and Paper Mills, Inc. ("RPPM") and Rustan Manufacturing Corporation ("RMC"). taxable years following the loss, and any portion of such loss
Under this agreement, the rights, properties, privileges, powers and franchises of which exceeds the taxable income of such first year shall be
RPPM and RMC were to be transferred, assigned and conveyed to Picop as the deducted in like manner from the taxable income of the next
surviving corporation. The entire subscribed and outstanding capital stock of remaining five years. The net operating loss shall be
RPPM and RMC would be exchanged for 2,891,476 fully paid up Class "A" computed in accordance with the provisions of the National
common stock of Picop (with a par value of P10.00) and 149,848 shares of Internal Revenue Code,any provision of this Act to the
preferred stock of Picop (with a par value of P10.00), to be issued by Picop , the contrary notwithstanding, except that income not taxable
result being that Picop would wholly own both RPPM and RMC while the either in whole or in part under this or other laws shall be
stockholders of RPPM and RMC would join the ranks of Picop 's shareholders. In included in gross income." (Emphasis supplied)
addition, Picop paid off the obligations of RPPM to the Development Bank of the
Philippines ("DBP") in the amount of P68,240,340.00, by issuing 6,824,034 Picop had secured a letter-opinion from the BOI dated 21 February 1977 that is,
shares of preferred stock (with a par value of P10.00) to the DBP. The merger after the date of the agreement of merger but before the merger became effective
agreement was approved in 1977 by the creditors and stockholders of Picop, relating to the deductibility of the previous losses of RPPM under Section 7 (c)
RPPM and RMC and by the Securities and Exchange Commission. Thereupon, on of R.A. No. 5186 as amended. The pertinent portions of this BOI opinion, signed
30 November 1977, apparently the effective date of merger, RPPM and RMC were by BOI Governor Cesar Lanuza; read as follows:
dissolved. The Board of Investments approved the merger agreement on 12
January 1978. "2) PICOP will not be allowed to carry over the losses of
Rustan prior to the legal dissolution of the latter because at
that time the two (2) companies still had separate legal
personalities;
It appears that RPPM and RMC were, like Picop, BOI-registered companies.
Immediately before merger effective date, RPPM had over preceding years 3) After BOI approval of the merger, PICOP can no longer
accumulated losses in the total amount of P81,159,904.00. In its 1977 Income Tax apply for the registration of the registered capacity of Rustan
Return, Picop claimed P44,196,106.00 of RPPM's accumulated losses as a because with the approved merger, such registered capacity
deduction against Picop's 1977 gross income. 24 of Rustan transferred to PICOP will have the same
registration date as that of Rustan. In this case, the precious
Upon the other hand, even before the effective date of merger, on 30 August 1977, losses of Rustan may be carried over by PICOP, because
Picop sold all the outstanding shares of RMC stock to San Miguel Corporation for with the merger, PICOP assumes all the rights and
the sum of P38,900,000.00, and reported a gain of P9,294,849.00 from this obligations of Rustan subject, however, to the period
transaction. 25 prescribed for carrying over such losses." 26 (Emphasis
supplied)

64
TAXATION II ATTY. ACOSTA -
CAJUSTIN
Curiously enough, Picop did not also seek a ruling on this matter, clearly a matter In respect of the above underscored portion of the CTA decision, we must note
of tax law, from the Bureau of Internal Revenue. Picop chose to rely solely on the that the CTA in fact overlooked the statement made by petitioner's counsel before
BOI letter-opinion. prLL the CTA that:
The CIR disallowed all the deductions claimed on the basis of RPPM's losses, "Among the attractions of the merger to Picop was the
apparently on two (2) grounds. Firstly, the previous losses were incurred by accumulated net operating loss carry-over of RMC that it
"another taxpayer," RPPM, and not by Picop in connection with Picop's own might possibly use to relieve it (Picop) from its income taxes,
registered operations. The CIR took the view that Picop, RPPM and RMC were under Section 7 (c) of R.A. 5186. Said section provides:
merged into one (1) corporate personality only on 12 January 1978, upon approval
of the merger agreement by the BOI. Thus, during the taxable year 1977, Picop on xxx xxx xxx
the one hand and RPPM and RMC on the other, still had their separate juridical
With this benefit in mind, Picop addressed three (3)
personalities. Secondly, the CIR alleged that these losses had been incurred by
questions to the BOI in a letter dated November 25, 1976.
RPPM "from the borrowing of funds" and not from carrying out of RPPM's
The BOI replied on February 21, 1977 directly answering the
registered operations. We focus on the first ground. 27
three (3) queries." 30 (Emphasis supplied)
The CTA upheld the deduction claimed by Picop; its reasoning, however, is less
The size of RPPM's accumulated losses as of the date of the merger more than
than crystal clear, especially in respect of its view of what the U.S. tax law was on
P81,000,000.00 must have constituted a powerful attraction indeed for Picop.
this matter. In any event, the CTA apparently fell back on the BOI opinion of 21
February 1977 referred to above. The CTA said: The Court of Appeals followed the result reached by the CTA. The Court of
Appeals, much like the CTA, concluded that since RPPM was dissolved on 30
"Respondent further averred that the incentives granted
November 1977, its accumulated losses were appropriately carried over by Picop
under Section 7 of R.A. No. 5186 shall be available only to the
in the latter's 1977 Income Tax Return "because by that time RPPMI and Picop
extent in which they are engaged in registered operations,
were no longer separate and different taxpayers." 31
citing Section 1 of Rule IX of the Basic Rules and Regulations
to Implement the Intent and Provisions of the Investment After prolonged consideration and analysis of this matter, the Court is unable to
Incentives Act, R.A. No. 5186. agree with the CTA and Court of Appeals on the deductibility of RPPM's
accumulated losses against Picop's 1977 gross income.
We disagree with respondent. The purpose of the merger
was to rationalize the container board industry and not to It is important to note at the outset that in our jurisdiction, the ordinary rule that
take advantage of the net losses incurred by RPPMI prior to is, the rule applicable in respect of corporations not registered with the BOI as a
the stock swap. Thus, when stock of a corporation is preferred pioneer enterprise is that net operating losses cannot be carried over.
purchased in order to take advantage of the corporation's net Under our Tax Code, both in 1977 and at present, losses may be deducted from
operating loss incurred in years prior to the purchase, the gross income only if such losses were actually sustained in the same year that they
corporation thereafter entering into a trade or business are deducted or charged off. Section 30 of the 1977 Tax Codeprovides:
different from that in which it was previously engaged, the
net operating loss carry-over may be entirely lost. [IRC "SECTION 30. Deductions from Gross Income. In
(1954), Sec. 382(a), Vol. 5, Mertens, Law of Federal Income computing net income, there shall be allowed as deduction
Taxation, Chap. 29.11a, p. 103]. 28 Furthermore, once the
BOI approved the merger agreement, the registered capacity xxx xxx xxx
of Rustan shall be transferred to PICOP , and the previous (d) Losses:
losses of Rustan may be carried over by PICOP by operation
of law. [BOI ruling dated February 21, 1977 (Exh. J-1)] It is (1) By Individuals. In the case of an individual,
clear therefrom, that the deduction availed of under Section losses actually sustained during the taxable year and not
7(c) of R.A. No. 5186 was only proper." (pp. 38-43, Rollo of compensated for by an insurance or otherwise
SP No. 20070)" 29 (Emphasis supplied)
(A) If incurred in trade or business;

65
TAXATION II ATTY. ACOSTA -
CAJUSTIN
xxx xxx xxx
(2) By Corporations. In a case of a corporation, all Thus it is that R.A. No. 5186 introduced the carry-over of net operating losses as
losses actually sustained and charged off within the taxable a very special incentive to be granted only to registered pioneer enterprises and
year and not compensated for by insurance or otherwise. only with respect to their registered operations. The statutory purpose here may
be seen to be the encouragement of the establishment and continued operation of
(3) By Non-resident Aliens or Foreign Corporations. In pioneer industries by allowing the registered enterprise to accumulate its
the case of a non-resident alien individual or a foreign operating losses which may be expected during the early years of the enterprise
corporation, the losses deductible are those actually and to permit the enterprise to offset such losses against income earned by it in
sustained during the year incurred in business or trade later years after successful establishment and regular operations. To promote its
conducted within the Philippines, . . . .." 32 (Emphasis economic development goals, the Republic foregoes or defers taxing the income of
supplied) the pioneer enterprise until after that enterprise has recovered or offset its earlier
losses. We consider that the statutory purpose can be served only if the
Section 76 of the Philippine Income Tax Regulations (Revenue Regulation
accumulated operating losses are carried over and charged off against income
No. 2, as amended) is even more explicit and detailed:
subsequently earned and accumulated by the same enterprise engaged in the
"Section. 76. When charges are deductible. Each year's same registered operations.
return, so far as practicable, both as to gross income and
deductions therefrom should be complete in itself, and In the instant case, to allow the deduction claimed by Picop would be to permit
taxpayers are expected to make every reasonable effort to one corporation or enterprise, Picop, to benefit from the operating losses
ascertain the facts necessary to make a correct return. The accumulated by another corporation or enterprise, RPPM. RPPM far from
expenses, liabilities, or deficit of one year cannot be used to benefitting from the tax incentive granted by the BOI statute, in fact gave up the
reduce the income of a subsequent year. A taxpayer has the struggle and went out of existence and its former stockholders joined the much
right to deduct all authorized allowances and it follows that if larger group of Picop's stockholders. To grant Picop's claimed deduction would be
he does not within any year deduct certain of his expenses, to permit Picop to shelter its otherwise taxable income (an objective which Picop
losses, interests, taxes, or other charges, he can not deduct had from the very beginning) which had not been earned by the registered
them from the income of the next or any succeeding year. . . . enterprise which had suffered the accumulated losses. In effect, to grant Picop's
claimed deduction would be to permit Picop to purchase a tax deduction and
xxx xxx xxx RPPM to peddle its accumulated operating losses. Under the CTA and Court of
Appeals decisions, Picop would benefit by immunizing P44,196,106.00 of its
. . .. If subsequent to its occurrence, however, a taxpayer first income from taxation thereof although Picop had not run the risks and incurred
ascertains the amount of a loss sustained during a prior the losses which had been encountered and suffered by RPPM. Conversely, the
taxable year which has not been deducted from gross income that would be shielded from taxation is not income that was, after much
income, he may render an amended return for such effort, eventually generated by the same registered operations which earlier had
preceding taxable year including such amount of loss in the sustained losses. We consider and so hold that there is nothing in Section 7 (c) of
deduction from gross income and may in proper cases file R. A. No. 5186 which either requires or permits such a result. Indeed, that result
a claim for refund of the excess paid by reason of the failure makes non-sense of the legislative purpose which may be seen clearly to be
to deduct such loss in the original return. A loss from theft or projected by Section 7 (c), R.A. No. 5186.
embezzlement occurring in one year and discovered in
another is ordinarily deductible for the year in which The CTA and the Court of Appeals allowed the offsetting of RPPM's accumulated
sustained." (Emphases supplied) operating losses against Picop's 1977 gross income, basically because towards the
end of the taxable year 1977, upon the arrival of the effective date of merger, only
It is thus clear that under our law, and outside the special realm of BOI-registered one (1) corporation, Picop, remained. The losses suffered by RPPM's registered
enterprises, there is no such thing as a carry-over of net operating loss. To the operations and the gross income generated by Picop's own registered operations
contrary, losses must be deducted against current income in the taxable year now came under one and the same corporate roof. We consider that this
when such losses were incurred. Moreover, such losses may be charged off only circumstance relates much more to form than to substance. We do not believe
against income earned in the same taxable year when the losses were incurred. that that single purely technical factor is enough to authorize and justify the

66
TAXATION II ATTY. ACOSTA -
CAJUSTIN
deduction claimed by Picop. Picop's claim for deduction is not only bereft of or official receipts of the Register of Deeds, these vouchers
statutory basis; it does violence to the legislative intent which animates the tax standing alone cannot prove that the payments made were
incentive granted by Section 7 (c) of R.A. No. 5186. In granting the extraordinary for the accrued expenses in question. The best evidence of
privilege and incentive of a net operating loss carry-over to BOI-registered payment is the official receipts issued by the Register of
pioneer enterprises, the legislature could not have intended to require the Deeds. The testimony of petitioner's witness that the official
Republic to forego tax revenues in order to benefit a corporation which had run receipts and cash vouchers were shown to the Bureau of
no risks and suffered no losses, but had merely purchased another's losses. prLL Internal Revenue will not suffice if no records could be
presented in court for proper marking and
Both the CTA and the Court of Appeals appeared much impressed not only with identification." 34 (Emphasis supplied)
corporate technicalities but also with the U.S. tax law on this matter. It should
suffice, however, simply to note that in U.S. tax law, the availability to companies The Court of Appeals added:
generally of operating loss carry-overs and of operating loss carry-backs
"The mere testimony of a witness for PICOP and the cash
is expressly provided and regulated in great detail by statute. 33 In our
vouchers do not suffice to establish its claim that registration
jurisdiction, save for Section 7 (c) of R.A. No. 5186, no statute recognizes or
fees were paid to the Register of Deeds for the registration of
permits loss carry-overs and loss carry-backs. Indeed, as already noted, our tax
real estate and chattel mortgages in favor of Development
law expressly rejects the very notion of loss carry-overs and carry-backs.
Bank of the Philippines and the Philippine National Bank as
We conclude that the deduction claimed by Picop in the amount of guarantors of PICOP's loans. The witness could very well
P44,196,106.00 in its 1977 Income Tax Return must be disallowed. have been merely repeating what he was instructed to say
regardless of the truth, while the cash vouchers, which we do
(3) Whether Picop is entitled to deduct against current income not find on file, are not said to provide the necessary details
certain claimed financial guarantee expenses. regarding the nature and purpose of the expenses reflected
therein. PICOP should have presented, through the
In its Income Tax Return for 1977, Picop also claimed a deduction in the amount guarantors, its owner's copy of the registered titles with the
of P1,237,421.00 as financial guarantee expenses. lien inscribed thereon as well as an official receipt from the
Register of Deeds evidencing payment of the registration
This deduction is said to relate to chattel and real estate mortgages required from
fee." 35 (Emphasis supplied)
Picop by the Philippine National Bank ("PNB") and DBP as guarantors of loans
incurred by Picop from foreign creditors. According to Picop, the claimed We must support the CTA and the Court of Appeals in their foregoing rulings. A
deduction represents registration fees and other expenses incidental to taxpayer has the burden of proving entitlement to a claimed deduction. 36 In the
registration of mortgages in favor of DBP and PNB. instant case, even Picop 's own vouchers were not submitted in evidence and the
BIR Examiners denied that such vouchers and other documents had been
In support of this claimed deduction, Picop allegedly showed its own vouchers to
exhibited to them. Moreover, cash vouchers can only confirm the fact of
BIR Examiners to prove disbursements to the Register of Deeds of Tandag,
disbursement but not necessarily the purpose thereof. 37 The best evidence that
Surigao del Sur, of particular amounts. In the proceedings before the CTA,
Picop should have presented to support its claimed deduction were the invoices
however, Picop did not submit in evidence such vouchers and instead presented
and official receipts issued by the Register of Deeds. Picop not only failed to
one of its employees to testify that the amount claimed had been disbursed for the
present such documents; it also failed to explain the loss thereof, assuming they
registration for chattel and real estate mortgages.
had existed before. 38 Under the best evidence rule, 39 therefore, the testimony
The CIR disallowed this claimed deduction upon the ground of insufficiency of of Picop's employee was inadmissible and was in any case entitled to very little, if
evidence. This disallowance was sustained by the CTA and the Court of Appeals. any, credence.
The CTA said:
We consider that entitlement to Picop's claimed deduction of P1,237,421.00 was
"No records are available to support the above mentioned not adequately shown and that such deduction must be disallowed.
expenses. The vouchers merely showed that the amounts
III
were paid to the Register of Deeds and simply cash
account. Without the Supporting papers such as the invoices

67
TAXATION II ATTY. ACOSTA -
CAJUSTIN
(1) Whether Picop had understated its sales and overstated its cost of Discrepancy P604,018.00
sales for 1977.
===========
In its assessment for deficiency income tax for 1977, the CIR claimed that Picop
had understated its sales by P2,391,644.00 and, upon the other hand, overstated Picop did not deny the existence of the above noted discrepancies. In the
its cost of sales by P604,018.00. Thereupon, the CIR added back both sums to proceedings before the CTA, Picop presented one of its officials to explain the
Picop's net income figure per its own return. LLpr foregoing discrepancies. That explanation is perhaps best presented in Picop's
own words as set forth in its Memorandum before this Court:
The 1977 Income Tax Return of Picop set forth the following figures:
Sales (per Picop's Income Tax Return):
". . . that the adjustment discussed in the testimony of the
Paper P537,656,719.00 witness, represent the best and most objective method of
determining in pesos the amount of the correct and actual
Timber P263,158,132.00 export sales during the year. It was this correct and actual
export sales and costs of sales that were reflected in the

income tax return and in the audited financial statements.
Total Sales P800,814,851.00 These corrections did not result in realization of income and
should not give rise to any deficiency tax.
===========
xxx xxx xxx
Upon the other hand, Picop's Books of Accounts reflected higher sales
figures: What are the facts of this case on this matter? Why were
adjustments necessary at the year-end?
Sales (per Picop 's Books of Accounts):
Because of PICOP's procedure of recording its export sales
Paper P537,656,719.00 (reckoned in U.S. dollars) on the basis of a fixed rate, day to
Timber P265,549,776.00 day and month to month, regardless of the actual exchange
rate and without waiting when the actual proceeds are
received. In other words, PICOP recorded its export sales at a
pre-determined fixed exchange rate. That pre-determined
Total Sales P803,206,495.00 rate was decided upon at the beginning of the year and
continued to be used throughout the year.
===========
At the end of the year, the external auditors made an
The above figures thus show a discrepancy between the sales figures reflected in examination. In that examination, the auditors determined
Picop's Books of Accounts and the sales figures reported in its 1977 Income Tax with accuracy the actual dollar proceeds of the export sales
Return, amounting to: P2,391,644.00. received. What exchange rate was used by the auditors to
The CIR also contended that Picop's cost of sales set out in its 1977 Income Tax convert these actual dollar proceeds into Philippine pesos?
Return, when compared with the cost figures in its Books of Accounts, was They used the average of the differences between (a) the
overstated: recorded fixed exchange rate and (b) the exchange rate at the
time the proceeds were actually received. It was this rate at
Cost of Sales (per Income Tax Return) P607,246,084.00 time of receipt of the proceeds that determined the amount of
pesos credited by the Central Bank (through the agent banks)
Cost of Sales (per Books of Accounts) P606,642,066.00 in favor of PICOP. These accumulated differences were
averaged by the external auditors and this was what was used

68
TAXATION II ATTY. ACOSTA -
CAJUSTIN
at the year-end for income tax and other government-report (2) Whether Picop is liable for the corporate development tax of five
purposes. (T.s.n., Oct. 17/85, pp. 20-25)" 40 percent (5%) of its income for 1977.
The above explanation, unfortunately, at least to the mind of the Court, raises The five percent (5%) corporate development tax is an additional corporate
more questions than it resolves. Firstly, the explanation assumes that all of income tax imposed in Section 24 (e) of the1977 Tax Code which reads in relevant
Picop's sales were export sales for which U.S. dollars (or other foreign exchange) part as follows:
were received. It also assumes that the expenses summed up as "cost of sales"
were all dollar expenses and that no peso expenses had been incurred. Picop's "(e) Corporate development tax. In addition to the tax
explanation further assumes that a substantial part of Picop's dollar proceeds for imposed in subsection (1) of this section, an additional tax in
its export sales were not actually surrendered to the domestic banking system and an amount equivalent to 5 per cent of the same taxable net
seasonably converted into pesos; had all such dollar proceeds been converted into income shall be paid by a domestic or a resident foreign
pesos then the peso figures could have been simply added up to reflect the actual corporation; Provided, That this additional tax shall be
peso value of Picop's export sales. Picop offered no evidence in respect of these imposed only if the net income exceeds 10 per cent of the net
assumptions, no explanation why and how a "pre-determined fixed exchange worth, in case of a domestic corporation, or net assets in the
rate" was chosen at the beginning of the year and maintained throughout. Philippines in case of a resident foreign corporation: . . . .
Perhaps more importantly, Picop was unable to explain why its Books of Accounts
The additional corporate income tax imposed in this
did not pick up the same adjustments that Picop's External Auditors were alleged
subsection shall be collected and paid at the same time and in
to have made for purposes of Picop's Income Tax Return. Picop attempted to
the same manner as the tax imposed in subsection (a) of this
explain away the failure of its Books of Accounts to reflect the same adjustments
section."
(no correcting entries, apparently) simply by quoting a passage from a case where
this Court refused to ascribe much probative value to the Books of Accounts of a Since this five percent (5%) corporate development tax is an income tax,
corporate taxpayer in a tax case. 41 What appears to have eluded Picop, however, Picop is not exempted from it under the provisions of Section 8 (a) of R.A.
is that its Books of Accounts, which are kept by its own employees and are No. 5186.
prepared under its control and supervision, reflect what may be deemed to be
admissions against interest in the instant case. For Picop's Books of Accounts For purposes of determining whether the net income of a corporation exceeds ten
precisely show higher sales figures and lower cost of sales figures than Picop's percent (10%) of its net worth, the term "net worth" means the stockholders'
Income Tax Return. equity represented by the excess of the total assets over liabilities as reflected in
the corporation's balance sheet provided such balance sheet has been prepared in
It is insisted by Picop that its Auditors' adjustments simply present the "best and accordance with generally accepted accounting principles employed in keeping
most objective" method of reflecting in pesos the "correct and ACTUAL export the books of the corporation. 43
sales" 42 and that the adjustments or "corrections" "did not result in realization
of [additional] income and should not give rise to any deficiency tax." The The adjusted net income of Picop for 1977, as will be seen below, is
correctness of this contention is not self-evident. So far as the record of this case P48,687,355.00. Its net worth figure or total stockholders' equity as reflected in
shows, Picop did not submit in evidence the aggregate amount of its U.S. dollar its Audited Financial Statements for 1977 is P464,749,528.00. Since its adjusted
proceeds of its export sales; neither did it show the Philippine pesos it had net income for 1977 thus exceeded ten percent (10%) of its net worth, Picop must
actually received or been credited for such U.S. dollar proceeds. It is clear to this be held liable for the five percent (5%) corporate development tax in the amount
Court that the testimonial evidence submitted by Picop fell far short of of P2,434,367.75.
demonstrating the correctness of its explanation. Recapitulating, we hold:
Upon the other hand, the CIR has made out at least a prima facie case that Picop (1) Picop is liable for the thirty five percent (35%) transaction tax in the amount of
had understated its sales and overstated its cost of sales as set out in its Income P3,578,543.51.
Tax Return. For the CIR has a right to assume that Picop's Books of Accounts
speak the truth in this case since, as already noted, they embody what must (2) Picop is not liable for interest and surcharge on unpaid transaction tax.
appear to be admissions against Picop's own interest.
(3) Picop is exempt from payment of documentary and science stamp taxes in the
Accordingly, we must affirm the findings of the Court of Appeals and the CTA. amount of P300,000.00 and the compromise penalty of P300.00.

69
TAXATION II ATTY. ACOSTA -
CAJUSTIN
(4) Picop is entitled to its claimed deduction of P42,840,131.00 for interest Tax Already Assessed per Return 80,358.00
payments on loans for, among other things, the purchase of machinery and
equipment.

(5) Picop's claimed deduction in the amount of P44,196,106.00 for the operating Deficiency Income Tax P16,560,216.00
losses previously incurred by RPPM, is disallowed for lack of merit.
Add:
(6) Picop's claimed deduction for certain financial guarantee expenses in the
amount P1,237,421.00 is disallowed for failure adequately to prove such expenses. Five percent (5%) Corporate
Development Tax P2,434,367.00
(7) Picop has understated its sales by P2,391,644.00 and overstated its cost of
sales by P604,018.00, for 1977. Total Deficiency Income Tax P18,994,583.00

(8) Picop is liable for the corporate development tax of five percent (5%) of its ===========
adjusted net income for 1977 in the amount of P2, 434,367.75.
Add:
Considering conclusions nos. 4, 5, 6, 7 and 8, the Court is compelled to hold Picop
liable for deficiency income tax for the year 1977 computed as follows: Five percent (5%) surcharge 45 P994.583.00

Deficiency Income Tax

Net Income Per Return P258,166.00 Total Deficiency Income Tax


with surcharge P19,944,312.15
Add:
Add:
Unallowable Deductions
Fourteen percent (14%)
(1) Deduction of net operating interest from 15 April 1978
losses incurred by RPPM P44,196,106.00 to 14 April 1981 46 P8,376,610.80

(2) Unexplained financial Fourteen percent (14%)


guarantee expenses P1,237,421.00 interest from 21 April 1983
to 20 April 1986 47 P11,894 787.00
(3) Understatement of Sales P2,391,644.00

(4) Overstatement of Cost of Sales P604,018.00
Total Deficiency Income Tax
Due and Payable P40,215.709.00
Total P48,429,189.00 ===========
WHEREFORE, for all the foregoing, the Decision of the Court of Appeals is
hereby MODIFIED and Picop is hereby ORDERED to pay the CIR the aggregate
Net Income as Adjusted P48,687,355.00 amount of P43,794,252.51 itemized as follows:
=========== (1) Thirty-five percent (35%) transaction tax P3, 578,543.51
Income Tax Due Thereon 44 P17,030,574.00 (2) Total Deficiency Income Tax Due 40,215,709.00

Less:

70
TAXATION II ATTY. ACOSTA -
CAJUSTIN
Aggregate Amount Due and Payable P43,794,252.51 instant case because while it implements sections 30(c) of the Tax Code
governing deduction of taxes, the respondent taxpayer seeks to come under
=========== section 30(b) of the same Code providing for deduction of interest on
No pronouncement as to costs. indebtedness.

SO ORDERED. ISSUE:
[G.R. No. L-13912. September 30, 1960.]
Whether or not such interest was paid upon an indebtedness within the
THE COMMISSIONER OF INTERNAL REVENUE, Petitioner, v. contemplation of section 30 (b) (1) of the Tax Code?
CONSUELO L. VDA. DE PRIETO, Respondent.
RULING:
Solicitor General Edilberto Barot, Solicitor F. R. Rosete and Special
Atty. B. Gatdula, Jr. for Petitioner. Yes. According to the Supreme Court, although interest payment for
delinquent taxes is not deductible as tax under Section 30(c) of the Tax Code
Formilleza & Latorre for Respondent. and section 80 of the Income Tax Regulations, the taxpayer is not precluded
thereby from claiming said interest payment as deduction under section 30(b)
of the same Code.
On December 4, 1945, the respondent conveyed by way of gifts to her four
SEC. 30 Deductions from gross income. In computing net income there
children, namely, Antonio, Benito, Carmen and Mauro, all surnamed Prieto,
shall be allowed as deductions
real property with a total assessed value of P892,497.50. After the filing of the
gift tax returns on or about February 1, 1954, the petitioner Commissioner of
Internal Revenue appraised the real property donated for gift tax purposes at (b) Interest:
P1,231,268.00, and assessed the total sum of P117,706.50 as donor's gift tax,
interest and compromises due thereon. Of the total sum of P117,706.50 paid by (1) In general. The amount of interest paid within the taxable year
respondent on April 29, 1954, the sum of P55,978.65 represents the total on indebtedness, except on indebtedness incurred or continued to
interest on account of deliquency. This sum of P55,978.65 was claimed as purchase or carry obligations the interest upon which is exempt from
deduction, among others, by respondent in her 1954 income tax return. taxation as income under this Title.
Petitioner, however, disallowed the claim and as a consequence of such
disallowance assessed respondent for 1954 the total sum of P21,410.38 as
deficiency income tax due on the aforesaid P55,978.65, including interest up to The term "indebtedness" as used in the Tax Code of the United States
March 31, 1957, surcharge and compromise for the late payment. containing similar provisions as in the above-quoted section has been defined
as an unconditional and legally enforceable obligation for the payment of
Under the law, for interest to be deductible, it must be shown that there be an money.
indebtedness, that there should be interest upon it, and that what is claimed as
an interest deduction should have been paid or accrued within the year. It is To give to the quoted portion of section 80 of our Income Tax Regulations the
here conceded that the interest paid by respondent was in consequence of the meaning that the petitioner gives it would run counter to the provision of
late payment of her donor's tax, and the same was paid within the year it is section 30(b) of the Tax Code and the construction given to it by courts in the
sought to be declared. United States. Such effect would thus make the regulation invalid for a
"regulation which operates to create a rule out of harmony with the statute, is a
To sustain the proposition that the interest payment in question is not mere nullity." As already stated, section 80 implements only section 30(c) of
deductible for the purpose of computing respondent's net income, petitioner the Tax Code, or the provision allowing deduction of taxes, while herein
relies heavily on section 80 of Revenue Regulation No. 2 (known as Income respondent seeks to be allowed deduction under section 30(b), which provides
Tax Regulation) promulgated by the Department of Finance, which provides for deduction of interest on indebtedness.
that "the word `taxes' means taxes proper and no deductions should be
allowed for amounts representing interest, surcharge, or penalties incident to
delinquency." The court below, however, held section 80 as inapplicable to the

71
TAXATION II ATTY. ACOSTA -
CAJUSTIN
SYLLABUS the petitioner Commissioner of Internal Revenue appraised the real property
donated for gift tax purposes at P1,231,268.00 and assessed the total sum of
1. TAXATION; DEFICIENCY INCOME TAX; REQUISITES IN ORDER THAT P117,706.50 as donors gift tax, interests and compromises due thereon. Of the
INTEREST MAY BE DEDUCTIBLE. For interest to be allowed as deduction total sum of P117,706.50 paid by respondent on April 29, 1954, the sum of
from gross income, it must be shown that there be indebtedness, that there P55,978.65 represents the total interest on account of delinquency. This sum of
should be interest upon it, and that what is claimed as an interest deduction P55,978.65 was claimed as deduction, among others, by respondent in her 1954
should have been paid or accrued within the year. income tax return. Petitioner, however, disallowed the claim and as a
consequence of such disallowance assessed respondent for 1954 the total sum of
2. ID.; ID.; ID.; TAX AS AN INDEBTEDNESS; INTEREST PAID FOR LATE P21,410.38 as deficiency income tax due on the aforesaid P55,978.65, including
PAYMENT OF DONORS TAX DEDUCTIBLE. The term "indebtedness" as interest up to March 31, 1957, surcharge and compromise for the late payment.
used in the Tax Code of the United States containing similar provisions as in
section 30 (b) (1) of our Tax Code, has been defined as an unconditional and Under the law, for interest to be deductible, it must be shown that there be an
legally enforceable obligation for the payment of money. Within the meaning indebtedness, that there should be interest upon it, and that what is claimed as an
of that definition a tax may be considered an indebtedness. Hence, interest interest deduction should have been paid or accrued within the year. It is here
paid for late payment of donors tax is deductible from gross income under said conceded that the interest paid by respondent was in consequence of the late
section. payment of her donors tax, and the same was paid within the year it is sought to
be deducted. The only question to be determined, as stated by the parties, is
3. ID.; ID.; ID.; WHEN SECTION 80 OF REVENUE REGULATION NO. 2 IS whether or not such interest was paid upon an indebtedness within the
NOT APPLICABLE. Although section 80 of Revenue Regulation No. 2 contemplation of section 30(b) (1) of the Tax Code, the pertinent part of which
(known as Income Tax Regulations) promulgated by the Department of reads:jgc:chanrobles.com.ph
Finance, which provides that "the word taxes means taxes proper and no
deductions should be allowed for amounts representing interest, surcharge, or "Sec. 30 Deductions from gross income. In computing net income there shall be
penalties incident to delinquency," implements section 30 (c) of the Tax Code allowed as deductions
governing deductions of taxes, the same is inapplicable to a case where the x x x
taxpayer seeks to come under section 30 (b) of the same Code providing for
deduction of interest on indebtedness.
"(b) Interest:
4. ID.; ID.; ID.; ID.; TAXPAYER NOT PRECLUDED FROM CLAIMING
INTEREST PAYMENT AS DEDUCTION. Although interest payment for "(1) In general. The amount of interest paid within the taxable year on
delinquency taxes is not deductible as tax under section 30 (c) of the Tax Code indebtedness, except on indebtedness incurred or continued to purchase or carry
and section 80 of the Income Tax Regulations, the taxpayer is not said interest obligations the interest upon which is exempt from taxation as income under this
payment as deduction under section 30 (b) of the same code. Title."cralaw virtua1aw library

The term "indebtedness" as used in the Tax Code of the United States containing
similar provisions as in the above-quoted section has been defined as an
unconditional and legally enforceable obligation for the payment of money.
This is an appeal from a decision of the Court of Tax Appeals reversing the
(Federal Taxes Vol. 2, p. 13,019, Prentice-Hall, Inc.; Mertens Law of Federal
decision of the Commissioner of Internal Revenue which held herein respondent
Income Taxation, Vol. 4, p. 542.) Within the meaning of that definition, it is
Consuelo L. Vda. de Prieto liable for the payment of the sum of P21,410.38 as
apparent that a tax may be considered an indebtedness. As stated by this Court in
deficiency income tax, plus penalties and monthly interest.
the case of Santiago Sambrano v. Court of Tax Appeals and Collector of Internal
Revenue (101 Phil., 1; 53 Off. Gaz., 4839)
The case was submitted for decision in the court below upon a stipulation of facts,
which for brevity is summarized as follows: On December 4, 1945, the respondent
conveyed by way of gifts to her four children, namely, Antonio, Benito, Carmen "Although taxes already due have not, strictly speaking, the same concept as
debts, they are, however, obligations that may be considered as such.
and Mauro, all surnamed Prieto, real property with a total assessed value of
P892,497.50. After the filing of the gift tax returns on or about February 1, 1954,

72
TAXATION II ATTY. ACOSTA -
CAJUSTIN
"The term "debt" is properly used in a comprehensive sense as embracing not ruling to be correct. Contrary to petitioners belief, the portion of section 80 of
merely money due by contract but whatever one is bound to render to another, Revenue Regulation No. 2 under consideration has been part and parcel of the
either for contract, or the requirement of the law. (Camben v. Fink Coule & Coke development to the law on deduction of taxes in the United States. (See Capital
Co. 61 LRA 584). Bldg. & Loan Assn. v. Comm., 23 BTA 848. Thus, Mertens in his treatise says:
"Penalties are to be distinguished from taxes and they are not deductible under
"Where statute imposes a personal liability for a tax, the tax becomes, at least in a the heading of taxes.." . . Interest on state taxes is not deductible as taxes." (Vol. 5,
board sense, a debt. (Idem). Law on Federal income Taxation, pp. 22-23, sec. 27.06, citing cases.) This
notwithstanding, courts in that jurisdiction, however, have invariably held that
"A tax is a debt for which a creditors bill may be brought in a proper case. (State interest on deficiency taxes are deductible, not as taxes, but as interest. (U.S. v.
v. Georgia Co., 19 LEA 485)."cralaw virtua1aw library Jaffray, Et Al., supra; see also Mertens, sec. 26.09, Vol. 4, p. 552, and cases cited
therein.) Section 80 of Revenue Regulation No. 2, therefore, merely incorporated
It follows that the interest paid by herein respondent for the late payment of her the established application of the tax deduction statute in the United States,
donors tax is deductible from her gross income under section 30 (b) of the Tax where deduction of "taxes" has always been limited to taxes proper and has never
Code above quoted. included interest on delinquent taxes, penalties and surcharges.

The above conclusion finds support in the established jurisprudence in the United To give to the quoted portion of section 80 of our Income Tax Regulations the
States after whose laws our Income Tax Law has been patterned. Thus, under sec. meaning that the petitioner gives it would run counter to the provision of section
23(b) of the Internal Revenue Code of 1939, as amended 1, which contains 30(b) of the Tax Code and the construction given to it by courts in the United
similarly worded provisions as sec. 30(b) of our Tax Code, the uniform ruling is States. Such effect would thus make the regulation invalid for a "regulation which
that interest on taxes is interest on indebtedness and is deductible. (U.S. v. operates to create a rule out of harmony with the statute, is a mere nullity."
Jaffray, 306 U.S. 276. See also Lustig v. U.S., 138 F. Supp. 870; Commissioner of (Lynch v. Tilden Produce Co., 265 U.S. 315; Miller v. U.S., 294 U.S. 435.) As
Internal Revenue v. Bryer, 151 F. 2d 267, 34 AFTR 151; Penrose v. U.S. 18 F. Supp. already stated, section 80 implements only section 30(c) of the Tax Code, or the
413, 18 AFTR 1289; Max Thomas Davis, Et. Al. v. Commissioner of Internal provision allowing deduction of taxes, while herein respondent seeks to be
Revenue, 46, U.S. Board of Tax Appeals Reports, p. 663, citing U.S. v. Jaffray, allowed deduction under section 30(b), which provides for deduction of interest
supra, Smith v. Commissioner of Internal Revenue, 6 Tax Court of limited States on indebtedness.
Reports, p. 255; Armour v. Commissioner of Internal Revenue, 6 Tax Court of the
United States Reports, p. 359; The Koppers Coal Co. v. Commissioner of Internal In conclusion, we are of the opinion and so hold that although interest payment
Revenue, 7 Tax Court of United States Reports, p. 1209; Toy v. Commissioner of for delinquent taxes is not deductible as tax under Section 30(c) of the Tax Code
Internal Revenue; Lucas v. Comm., 34 U.S. Board of Tax Appeals Reports, 877; and section 80 of the Income Tax Regulations, the taxpayer is not precluded
Evens & Howard Fire Brick Co. v. Commissioner of Internal Revenue, 8 U.S. thereby from claiming said interest payment as deduction under section 30(b) of
Board of Tax Appeals Reports, 867; Koppers Co. v. Commissioner of Internal the same Code.
Revenue, 3 Tax Court of United States Reports, p. 62). The rule applies even
though the tax is nondeductible. (Federal Taxes, Vol. 2, Prentice Hall, sec. 163, In view of the foregoing, the decision sought to be reviewed is affirmed, without
13,022; see also Mertens Law of Federal Income Taxation, Vol. 5, pp. 23-24.) pronouncement as to costs.

To sustain the proposition that the interest payment in question is not deductible
for the purpose of computing respondents net income, petitioner relies heavily on
section 80 of Revenue Regulation No. 2 (known as Income Tax Regulation)
promulgated by the Department of Finance, which provides that "the word taxes
means taxes proper and no deductions should be allowed for amounts
representing interest, surcharge, or penalties incident to delinquency." The court
below, however, held section 80 as inapplicable to the instant case because while
it implements sections 30(c) of the Tax Code governing deduction of taxes, the
respondent taxpayer seeks to come under section 30(b) of the same Code
providing for deduction of interest on indebtedness. We find the lower courts TOPIC: DEDUCTIONS TAXES

73
TAXATION II ATTY. ACOSTA -
CAJUSTIN
G.R. Nos. L-18169, L-18262 & L-21434 July 31, 1964 his income is derived wholly from sources within the Philippines.

COMMISSIONER OF INTERNAL REVENUES, petitioner, Double taxation becomes obnoxious only where the taxpayer is taxed twice
vs. for the benefit of the same governmental entity. In the present case, although
V.E. LEDNICKY and MARIA VALERO LEDNICKY, respondents. the taxpayer would have to pay two taxes on the same income but the
Philippine government only receives the proceeds of one tax, there is no
obnoxious double taxation.
Principle/s:
- Alien residents deduction of Income Taxation from Gross Income paid in
their home country SYLLABUS
- Double Taxation
1. TAXATION; INCOME TAX; DEDUCTIONS FROM GROSS INCOME;
FOREIGN INCOME TAX PAID BY ALIEN RESIDENT. An alien resident
FACTS: who derives income wholly from sources within the Philippines may not
Spouses are both American citizens residing in the Philippines and have deduct from gross income the income taxes he paid to his home country for the
derived all their income from Philippine sources for taxable years in question. taxable year.

On March, 1957, filed their ITR for 1956, reporting gross income of 2. ID.; ID.; ID.; RIGHT TO DEDUCT FOREIGN TAXES PAID GIVEN ONLY
P1,017,287.65 and a net income of P 733,809.44. On March 1959, file an WHERE ALTERNATIVE RIGHT TO TAX CREDIT EXISTS. An alien
amended claimed deduction of P 205,939.24 paid in 1956 to the resident's right to deduct from gross income the income taxes he paid to a
United States government as federal income tax of 1956. foreign government is given only as an alternative to his right to claim a tax
credit for such foreign income taxes; so that unless he has a right to claim such
tax credit if he chooses, he is precluded from said deduction.
ISSUE:
Whether a citizen of the United States residing in the Philippines, who derives
wholly from sources within the Philippines, may deduct his gross income from 3. ID.; ID; ID.; WHEN ALIEN RESIDENT NOT ENTITLED TO TAX CREDIT.
the income taxes he has paid to the United States government for the said An alien resident is not entitled to tax credit for foreign income taxes paid
taxable year? when his income is derived wholly from sources within the Philippines.

4. ID.; ID.; DOUBLE TAXATION; NOT OBNOXIOUS WHERE TAXES PAID


HELD: TO DIFFERENT JURISDICTION; CASE AT BAR. Double taxation becomes
An alien resident who derives income wholly from sources within the obnoxious only where the taxpayer is taxed twice for the benefit of the same
Philippines may not deduct from gross income the income taxes he paid to his governmental entity. In the present case, although the taxpayer would have to
home country for the taxable year. The right to deduct foreign income taxes pay two taxes on the same income but the Philippine government only receives
paid given only where alternative right to tax credit exists. the proceeds of one tax, there is no obnoxious double taxation.

Section 30 of the NIRC, Gross Income Par. C (3): Credits against tax per taxes
of foreign countries.
The above-captioned cases were elevated to this Court under separate petitions by
If the taxpayer signifies in his return his desire to have the benefits of the Commissioner for review of the corresponding decisions of the Court of Tax
this paragraph, the tax imposed by this shall be credited with: Paragraph Appeals. Since these cases involve the same parties and issues akin to each case
(B), Alien resident of the Philippines; and, Paragraph C (4), Limitation presented, they are herein decided jointly.
on credit.

An alien resident not entitled to tax credit for foreign income taxes paid when

74
TAXATION II ATTY. ACOSTA -
CAJUSTIN
The respondents, V. E. Lednicky and Maria Valero Lednicky, are husband and adducing other evidence to prove their case not covered by this stipulation of
wife, respectively, both American citizens residing in the Philippines, and have facts. 1wph1.t
derived all their income from Philippine sources for the taxable years in question.
On 11 August 1958, the said respondents amended their Philippine income tax
In compliance with local law, the aforesaid respondents, on 27 March 1957, filed return for 1955 to include the following deductions:
their income tax return for 1956, reporting therein a gross income of P1,017,287.
65 and a net income of P733,809.44 on which the amount of P317,395.4 was
assessed after deducting P4,805.59 as withholding tax. Pursuant to the U.S. Federal income taxes P471,867.32
petitioner's assessment notice, the respondents paid the total amount of
P326,247.41, inclusive of the withheld taxes, on 15 April 1957. Interest accrued up to May 15, 1955 40,333.92

Exchange and bank charges 4,143.91


On 17 March 1959, the respondents Lednickys filed an amended income tax
return for 1956. The amendment consists in a claimed deduction of P205,939.24
paid in 1956 to the United States government as federal income tax for 1956.
Simultaneously with the filing of the amended return, the respondents requested Total P516,345.15
the refund of P112,437.90.
and therewith filed a claim for refund of the sum of P166,384.00, which was later
When the petitioner Commissioner of Internal Revenue failed to answer the claim reduced to P150,269.00.
for refund, the respondents filed their petition with the Tax Court on 11 April 1959
as CTA Case No. 646, which is now G. R. No. L-18286 in the Supreme Court.
The respondents Lednicky brought suit in the Tax Court, which was docketed
therein as CTA Case No. 570.
G. R. No. L-18169 (formerly CTA Case No. 570) is also a claim for refund in the
amount of P150,269.00, as alleged overpaid income tax for 1955, the facts of
which are as follows: In G. R. No. 21434 (CTA Case No. 783), the facts are similar, but refer to
respondents Lednickys' income tax return for 1957, filed on 28 February 1958,
and for which respondents paid a total sum of P196,799.65. In 1959, they filed an
On 28 February 1956, the same respondents-spouses filed their domestic income amended return for 1957, claiming deduction of P190,755.80, representing taxes
tax return for 1955, reporting a gross income of P1,771,124.63 and a net income of paid to the U.S. Government on income derived wholly from Philippine sources.
P1,052,550.67. On 19 April 1956, they filed an amended income tax return, the On the strength thereof, respondents seek refund of P90 520.75 as overpayment.
amendment upon the original being a lesser net income of P1,012,554.51, and, on The Tax Court again decided for respondents.
the basis of this amended return, they paid P570,252.00, inclusive of withholding
taxes. After audit, the petitioner determined a deficiency of P16,116.00, which
amount, the respondents paid on 5 December 1956. The common issue in all three cases, and one that is of first impression in this
jurisdiction, is whether a citizen of the United States residing in the Philippines,
who derives income wholly from sources within the Republic of the Philippines,
Back in 1955, however, the Lednickys filed with the U.S. Internal Revenue Agent may deduct from his gross income the income taxes he has paid to the United
in Manila their federal income tax return for the years 1947, 1951, 1952, 1953, and States government for the taxable year on the strength of section 30 (C-1) of the
1954 on income from Philippine sources on a cash basis. Payment of these federal Philippine Internal Revenue Code, reading as follows:
income taxes, including penalties and delinquency interest in the amount of
P264,588.82, were made in 1955 to the U.S. Director of Internal Revenue,
Baltimore, Maryland, through the National City Bank of New York, Manila SEC. 30. Deduction from gross income. In computing net income
Branch. Exchange and bank charges in remitting payment totaled P4,143.91. there shall be allowed as deductions

Wherefore, the parties respectfully pray that the foregoing stipulation of facts be (a) ...
admitted and approved by this Honorable Court, without prejudice to the parties

75
TAXATION II ATTY. ACOSTA -
CAJUSTIN
(b) ... such taxes, allows a similar credit to citizens of the
Philippines residing in such country;
(c) Taxes:
It is well to note that the tax credit so authorized is limited under
(1) In general. Taxes paid or accrued within the paragraph 4 (A and B) of the same subsection, in the following terms:
taxable year, except
Par. (c) (4) Limitation on credit. The amount of the credit
(A) The income tax provided for under this taken under this section shall be subject to each of the following
Title; limitations:

(B) Income, war-profits, and excess profits (A) The amount of the credit in respect to the tax paid
taxes imposed by the authority of any foreign or accrued to any country shall not exceed the same
country; but this deduction shall be allowed proportion of the tax against which such credit is
in the case of a taxpayer who does not taken, which the taxpayer's net income from sources
signify in his return his desire to have to any within such country taxable under this Title bears to
extent the benefits of paragraph (3) of this his entire net income for the same taxable year; and
subsection (relating to credit for foreign
countries); (B) The total amount of the credit shall not exceed the
same proportion of the tax against which such credit is
(C) Estate, inheritance and gift taxes; and taken, which the taxpayer's net income from sources
without the Philippines taxable under this Title bears
to his entire net income for the same taxable year.
(D) Taxes assessed against local benefits of a
kind tending to increase the value of the
property assessed. (Emphasis supplied) We agree with appellant Commissioner that the Construction
and wording of Section 30 (c) (1) (B) of the Internal Revenue
Act shows the law's intent that the right to deduct income taxes
The Tax Court held that they may be deducted because of the undenied paid to foreign government from the taxpayer's gross income is
fact that the respondent spouses did not "signify" in their income tax given only as an alternative or substitute to his right to claim a
return a desire to avail themselves of the benefits of paragraph 3 (B) of tax credit for such foreign income taxes under section 30 (c) (3)
the subsection, which reads: and (4); so that unless the alien resident has a right to claim
such tax credit if he so chooses, he is precluded from deducting
Par. (c) (3) Credits against tax for taxes of foreign countries. the foreign income taxes from his gross income. For it is
If the taxpayer signifies in his return his desire to have the obvious that in prescribing that such deduction shall be allowed
benefits of this paragraph, the tax imposed by this Title shall be in the case of a taxpayer who does not signify in his return his
credited with desire to have to any extent the benefits of paragraph (3)
(relating to credits for taxes paid to foreign countries), the
statute assumes that the taxpayer in question also may
(A) ...;
signify his desire to claim a tax credit and waive the deduction;
otherwise, the foreign taxes would always be deductible, and
(B) Alien resident of the Philippines. In the case of their mention in the list of non-deductible items in Section
an alien resident of the Philippines, the amount of any 30(c) might as well have been omitted, or at least expressly
such taxes paid or accrued during the taxable year to limited to taxes on income from sources outside the Philippine
any foreign country, if the foreign country of which Islands.
such alien resident is a citizen or subject, in imposing

76
TAXATION II ATTY. ACOSTA -
CAJUSTIN
Had the law intended that foreign income taxes could be (D) Taxes assessed against local benefits of a
deducted from gross income in any event, regardless of the kind tending to increase the value of the
taxpayer's right to claim a tax credit, it is the latter right that property assessed.
should be conditioned upon the taxpayer's waiving the
deduction; in which Case the right to reduction under would have merely provided:
subsection (c-1-B) would have been made absolute or
unconditional (by omitting foreign taxes from the enumeration
of non-deductions), while the right to a tax credit under SEC. 30. Decision from grow income. In computing net income there
subsection (c-3) would have been expressly conditioned upon shall be allowed as deductions:
the taxpayer's not claiming any deduction under subsection (c-
1). In other words, if the law had been intended to operate as (a) ...
contended by the respondent taxpayers and by the Court of Tax
Appeals section 30 (subsection (c-1) instead of providing as at
(b) ...
present:

(c) Taxes paid or accrued within the taxable year, EXCEPT


SEC. 30. Deduction from gross income. In computing net income
there shall be allowed as deductions
(A) The income tax provided for in this Title;
(a) ...
(B) Omitted or else worded as follows:
(b) ...
Income, war profits and excess profits taxes imposed by
authority of any foreign country on income earned within the
(c) Taxes:
Philippines if the taxpayer does not claim the benefits under
paragraph 3 of this subsection;
(1) In general. Taxes paid or accrued within the
taxable year, except
(C) Estate, inheritance or gift taxes;

(A) The income tax provided for under this


(D) Taxes assessed against local benefits of a kind tending to
Title;
increase the value of the property assessed.

(B) Income, war-profits, and excess profits


while subsection (c-3) would have been made conditional in the following or
taxes imposed by the authority of any foreign
equivalent terms:
country; but this deduction shall be allowed
in the case of a taxpayer who does not signify
in his return his desire to have to any extent (3) Credits against tax for taxes of foreign countries. If the
the benefits of paragraph (3) of this taxpayer has not deducted such taxes from his gross income
subsection (relating to credit for taxes of but signifies in his return his desire to have the benefits of this
foreign countries); paragraph, the tax imposed by Title shall be credited with ... (etc.).

(C) Estate, inheritance and gift taxes; and Petitioners admit in their brief that the purpose of the law is to prevent the
taxpayer from claiming twice the benefits of his payment of foreign taxes, by
deduction from gross income (subs. c-1) and by tax credit (subs. c-3). This danger

77
TAXATION II ATTY. ACOSTA -
CAJUSTIN
of double credit certainly can not exist if the taxpayer can not claim benefit under IN VIEW OF THE FOREGOING, the decisions of the Court of Tax Appeals are
either of these headings at his option, so that he must be entitled to a tax credit reversed, and, the disallowance of the refunds claimed by the respondents
(respondent taxpayers admittedly are not so entitled because all their income is Lednicky is affirmed, with costs against said respondents-appellees.
derived from Philippine sources), or the option to deduct from gross income
disappears altogether. G.R. No. L-19537 May 20, 1965

Much stress is laid on the thesis that if the respondent taxpayers are not allowed The late LINO GUTIERREZ substituted by ANDREA C. VDA. DE
to deduct the income taxes they are required to pay to the government of the GUTIERREZ, ANTONIO D. GUTIERREZ, GUILLERMO D.
United States in their return for Philippine income tax, they would be subjected to GUTIERREZ, SANTIAGO D. GUTIERREZ and TOMAS D.
double taxation. What respondents fail to observe is that double taxation becomes GUTIERREZ,petitioners,
obnoxious only where the taxpayer is taxed twice for the benefit of the same vs.
governmental entity (cf. Manila vs. Interisland Gas Service, 52 Off. Gaz. 6579; COLLECTOR (now COMMISSIONER) OF INTERNAL
Manuf. Life Ins. Co. vs. Meer, 89 Phil. 357). In the present case, while the REVENUE, respondent.
taxpayers would have to pay two taxes on the same income, the Philippine
government only receives the proceeds of one tax. As between the Philippines,
where the income was earned and where the taxpayer is domiciled, and the SYLLABUS
United States, where that income was not earned and where the taxpayer did not
reside, it is indisputable that justice and equity demand that the tax on the income
should accrue to the benefit of the Philippines. Any relief from the alleged double 1. TAXATION; INCOME TAX; DEDUCTIONS; TRANSPORTATION TO
taxation should come from the United States, and not from the Philippines, since ATTEND FUNERAL AND IRON DOOR OF RESIDENCE NOT DEDUCTIBLE.
the former's right to burden the taxpayer is solely predicated on his citizenship, The transportation expenses which a businessman incurred to attend the
without contributing to the production of the wealth that is being taxed. funeral of his friends and the cost of admission tickets to operas were expenses
relative to his personal and social activities rather than to his business of
Aside from not conforming to the fundamental doctrine of income taxation that leasing real estate. Likewise, the procurement and installation of an iron door
the right of a government to tax income emanates from its partnership in the to his residence is purely a personal expense. Personal, living, or family
production of income, by providing the protection, resources, incentive, and expenses are not deductible.
proper climate for such production, the interpretation given by the respondents to
the revenue law provision in question operates, in its application, to place a 2. ID.; ID.; ID.; COMMISSIONS AND EXPENSES IN ASSOCIATION TO
resident alien with only domestic sources of income in an equal, if not in a better, ENHANCE BUSINESS DEDUCTIBLE. The cost of furniture given by the
position than one who has both domestic and foreign sources of income, a taxpayer as commission in furtherance of a business transaction, the expenses
situation which is manifestly unfair and short of logic. incurred in attending a businessmen luncheon meeting and cruise of a
business Association, when shown to have been made in the pursuit of his
business, are deductible.
Finally, to allow an alien resident to deduct from his gross income whatever taxes
he pays to his own government amounts to conferring on the latter the power to 3. ID.; ID.; ID.; MAINTENANCE OF CAR USED BOTH FOR PERSONAL AND
reduce the tax income of the Philippine government simply by increasing the tax BUSINESS NEEDS PARTLY DEDUCTIBLE. Where there is no clear
rates on the alien resident. Everytime the rate of taxation imposed upon an alien showing that the taxpayers car was devoted more for the taxpayers business
resident is increased by his own government, his deduction from Philippine taxes than for his personal needs, but according to the evidence, the taxpayers car
would correspondingly increase, and the proceeds for the Philippines diminished, was utilized both for personal and business needs, it is held that it is
thereby subordinating our own taxes to those levied by a foreign government. reasonable to allow as deduction one-half of the drivers salary, car expenses
Such a result is incompatible with the status of the Philippines as an independent and depreciation.
and sovereign state.
4. ID.; ID.; ID.; ORDINARY REPAIRS FOR MAINTENANCE OF RENTAL
APARTMENTS DEDUCTIBLE. Where electrical supplies, paint, lumber,
plumbing, cement, tiles, gravel, masonry and labor used to repair the

78
TAXATION II ATTY. ACOSTA -
CAJUSTIN
taxpayers rental apartments, did not increase the value of such apartments, or 11. ID.; ID.; ID.; REAL PROPERTIES USED IN TAXPAYERS BUSINESS ARE
prolong their life but merely kept the apartments in an ordinary operating ORDINARY ASSETS. Real property used in the trade or business of the
condition, its is held that the expenses incurred therefor are deductible as taxpayer is ordinary asset, and any gain or loss from the sale or exchange
necessary expenditure for the maintenance of the taxpayers business. thereof should be treated as ordinary, not capital gain or loss.

5. ID.; ID.; ID.; LITIGATION EXPENSES TO COLLECT APARTMENT 12. ID.; ID.; PRESCRIPTION PERIOD OF COLLECTION OF TAX STARTS
RENTALS DEDUCTIBLE. The litigation expenses defrayed by a taxpayer to FROM ASSESSMENT. The period of limitation to collect income tax is
collect apartment rentals and to eject delinquent tenants are ordinary and counted from the assessment of the tax.
necessary expenses in pursuing his business.
13. ID.; ID.; PRESCRIPTION PERIOD TO COLLECT TAX BY DISTRAINT
6. ID.; ID.; ID.; CAPITAL EXPENDITURES NOT DEDUCTIBLE. The AND LEVY STARTS FROM FILING OF RETURNS. Where the taxpayer has
following are not deductible business expenses but should be integrated into filed an income tax return, the three-year prescriptive period to collect the tax
the cost of the capital assets for which they were incurred and depreciated by distraint and levy should be counted from the time he filed such return.
yearly: (1) Expenses in watching over laborers in construction work. Watching
over laborers is an activity more akin to the construction work than to running
the taxpayers business. Hence, the expenses incurred therefor should form
part of the construction cost. (2) Real estate tax which remained unpaid by the
former owner of the taxpayers rental property but which the latter paid, is an Lino Gutierrez was primarily engaged in the business of leasing real property for
additional cost to acquire such property and ought therefore to be treated as which he paid estate broker's privilege tax. He filed his income tax returns for the
part of the propertys purchase price. (3) The iron bars, venetian blind and years 1951, 1952, 1953 and 1954 on the following dates:
water pump augmented the value of the apartments where they were installed.
Their cost is not a maintenance charge, hence, not deductible. (4) Expenses for
the relocation, survey and registration of property tend to strengthen title over Year Date Filed
the property, hence, they should be considered as addition to the cost of such
property. (5) The set of "Comments on the Rules of Court" having a life span of 1951 March 1, 1952
more than one year should be depreciated ratably during its whole life span
instead of its total cost being deducted in one year. 1952 February 28, 1953

1953 February 22, 1954


7. ID.; ID.; ID.; DEPRECIATION OF RESIDENCE NOT DEDUCTIBLE. The
claim for depreciation of taxpayers residence is not deductible.
1954 February 23, 1955
8. ID.; ID.; ID.; FINES AND PENALTIES NOT DEDUCTIBLE. Fines and
penalties paid for late payment of taxes are not deductible. and paid the corresponding tax declared therein.
9. ID.; ID.; ID.; CONTRIBUTIONS TO PERSONS NOT SPECIFIED BY LAW
NOT DEDUCTIBLE. The aims to an indigent family and various individuals, On July 10, 1956 the Commissioner (formerly Collector) of Internal Revenue
and a donation consisting of officers jewels and aprons to Biak-na-Bato Lodge assessed against Gutierrez the following defiency income tax:
No. 7, are not deductible from gross income inasmuch as their recipients have
not been shown to be among those specified by law.
1951 . . . . . . . . . . . . . . P 1,400.00
10. ID.; ID.; BALLANTYNE SCALE OF VALUES AS BASIS OF COST OF
PROPERTY SOLD. When property bought during the Japanese occupation 1952 . . . . . . . . . . . . . . 672.00
is sold after the war, the Ballantyne scale of value is used to compute the tax
1953 . . . . . . . . . . . . . . 5,161.00
based upon which the tax is to be imposed.

79
TAXATION II ATTY. ACOSTA -
CAJUSTIN

1954 . . . . . . . . . . . . . . 4,608.00
Fines and penalties for late payment of taxes P64.48

1952
Total . . . . . . . . . . . . . . P 11,841.00
========== Personal expenses:chanrob1es virtual 1aw library
The above defiency tax came about by the disallowance of deductions from gross
income representing depreciation, expenses Gutierrez allegely incurred in Car expenses, salary of driver and car
carrying on his business, and the addition to gross income of receipts which he
did not report in his income tax returns. The disallowed business expenses which depreciation P1,454.87
were considered by the Commissioner either as personal or capital expenditures
consisted of: Contribution to Lydia Yamson and G. Trinidad 52.00
1951
Officers jewels and apron donated to Biak-na-
Personal Expenses:chanrob1es virtual 1aw library
Bato Lodge No. 7, Free Masons 280.00
Transportation expenses to attend funeral of
Luncheon of Homeowners Association 5.50
various persons P96.50
Ticket to opera "Aida" 15.00
Repair of car and salary of driver 59.80
1953
Expenses in attending National Convention of
Personal expenses:chanrob1es virtual 1aw library
Filipino Businessmen in Baguio 121.35
Car expenses, salary of driver, car depreciation P1,409.24
Alms to indigent family 15.00
Cruise to Corregidor with Homeowners
Capital expenditures:chanrob1es virtual 1aw library
Association 43.00
Electrical fixtures and supplies P100.00
Contribution to alms to various individuals 70.00
Transportation and other expenses to watch
Tickets to operas 28.00
laborers in construction work 516.00
Capital expenditures:chanrob1es virtual 1aw library
Realty tax not paid by former owner of
Cost of one set of Comments on the Rules of
property acquired by Gutierrez 350.00
Court by Moran P145.00
Litigation expenses to collect rental and eject
1954
lessee 702.65
Personal expenses:chanrob1es virtual 1aw library
Other disallowed deductions:chanrob1es virtual 1aw library

80
TAXATION II ATTY. ACOSTA -
CAJUSTIN
Car expenses, salary of driver and car
1953
depreciation P1,413.67
Overstatement of purchase price of real estate P8,476.92
Furniture given as commission in connection
Understatement of profits from sale of real estate 5,803.74
with business transaction 115.00
1954
Cost of iron door of Gutierrez residence 55.00
Understatement of profits from sale of real estate P5,444.24
Capital expenditures:chanrob1es virtual 1aw library

Painting of rental apartments P908.00 The overstatement of purchase price of real estate refers to the sale of two pieces
of property in 1953. In 1943 Gutierrez bought a parcel of land situated along
Carpentry and lumber for rental apartments 335.83 Padre Faura St. in Manila for P35,000.00. Sometime in 1953, he sold the same for
P30,400.00. Expenses of sale amounted to P631.80. In his return he claimed a
Tinsmith and plumbing for rental apartments 605.25 loss of P5,231.80 1 However, the Commissioner, concluding that said property
was bought in Japanese military notes, converted the buying price to its
Cement, tiles, gravel, sand and masonry for equivalent in Philippine Commonwealth peso by the use of the Ballantyne Scale of
Values. At P1.30 Japanese military notes per Commonwealth peso, the acquisition
rental apartments 199.48 cost of P35,000.00 Japanese military notes was valued at P26,823.08 Philippine
Commonwealth peso. Accordingly, the Commissioner determined a profit of
Iron bars, venetian blind, water pumps for P3,476.92 after restoring to Gutierrez gross income the P5,231.80 deduction for
loss.
rental apartments P1,340.00
In another transaction, Gutierrez sold a piece of land for P1,200.00. Alleging that
Relocation and registration of property used in said property was purchased for P1,200.00, he reported no profit thereunder.
However, after verifying the deed of acquisition, the Commissioner discovered the
tax-payers business 1,758.12 purchase price to be only P800.00. Consequently, he determined a profit of
P400.00 which was added to the gross income for 1953.
He also claimed the depreciation of his residence as follows:chanrob1es virtual
1aw library The understatement of profit from the sale of real estate may be explained thus:
In 1953 and 1954 Gutierrez sold four other properties upon which he made
1952 P992.22 substantial profits. 2 Convinced that said properties were capital assets, he
declared only 50% of the profits from their sale. However, treating said properties
1953 942.61 as ordinary assets (as property held and used by Gutierrez in his business), the
Commissioner taxed 100% of the profits from their disposition pursuant to
1954 895.48 Section 35 of the Tax Code.

The following are the items of income which Gutierrez did not declare in his Having unsuccessfully questioned the legality and correctness of the aforesaid
income tax returns:chanrob1es virtual 1aw library assessment, Gutierrez instituted on February 17, 1958 an appeal to the Court of
Tax Appeals. Later, on February 21, 1958, the Commissioner issued a warrant of
1951 distraint and levy on one of Gutierrez real properties but desisted from enforcing
the same when Gutierrez filed a bond to assure payment of his tax liability.
Income of wife (admitted by Gutierrez) P2,749.90

81
TAXATION II ATTY. ACOSTA -
CAJUSTIN
In a decision dated January 28, 1962, the Court of Tax Appeals upheld in toto the 4
assessment of the Commissioner of Internal Revenue. Hence, this appeal.
On the other hand, the cost of furniture given by the taxpayer as commission in
On October 18, 1962 Lino Gutierrez died and he was substituted by Andrea C. furtherance of a business transaction, the expenses incurred in attending the
Vda. de Gutierrez, Antonio D. Gutierrez, Santiago D. Gutierrez, Guillermo D. National Convention of Filipino Businessmen, luncheon meeting and cruise to
Gutierrez and Thomas D. Gutierrez, his heirs, as party petitioners. Corregidor of the Homeowners Association, were shown to have been made in
the pursuit of his business. Commissions given in consideration for bringing
The issues are: (1) Are the taxpayers aforementioned claims for deduction proper about a profitable transaction are part of the cost of the business transaction and
and allowable? (2) May the Ballantyne Scale of Values be applied in determining are deductible.
the acquisition cost in 1943 of a real property sold in 1953 for income tax
purposes? (3) Are real properties used in the trade or business of the taxpayer The record shows that Gutierrez was an officer of the Junior Chamber of
capital or ordinary assets? (4) Has the right of the Commissioner of Internal Commerce which sponsored the National Convention of Filipino Businessmen.
Revenue to collect the deficiency income tax for the years 1951 and 1952 He was also the president of the Homeowners Association, an organization
prescribed? (5) Has the right of the Commissioner of Internal Revenue to collect established by those engaged in the real estate trade. Having proved that his
by distraint and levy the deficiency income tax for 1954 prescribed? If not, may membership thereof and activities in connection therewith were solely to enhance
the taxpayers real property be distrained and levied upon, without first his business, the expenses incurred thereunder are deductible as ordinary and
exhausting his personal property? necessary business expenses.

We come first to the question whether or not the deductions claimed by Gutierrez With respect to the taxpayers claim for deduction for car expenses, salary of his
are allowable. Section 30(a) of the Tax Code allows business expenses to be driver and car depreciation, one-third of the same was disallowed by the
deducted from gross income. We quote:chanrob1es virtual 1aw library Commissioner on the ground that the taxpayer used his car and driver both for
personal and business purposes. There is no clear showing, however, that the car
SEC. 30. Deductions from gross income. In computing net income there shall was devoted more for the taxpayers business than for his personal needs. 5
be allowed as deductions According to the evidence, the taxpayers car was utilized both for personal and
business needs. We therefore find it reasonable to allow as deduction one-half of
"(a) Expenses:jgc:chanrobles.com.ph the drivers salary, car expenses and depreciation.

"(1) In general. All the ordinary and necessary expenses paid or incurred during The electrical supplies, paint, lumber, plumbing, cement, tiles, gravel, masonry
the taxable year in carrying on any trade or business, including a reasonable and labor used to repair the taxpayers rental apartments, did not increase the
allowance for salaries or other compensation for personal services actually value of such apartments, or prolong their life. They merely kept the apartments
rendered; traveling expenses while away from home in the pursuit of a trade or in an ordinary operating condition. Hence, the expenses incurred therefor are
business; and rentals or other payments required to be made as a condition to the deductible as necessary expenditures for the maintenance of the taxpayers
continued use or possession, for the purposes of the trade or business, or property business.
to which the taxpayer has not taken or is not taking title of in which he has no
equity."cralaw virtua1aw library Similarly, the litigation expenses defrayed by Gutierrez to collect apartment
rentals and to eject delinquent tenants are ordinary and necessary expenses in
To be deductible, therefore, an expense must be (1) ordinary and necessary; (2) pursuing his business. It is routinary and necessary for one in the leasing business
paid or incurred within the taxable year; and, (3) paid or incurred in carrying on a to collect rentals and to eject tenants who refuse to pay their accounts.
trade or business. 3
The following are not deductible business expenses but should be integrated into
The transportation expenses which petitioner incurred to attend the funeral of his the cost of the capital assets for which they were incurred and depreciated yearly:
friends and the cost of admission tickets to operas were expenses relative to his (1) Expenses in watching over laborers in construction work. Watching over
personal and social activities rather than to his business of leasing real estate. laborers is an activity more akin to the construction work than to running the
Likewise, the procurement and installation of an iron door to his residence is taxpayers business. Hence, the expenses incurred therefor should form part of
purely a personal expense. Personal, living, or family expenses are not deductible. the construction cost. (2) Real estate tax which remained unpaid by the former

82
TAXATION II ATTY. ACOSTA -
CAJUSTIN
owner of Gutierrez rental property but which the latter paid, is an additional cost into Philippine Commonwealth pesos by the use of the Ballantyne Scale of Values.
to acquire such property and ought therefore to be treated as part of the As a result, the Commissioner found Gutierrez to have profited, instead of lost in
propertys purchase price. (3) The iron bars, venetian blind and water pump the sale.
augmented the value of the apartments where they were installed. Their cost is
not a maintenance charge, 6 hence, not deductible. 7 (4) Expenses for the Firstly, Gutierrez maintains that the purchase price was paid for in
relocation, survey and registration of property tend to strengthen title over the Commonwealth pesos. On the other hand the Commissioner insists that
property, hence, they should be considered as addition to the cost of such inasmuch as the prevailing currency in the City of Manila in 1943 was the
property. (5) The set of "Comments on the Rules of Court" having a life span of Japanese military issue, the transaction could have been in said military notes.
more than one year should be depreciated ratably during its whole life span The evidence offered by Gutierrez, consisting of the testimony of his son to the
instead of its total cost being deducted in one year. effect that it was he who carried the bundle of Commonwealth pesos and
Japanese military notes when his father purchased the property, did not convince
Coming to the claim for depreciation of Gutierrez residence, we find the same not the Tax Court. No cogent reason to alter the court a quos finding of fact in this
deductible. A taxpayer may deduct from gross income a reasonable allowance for regard has been given. There is no definite showing that Gutierrez paid for the
deterioration of property arising out of its use or employment in business or property in Commonwealth pesos. Considering that in 1943 the medium of
trade. 8 Gutierrez residence was not used in his trade or business. exchange in Manila was the Japanese military notes, the use of which the
Japanese Military Government enforced with stringent measures, we are inclined
Gutierrez also claimed for deduction the fines and penalties which he paid for late to concur with the finding that the purchase price was in Japanese military notes.
payment of taxes. While Section 30 allows taxes to be deducted from gross We are specifically mindful of the fact that Gutierrez sold the property in 1953 for
income, it does not specifically allow fines and penalties to be so deducted. only P30,400.00 at a time when the price of real estate in the City of Manila was
Deductions from gross income are matters of legislative grace; what is not much greater than in 1943.
expressly granted by Congress is withheld. Moreover, when acts are condemned
by law and their commission is made punishable by fines or forfeitures, to allow It is further contended by Gutierrez that the money he used to pay for the
them to be deducted from the wrongdoers gross income, reduces, and so in part purchase of the property in question came from the proceeds of merchandise
defeats, the prescribed punishment. 9 acquired prior to World War II but which he sold after Manila was occupied by
the Japanese military forces, hence the purchase price should be deemed to have
As regards the alms to an indigent family and various individuals, contributions been made in Commonwealth pesos inasmuch as the aforesaid merchandise was
to Lydia Yamson and G. Trinidad and a donation consisting of officers jewels and purchased in Commonwealth pesos. This contention, if true, strengthens our
aprons to Biak-na-Bato Lodge No. 7, the same are not deductible from gross conclusion that the real estate in question was bought in Japanese military notes.
income inasmuch as their recipients have not been shown to be among those For, at the time Gutierrez sold his merchandise, the prevailing currency in the
specified by law. Contributions are deductible when given to the Government of City of Manila was the Japanese military money. Consequently, the proceeds
the Philippines, or any of its political subdivisions for exclusively public purposes, therefrom, which was used to buy the real estate in question, were Japanese
to domestic corporations or associations organized and operated exclusively for military notes.
religious, charitable, scientific, athletic, cultural or educational purposes, or for
the rehabilitation of veterans, or to societies for the prevention of cruelty to Gutierrez assails the use of the Ballantyne Scale of Values in converting the
children or animals, no part of the net income of which inures to the benefit of purchase price of the real estate in question from Japanese military notes to
any private stockholder or individual. 10 Philippine Commonwealth pesos on the ground that (1) the Ballantyne Scale of
Values was intended only for transactions entered into by parties voluntarily
We come to the question whether or not the Ballantyne Scale of Values can be during the Japanese occupation, wherein a portion of the contract was left
applied to tax cases. unperformed until liberation of the Philippines by the Americans; (2) that such
Scale of Values cannot be the basis of a tax, for it is not a law.
Sometime in 1943 Gutierrez bought a piece of real estate in Manila for a price of
P35,000.00. In 1953 he sold said property for P30,400.00, thereby incurring a In determining the gain or loss from the sale of property the purchase price and
loss which he claimed as deduction in his income tax return for 1953. The the selling price ought to be in the same currency. Since in this case the purchase
Commissioner of Internal Revenue, convinced that the purchase price of the price was in Japanese military notes and the selling price was in our present legal
property in 1943 was in Japanese military notes, converted said purchase price tender, the Japanese military notes should be converted to the present currency.

83
TAXATION II ATTY. ACOSTA -
CAJUSTIN
Since the only standard scale recognized by courts for the purpose is the twelve months;
Ballantyne Scale of Values, we find it compelling to use such table of values rather
than adopt an arbitrary scale. It may not be amiss to state in this connection that "(2) Fifty per centum if the capital asset has been held for more than twelve
the Ballantyne Scale of Values is not being used herein as the authority to impose months."cralaw virtua1aw library
the tax, but only as a medium of computing the tax base upon which the tax is to
be imposed. Section 34, before it was amended by Republic Act 82 in 1947, considered as
capital assets real property used in the trade or business of a taxpayer. However,
It is furthermore proffered by the taxpayer that in determining gain or loss, the with the passage of Republic Act 82, Congress classified "real property used in the
real value of the Commonwealth peso at the time the property was purchased and trade or business of the taxpayer" as ordinary asset. The explanatory note to
the value of the Republic peso at the time the same property was sold should be Republic Act 82 says: ". . . the words `or real property used in the trade or
considered. The Commonwealth peso and the Republic peso are the same business of the taxpayer have been included among the non-capital assets. This
currency, with the same intrinsic value, sanctioned by the same authorities. Both has the effect of withdrawing the gain or loss from the sale or exchange of real
are legal tender and accepted at face value regardless of fluctuation in their property used in the trade or business of the taxpayer from the operation of the
buying power. The 1941 Commonwealth peso when used to buy in 1953 or in 1965 capital gains and losses provisions. As such real property is used in the trade or
is accorded the same value: one peso. business of the taxpayer, it is logical that the gain or loss from the sale or
exchange thereof should be treated as ordinary income or loss." 11 Accordingly,
In his income tax returns for 1953 and 1954, Gutierrez reported only 50% of the real estate, admittedly used by Gutierrez in his business, which he sold in 1953
profits he realized from the sale of real properties during the years 1953 and 1954 and 1954 should be treated as ordinary assets and the gain from the sale thereof,
on the ground that said properties were capital assets. Profits from the sale of as ordinary gain, hence, fully taxable. 12
capital assets are taxable to the extent of 50% thereof pursuant to Section 34 of
the Tax Code. With regard to the issue of the prescription of the Commissioners right to collect
deficiency tax for 1951 and 1952, Gutierrez claims that the counting of the 5-year
Section 34 provides:jgc:chanrobles.com.ph period to collect income tax should start from the time the income tax returns
were filed. He, therefore, urges us to declare the Commissioners right to collect
"SEC. 34. Capital gains and losses.(a) Definitions.- As used in this title the deficiency tax for 1951 and 1952 to have prescribed, the income tax returns for
1951 and 1952 having been filed in March 1952 and on February 28, 1953,
"(1) Capital assets. The term `capital assets means property held by the respectively, and the action to collect the tax having been instituted on March 5,
taxpayer (whether or not connected with his trade or business), but does not 1958 when the Commissioner filed his answer to the petitioner for review in
include stock in trade of the taxpayer or other property of a kind which would C.T.A. Case No. 504. On the other hand, the Commissioner argues that the
properly be included in the inventory of the taxpayer if on hand at the close of the running of the prescriptive period to collect commences from the time of
taxable year, or property held by the taxpayer primarily for sale to customers in assessment. Inasmuch as the tax for 1951 and 1952 were assessed only on July 10,
the ordinary course of his trade or business, or property, used in the trade or 1956, less than five years lapsed when he filed his answer on March 5, 1958.
business, of a character which is subject to the allowance for depreciation
provided in subsection (f) of section thirty; or real property used in the trade or The period of limitation to collect income tax is counted from the assessment of
business of the taxpayer. the tax as provided for in paragraph (c) of Section 332 quoted
below:jgc:chanrobles.com.ph
x x x
"SEC. 332(c). Where the assessment of any internal-revenue tax has been made
with the period of limitation above prescribed such tax may be collected by
(b) Percentage taken into account. In the case of a taxpayer, other than a distraint or levy or by a proceeding in court, but only if begin (1) within five years
corporation, only the following percentages of the gain or loss recognized upon after the assessment of the tax, or (2) prior to the expiration of any period for
the sale or exchange of capital asset shall be taken into account in computing net collection agreed upon in writing by the Collector of Internal Revenue and the
capital gain, net capital loss, and net income:jgc:chanrobles.com.ph taxpayer before the expiration of such five-year period. The period so agreed upon
may be extended by subsequent agreements in writing made before the expiration
"(1) One hundred per centum if the capital asset has been held for not more than of the period previously agreed upon."cralaw virtua1aw library

84
TAXATION II ATTY. ACOSTA -
CAJUSTIN
days, the warrant issued is null and void.
Inasmuch as the assessment for deficiency income tax was made on July 10, 1956
which is 7 months and 25 days prior to the action for collection, the right of the The above finding has made academic the question of whether or not the warrant
Commissioner to collect such tax has not prescribed. of distraint and levy can be enforced against the taxpayers real property without
first exhausting his personal properties.
The next issue relates to the prescription of the right of the Commissioner of
Internal Revenue to collect the deficiency tax for 1954 by distraint and levy. In resum, the tax liability of Lino Gutierrez 1951, 1952, 1953 and 1954 may be
computed as follows:
The pertinent provision of the Tax Code states:jgc:c chanrob1es virtual 1aw library
hanrobles.com.ph
1951
"SEC. 51(d) Refusal or neglect to make returns; fraudulent returns, etc. In cases
of refusal or neglected to make a return and in cases of erroneous, false, or Net income per investigation P29,471.81
fraudulent returns, the Collector of Internal Revenue shall, upon the discovery
thereof at any time within three years after said return is due, or has been made, Add: Disallowed deductions for salary of driver
make a return upon information obtained as provided for in this code or by
existing law, or require the necessary corrections to be made, and the assessment and car expenses 29.90
made by the Collector of Internal Revenue thereon shall be paid by such person or
corporation immediately upon notification of the amount of such P29,501.71
assessment."cralaw virtua1aw library
Less: Allowable deductions:chanrob1es virtual 1aw library
On February 23, 1955 Gutierrez filed his income tax return for 1954 and on
February 24, 1958 the Commissioner of Internal Revenue issued a warrant of Expenses in attending National Convention of
distraint and levy to collect the tax due thereunder. Gutierrez contends that the
Commissioners right to issue said warrant is barred, for the same was issued Filipino businessmen P121.35
more than 3 years from the time he filed his income tax return. On the other
hand, the Commissioner of Internal Revenue maintains that his right did not Repair of rental apartments 802.65 924.00
lapse inasmuch as from the last day prescribed by law for the filing of the 1954
return to the date when he issued the warrant of distraint and levy less than 3 Net income P30,425.71
years passed. The question now is: should the counting of the prescriptive period
commence from the actual filing of the return or from the last day prescribed by Less: Personal exemption 3,600.00
law for the filing thereof?
Amount subject to tax P26,825.71
We observe that Section 51 (d) speaks of erroneous, false or fraudulent returns
and refusal or neglect of the taxpayer to file a return. It also provides for two dates Tax due thereon P 5,668.00
from which to count the three- year prescriptive period, namely, the date when
the return is due and the date the return has been made. We are inclined to Less tax already paid 3,981.00
conclude that the date when the return is due refers to cases where the taxpayer
refused or neglected to file a return, and the date when the return has been made Deficiency income tax due P 1,687.00
refers to instances where the taxpayer filed erroneous, false or fraudulent returns.
Since Gutierrez filed an income tax return, the three-year prescriptive period 1952
should be counted from the time he filed such return. From February 23, 1955
when the income tax return for 1954 was filed, to February 24, 1958, when the Net income per investigation P21,632.22
warrant of distraint and levy was issued, 3 years and 2 days elapsed. The right of
the Commissioner to issue said warrant of distraint and levy having lapsed by two Add: Disallowed deductions:chanrob1es virtual 1aw library

85
TAXATION II ATTY. ACOSTA -
CAJUSTIN
Net income P69,828.40
Salary of driver P260.67
Less: Personal exemption 3,600.00
Car expenses 401.51
Amount subject to tax P66,228.40
Car depreciation 65.00 727.18
Tax due thereon P15,179.00
P22,359.40
Less tax already paid 9,805.00
Less: Allowable deduction:chanrob1es virtual 1aw library
Deficiency income tax due P 5,374.00
Luncheon, Homeowners Association 5.50
1954
Net income P22,364.90
Net income per investigation P43,881.92
Less: Personal exemption 3,600.00
Add: Disallowed deductions:chanrob1es virtual 1aw library
Amount subject to tax P18,764.90
Salary of driver P140.00
Tax due thereon P 3,324.00
Car expenses 414.18
Less tax already paid 2,476.00
Car depreciation 72.65 626.83
Deficiency income tax due P 848.00
P44,508.75
1953
Less: Allowable deductions:chanrob1es virtual 1aw library
Net income per investigation P69,180.91
Furniture given in connection with business
Add: Disallowed deductions:chanrob1es virtual 1aw library
transaction P115.00
Salary of driver P140.00
Repairs of rental apartments 2,048.56
Car expenses 406.00
P 2,163.56
Car depreciation 58.50 604.50
Net income P42,345.19
P69,785.40
Less: Personal exemption 3,000.00
Less: Allowable deduction:chanrob1es virtual 1aw library
Amount subject to tax P39,345.19
Cruise to Corregidor with Homeowners
Tax due thereon P 9,984.00
Association 42.00
Less tax already paid 5,964.00

86
TAXATION II ATTY. ACOSTA -
CAJUSTIN

Deficiency income tax due P 4,020.00

SUMMARY

1951 P 1,687.00

1952 848.00

1953 5,374.00

1954 4,020.00

Total P11,929.00

WHEREFORE, the decision appealed from is modified and Lino Gutierrez and/or
his heirs, namely, Andrea C. Vda. de Gutierrez, Antonio D. Gutierrez, Santiago D.
Gutierrez, Guillermo D. Gutierrez and Tomas D. Gutierrez, are ordered to pay the
sums of P1,687.00, P848.00, P5,374.00, and P4,020.00, as deficiency income tax
for the years 1951, 1952, 1953, 1954, respectively, or a total of P11,929.00, plus the
statutory penalties in case of delinquency. No costs. So ordered.

87
TAXATION II ATTY. ACOSTA -
CAJUSTIN
However, Petitioner, who had the burden of proof, failed to adduce evidence
that there was a charge-off in connection with the P44,490.00 or
P30,600.00 which it paid to Galang Machinery.

3. ID.; ID.; INTEREST; PRACTICE AND PROCEDURE; CASE AT BAR. In


connection with the claimed interest deduction of P10,000.00, the Solicitor
General correctly points out that this question was never raised before the Tax
Court. The alleged interest deduction not having been properly litigated as an
issue before the Tax Court, it is now too late to raise and assert it before this
Court.
TOPIC: DEDUCTIONS LOSSES
Petitioner Plaridel Surety & Insurance Co., is a domestic corporation engaged in
the bonding business. On November 9, 1950, Petitioner, as surety, and
[G.R. No. L-21520. December 11, 1967.] Constancio San Jose, as principal, solidarily executed a performance bond in the
penal sum of P30,600.00 in favor of the P.L. Galang Machinery Co., Inc., to
PLARIDEL SURETY & INSURANCE COMPANY, Petitioner, v. secure the performance of San Joses contractual obligation to produce and
COMMISSIONER OF INTERNAL REVENUE, Respondent. supply logs to the latter.

Gil R. Carlos & Associates for Petitioner. To afford itself adequate protection against loss or damage on the performance
bond, petitioner required San Jose and one Ramon Cuervo to execute an
The Solicitor General for Respondent. indemnity agreement obligating themselves, solidarily, to indemnify petitioner for
whatever liability it may incur by reason of said performance bond. Accordingly,
San Jose constituted a chattel mortgage on logging machineries and other
movables in petitioners favor 1 while Ramon Cuervo executed a real estate
SYLLABUS mortgage. 2

1. INCOME TAX; DEDUCTIONS; LOSS. Loss is deductible only in the San Jose later failed to deliver the logs to Galang Machinery 3 and the latter sued
taxable year it actually happens or is sustained. However, if the loss is on the performance bond. On October 1, 1952, the Court of First Instance
compensable otherwise than by insurance, deduction for the loss suffered is adjudged San Jose and petitioner liable; it also directed San Jose and Cuervo to
postponed to a subsequent year, which to be precise, is that year in which it reimburse petitioner for whatever amount it would pay Galang Machinery. The
appears that no compensation at all can be had, or that there is a remaining or Court of Appeals, on June 17, 1955 affirmed the judgment of the lower court. The
net loss, i.e., no full compensation. The rule is that loss deduction will be same judgment was likewise affirmed by this Court 4 on January 11, 1957 except
denied if there is a measurable right to compensation for the loss, with for a slight modification apropos the award of attorneys fees.
ultimate collection reasonably clear. So where there is reasonable ground for
reimbursement, the taxpayer must seek his redress and may not secure a loss On February 19 and March 20, 1957, petitioner effected payment in favor of
deduction until he establishes that no recovery may be had. In other words, the Galang Machinery in the total sum of P44,490.00 pursuant to the final decision.
taxpayer must first exhaust his remedies to recover or reduce his loss. Where
the evidence on record reveals that petitioner had not exhausted its remedies, In its income tax return for the year 1957, petitioner claimed the said amount of
then it was too premature for petitioner to claim a loss deduction. P44,490.00 as deductible loss from its gross income and, accordingly, paid the
amount of P136.00 as its income tax for 1957.
2. ID.; ID.; ID.; PROOF OF CHARGE-OFF ESSENTIAL; CASE AT BAR.
Assuming that there was no reasonable expectation of recovery of the loss The Commissioner of Internal Revenue disallowed the claimed deduction of
sustained by petitioner, still no loss deduction can be had. Sec. 30(d)(2) of the P44,490.00 and assessed against petitioner the sum of P8,898.00, plus interest,
Tax Code requires a charge off as one of the conditions for loss deduction, as deficiency income tax for the year 1957. Petitioner filed its protest which was

88
TAXATION II ATTY. ACOSTA -
CAJUSTIN
denied. Whereupon, appeal was taken to the Tax Court, petitioner insisting that (petitioner) must exhaust his remedies first to recover or reduce his loss. It is on
the P44,490.00 which it paid to Galang Machinery was a deductible loss. record that petitioner had not exhausted its remedies, especially against Ramon
Cuervo who was solidarily liable with San Jose for reimbursement to it. Upon
The Tax Court dismissed the appeal, ruling that petitioner was duly compensated being prodded by the Tax Court to go after Cuervo, Hermogenes Dimaguiba,
for otherwise than by insurance thru the mortgages in its favor executed by San president of petitioner corporation, said that they would 10 but no evidence was
Jose and Cuervo and it had not yet exhausted all its available remedies, submitted that anything was really done on the matter. Moreover, petitioners
especially as against Cuervo, to minimize its loss. When its motion to reconsider evidence on remote possibility of recovery is fatally wanting. Its right to
was denied, petitioner elevated the present appeal. reimbursement is not only secured by the mortgages executed by San Jose and
Cuervo but also by a final and executory judgment in the civil case itself. Thus,
Of the sum of P44,490.00, the amount of P30,600.00 which is the principal other properties of San Jose and Cuervo were subject to levy and execution. But
sum stipulated in the performance bond is being claimed as loss deduction no writ of execution, satisfied or unsatisfied, was ever submitted. Neither has it
under Sec. 30(d)(2) of the Tax Code and P10,000.00 which is the interest that been established that Cuervo was insolvent. The only evidence on record on the
had accrued on the principal sum is now being claimed as interest deduction point is Dimaguibas testimony that he does not really know if Cuervo has other
under Sec. 30(b)(1). properties. 11 This is not substantial proof of insolvency. Thus, it was too
premature for petitioner to claim a loss deduction.
Loss is deductible only in the taxable year it actually happens or is sustained.
However, if it is compensable by insurance or otherwise, deduction for the loss But assuming that there was no reasonable expectation of recovery, still no loss
suffered is postponed to a subsequent year, which, to be precise, is that year in deduction can be had. Sec. 30(d)(2) of the Tax Code requires a charge-off as one
which it appears that no compensation at all can be had, or that there is a of the conditions for loss deduction:jgc:chanrobles.com.ph
remaining or net loss, i.e., no full compensation. 5
"In the case of a corporation, all losses actually sustained and charged-off within
There is no question that the year in which the petitioner Insurance Co. effected the taxable year and not compensated for by insurance or otherwise." (Stress
payment to Galang Machinery pursuant to a final decision occurred in 1957. supplied)
However, under the same court decision, San Jose and Cuervo were obligated to
reimburse petitioner for whatever payments it would make to Galang Machinery. Mertens 12 states only four (4) requisites because the United States Internal
Clearly, petitioners loss is compensable otherwise (than by insurance). It should Revenue Code of 1939 13 has no charge-off requirement. Sec. 23(f) thereof
follow, then, that the loss deduction can not be claimed in 1957. provides merely:jgc:chanrobles.com.ph

Now, petitioners submission is that its case is an exception. Citing Cu Unjieng "In the case of a corporation, losses sustained during the taxable year and not
Sons, Inc. v. Board of Tax Appeals, 6 and American cases also, petitioner argues compensated for by insurance or otherwise."cralaw virtua1aw library
that even if there is a right to compensation by insurance or otherwise, the
deduction can be taken in the year of actual loss where the possibility of recovery Petitioner, who had the burden of proof 14 failed to adduce evidence that there
is remote. The pronouncement, however, to this effect in the Cu Unjieng case is was a charge-off in connection with the P44,490.00 or P30,600.00 which it
not as authoritative as petitioner would have it since it was there found that the paid to Galang Machinery.
taxpayer had no legal right to compensation either by insurance or otherwise. 7
And the American cases cited 8 are not in point. None of them involved a taxpayer In connection with the claimed interest deduction of P10,000.00, the Solicitor
who had, as in the present case, obtained a final judgment against third persons General correctly points out that this question was never raised before the Tax
for reimbursement of payments made. In those cases, there was either no legally Court. Petitioner, thru counsel, had admitted before said court 15 and in the
enforceable right at all or such claimed right was still to be, or being, litigated. memorandum it filed 16 that the only issue in the case was whether the entire
P44,490.00 paid by it was or was not a deductible loss under Sec. 30(d)(2) of the
On the other hand, the rule is that loss deduction will be denied if there is a Tax Code. Even in petitioners return, the P44,490.00 was claimed wholly as
measurable right to compensation for the loss, with ultimate collection reasonably losses on its bonds. 17 The alleged interest deduction not having been properly
clear. So where there is reasonable ground for reimbursement, the taxpayer must litigated as an issue before the Tax Court, it is now too late to raise and assert it
seek his redress and may not secure a loss deduction until he establishes that no before this Court.
recovery may be had. 9 In other words, as the Tax Court put it, the taxpayer

89
TAXATION II ATTY. ACOSTA -
CAJUSTIN
WHEREFORE, the appealed decision is, as it is hereby, affirmed. Costs against Rafael Dinglasan for respondent Fernandez Hermanos, Inc.
petitioner Plaridel Surety & Insurance Co. So ordered.
L-24978:chanrob1es virtual 1aw library
[G.R. No. L-21551. September 30, 1969.]
Rafael Dinglasan for Petitioner.
FERNANDEZ HERMANOS, INC., Petitioner, v. COMMISSIONER OF
INTERNAL REVENUE and COURT OF TAX APPEALS., Respondents. Solicitor General Antonio P. Barredo, Assistant Solicitor General
Antonio G. Ibarra and Special Attorney Virgilio G. Saldajeno
[G.R. No. L-21557. September 30, 1969.] for Respondent.

COMMISSIONER OF INTERNAL REVENUE, Petitioner, v.


FERNANDEZ HERMANOS, INC., and COURT OF TAX SYLLABUS
APPEALS, Respondents.

[G.R. No. L-24972. September 30, 1969.]


1. TAXATION; NATIONAL INTERNAL REVENUE CODE; INCOME TAX
COMMISSIONER OF INTERNAL REVENUE, Petitioner, v. ALLOWABLE DEDUCTIONS; WORTHLESS SECURITIES; WRITING-OFF
FERNANDEZ HERMANOS, INC., and the COURT OF TAX PROPER IN INSTANT CASE. We find no reason to disturb the Tax Courts
APPEALS, Respondents. allowance of the taxpayers writing-off as worthless securities in its 1950 return
the sum of P8,050.00 representing the cost of shares of stock of Mati Lumber
[G.R. No. L-24978. September 30, 1969.] Co. acquired by the taxpayer on January 1, 1948. There was adequate basis for
the writing off of the stock as worthless securities. As found by the Tax Court,
FERNANDEZ HERMANOS, INC., Petitioner, v. THE COMMISSIONER the Mati Lumber Co. ceased operation in 1949 when its manager and owner
OF INTERNAL REVENUE, and HON. ROMAN A. UMALI, COURT OF left for Spain where he subsequently died. When the company ceased to
TAX APPEALS, Respondents. operate, it had no assets. Assuming that the Company would later somehow
realize some proceeds from its sawmill and equipment, which were still
L-21551 existing as claimed by the Commissioner, and that such proceeds would later
be distributed to its stockholders such as the taxpayer, the amount so received
Rafael Dinglasan for Petitioner. by the taxpayer would then properly be reportable as income of the taxpayer
on the year it is received.
Solicitor General Arturo A. Alafriz, Solicitor Alejandro B. Afurong and
Special Attorney Virgilio G. Saldajeno for Respondent. 2. ID.; ID.; ID.; ID.; BAD DEBT, WHEN CONSIDERED. The Tax Courts
disallowance of the writing-off in 1951 as a loss or bad debt the sum of
L-21557:chanrob1es virtual 1aw library P353,134.25, which it had advanced or loaned to Palawan Manganese Mines,
Inc., was proper. The Solicitor General has rightly pointed out that the
Solicitor General for Petitioner. taxpayer has taken an "ambiguous position" and "has not definitely taken a
stand on whether the amount involved is claimed as losses or as bad debts but
Rafael Dinglasan for respondent Fernandez Hermanos, Inc. insists that it is either a loss or a bad debt. "We sustain the governments
position that the advances made by the taxpayer to its 100% subsidiary,
L-24972:chanrob1es virtual 1aw library Palawan Manganese Mines, Inc. amounting to P587,308.07 as of 1951 were
investments and not loans. The evidence on record shows that the board of
Solicitor General Antonio P. Barredo, Assistant Solicitor General directors of the two companies since August, 1945 were identical and that the
Felicisimo R. Rosete and Special Attorney Virgilio G. Saldajeno only capital of Palawan Manganese Mines, Inc. is the amount of P100,000.00
for Petitioner. entered in the taxpayers balance sheet as its investment in its subsidiary
company. This fact explains the liberality with which the taxpayer made such

90
TAXATION II ATTY. ACOSTA -
CAJUSTIN
large advances to the subsidiary, despite the latters admittedly poor financial 6. ID.; ID.; ID.; ID.; DEPRECIATION OF BUILDINGS; ANNUAL RATE OF
condition. 10% DEPRECIATION, EXCESSIVE; DISALLOWANCE PROPER. During
the year 1950 to 1954, the taxpayer claimed a depreciation allowance for its
3. ID.; ID.; ID.; ID.; DEBT OR INVESTMENT OF CORPORATION NOT buildings at the annual rate of 10%. The Commissioner claimed that the
WORTHLESS IF CORPORATION IS STILL IN OPERATION. The Tax Court reasonable depreciation rate is only 3% annually. We sustain the Tax Courts
correctly held that the subsidiary company was still in operation in 1951 and finding that the taxpayer did not submit adequate proof of the correctness of
1952 and the taxpayer continued to give it advances in those years, and the taxpayers claim that the depreciable assets or buildings in question had a
therefore, the alleged debt or investment could not properly be considered useful life only of 10 years so as to justify its 10% depreciation per annum
worthless and deductible in 1951, as claimed by the taxpayer. Furthermore, claim, such finding being supported by the record. The taxpayers contention
neither under Section 30(d)(c) of our Tax Code providing for deduction by that it has many zero or one-peso assets, support the Commissioners position
corporations of losses actually sustained and charged off during the taxable that a 10% annual depreciation rate was excessive.
year nor under Section 30 (e)(1) thereof providing for deduction of bad debts
actually ascertained to be worthless and charged off within the taxable year, 7. ID.; ID.; ID.; INCREASE IN NET WORTH NOT TAXABLE IF NOT DUE TO
can there be a partial writing-off of a loss or bad debt, as was sought to be done TAXABLE RECEIPT. Where it is shown that the increase in the taxpayers
here by the taxpayer. For such losses or bad debts must be ascertained to be so net worth were not the result of the receipt by it of unreported or unexplained
and written-off during the taxable year, are therefore deductible in full or not taxable income but were merely the result of the correction of errors in its
at all, in the absence of any express provision in the Tax Code authorizing entries in its books relating to its debtedness to certain creditors, which had
partial deductions. been erroneously overstated or listed as outstanding when they had in fact duly
paid, these increase in the taxpayers net worth were not taxable increases in
4. ID.; ID.; ID.; ID.; LOSSES; DISALLOWANCE THEREOF PROPER IN net worth.
INSTANT CASE. The Court sustains the Tax Courts disallowance of the
sums of P8,989.76 and P27,732.66 spent by the taxpayer for the operation of 8. ID.; ID.; ID. ALLEGED UNREPORTED GAIN FROM SALE OF REAL
its Balamban coal mine in Cebu in 1950 and 1951, respectively, ad claimed as PROPERTY, NO BASIS. Where it was sufficiently proved from the taxpayers
losses in the taxpayers returns for said years. The Tax Court correctly held that books that after acquiring the property in 1926 for P11,852.74, the gain derived
the losses "are deductible in 1952, when the mines were abandoned, and not in from the sale of the said property for P60,000.00 was correctly reported by the
1950 and 1951, when they were still in operation." The taxpayers claim that taxpayer at P37,000.00.
these expenditures should be allowed as losses for the corresponding years that
they were incurred, because it made no sales of coal during said years, since 9. ID.; ID.; ID.; CAPITAL INVESTMENT, NOT BASIS FOR DEPLETION.
the promised road or outlet through which the coal could be transported from The taxpayer insists in this appeal that it could use as a method for depletion
the mines to the provincial road was not constructed, cannot be sustained. under the pertinent provision of the Tax Code its "capital investment"
Some definite event must fix the time when the loss is sustained, and there it representing the alleged value of its contractual rights and titles to mining
was the event of actual abandonment of the mines in 1952. claims in the sum of P242,408.10 and thus deduct outright one-fifth (1/5) of
this "Capital investment" ever year, regardless of whether it had actually mined
5. ID.; ID.; ID.; ID.; LOSSES BY INVENTORY METHOD; DISALLOWANCE the product and sold the products. HELD: The alleged "capital investment"
OF LOSSES THEREOF, NOT PROPER. Where respondent Commissioner method invoked by the taxpayer is not a method of depletion, but the Tax Code
concedes that under Section 100 of Revenue Regulations No. 2, it does not provision, prior to its amendment by Section 1 of Republic Act No. 2698, which
specify how the inventories are to be made and the Tax Court is satisfied with took effect on June 18, 1960, expressly provided that "when the allowances
the evidence presented by the taxpayer ... which merely consisted of an alleged shall equal capital invested . . . no further allowances shall be made;" in other
physical count of the number of the livestock in Hacienda Dalupiri for the words, the "capital investment" was but the limitation of the amount of
years involved and the method adopted by the taxpayer as a farmer breeding depletion that could be claimed. The outright deduction by the taxpayer of 1/5
livestock, reporting on the basis of receipts and disbursements, there is no of the cost of the mines, as if it were a "straight line" rate of depreciations was
compelling reason to disturb the ruling of the Tax Court overruling the correctly held by the Tax Court not to be authorized by the Tax Code.
Commissioners disallowance of losses in Hacienda Dalupiri (1950 to 1954)
and Hacienda Samal (1951-1952). 10. ID.; ID.; ASSESSMENT AND COLLECTION OF INCOME TAX;
PRESCRIPTION; ACTION FOR COLLECTION IN INSTANT CASE HAS NOT

91
TAXATION II ATTY. ACOSTA -
CAJUSTIN
PRESCRIBED. In the present case, regardless of whether the assessments d. Losses in Hacienda Dalupiri
were made on February 24 and 27, 1956, as claimed by the Commissioner, or
on December 27, 1955 as claimed by the taxpayer, the governments right to 1950 17,418.95
collect the taxes due has clearly not prescribed, as the taxpayers appeal or
1951 29,125.82
petition for review was filed with the Tax Court on May 4, 1960, with the
Commissioner filing on May 20, 1960 his answer with a prayer for payment of 1952 26,744.81
the taxes due, long before the expiration of the five-year period to effect 1953 21,932.62
collection by judicial action counted from the date of assessment 1954 42,938.56

These four appears involve two decisions of the Court of Tax Appeals determining e. Losses in Hacienda Samal
the taxpayer's income tax liability for the years 1950 to 1954 and for the year 1957.
Both the taxpayer and the Commissioner of Internal Revenue, as petitioner and 1951 8,380.25
respondent in the cases a quo respectively, appealed from the Tax Court's
decisions, insofar as their respective contentions on particular tax items were 1952 7,621.73
therein resolved against them. Since the issues raised are interrelated, the Court
resolves the four appeals in this joint decision. 2. Excessive depreciation of Houses

Cases L-21551 and L-21557 1950 P 8,180.40


1951 8,768.11
The taxpayer, Fernandez Hermanos, Inc., is a domestic corporation organized for 1952 18,002.16
the principal purpose of engaging in business as an "investment company" with 1953 13,655.25
main office at Manila. Upon verification of the taxpayer's income tax returns for 1954 29,314.98
the period in question, the Commissioner of Internal Revenue assessed against
the taxpayer the sums of P13,414.00, P119,613.00, P11,698.00, P6,887.00 and
P14,451.00 as alleged deficiency income taxes for the years 1950, 1951, 1952, 1953 3. Taxable increase in net worth
and 1954, respectively. Said assessments were the result of alleged discrepancies
found upon the examination and verification of the taxpayer's income tax returns 1950 P 30,050.00
for the said years, summarized by the Tax Court in its decision of June 10, 1963 in 1951 1,382.85
CTA Case No. 787, as follows:

4. Gain realized from sale of real property in 1950 P


1. Losses 11,147.2611

a. Losses in Mati Lumber Co. (1950) P 8,050.00 The Tax Court sustained the Commissioner's disallowances of Item 1,
sub-items (b) and (e) and Item 2 of the above summary, but overruled
b. Losses in or bad debts of Palawan Manganese Mines, Inc. the Commissioner's disallowances of all the remaining items. It therefore
(1951) 353,134.25 modified the deficiency assessments accordingly, found the total
deficiency income taxes due from the taxpayer for the years under review
c. Losses in Balamban Coal Mines to amount to P123,436.00 instead of P166,063.00 as originally assessed
by the Commissioner, and rendered the following judgment:

1950 8,989.76
RESUME
1951 27,732.66

92
TAXATION II ATTY. ACOSTA -
CAJUSTIN
1950 P2,748.00 1951 108,724.00
1952 3,600.00
1953 2,501.00
1954 5,863.00

Total P123,436.00

WHEREFORE, the decision appealed from is hereby modified, and


petitioner is ordered to pay the sum of P123,436.00 within 30 days from
the date this decision becomes final. If the said amount, or any part
thereof, is not paid within said period, there shall be added to the unpaid
amount as surcharge of 5%, plus interest as provided in Section 51 of the
National Internal Revenue Code, as amended. With costs against
petitioner. (Pp. 75, 76, Taxpayer's Brief as appellant)

Both parties have appealed from the respective adverse rulings against them in
the Tax Court's decision. Two main issues are raised by the parties: first, the
correctness of the Tax Court's rulings with respect to the disputed items of
disallowances enumerated in the Tax Court's summary reproduced above, and
second, whether or not the government's right to collect the deficiency income
taxes in question has already prescribed.

On the first issue, we will discuss the disputed items of disallowances seriatim.

1. Re allowances/disallowances of losses.

(a) Allowance of losses in Mati Lumber Co. (1950). The Commissioner of


Internal Revenue questions the Tax Court's allowance of the taxpayer's writing off
as worthless securities in its 1950 return the sum of P8,050.00 representing the
cost of shares of stock of Mati Lumber Co. acquired by the taxpayer on January 1,
1948, on the ground that the worthlessness of said stock in the year 1950 had not
been clearly established. The Commissioner contends that although the said
Company was no longer in operation in 1950, it still had its sawmill and
equipment which must be of considerable value. The Court, however, found that
"the company ceased operations in 1949 when its Manager and owner, a certain
Mr. Rocamora, left for Spain ,where he subsequently died. When the company
eased to operate, it had no assets, in other words, completely insolvent. This
information as to the insolvency of the Company reached (the taxpayer) in
1950," when it properly claimed the loss as a deduction in its 1950 tax return,
pursuant to Section 30(d) (4) (b) or Section 30 (e) (3) of the National Internal
Revenue Code. 2

93
TAXATION II ATTY. ACOSTA -
CAJUSTIN
We find no reason to disturb this finding of the Tax Court. There was adequate to pay to the FIRST PARTY fifteen per centum (15%) of its net
basis for the writing off of the stock as worthless securities. Assuming that the profits." (Exh. H-2)
Company would later somehow realize some proceeds from its sawmill and
equipment, which were still existing as claimed by the Commissioner, and that Pursuant to the agreement mentioned above, petitioner gave to Palawan
such proceeds would later be distributed to its stockholders such as the taxpayer, Manganese Mines, Inc. yearly advances starting from 1945, which advances
the amount so received by the taxpayer would then properly be reportable as amounted to P587,308.07 by the end of 1951. Despite these advances and the
income of the taxpayer in the year it is received. resumption of operations by Palawan Manganese Mines, Inc., it continued to
suffer losses. By 1951, petitioner became convinced that those advances could no
(b) Disallowance of losses in or bad debts of Palawan Manganese Mines, longer be recovered. While it continued to give advances, it decided to write off as
Inc. (1951). The taxpayer appeals from the Tax Court's disallowance of its worthless the sum of P353,134.25. This amount "was arrived at on the basis of the
writing off in 1951 as a loss or bad debt the sum of P353,134.25, which it had total of advances made from 1945 to 1949 in the sum of P438,981.39, from which
advanced or loaned to Palawan Manganese Mines, Inc. The Tax Court's findings amount the sum of P85,647.14 had to be deducted, the latter sum representing its
on this item follow: pre-war assets. (t.s.n., pp. 136-139, Id)." (Page 4, Memorandum for Petitioner.)
Petitioner decided to maintain the advances given in 1950 and 1951 in the hope
Sometime in 1945, Palawan Manganese Mines, Inc., the controlling that it might be able to recover the same, as in fact it continued to give advances
stockholders of which are also the controlling stockholders of petitioner up to 1952. From these facts, and as admitted by petitioner itself, Palawan
corporation, requested financial help from petitioner to enable it to Manganese Mines, Inc., was still in operation when the advances corresponding
resume it mining operations in Coron, Palawan. The request for financial to the years 1945 to 1949 were written off the books of petitioner. Under the
assistance was readily and unanimously approved by the Board of circumstances, was the sum of P353,134.25 properly claimed by petitioner as
Directors of petitioner, and thereafter a memorandum agreement was deduction in its income tax return for 1951, either as losses or bad debts?
executed on August 12, 1945, embodying the terms and conditions under
which the financial assistance was to be extended, the pertinent It will be noted that in giving advances to Palawan Manganese Mine Inc.,
provisions of which are as follows: petitioner did not expect to be repaid. It is true that some testimonial evidence
was presented to show that there was some agreement that the advances would be
"WHEREAS, the FIRST PARTY, by virtue of its resolution repaid, but no documentary evidence was presented to this effect. The
adopted on August 10, 1945, has agreed to extend to the memorandum agreement signed by the parties appears to be very clear that the
SECOND PARTY the requested financial help by way of consideration for the advances made by petitioner was 15% of the net profits of
accommodation advances and for this purpose has authorized Palawan Manganese Mines, Inc. In other words, if there were no earnings or
its President, Mr. Ramon J. Fernandez to cause the release of profits, there was no obligation to repay those advances. It has been held that the
funds to the SECOND PARTY. voluntary advances made without expectation of repayment do not result in
deductible losses. 1955 PH Fed. Taxes, Par. 13, 329, citing W. F. Young, Inc. v.
Comm., 120 F 2d. 159, 27 AFTR 395; George B. Markle, 17 TC. 1593.
"WHEREAS, to compensate the FIRST PARTY for the advances
that it has agreed to extend to the SECOND PARTY, the latter
has agreed to pay to the former fifteen per centum (15%) of its Is the said amount deductible as a bad debt? As already stated, petitioner gave
net profits. advances to Palawan Manganese Mines, Inc., without expectation of repayment.
Petitioner could not sue for recovery under the memorandum agreement because
the obligation of Palawan Manganese Mines, Inc. was to pay petitioner 15% of its
"NOW THEREFORE, for and in consideration of the above net profits, not the advances. No bad debt could arise where there is no valid and
premises, the parties hereto have agreed and covenanted that in subsisting debt.
consideration of the financial help to be extended by the FIRST
PARTY to the SECOND PARTY to enable the latter to resume
its mining operations in Coron, Palawan, the SECOND PARTY Again, assuming that in this case there was a valid and subsisting debt and that
has agreed and undertaken as it hereby agrees and undertakes the debtor was incapable of paying the debt in 1951, when petitioner wrote off the
advances and deducted the amount in its return for said year, yet the debt is not
deductible in 1951 as a worthless debt. It appears that the debtor was still in

94
TAXATION II ATTY. ACOSTA -
CAJUSTIN
operation in 1951 and 1952, as petitioner continued to give advances in those sustaining the disallowance of the write-off in 1951 of the taxpayer's claimed
years. It has been held that if the debtor corporation, although losing money or losses is borne out by subsequent events shown in Cases L-24972 and L-24978
insolvent, was still operating at the end of the taxable year, the debt is not involving the taxpayer's 1957 income tax liability. (Infra, paragraph 6.) It will
considered worthless and therefore not deductible. 3 there be seen that by 1956, the obligation of the taxpayer's subsidiary to it had
been reduced from P587,398.97 in 1951 to P442,885.23 in 1956, and that it was
The Tax Court's disallowance of the write-off was proper. The Solicitor General only on January 1, 1956 that the subsidiary decided to cease operations. 8
has rightly pointed out that the taxpayer has taken an "ambiguous position " and
"has not definitely taken a stand on whether the amount involved is claimed as (c) Disallowance of losses in Balamban Coal Mines (1950 and 1951). The Court
losses or as bad debts but insists that it is either a loss or a bad debt." 4 We sustain sustains the Tax Court's disallowance of the sums of P8,989.76 and P27,732.66
the government's position that the advances made by the taxpayer to its 100% spent by the taxpayer for the operation of its Balamban coal mines in Cebu in
subsidiary, Palawan Manganese Mines, Inc. amounting to P587,308,07 as of 1951 1950 and 1951, respectively, and claimed as losses in the taxpayer's returns for
were investments and not loans. 5 The evidence on record shows that the board of said years. The Tax Court correctly held that the losses "are deductible in 1952,
directors of the two companies since August, 1945, were identical and that the when the mines were abandoned, and not in 1950 and 1951, when they were still
only capital of Palawan Manganese Mines, Inc. is the amount of P100,000.00 in operation." 9 The taxpayer's claim that these expeditions should be allowed as
entered in the taxpayer's balance sheet as its investment in its subsidiary losses for the corresponding years that they were incurred, because it made no
company. 6 This fact explains the liberality with which the taxpayer made such sales of coal during said years, since the promised road or outlet through which
large advances to the subsidiary, despite the latter's admittedly poor financial the coal could be transported from the mines to the provincial road was not
condition. constructed, cannot be sustained. Some definite event must fix the time when the
loss is sustained, and here it was the event of actual abandonment of the mines in
The taxpayer's contention that its advances were loans to its subsidiary as against 1952. The Tax Court held that the losses, totalling P36,722.42 were properly
the Tax Court's finding that under their memorandum agreement, the taxpayer deductible in 1952, but the appealed judgment does not show that the taxpayer
did not expect to be repaid, since if the subsidiary had no earnings, there was no was credited therefor in the determination of its tax liability for said year. This
obligation to repay those advances, becomes immaterial, in the light of our additional deduction of P36,722.42 from the taxpayer's taxable income in 1952
resolution of the question. The Tax Court correctly held that the subsidiary would result in the elimination of the deficiency tax liability for said year in the
company was still in operation in 1951 and 1952 and the taxpayer continued to sum of P3,600.00 as determined by the Tax Court in the appealed judgment.
give it advances in those years, and, therefore, the alleged debt or investment
could not properly be considered worthless and deductible in 1951, as claimed by (d) and (e) Allowance of losses in Hacienda Dalupiri (1950 to 1954) and
the taxpayer. Furthermore, neither under Section 30 (d) (2) of our Tax Code Hacienda Samal (1951-1952). The Tax Court overruled the Commissioner's
providing for deduction by corporations of losses actually sustained and charged disallowance of these items of losses thus:
off during the taxable year nor under Section 30 (e) (1) thereof providing for
deduction of bad debts actually ascertained to be worthless and charged off within Petitioner deducted losses in the operation of its Hacienda Dalupiri the
the taxable year, can there be a partial writing off of a loss or bad debt, as was sums of P17,418.95 in 1950, P29,125.82 in 1951, P26,744.81 in 1952,
sought to be done here by the taxpayer. For such losses or bad debts must be P21,932.62 in 1953, and P42,938.56 in 1954. These deductions were
ascertained to be so and written off during the taxable year, are therefore disallowed by respondent on the ground that the farm was operated
deductible in full or not at all, in the absence of any express provision in the Tax solely for pleasure or as a hobby and not for profit. This conclusion is
Code authorizing partial deductions. based on the fact that the farm was operated continuously at a
loss.1awphl.nt
The Tax Court held that the taxpayer's loss of its investment in its subsidiary
could not be deducted for the year 1951, as the subsidiary was still in operation in From the evidence, we are convinced that the Hacienda Dalupiri was
1951 and 1952. The taxpayer, on the other hand, claims that its advances were operated by petitioner for business and not pleasure. It was mainly a
irretrievably lost because of the staggering losses suffered by its subsidiary in 1951 cattle farm, although a few race horses were also raised. It does not
and that its advances after 1949 were "only limited to the purpose of salvaging appear that the farm was used by petitioner for entertainment, social
whatever ore was already available, and for the purpose of paying the wages of the activities, or other non-business purposes. Therefore, it is entitled to
laborers who needed help." 7 The correctness of the Tax Court's ruling in deduct expenses and losses in connection with the operation of said

95
TAXATION II ATTY. ACOSTA -
CAJUSTIN
farm. (See 1955 PH Fed. Taxes, Par. 13, 63, citing G.C.M. 21103, CB 2. Disallowance of excessive depreciation of buildings (1950-1954). During the
1939-1, p.164) years 1950 to 1954, the taxpayer claimed a depreciation allowance for its buildings
at the annual rate of 10%. The Commissioner claimed that the reasonable
Section 100 of Revenue Regulations No. 2, otherwise known as the depreciation rate is only 3% per annum, and, hence, disallowed as excessive the
Income Tax Regulations, authorizes farmers to determine their gross amount claimed as depreciation allowance in excess of 3% annually. We sustain
income on the basis of inventories. Said regulations provide: the Tax Court's finding that the taxpayer did not submit adequate proof of the
correctness of the taxpayer's claim that the depreciable assets or buildings in
question had a useful life only of 10 years so as to justify its 10% depreciation per
"If gross income is ascertained by inventories, no deduction can annum claim, such finding being supported by the record. The taxpayer's
be made for livestock or products lost during the year, whether contention that it has many zero or one-peso assets, 12 representing very old and
purchased for resale, produced on the farm, as such losses will fully depreciated assets serves but to support the Commissioner's position that a
be reflected in the inventory by reducing the amount of 10% annual depreciation rate was excessive.
livestock or products on hand at the close of the year."
3. Taxable increase in net worth (1950-1951). The Tax Court set aside the
Evidently, petitioner determined its income or losses in the operation of Commissioner's treatment as taxable income of certain increases in the taxpayer's
said farm on the basis of inventories. We quote from the memorandum net worth. It found that:
of counsel for petitioner:
For the year 1950, respondent determined that petitioner had an
"The Taxpayer deducted from its income tax returns for the increase in net worth in the sum of P30,050.00, and for the year 1951,
years from 1950 to 1954 inclusive, the corresponding yearly the sum of P1,382.85. These amounts were treated by respondent as
losses sustained in the operation of Hacienda Dalupiri, which taxable income of petitioner for said years.
losses represent the excess of its yearly expenditures over the
receipts; that is, the losses represent the difference between the
sales of livestock and the actual cash disbursements or It appears that petitioner had an account with the Manila Insurance
expenses." (Pages 21-22, Memorandum for Petitioner.) Company, the records bearing on which were lost. When its records were
reconstituted the amount of P349,800.00 was set up as its liability to the
Manila Insurance Company. It was discovered later that the correct
As the Hacienda Dalupiri was operated by petitioner for business and liability was only 319,750.00, or a difference of P30,050.00, so that the
since it sustained losses in its operation, which losses were determined records were adjusted so as to show the correct liability. The correction
by means of inventories authorized under Section 100 of Revenue or adjustment was made in 1950. Respondent contends that the
Regulations No. 2, it was error for respondent to have disallowed the reduction of petitioner's liability to Manila Insurance Company resulted
deduction of said losses. The same is true with respect to loss sustained in the increase of petitioner's net worth to the extent of P30,050.00
in the operation of the Hacienda Samal for the years 1951 and 1952. 10 which is taxable. This is erroneous. The principle underlying the
taxability of an increase in the net worth of a taxpayer rests on the theory
The Commissioner questions that the losses sustained by the taxpayer were that such an increase in net worth, if unreported and not explained by
properly based on the inventory method of accounting. He concedes, however, the taxpayer, comes from income derived from a taxable source. (See
"that the regulations referred to does not specify how the inventories are to be Perez v. Araneta, G.R. No. L-9193, May 29, 1957; Coll. vs. Reyes, G.R.
made. The Tax Court, however, felt satisfied with the evidence presented by the Nos. L- 11534 & L-11558, Nov. 25, 1958.) In this case, the increase in the
taxpayer ... which merely consisted of an alleged physical count of the number of net worth of petitioner for 1950 to the extent of P30,050.00 was not the
the livestock in Hacienda Dalupiri for the years involved." 11The Tax Court was result of the receipt by it of taxable income. It was merely the outcome of
satisfied with the method adopted by the taxpayer as a farmer breeding livestock, the correction of an error in the entry in its books relating to its
reporting on the basis of receipts and disbursements. We find no Compelling indebtedness to the Manila Insurance Company. The Income Tax Law
reason to disturb its findings. imposes a tax on income; it does not tax any or every increase in net
worth whether or not derived from income. Surely, the said sum of

96
TAXATION II ATTY. ACOSTA -
CAJUSTIN
P30,050.00 was not income to petitioner, and it was error for authority to pronounce judgment as to the taxpayer's liability to the exclusion of
respondent to assess a deficiency income tax on said amount. any other court. In the present case, regardless of whether the assessments were
made on February 24 and 27, 1956, as claimed by the Commissioner, or on
The same holds true in the case of the alleged increase in net worth of petitioner December 27, 1955 as claimed by the taxpayer, the government's right to collect
for the year 1951 in the sum of P1,382.85. It appears that certain items (all the taxes due has clearly not prescribed, as the taxpayer's appeal or petition for
amounting to P1,382.85) remained in petitioner's books as outstanding liabilities review was filed with the Tax Court on May 4, 1960, with the Commissioner filing
of trade creditors. These accounts were discovered in 1951 as having been paid in on May 20, 1960 his Answer with a prayer for payment of the taxes due, long
prior years, so that the necessary adjustments were made to correct the errors. If before the expiration of the five-year period to effect collection by judicial action
there was an increase in net worth of the petitioner, the increase in net worth was counted from the date of assessment.
not the result of receipt by petitioner of taxable income." 13 The Commissioner
advances no valid grounds in his brief for contesting the Tax Court's findings. Cases L-24972 and L-24978
Certainly, these increases in the taxpayer's net worth were not taxable increases in
net worth, as they were not the result of the receipt by it of unreported or These cases refer to the taxpayer's income tax liability for the year 1957. Upon
unexplained taxable income, but were shown to be merely the result of the examination of its corresponding income tax return, the Commissioner assessed it
correction of errors in its entries in its books relating to its indebtednesses to for deficiency income tax in the amount of P38,918.76, computed as follows:
certain creditors, which had been erroneously overstated or listed as outstanding
when they had in fact been duly paid. The Tax Court's action must be affirmed.
Net income per return
4. Gain realized from sale of real property (1950). We likewise sustain as being
in accordance with the evidence the Tax Court's reversal of the Commissioner's Add: Unallowable deductions:
assessment on all alleged unreported gain in the sum of P11,147.26 in the sale of a (1) Net loss claimed on Ha. Dalupiri
certain real property of the taxpayer in 1950. As found by the Tax Court, the (2) Amortization of Contractual right claimed as an expense under Mines Operations
evidence shows that this property was acquired in 1926 for P11,852.74, and was
sold in 1950 for P60,000.00, apparently, resulting in a gain of P48,147.26. 14 The
taxpayer reported in its return a gain of P37,000.00, or a discrepancy of Net income per investigation
P11,147.26. 15 It was sufficiently proved from the taxpayer's books that after Tax due thereon
acquiring the property, the taxpayer had made improvements totalling
P11,147.26, 16 accounting for the apparent discrepancy in the reported gain. In Less: Amount already assessed
other words, this figure added to the original acquisition cost of P11,852.74 results Balance
in a total cost of P23,000.00, and the gain derived from the sale of the property
Add: 1/2% monthly interest from 6-20-59 to 6-20-62
for P60,000.00 was correctly reported by the taxpayer at P37,000.00.

On the second issue of prescription, the taxpayer's contention that the TOTAL AMOUNT DUE AND COLLECTIBLE 18

Commissioner's action to recover its tax liability should be deemed to have


prescribed for failure on the part of the Commissioner to file a complaint for The Tax Court overruled the Commissioner's disallowance of the taxpayer's losses
collection against it in an appropriate civil action, as contradistinguished from the in the operation of its Hacienda Dalupiri in the sum of P89,547.33 but sustained
answer filed by the Commissioner to its petition for review of the questioned the disallowance of the sum of P48,481.62, which allegedly represented 1/5 of the
assessments in the case a quo has long been rejected by this Court. This Court has cost of the "contractual right" over the mines of its subsidiary, Palawan
consistently held that "a judicial action for the collection of a tax is begun by the Manganese Mines, Inc. which the taxpayer had acquired. It found the taxpayer
filing of a complaint with the proper court of first instance, or where the liable for deficiency income tax for the year 1957 in the amount of P9,696.00,
assessment is appealed to the Court of Tax Appeals, by filing an answer to the instead of P32,982.00 as originally assessed, and rendered the following
taxpayer's petition for review wherein payment of the tax is prayed for." 17 This judgment:
is but logical for where the taxpayer avails of the right to appeal the tax
assessment to the Court of Tax Appeals, the said Court is vested with the

97
TAXATION II ATTY. ACOSTA -
CAJUSTIN
WHEREFORE, the assessment appealed from is hereby modified. The Tax Court, having satisfied itself with the adequacy of the taxpayer's
Petitioner is hereby ordered to pay to respondent the amount of accounting method and procedure as properly reflecting the taxpayer's
P9,696.00 as deficiency income tax for the year 1957, plus the income or losses, and the Commissioner having failed to show the
corresponding interest provided in Section 51 of the Revenue Code. If contrary, we reiterate our ruling [supra, paragraph 1 (d) and (e)] that we
the deficiency tax is not paid in full within thirty (30) days from the date find no compelling reason to disturb its findings.
this decision becomes final and executory, petitioner shall pay a
surcharge of five per cent (5%) of the unpaid amount, plus interest at the 6. Disallowance of amortization of alleged "contractual rights." The reasons
rate of one per cent (1%) a month, computed from the date this decision for sustaining this disallowance are thus given by the Tax Court:
becomes final until paid, provided that the maximum amount that may
be collected as interest shall not exceed the amount corresponding to a
period of three (3) years. Without pronouncement as to costs. 19 It appears that the Palawan Manganese Mines, Inc., during a special
meeting of its Board of Directors on January 19, 1956, approved a
resolution, the pertinent portions of which read as follows:
Both parties again appealed from the respective adverse rulings against them in
the Tax Court's decision.
"RESOLVED, as it is hereby resolved, that the corporation's
current assets composed of ores, fuel, and oil, materials and
5. Allowance of losses in Hacienda Dalupiri (1957). The Tax Court cited its supplies, spare parts and canteen supplies appearing in the
previous decision overruling the Commissioner's disallowance of losses suffered inventory and balance sheet of the Corporation as of December
by the taxpayer in the operation of its Hacienda Dalupiri, since it was convinced 31, 1955, with an aggregate value of P97,636.98, contractual
that the hacienda was operated for business and not for pleasure. And in this rights for the operation of various mining claims in Palawan
appeal, the Commissioner cites his arguments in his appellant's brief in Case No. with a value of P100,000.00, its title on various mining claims
L-21557. The Tax Court, in setting aside the Commissioner's principal objections, in Palawan with a value of P142,408.10 or a total value of
which were directed to the accounting method used by the taxpayer found that: P340,045.02 be, as they are hereby ceded and transferred to
Fernandez Hermanos, Inc., as partial settlement of the
It is true that petitioner followed the cash basis method of reporting indebtedness of the corporation to said Fernandez Hermanos
income and expenses in the operation of the Hacienda Dalupiri and used Inc. in the amount of P442,895.23." (Exh. E, p. 17, CTA rec.)
the accrual method with respect to its mine operations. This method of
accounting, otherwise known as the hybrid method, followed by On March 29, 1956, petitioner's corporation accepted the above offer of
petitioner is not without justification. transfer, thus:

... A taxpayer may not, ordinarily, combine the cash and accrual "WHEREAS, the Palawan Manganese Mines, Inc., due to its
bases. The 1954 Code provisions permit, however, the use of a yearly substantial losses has decided to cease operation on
hybrid method of accounting, combining a cash and accrual January 1, 1956 and in order to satisfy at least a part of its
method, under circumstances and requirements to be set out in indebtedness to the Corporation, it has proposed to transfer its
Regulations to be issued. Also, if a taxpayer is engaged in more current assets in the amount of NINETY SEVEN THOUSAND
than one trade or business he may use a different method of SIX HUNDRED THIRTY SIX PESOS & 98/100 (P97,636.98) as
accounting for each trade or business. And a taxpayer may per its balance sheet as of December 31, 1955, its contractual
report income from a business on accrual basis and his personal rights valued at ONE HUNDRED THOUSAND PESOS
income on the cash basis.' (See Mertens, Law of Federal Income (P100,000.00) and its title over various mining claims valued
Taxation, Zimet & Stanley Revision, Vol. 2, Sec. 12.08, p. 26.) 20 at ONE HUNDRED FORTY TWO THOUSAND FOUR
HUNDRED EIGHT PESOS & 10/100 (P142,408.10) or a total
evaluation of THREE HUNDRED FORTY THOUSAND FORTY
FIVE PESOS & 08/100 (P340,045.08) which shall be applied in
partial settlement of its obligation to the Corporation in the

98
TAXATION II ATTY. ACOSTA -
CAJUSTIN
amount of FOUR HUNDRED FORTY TWO THOUSAND years. The said memorandum merely showed the estimated ore reserves
EIGHT HUNDRED EIGHTY FIVE PESOS & 23/100 of the mines and it probable selling price. No evidence whatsoever was
(P442,885.23)," (Exh. E-1, p. 18, CTA rec.) presented to show the produced mine and for how much they were sold
during the year for which the return and computation were made. This is
Petitioner determined the cost of the mines at P242,408.10 by adding necessary in order to determine the amount of depletion that can be
the value of the contractual rights (P100,000.00) and the value of its legally deducted from petitioner's gross income. The method employed
mining claims (P142,408.10). Respondent disallowed the deduction on by petitioner in making an outright deduction of 1/5 of the cost of the
the following grounds: (1) that the Palawan Manganese Mines, Inc. could mines is not authorized under Section 30(g) (1) (B) of the Revenue Code.
not transfer P242,408.10 worth of assets to petitioner because the Respondent's disallowance of the alleged "contractual rights" amounting
balance sheet of the said corporation for 1955 shows that it had only to P48,481.62 must therefore be sustained. 21
current as worth P97,636.96; and (2) that the alleged amortization of
"contractual rights" is not allowed by the Revenue Code. The taxpayer insists in this appeal that it could use as a method for depletion
under the pertinent provision of the Tax Code its "capital investment,"
The law in point is Section 30(g) (1) (B) of the Revenue Code, before its representing the alleged value of its contractual rights and titles to mining claims
amendment by Republic Act No. 2698, which provided in part: in the sum of P242,408.10 and thus deduct outright one-fifth (1/5) of this "capital
investment" every year. regardless of whether it had actually mined the product
and sold the products. The very authorities cited in its brief give the correct
"(g) Depletion of oil and gas wells and mines.: concept of depletion charges that they "allow for the exhaustion of the capital
value of the deposits by production"; thus, "as the cost of the raw materials must
"(1) In general. ... (B) in the case of mines, a reasonable be deducted from the gross income before the net income can be determined, so
allowance for depletion thereof not to exceed the market value the estimated cost of the reserve used up is allowed." 22 The alleged "capital
in the mine of the product thereof, which has been mined and investment" method invoked by the taxpayer is not a method of depletion, but the
sold during the year for which the return and computation are Tax Code provision, prior to its amendment by Section 1, of Republic Act No.
made. The allowances shall be made under rules and 2698, which took effect on June 18, 1960, expressly provided that "when the
regulations to be prescribed by the Secretary of allowances shall equal the capital invested ... no further allowances shall be
Finance: Provided, That when the allowances shall equal the made;" in other words, the "capital investment" was but the limitation of the
capital invested, ... no further allowance shall be made." amount of depletion that could be claimed. The outright deduction by the
taxpayer of 1/5 of the cost of the mines, as if it were a "straight line" rate of
depreciation, was correctly held by the Tax Court not to be authorized by the Tax
Assuming, arguendo, that the Palawan Manganese Mines, Inc. had
Code.
assets worth P242,408.10 which it actually transferred to the petitioner
in 1956, the latter cannot just deduct one-fifth (1/5) of said amount from
its gross income for the year 1957 because such deduction in the form of ACCORDINGLY, the judgment of the Court of Tax Appeals, subject of the appeals
depletion charge was not sanctioned by Section 30(g) (1) (B) of the in Cases Nos. L-21551 and L-21557, as modified by the crediting of the losses of
Revenue Code, as above-quoted. P36,722.42 disallowed in 1951 and 1952 to the taxpayer for the year 1953 as
directed in paragraph 1 (c) of this decision, is hereby affirmed. The judgment of
the Court of Tax Appeals appealed from in Cases Nos. L-24972 and L-24978 is
xxx xxx xxx
affirmed in toto. No costs. So ordered.

The sole basis of petitioner in claiming the amount of P48,481.62 as a


deduction was the memorandum of its mining engineer (Exh. 1, pp. 31-
32, CTA rec.), who stated that the ore reserves of the Busuange Mines
(Mines transferred by the Palawan Manganese Mines, Inc. to the [G.R. No. 125508. July 19, 2000]
petitioner) would be exhausted in five (5) years, hence, the claim for
P48,481.62 or one-fifth (1/5) of the alleged cost of the mines
corresponding to the year 1957 and every year thereafter for a period of 5

99
TAXATION II ATTY. ACOSTA -
CAJUSTIN
CHINA BANKING CORPORATION, petitioner, vs. COURT OF *The loss cannot be deductible as bad debt since the shares of stock do not
APPEALS, COMMISSIONER OF INTERNAL REVENUE and constitute a loan extended by it to its subsidiary or a debt subject to obligatory
COURT OF TAX APPEALS, respondents. repayment by the latter.
Facts:
China Banking Corporation made a 53% equity investment (P16,227,851.80) in SYNOPSIS
the First CBC Capital a Hongkong subsidiary engaged in financing and
investment with deposit-taking function. Petitioner China Banking Corporation made a 53% equity investment in the
First CBC Capital (Asia) Ltd., a Hongkong subsidiary engaged in financing and
It was shown that CBC has become insolvent so China Banking wrote-off its investment with "deposit-taking" function. First CBC Capital (Asia), Ltd.
investment as worthless and treated it as a bad debt or as an ordinary loss became insolvent. With the approval of Bangko Sentral, petitioner wrote-off as
deductible from its gross income. being worthless its investment in First CBC Capital (Asia), Ltd., in its 1987
Income Tax Return and treated it as a bad debt or as an ordinary loss
CIR disallowed the deduction on the ground that the investment should not be deductible from its gross income. Respondent Commissioner of Internal
classified as being worthless. It also held that assuming that the securities were Revenue disallowed the deduction. The Commissioner held the view that the
worthless, then they should be classified as a capital loss and not as a bad debt shares should be classified as "capital loss," and not as a bad debt expense
since there was no indebtedness between China Banking and CBC. there being no indebtedness to speak of between petitioner and its subsidiary.
Petitioner contested the ruling of respondent Commissioner before the Court
Issue: of Tax Appeals (CTA). The tax court sustained the Commissioner. When the
Whether or not the investment should be classified as a capital loss. decision was appealed to the Court of Appeals, the latter upheld the CTA.
Hence, the present petition. Petitioner bank assailed the appellate court in
Held: affirming the CTA's decision.
Yes. Section 29.d.4.B of the NIRC contains provisions on securities becoming
worthless. It conveys that capital loss normally requires the concurrence of 2 The Supreme Court affirmed the decision of the Court of Appeals. The Court
conditions: ruled that shares of stock held by way of an investment are considered as
there is a sale or exchange capital assets under the law and when said shares held by such investor
b. the thing sold or exchanges is a capital asset. become worthless, the loss is deemed to be a loss from the sale or exchange of
capital assets under Section 29(d)(4)(B) of the National Internal Revenue
When securities become worthless, there is strictly no sale or exchange but the Code. In the case at bar, First CBC Capital (Asia), Ltd., the investee
law deems it to be a loss. These are allowed to be deducted only to the extent of corporation, is a subsidiary corporation of petitioner bank whose shares in said
capital gains and not from any other income of the taxpayer. A similar kind of investee corporation are not intended for purchase or sale but as an
treatment is given by the NIRC on the retirement of certificates of investment. Unquestionably then, any loss therefrom would be a capital loss,
indebtedness with interest coupons or in registered form, short sales and not an ordinary loss to the investor. The Court also ruled that equity holdings
options to buy or sell property where no sale or exchange strictly exists. In cannot come close to being, within the purview of "evidence of indebtedness"
these cases, The NIRC dispenses with the standard requirements. under Section 33 of the NIRC. The loss of petitioner bank in its equity
investment in the Hongkong subsidiary cannot also be deductible as a bad
There is ordinary loss when the property sold is not a capital asset. debt. The shares of stock in question did not constitute a loan extended by it to
its subsidiary (First CBC Capital) or a debt subject to obligatory repayment by
In the case, CBC as an investee corporation, is a subsidiary corporation of the latter, essential elements to constitute a bad debt, but a long term
China Banking whose shares in CBC are not intended for purchase or sale but investment made by CBC.
as an investment. An equity investment is a capital asset of the investor.
Unquestionably, any loss is a capital loss to the investor.
SYLLABUS
--
Additional notes: 1. TAXATION; NATIONAL INTERNAL REVENUE CODE; INCOME

100
TAXATION II ATTY. ACOSTA -
CAJUSTIN

TAXATION; GROSS INCOME; DEDUCTIONS FROM GROSS INCOME; operations of a financial institution. Equity holdings cannot come close to
SHARES OF STOCK HELD BY INVESTOR BECOMING WORTHLESS; NOT being within the purview of "evidence of indebtedness" under the second
DEDUCTIBLE, LOSS SUSTAINED DEEMED TO BE A LOSS FROM THE sentence of the aforequoted paragraph. Verily, it is for a like thesis that the loss
SALE OR EXCHANGE OF CAPITAL ASSETS. An equity investment is a of petitioner bank in its equity investment in the Hongkong subsidiary cannot
capital, not ordinary, asset of the investor the sale or exchange of which results also be deductible as a bad debt. The shares of stock in question do not
in either a capital gain or a capital loss. The gain or the loss is ordinary when constitute a loan extended by it to its subsidiary (First CBC Capital) or a debt
the property sold or exchanged is not a capital asset. Shares of stock, like the subject to obligatory repayment by the latter, essential elements to constitute a
other securities defined in Section 20(t) of the NIRC, would be ordinary assets bad debt, but a long term investment made by CBC.
only to a dealer in securities or a person engaged in the purchase and sale of, or
an active trader (for his own account) in, securities. In the hands, however, of
another who holds the shares of stock by way of an investment, the shares to
him would be capital assets. When the shares held by such investor become
worthless, the loss is deemed to be a loss from the sale or exchange of capital
The Commissioner of Internal Revenue denied the deduction from gross
assets. Section 29(d)(4)(B) of the NIRC. - Provides that the loss sustained by
income of "securities becoming worthless" claimed by China Banking Corporation
the holder of the securities, which are capital assets (to him), is to be treated as
(CBC). The Commissioners disallowance was sustained by the Court of Tax
a capital loss as if incurred from a sale or exchange transaction. A capital gain
Appeals ("CTA"). When the ruling was appealed to the Court of Appeals ("CA"),
or a capital loss normally requires the concurrence of two conditions for it to
the appellate court upheld the CTA. The case is now before us on a Petition for
result: (1) There is a sale or exchange; and (2) the thing sold or exchanged is a
Review on Certiorari.
capital asset. When securities become worthless, there is strictly no sale or
exchange but the law deems the loss anyway to be "a loss from the sale or Sometime in 1980, petitioner China Banking Corporation made a 53%
exchange of capital assets." A similar kind of treatment is given by the NIRC on equity investment in the First CBC Capital (Asia) Ltd., a Hongkong subsidiary
the retirement of certificates of indebtedness with interest coupons or in engaged in financing and investment with "deposit-taking" function. The
registered form, short sales and options to buy or sell property where no sale or investment amounted to P16,227,851.80, consisting of 106,000 shares with a par
exchange strictly exists. In these cases, the NIRC dispenses, in effect, with the Value of P100 per share.
standard requirement of a sale or exchange for the application of the capital
gain and loss provisions of the code. Capital losses are allowed to be deducted In the course of the regular examination of the financial books and
only to the extent of capital gains, i.e., gains derived from the sale or exchange investment portfolios of petitioner conducted by Bangko Sentralin 1986, it was
of capital assets, and not from any other income of the taxpayer. In the case at shown that First CBC Capital (Asia), Ltd., has become insolvent. With the
bar, First CBC Capital (Asia), Ltd., the investee corporation, is a subsidiary approval of Bangko Sentral, petitioner wrote-off as being worthless its
corporation of petitioner bank whose shares in said investee corporation are investment in First CBC Capital (Asia), Ltd., in its 1987 Income Tax Return and
not intended for purchase or sale but as an investment. Unquestionably then, treated it as a bad debt or as an ordinary loss deductible from its gross income.
any loss therefrom would be a capital loss, not an ordinary loss, to the
investor. cTCaEA Respondent Commissioner of internal Revenue disallowed the deduction
and assessed petitioner for income tax deficiency in the amount of P8,533,328.04,
inclusive of surcharge, interest and compromise penalty. The disallowance of the
2. ID.; ID.; ID.; LOSSES IN EQUITY INVESTMENT LIKE SHARES OF STOCK deduction was made on the ground that the investment should not be classified as
NOT DEDUCTIBLE AS BAD DEBT; EQUITY HOLDINGS SUCH AS SHARES being "worthless" and that, although the Hongkong Banking Commissioner had
OF STOCK NOT WITHIN THE PURVIEW OF "EVIDENCE OF revoked the license of First CBC Capital as a "deposit-taping" company, the latter
INDEBTEDNESS" UNDER OF SECTION 33 (c) OF THE NATIONAL could still exercise, however, its financing and investment activities. Assuming
INTERNAL REVENUE CODE. The exclusionary clause found in Section that the securities had indeed become worthless, respondent Commissioner of
33(c) of the National Internal Revenue Code does not include all forms of Internal Revenue held the view that they should then be classified as "capital
securities but specifically covers only bonds, debentures, notes, certificates or loss," and not as a bad debt expense there being no indebtedness to speak of
other evidence of indebtedness, with interest coupons or in registered form, between petitioner and its subsidiary.
which are the instruments of credit normally dealt with in the usual lending

101
TAXATION II ATTY. ACOSTA -
CAJUSTIN
Petitioner contested the ruling of respondent Commissioner before the Thus, shares of stock; like the other securities defined in Section 20(t) [4] of
CTA. The tax court sustained the Commissioner, holding that the securities had the NIRC, would be ordinary assets only to a dealer in securities or a
not indeed become worthless and ordered petitioner to pay its deficiency income person engaged in the purchase and sale of, or an active trader (for his
tax for 1987 of P8,533,328.04 plus 20% interest per annum until fully paid. When own account) in, securities. Section 20(u) of the NIRC defines a dealer in
the decision was appealed to the Court of Appeals, the latter upheld the CTA. In securities thus:
its instant petition for review on certiorari, petitioner bank assails the CA
decision. "(u) The term 'dealer in securities' means a merchant of stocks or securities,
The petition must fail. whether an individual, partnership or corporation, with an established place of
business, regularly engaged in the purchase of securities and their resale to
The claim of petitioner that the shares of stock in question have become customers; that is, one who as a merchant buys securities and sells them to
worthless is based on a Profit and Loss Account for the Year-End 31 December customers with a view to the gains and profits that may be derived therefrom."
1987, and the recommendation of Bangko Sentral that the equity investment be
written-off due to the insolvency of the subsidiary. While the matter may not be In the hands, however, of another who holds the shares of stock by way of an
indubitable (considering that certain classes of intangibles, like franchises and investment, the shares to him would be capital assets.When the shares held
goodwill, are not always given corresponding values in financial statements [1], by such investor become worthless, the loss is deemed to be a loss
there may really be no need, however, to go of length into this issue since, even to from the sale or exchange of capital assets. Section 29(d)(4)(B) of the
assume the worthlessness of the shares, the deductibility thereof would still be nil NIRC states:
in this particular case. At all events, the Court is not prepared to hold that both
the tax court and the appellate court are utterly devoid of substantial basis for
their own factual findings. "(B) Securities becoming worthless. - If securities as defined in Section 20 become
worthless during the tax" year and are capital assets, the loss resulting therefrom
Subject to certain exceptions, such as the compensation income of shall, for the purposes of his Title, be considered as a loss from the sale or
individuals and passive income subject to final tax, as well as income of non- exchange, on the last day of such taxable year, of capital assets."
resident aliens and foreign corporations not engaged in trade or business in the
Philippines, the tax on income is imposed on the net income allowing certain The above provision conveys that the loss sustained by the holder of the
specified deductions from gross income to be claimed by the taxpayer. Among the securities, which are capital assets (to him), is to be treated as a capital loss as
deductible items allowed by the National Internal Revenue Code ("NIRC") if incurred from a sale or exchange transaction. A capital gain or a capital
are bad debts and losses.[2] loss normally requires the concurrence of two conditions for it to result: (1) There
An equity investment is a capital, not ordinary, asset of the investor the is a sale or exchange; and (2) the thing sold or exchanged is a capital asset. When
sale or exchange of which results in either a capital gain or a capital loss. The gain securities become worthless, there is strictly no sale or exchange but the law
or the loss is ordinary when the property sold or exchanged is not a capital deems the loss anyway to be "a loss from the sale or exchange of capital assets. [5]A
asset.[3] A capital asset is defined negatively in Section 33(1) of the NIRC; viz: similar kind of treatment is given, by the NIRC on the retirement of certificates of
indebtedness with interest coupons or in registered form, short sales and options
to buy or sell property where no sale or exchange strictly exists. [6] In these cases,
(1) Capital assets. - The term 'capital assets' means property held by the taxpayer the NIRC dispenses, in effect, with the standard requirement of a sale or exchange
(whether or not connected with his trade or business), but does not include stock for the application of the capital gain and loss provisions of the code.
in trade of the taxpayer or other property of a kind which would properly be
included in the inventory of the taxpayer if on hand at the close of the taxable Capital losses are allowed to be deducted only to the extent of
year, or property held by the taxpayer primarily for sale to customers in the capital gains, i.e., gains derived from the sale or exchange of capital
ordinary course of his trade or business, or property used in the trade or business, assets, and not from any other income of the taxpayer.
of a character which is subject to the allowance for depreciation provided in
subsection (f) of section twenty-nine; or real property used in the trade or In the case at bar, First CBC Capital (Asia), Ltd., the investee corporation, is
business of the taxpayer. a subsidiary corporation of petitioner bank whose shares in said investee
corporation are not intended for purchase or sale but as an

102
TAXATION II ATTY. ACOSTA -
CAJUSTIN
investment. Unquestionably then, any loss therefrom would be a capital loss, not SECTION 34. Determination of amount of and recognition of gain or loss.-
an ordinary loss, to the investor.
Section 29(d)(4)(A), of the NIRC expresses: "(a) Computation of gain or loss. - The gain from the sale or other disposition of
property shall be the excess of the amount realized therefrom over the basis or
adjusted basis for determining gain and the loss shall be the excess of the basis or
"(A) Limitations. - Losses from sales or exchanges of capital assets shall be adjusted basis for determining loss over the amount realized. The amount realized
allowed only to the extent provided in Section 33." from the sale or other disposition of property shall be to sum of money received
plus the fair market value of the property (other than money) received. (As
The pertinent provisions of Section 33 of the NIRC referred to in the aforesaid amended by E.O. No. 37)
Section 29(d)(4)(A), read:
"(b) Basis for determining gain or loss from sale or disposition of property. - The
"Section 33. Capital gains and losses. - basis of property shall be - (1) The cost thereof in cases of property acquired on or
before March 1, 1913, if such property was acquired by purchase; or
x x x x x x x x x.
"(2) The fair market price or value as of the date of acquisition if the same was
"(c) Limitation on capital losses. - Losses from sales or exchange of capital acquired by inheritance; or
assets shall be allowed only to the extent of the gains from such sales
or exchanges. If a bank or trust company incorporated under the laws of the "(3) If the property was acquired by gift the basis shall be the same as if it would
Philippines, a substantial part of whose business is the receipt of deposits, sells be in the hands of the donor or the last preceding owner by whom it was not
any bond, debenture, note, or certificate or other evidence of acquired by gift, except that if such basis is greater than the fair market value of
indebtedness issued by any corporation (including one issued by a government the property at the time of the gift, then for the purpose of determining loss the
or political subdivision thereof), with interest coupons or in registered basis shall be such fair market value; or
form, any loss resulting from such sale shall not be subject to the foregoing
limitation an shall not be included in determining the applicability of such "(4) If the property, other than capital asset referred to in Section 21 (e), was
limitation to other losses. acquired for less than an adequate consideration in money or moneys worth, the
basis of such property is (i) the amount paid by the transferee for the property or
The exclusionary clause found in the foregoing text of the law does not (ii) the transferor's adjusted basis at the time of the transfer whichever is greater.
include all forms of securities but specifically covers only bonds, debentures,
notes, certificates or other evidence of indebtedness, with interest "(5) The basis as defined in paragraph (c) (5) of this section if the property was
coupons or in registered form, which are the instruments of credit normally acquired in a transaction where gain or loss is not recognized under paragraph (c)
dealt with in the usual lending operations of a financial institution. Equity (2) of this section. (As amended by E.O. No. 37)
holdings cannot come close to being, within the purview of "evidence of
indebtedness" under the second sentence of the aforequoted paragraph. Verily,
it is for a like thesis that the loss of petitioner bank in its equity in vestment in (c) Exchange of property.
the Hongkong subsidiary cannot also be deductible as a bad debt. The shares
of stock in question do not constitute a loan extended by it to its subsidiary (First "(1) General rule.- Except as herein provided, upon the sale or exchange of
CBC Capital) or a debt subject to obligatory repayment by the latter, essential property, the entire amount of the gain or loss, as the case may be, shall be
elements to constitute a bad debt, but a long term investment made by CBC. recognized.
One other item. Section 34(c)(1) of the NIRC , states that the entire amount
of the gain or loss upon the sale or exchange of property, as the case may be, shall "(2) Exception. - No gain or loss shall be recognized if in pursuance of a plan of
be recognized. The complete text reads: merger or consolidation (a) a corporation which is a party to a merger or
consolidation exchanges property solely for stock in a corporation which is, a
party to the merger or consolidation, (b) a shareholder exchanges stock in a

103
TAXATION II ATTY. ACOSTA -
CAJUSTIN
corporation which is a party to the merger or consolidation solely for the stock in
another corporation also a party to the merger or consolidation, or (c) a security
holder of a corporation which is a party to the merger or consolidation exchanges
his securities in such corporation solely for stock or securities in another
corporation, a party to the merger or consolidation.

"No gain or loss shall also be recognized if property is transferred to a corporation


by a person in exchange for stock in such corporation of which as a result of such
exchange said person, alone or together with others, not exceeding four persons,
gains control of said corporation: Provided, That stocks issued for services shall
not be considered as issued in return of property."

The above law should be taken within context on the general subject of the
determination, and recognition of gain or loss; it is not preclusive of, let alone
renders completely inconsequential, the more specific provisions of the
code. Thus, pursuant, to the same section of the law, no such recognition shall be
made if the sale or exchange is made in pursuance of a plan of corporate merger
or consolidation or, if as a result of an exchange of property for stocks, the
exchanger, alone or together with others not exceeding four, gains control of the
corporation.[7] Then, too, how the resulting gain might be taxed, or whether or not
the loss would be deductible and how, are matters properly dealt with elsewhere
in various other sections of the NIRC. [8] At all events, it may not be amiss to once
again stress that the basic rule is still that any capital loss can be deducted
only from capital gains under Section 33(c) of the NIRC.
In sum -
(a) The equity investment in shares of stock held by CBC of approximately
53% in its Hongkong subsidiary, the First CBC Capital (Asia), Ltd., is not an
indebtedness, and it is a capital, not an ordinary, asset.[9]
TOPIC: DEDUCTIONS BAD
(b) Assuming that the equity investment of CBC has indeed become
"worthless," the loss sustained is a capital, not an ordinary, loss.[10] DEBTS
(c) The capital loss sustained by CBC can only be deducted from capital
gains if any derived by it during the same taxable year that the securities have
[G.R. No. L-22265. December 22, 1967.]
become "worthless."[11]
WHEREFORE, the Petition is DENIED. The decision of the Court of COLLECTOR OF INTERNAL REVENUE, Petitioner, v. GOODRICH
Appeals disallowing the claimed deduction of P16,227,851.80 is AFFIRMED. INTERNATIONAL RUBBER CO., Respondent.

SO ORDERED. Solicitor General for Petitioner.

Manuel O. Chan for Respondent.

104
TAXATION II ATTY. ACOSTA -
CAJUSTIN
SYLLABUS Goodrich, on November 19, 1963, the Court of Tax Appeals amended its
aforementioned decision and allowed said deductions for representation
expenses. Hence, this appeal by the Government.
1. TAXATION; CLAIMS FOR DEDUCTION MUST BE BASED ON RECEIPTS
ISSUED BY ENTITIES WHERE EXPENSES INCURRED NOT BY OFFICERS The alleged representation expenses are:
OF TAXPAYING CLAIMANT. The claim for deduction must be based not on
the receipts issued by the officers of the taxpaying entity but on receipts issued
by the entities where the alleged expenses had been incurred. Receipts or chits
would be issued by such entities if the expenses had actually been incurred. 1. Expenses at Elks Club P10,959.21
The receipts issued by the officers of the taxpayer merely attest to their claim
that they had incurred and paid such expenses; they do not establish payment
of said alleged expenses to the entities where the same are said to have been
incurred.

2. ID.; "BAD DEBTS" ; CRITERIA FOR ASCERTAINING WORTHLESSNESS 2. Manila Polo Club 4,947.35
OF DEBTS. The statute permits the deduction of debts "actually ascertained
to be worthless within the taxable year" obviously to prevent arbitrary action
by the taxpayer to unduly avoid tax liability. The ascertainment of
worthlessness of bad debts requires proof of two facts: (1) that the taxpayer did
in fact ascertain the debt to be worthless in the year the deduction is sought; 3. Army and Navy Club 2,812.95
and (2) in so doing, he acted in good faith. Good faith is not enough. The
taxpayer must show that he had reasonably investigated the relevant facts and
had drawn a reasonable inference from the information thus obtained by him.

3. ID., ID.; WHERE SMALL AMOUNTS ARE INVOLVED WRITING THEM 4. Manila Golf Club 4,478.45
OFF, WHEN JUSTIFIED. Considering the small amounts involved, the
taxpayer may be justified in feeling that the unsuccessful efforts therefor
exerted to collect the same would suffice to warrant their being written off. "It
is foolish to spend good money after bad."
5. Wack Wack Golf Club, Casino Espaol, etc. 6,940.92

Appeal by the Government from a decision of the Court of Tax Appeals, setting
aside the assessments made by the Commissioner of Internal Revenue, in the
sums of P14,128.00 and P8,439.00, as deficiency income taxes allegedly due from
respondent Goodrich International Rubber Company hereinafter referred to as TOTAL P30,138.88
Goodrich for the years 1951 and 1952, respectively.

These assessments were based on disallowed deductions, claimed by Goodrich,


consisting of several alleged bad debts, in the aggregate sum of P50,455.41, for the The claim for deduction thereof is based upon receipts issued, not by the entities
year 1951, and the sum of P30,138.88, as representation expenses allegedly in which the alleged expenses had been incurred, but by the officers of
incurred in the year 1952. Goodrich had appealed from said assessments to the Goodrich who allegedly paid them.
Court of Tax Appeals, which, after appropriate proceedings, rendered, on June 8,
1963, a decision allowing the deduction for bad debts, but disallowing the alleged The claim must be rejected. If the expenses had really been incurred, receipts or
representation expenses. On motion for reconsideration and new trial, filed by chits would have been issued by the entities to which the payments had been

105
TAXATION II ATTY. ACOSTA -
CAJUSTIN
made, and it would have been easy for Goodrich or its officers to produce such
receipts.lawphil These issued by said officers merely attest to their claim that
they had incurred and paid said expenses. They do not establish payment of said 8. National land Settlement Administration 3,020.76
alleged expenses to the entities in which the same are said to have been incurred.
The Court of Tax Appeals erred, therefore, in allowing the deduction thereof.

The alleged bad debts are:


9. National Coconut Corporation 644.74

1. Portillo's Auto Seat Cover P630.31


10. Interior Caltex Service Station 1,505.87

2. Visayan Rapid Transit 17,810.26


11. San Juan Auto Supply 4,530.64

3. Bataan Auto Seat Cover 373.13


12. P A C S A 45.36

4. Tres Amigos Auto Supply 1,370.31


13. Philippine Naval Patrol 14.18

5. P. C. Teodorolawphil 650.00
14. Surplus Property Commission 277.68

6. Ordnance Service, P.A. 386.42


15. Alverez Auto Supply 285.62

7. Ordnance Service, P.C. 796.26


16. Lion Shoe Store 1,686.93

106
TAXATION II ATTY. ACOSTA -
CAJUSTIN
This is the balance of a debt of P474.13 contracted in 1949. In 1951, the debtor
paid P100.00. That same year, the balance of P373.13 was charged off as bad debt.
The next year, the debtor paid the additional sum of P50.00.

Tres Amigos Auto Supply (P1,370.31):

This account had been outstanding since 1949. Counsel for the taxpayer had
17. Ruiz Highway Transit 2,350.00 merely sent demand letters (Exh. B-13) without success.

P. C. Teodoro (P650.00):

18. Esquire Auto Seat Cover 3,536.94 In 1949, the account was P751.91. In 1951, the debtor paid P101.91, thus leaving a
balance of P650.00, which the taxpayer charged off as bad debt in the same year.
In 1952, the debtor made another payment of P150.00.

Ordinance Service, P.A. (P386.42):


TOTAL P50,455.41*
In 1949, the outstanding account of this government agency was P817.55.
Goodrich's counsel sent demand letters (Exh. B-8). In 1951, it paid Goodrich
P431.13. The balance of P386.42 was written off as bad debt that same year.
The issue, in connection with these debts is whether or not the same had been
properly deducted as bad debts for the year 1951. In this connection, we find: Ordinance Service, P.C. (P796.26):

Portillo's Auto Seat Cover (P730.00): In 1950, the account was P796.26.lawphil It was referred to counsel for collection.
In 1951, the account was written off as a debt. In 1952, the debtor paid it in full.
This debt was incurred in 1950. In 1951, the debtor paid P70.00, leaving a balance
of P630.31. That same year, the account was written off as bad debt (Exhibit 3-C- National Land Settlement Administration (P3,020.76):
4). Counsel for Goodrich had merely sent two (2) letters of demand in 1951 (Exh.
B-14). In 1952, the debtor paid the full balance (Exhibit A).
The outstanding account in 1949 was P7,041.51. Collection letters were sent (Exh.
B-7). In 1951, the debtor paid P4,020.75, leaving a balance of P3,020.76, which
Visayan Rapid Transit (P17,810.26): was written off, that same year, as a bad debt. This office was under liquidation,
and its Board of Liquidators promised to pay when funds shall become available.
This debt was, also, incurred in 1950. In 1951, it was charged off as bad debt, after
the debtor had paid P275.21. No other payment had been National Coconut Corporation (P644.74):
made.lawphil Taxpayer's Accountant testified that, according to its branch
manager in Cebu, he had been unable to collect the balance. The debtor had
merely promised and kept on promising to pay. Taxpayer's counsel stated that the This account had been outstanding since 1949. Collection letters were sent (Exh.
debtor had gone out of business and became insolvent, but no proof to this effect. B-12) without success. It was written off as bad debt in 1951, while the corporation
was introduced. was under a Board of Liquidators, which promised to pay upon availability of
funds. In 1961, the debt was fully paid.

Bataan Auto Seat Cover (P373.13):

107
TAXATION II ATTY. ACOSTA -
CAJUSTIN
Interior Caltex Service Station (P1,505.87): This account was contracted in 1950. Referred, for collection, to respondent's
counsel, the latter secured no payment. In November, 1950, the corresponding
The original account was P2,705.87, when, in 1950, it was turned over for suit for collection was filed (Exh. C). The debtor's counsel was allowed to
collection to counsel for Goodrich (p. 156, CTA Records). Counsel began sending withdraw, as such, the debtor having failed to meet him. In fact, the debtor did
letters of collection in April 1950. Interior Caltex made partial payments, so that not appear at the hearing of the case.lawphil.net Judgment was rendered in 1951
as of December, 1951, the balance outstanding was P1,505.87.lawphil.net The for the creditor (Exh. C-2). The corresponding writ of execution (Exh. C-3) was
debtor paid P200, in 1952; P113.20, in 1954; P750.00, in 1961; and P300.00.00 in returned unsatisfied, for no properties could be attached or levied upon.
1962. The account had been written off as bad debt in 1951.

The claim for deduction of these ten (10) debts should be rejected. Goodrich has
not established either that the debts are actually worthless or that it had PACSA (P45.36),
reasonable grounds to believe them to be so in 1951. Our statute permits the
deduction of debts "actually ascertained to be worthless within the taxable year,"
obviously to prevent arbitrary action by the taxpayer, to unduly avoid tax liability.

Philippine Naval Patrol (P14.18),


The requirement of ascertainment of worthlessness requires proof of two facts: (1)
that the taxpayer did in fact ascertain the debt to be worthlessness, in the year for
which the deduction is sought; and (2) that, in so doing, he acted in good faith. 1

Good faith on the part of the taxpayer is not enough. He must show, also, that he Surplus Property Commission (P277.68),
had reasonably investigated the relevant facts and had drawn a reasonable
inference from the information thus obtained by him. 2 Respondent herein has not
adequately made such showing.

The payments made, some in full, after some of the foregoing accounts had been Alvarez Auto Supply (P285.62):
characterized as bad debts, merely stresses the undue haste with which the same
had been written off. At any rate, respondent has not proven that said debts were
worthless. There is no evidence that the debtors can not pay them.lawphil.net It
should be noted also that, in violation of Revenue Regulations No. 2, Section 102, These four (4) accounts were 2 or 3 years old in 1951. After the collectors of the
respondent had not attached to its income tax returns a statement showing the creditor had failed to collect the same, its counsel wrote letters of demand (Exhs.
propriety of the deductions therein made for alleged bad debts. B-10, B-11, B-6 and B-2) to no avail. Considering the small amounts involved in
these accounts, the taxpayer was justified in feeling that the unsuccessful efforts
Upon the other hand, we find that the following accounts were properly written therefore exerted to collect the same sufficed to warrant their being written off. 3
off:

San Juan Auto Supply (P4,530.64):


Lion Shoe Store (P11,686.93),

Ruiz Highway Transit (P2,350.00), and

108
TAXATION II ATTY. ACOSTA -
CAJUSTIN
COMMISSIONER OF INTERNAL REVENUE, Petitioner, v.
FERNANDEZ HERMANOS, INC., and COURT OF TAX
APPEALS, Respondents.

[G.R. No. L-24972. September 30, 1969.]

COMMISSIONER OF INTERNAL REVENUE, Petitioner, v.


FERNANDEZ HERMANOS, INC., and the COURT OF TAX
Esquire Auto Seat Cover (P3,536.94):
APPEALS, Respondents.

[G.R. No. L-24978. September 30, 1969.]

These three (3) accounts were among those referred to counsel for Goodrich for FERNANDEZ HERMANOS, INC., Petitioner, v. THE COMMISSIONER
collection. Up to 1951, when they were written off, counsel had sent 17 Letters of OF INTERNAL REVENUE, and HON. ROMAN A. UMALI, COURT OF
demand to Lion Shoe Store (Exh. B); 16 demand letters to Ruiz Highway Transit TAX APPEALS, Respondents.
(Exh. B-1); and 6 letters of demand to Esquire Auto Seat Cover (Exit. B-5) In 1951,
Lion Shoe Store, Ruiz Highway Transit, and Esquire Auto Seat Cover had made L-21551
partial payments in the sums of P1,050.00, P400.00, and P300.00 respectively.
Subsequent to the write-off, additional small payments were made and accounted Rafael Dinglasan for Petitioner.
for as income of Goodrich. Counsel interviewed the debtors, investigated their
ability to pay and threatened law suits. He found that the debtors were in strained Solicitor General Arturo A. Alafriz, Solicitor Alejandro B. Afurong and
financial condition and had no attachable or leviable property. Moreover, Lion Special Attorney Virgilio G. Saldajeno for Respondent.
Shoe Store was burned twice, in 1948 and 1949. Thereafter, it continued to do
business on limited scale. Later; it went out of business. Ruiz Highway Transit, L-21557:chanrob1es virtual 1aw library
had more debts than assets. Counsel, therefore, advised respondent to write off
these accounts as bad debts without going to court, for it would be "foolish to Solicitor General for Petitioner.
spend good money after bad."
Rafael Dinglasan for respondent Fernandez Hermanos, Inc.
The deduction of these eight (8) accounts, aggregating P22,627.35, as bad debts
should be allowed. L-24972:chanrob1es virtual 1aw library

Solicitor General Antonio P. Barredo, Assistant Solicitor General


WHEREFORE, the decision appealed from should be, as it is hereby, modified, in Felicisimo R. Rosete and Special Attorney Virgilio G. Saldajeno
the sense that respondent's alleged representation expenses are totally for Petitioner.
disallowed, and its claim for bad debts allowed up to the sum of P22,627.35 only.
Without special pronouncement as to costs. It is so ordered. Rafael Dinglasan for respondent Fernandez Hermanos, Inc.

L-24978:chanrob1es virtual 1aw library


[G.R. No. L-21551. September 30, 1969.]
Rafael Dinglasan for Petitioner.
FERNANDEZ HERMANOS, INC., Petitioner, v. COMMISSIONER OF
INTERNAL REVENUE and COURT OF TAX APPEALS., Respondents. Solicitor General Antonio P. Barredo, Assistant Solicitor General
Antonio G. Ibarra and Special Attorney Virgilio G. Saldajeno
[G.R. No. L-21557. September 30, 1969.] for Respondent.

109
TAXATION II ATTY. ACOSTA -
CAJUSTIN
neither under Section 30(d)(c) of our Tax Code providing for deduction by
corporations of losses actually sustained and charged off during the taxable
SYLLABUS year nor under Section 30 (e)(1) thereof providing for deduction of bad debts
actually ascertained to be worthless and charged off within the taxable year,
can there be a partial writing-off of a loss or bad debt, as was sought to be done
here by the taxpayer. For such losses or bad debts must be ascertained to be so
1. TAXATION; NATIONAL INTERNAL REVENUE CODE; INCOME TAX and written-off during the taxable year, are therefore deductible in full or not
ALLOWABLE DEDUCTIONS; WORTHLESS SECURITIES; WRITING-OFF at all, in the absence of any express provision in the Tax Code authorizing
PROPER IN INSTANT CASE. We find no reason to disturb the Tax Courts partial deductions.
allowance of the taxpayers writing-off as worthless securities in its 1950 return
the sum of P8,050.00 representing the cost of shares of stock of Mati Lumber 4. ID.; ID.; ID.; ID.; LOSSES; DISALLOWANCE THEREOF PROPER IN
Co. acquired by the taxpayer on January 1, 1948. There was adequate basis for INSTANT CASE. The Court sustains the Tax Courts disallowance of the
the writing off of the stock as worthless securities. As found by the Tax Court, sums of P8,989.76 and P27,732.66 spent by the taxpayer for the operation of
the Mati Lumber Co. ceased operation in 1949 when its manager and owner its Balamban coal mine in Cebu in 1950 and 1951, respectively, ad claimed as
left for Spain where he subsequently died. When the company ceased to losses in the taxpayers returns for said years. The Tax Court correctly held that
operate, it had no assets. Assuming that the Company would later somehow the losses "are deductible in 1952, when the mines were abandoned, and not in
realize some proceeds from its sawmill and equipment, which were still 1950 and 1951, when they were still in operation." The taxpayers claim that
existing as claimed by the Commissioner, and that such proceeds would later these expenditures should be allowed as losses for the corresponding years that
be distributed to its stockholders such as the taxpayer, the amount so received they were incurred, because it made no sales of coal during said years, since
by the taxpayer would then properly be reportable as income of the taxpayer the promised road or outlet through which the coal could be transported from
on the year it is received. the mines to the provincial road was not constructed, cannot be sustained.
Some definite event must fix the time when the loss is sustained, and there it
2. ID.; ID.; ID.; ID.; BAD DEBT, WHEN CONSIDERED. The Tax Courts was the event of actual abandonment of the mines in 1952.
disallowance of the writing-off in 1951 as a loss or bad debt the sum of
P353,134.25, which it had advanced or loaned to Palawan Manganese Mines, 5. ID.; ID.; ID.; ID.; LOSSES BY INVENTORY METHOD; DISALLOWANCE
Inc., was proper. The Solicitor General has rightly pointed out that the OF LOSSES THEREOF, NOT PROPER. Where respondent Commissioner
taxpayer has taken an "ambiguous position" and "has not definitely taken a concedes that under Section 100 of Revenue Regulations No. 2, it does not
stand on whether the amount involved is claimed as losses or as bad debts but specify how the inventories are to be made and the Tax Court is satisfied with
insists that it is either a loss or a bad debt. "We sustain the governments the evidence presented by the taxpayer ... which merely consisted of an alleged
position that the advances made by the taxpayer to its 100% subsidiary, physical count of the number of the livestock in Hacienda Dalupiri for the
Palawan Manganese Mines, Inc. amounting to P587,308.07 as of 1951 were years involved and the method adopted by the taxpayer as a farmer breeding
investments and not loans. The evidence on record shows that the board of livestock, reporting on the basis of receipts and disbursements, there is no
directors of the two companies since August, 1945 were identical and that the compelling reason to disturb the ruling of the Tax Court overruling the
only capital of Palawan Manganese Mines, Inc. is the amount of P100,000.00 Commissioners disallowance of losses in Hacienda Dalupiri (1950 to 1954)
entered in the taxpayers balance sheet as its investment in its subsidiary and Hacienda Samal (1951-1952).
company. This fact explains the liberality with which the taxpayer made such
large advances to the subsidiary, despite the latters admittedly poor financial 6. ID.; ID.; ID.; ID.; DEPRECIATION OF BUILDINGS; ANNUAL RATE OF
condition. 10% DEPRECIATION, EXCESSIVE; DISALLOWANCE PROPER. During
the year 1950 to 1954, the taxpayer claimed a depreciation allowance for its
3. ID.; ID.; ID.; ID.; DEBT OR INVESTMENT OF CORPORATION NOT buildings at the annual rate of 10%. The Commissioner claimed that the
WORTHLESS IF CORPORATION IS STILL IN OPERATION. The Tax Court reasonable depreciation rate is only 3% annually. We sustain the Tax Courts
correctly held that the subsidiary company was still in operation in 1951 and finding that the taxpayer did not submit adequate proof of the correctness of
1952 and the taxpayer continued to give it advances in those years, and the taxpayers claim that the depreciable assets or buildings in question had a
therefore, the alleged debt or investment could not properly be considered useful life only of 10 years so as to justify its 10% depreciation per annum
worthless and deductible in 1951, as claimed by the taxpayer. Furthermore, claim, such finding being supported by the record. The taxpayers contention

110
TAXATION II ATTY. ACOSTA -
CAJUSTIN
that it has many zero or one-peso assets, support the Commissioners position
that a 10% annual depreciation rate was excessive.

7. ID.; ID.; ID.; INCREASE IN NET WORTH NOT TAXABLE IF NOT DUE TO
TAXABLE RECEIPT. Where it is shown that the increase in the taxpayers
net worth were not the result of the receipt by it of unreported or unexplained
taxable income but were merely the result of the correction of errors in its
entries in its books relating to its debtedness to certain creditors, which had
been erroneously overstated or listed as outstanding when they had in fact duly
paid, these increase in the taxpayers net worth were not taxable increases in
net worth.

8. ID.; ID.; ID. ALLEGED UNREPORTED GAIN FROM SALE OF REAL


PROPERTY, NO BASIS. Where it was sufficiently proved from the taxpayers
books that after acquiring the property in 1926 for P11,852.74, the gain derived
from the sale of the said property for P60,000.00 was correctly reported by the
taxpayer at P37,000.00.

9. ID.; ID.; ID.; CAPITAL INVESTMENT, NOT BASIS FOR DEPLETION.


The taxpayer insists in this appeal that it could use as a method for depletion
under the pertinent provision of the Tax Code its "capital investment"
representing the alleged value of its contractual rights and titles to mining
claims in the sum of P242,408.10 and thus deduct outright one-fifth (1/5) of
this "Capital investment" ever year, regardless of whether it had actually mined
the product and sold the products. HELD: The alleged "capital investment"
method invoked by the taxpayer is not a method of depletion, but the Tax Code
provision, prior to its amendment by Section 1 of Republic Act No. 2698, which
took effect on June 18, 1960, expressly provided that "when the allowances
shall equal capital invested . . . no further allowances shall be made;" in other
words, the "capital investment" was but the limitation of the amount of
depletion that could be claimed. The outright deduction by the taxpayer of 1/5
of the cost of the mines, as if it were a "straight line" rate of depreciations was
correctly held by the Tax Court not to be authorized by the Tax Code.

10. ID.; ID.; ASSESSMENT AND COLLECTION OF INCOME TAX;


PRESCRIPTION; ACTION FOR COLLECTION IN INSTANT CASE HAS NOT
PRESCRIBED. In the present case, regardless of whether the assessments
were made on February 24 and 27, 1956, as claimed by the Commissioner, or
on December 27, 1955 as claimed by the taxpayer, the governments right to
collect the taxes due has clearly not prescribed, as the taxpayers appeal or
petition for review was filed with the Tax Court on May 4, 1960, with the
Commissioner filing on May 20, 1960 his answer with a prayer for payment of
the taxes due, long before the expiration of the five-year period to effect
collection by judicial action counted from the date of assessment

111
TAXATION II ATTY. ACOSTA -
CAJUSTIN
the right of a company to make such a provision, but it is its duty to its bond
and stockholders, and, in the case of a public service corporation, at least, its
plain duty to the public (Knoxville v. Knoxville Water Co., 212 U.S. 1, 53 L. Ed.
371). Accordingly, the law permits the taxpayer to recover gradually his capital
TOPIC: DEDUCTIONS investment in wasting assets free from income tax (Detroit Edison Co. v.
Commissioner, 131 F 2d. 619). Precisely, Section 30(f) (1) of the Tax Code
DEPRECIATION allows a deduction from gross income for depreciation but limits the recovery
to the capital invested in the asset being depreciated.

4. ID.; BASIS OF DEPRECIATION. The income tax law does not authorize
[G.R. No. L-22492. September 5, 1967.] the depreciation of an asset beyond its acquisition cost. Hence, a deduction
over and above such cost cannot be claimed and allowed. The reason is that
BASILAN ESTATES, INC., Petitioner, v. THE COMMISSIONER OF deduction from gross income are privileges (Palmer v. State Commission of
INTERNAL REVENUE and THE COURT OF TAX Revenue & Taxation, 156 Kan. 690, 135 P. 2d. 899), not matters of right
APPEALS, Respondents. (Souther Weaving Co. v. Query, 206 SC 307, 34 SE 2d. 51). They are not
created by implication but upon clear expression in the law (Gutierrez v.
Felix A. Gulfin and Antonio S. Alano for Petitioner. Collector of Internal Revenue, L-19537, May 20, 1965). Moreover, the recovery,
free of income tax, of an amount more than the invested capital in an asset will
The Solicitor General for Respondents. transgress the underlying purpose of a depreciation allowance. For then what
the taxpayer would recover will be, not only the acquisition cost, but also some
profit. Recovery in due time through depreciation of investment made is the
SYLLABUS philosophy behind depreciation allowance; the idea of profit on the investment
made has never been the underlying reason for the allowance of a deduction
for depreciation.
1. NOTICE OF ASSESSMENT, WHEN DEEMED MADE. Under Section 331
5. ID.; TRAVELING EXPENSES; PERIOD WITHIN WHICH TO KEEP
of the Tax Code requiring 5 years within which to assess deficiency taxes, the
SUPPORTING PAPERS; CASE AT BAR. Under Section 337 of the National
assessment is deemed made when notice to this effect is released, mailed or
Internal Revenue Code, receipts and papers supporting traveling expenses
sent by the Collector of Internal Revenue to the taxpayer, and it is not required
need be kept by the taxpayer for a period of five years from the last entry.
that the notice be received by the taxpayer within the aforementioned 5-year
period (Collector of Internal Revenue v. Bautista, L-12250 & L-12259, May 27,
6. ID.; SURTAX ON UNREASONABLY ACCUMULATED PROFITS; TEST TO
1959)
DETERMINE REASONABLENESS ACCUMULATION OF PROFITS.
Persuasive jurisprudence on the matter such as those in the United States from
2. ID.; DEPRECIATION; DEFINITION. Depreciation is the gradual
where our tax law was deprived (Collector of Internal Revenue v. Binalbagan
diminution in the useful value of tangible property resulting from wear and
Estate, Inc., L-12752, Jan. 30, 1965), has it that: "In order to determine
tear and normal obsolescence. The term is also applied to amortization of the
whether profits were accumulated for the reasonable needs of the business or
value of intangible assets, the use of which in the trade or business is definitely
to avoid the surtax upon shareholders, the controlling intention of the taxpayer
limited in duration (Jose Aranas, Annotation and Jurisprudence on the
is that which is manifested at the time of the accumulation, not subsequently
National Internal Revenue Code, as Amended, 2nd Ed., Vol. 1, p. 263).
declared intentions which are merely the products of afterthought (Jacob
Mertens, Jr., The Law of Federal Income Taxation, Vol. 7, Cumulative
3. ID.; ID.; WHEN DEPRECIATION COMMENCES. Depreciation
Supplement, p. 213). In determining whether accumulations of earnings or
commences with the acquisition of the property and its owner is not bound to
profits in a particular year are within the reasonable needs of a corporation, it
see his property gradually waste, without making provision out of earnings for
is necessary to take unto account prior accumulations, since accumulations
its replacement. It is entitled to see that from earnings the value of the
prior to the year involved may have been sufficient to cover the business needs
property invested is kept unimpaired, so that at the end of any given term of
and additional accumulations during the year involved would not reasonably
years, the original investment remains as it was in the beginning. It is not only

112
TAXATION II ATTY. ACOSTA -
CAJUSTIN
be necessary. (Ibid, p. 202). assessment or if it did, it received the notice beyond the five-year prescriptive
A Philippine corporation engaged in the coconut industry, Basilan Estates, Inc., period. To show prescription, the annotation on the notice (Exhibit 10, No. 52,
with principal offices in Basilan City, filed on March 24, 1954 its income tax ACR, p. 54-A of the BIR records) "No accompanying letter 11/25/" is advanced as
returns for 1953 and paid an income tax of P8,028. On February 26, 1959, the indicative of the fact that receipt of the notice was after March 24, 1959, the last
Commissioner of Internal Revenue, per examiners' report of February 19, 1959, date of the five-year period within which to assess deficiency tax, since the
assessed Basilan Estates, Inc., a deficiency income tax of P3,912 for 1953 and original returns were filed on March 24, 1954.
P86,876.85 as 25% surtax on unreasonably accumulated profits as of 1953
pursuant to Section 25 of the Tax Code. On non-payment of the assessed amount, Although the evidence is not clear on this point, We cannot accept this
a warrant of distraint and levy was issued but the same was not executed because interpretation of the petitioner, considering the presence of circumstances that
Basilan Estates, Inc. succeeded in getting the Deputy Commissioner of Internal lead Us to presume regularity in the performance of official functions. The notice
Revenue to order the Director of the district in Zamboanga City to hold execution of assessment shows the assessment to have been made on February 26, 1959,
and maintain constructive embargo instead. Because of its refusal to waive the well within the five-year period. On the right side of the notice is also stamped
period of prescription, the corporation's request for reinvestigation was not given "Feb. 26, 1959" denoting the date of release, according to Bureau of Internal
due course, and on December 2, 1960, notice was served the corporation that the Revenue practice. The Commissioner himself in his letter (Exh. H, p. 84 of BIR
warrant of distraint and levy would be executed. records) answering petitioner's request to lift, the warrant of distraint and levy,
asserts that notice had been sent to petitioner. In the letter of the Regional
On December 20, 1960, Basilan Estates, Inc. filed before the Court of Tax Appeals Director forwarding the case to the Chief of the Investigation Division which the
a petition for review of the Commissioner's assessment, alleging prescription of latter received on March 10, 1959 (p. 71 of the BIR records), notice of assessment
the period for assessment and collection; error in disallowing claimed was said to have been sent to petitioner. Subsequently, the Chief of the
depreciations, travelling and miscellaneous expenses; and error in finding the Investigation Division indorsed on March 18, 1959 (p. 24 of the BIR records) the
existence of unreasonably accumulated profits and the imposition of 25% surtax case to the Chief of the Law Division. There it was alleged that notice was already
thereon. On October 31, 1963, the Court of Tax Appeals found that there was no sent to petitioner on February 26, 1959. These circumstances pointing to official
prescription and affirmed the deficiency assessment in toto. performance of duty must necessarily prevail over petitioner's contrary
interpretation. Besides, even granting that notice had been received by the
petitioner late, as alleged, under Section 331 of the Tax Code requiring five years
On February 21, 1964, the case was appealed to Us by the taxpayer, upon the
within which to assess deficiency taxes, the assessment is deemed made when
following issues:
notice to this effect is released, mailed or sent by the Collector to the taxpayer and
it is not required that the notice be received by the taxpayer within the
1. Has the Commissioner's right to collect deficiency income tax prescribed? aforementioned five-year period.1

2. Was the disallowance of items claimed as deductible proper? ASSESSMENT

3. Have there been unreasonably accumulated profits? If so, should the 25% The questioned assessment is as follows:
surtax be imposed on the balance of the entire surplus from 1947-1953, or only for
1953?
Net Income
P40,142.90
per return
4. Is the petitioner exempt from the penalty tax under Republic Act 1823 Add: Over-claimed
amending Section 25 of the Tax Code? P10,500.49
depreciation
Mis. expenses
PRESCRIPTION 6,759.17
disallowed
Officer's travelling
There is no dispute that the assessment of the deficiency tax was made on expenses disallowed 2,300.40 19,560.06
February 26, 1959; but the petitioner claims that it never received notice of such

113
TAXATION II ATTY. ACOSTA -
CAJUSTIN
Net Income per The question for resolution therefore is whether depreciation shall be determined
P59,702.96 on the acquisition cost or on the reappraised value of the assets.
Investigation
20% tax on P59,702.96 11,940.00
Less: Tax already 8,028.00 Depreciation is the gradual diminution in the useful value of tangible property
assessed resulting from wear and tear and normal obsolescense. The term is also applied to
amortization of the value of intangible assets, the use of which in the trade or
Deficiency income tax P3,912.00
business is definitely limited in duration. 2 Depreciation commences with the
Add: Additional tax of acquisition of the property and its owner is not bound to see his property
86,876.75
25% on P347,507.01 gradually waste, without making provision out of earnings for its replacement. It
is entitled to see that from earnings the value of the property invested is kept
Tax Due & Collectible P90,788.75 unimpaired, so that at the end of any given term of years, the original investment
========= remains as it was in the beginning. It is not only the right of a company to make
The Commissioner disallowed: such a provision, but it is its duty to its bond and stockholders, and, in the case of
a public service corporation, at least, its plain duty to the public. 3 Accordingly, the
law permits the taxpayer to recover gradually his capital investment in wasting
Over-claimed depreciation P10,500.49 assets free from income tax.4 Precisely, Section 30 (f) (1) which states:
Miscellaneous expenses 6,759.17
Officer's travelling expenses 2,300.40 (1)In general. A reasonable allowance for deterioration of property
DEDUCTIONS arising out of its use or employment in the business or trade, or out of its
not being used: Provided, That when the allowance authorized under
A. Depreciation. Basilan Estates, Inc. claimed deductions for the depreciation this subsection shall equal the capital invested by the taxpayer . . . no
of its assets up to 1949 on the basis of their acquisition cost. As of January 1, 1950 further allowance shall be made. . . .
it changed the depreciable value of said assets by increasing it to conform with the
increase in cost for their replacement. Accordingly, from 1950 to 1953 it deducted allows a deduction from gross income for depreciation but limits the recovery to
from gross income the value of depreciation computed on the reappraised value. the capital invested in the asset being depreciated.

In 1953, the year involved in this case, taxpayer claimed the following The income tax law does not authorize the depreciation of an asset beyond its
depreciation deduction: acquisition cost. Hence, a deduction over and above such cost cannot be claimed
and allowed. The reason is that deductions from gross income are privileges, 5 not
Reappraised assets P47,342.53 matters of right.6 They are not created by implication but upon clear expression in
the law.7
New assets consisting of hospital building and equipment 3,910.45
Total depreciation
Moreover, the recovery, free of income tax, of an amount more than the invested
P51,252.98 capital in an asset will transgress the underlying purpose of a depreciation
Upon investigation and examination of taxpayer's books and papers, the allowance. For then what the taxpayer would recover will be, not only the
Commissioner of Internal Revenue found that the reappraised assets depreciated acquisition cost, but also some profit. Recovery in due time thru depreciation of
in 1953 were the same ones upon which depreciation was claimed in 1952. And for investment made is the philosophy behind depreciation allowance; the idea of
the year 1952, the Commissioner had already determined, with taxpayer's profit on the investment made has never been the underlying reason for the
concurrence, the depreciation allowable on said assets to be P36,842.04, allowance of a deduction for depreciation.
computed on their acquisition cost at rates fixed by the taxpayer. Hence, the
Commissioner pegged the deductible depreciation for 1953 on the same old assets
Accordingly, the claim for depreciation beyond P36,842.04 or in the amount of
at P36,842.04 and disallowed the excess thereof in the amount of P10,500.49.
P10,500.49 has no justification in the law. The determination, therefore, of the

114
TAXATION II ATTY. ACOSTA -
CAJUSTIN
Commissioner of Internal Revenue disallowing said amount, affirmed by the of permitting its gains and profits to accumulate instead of being divided
Court of Tax Appeals, is sustained. or distributed, there is levied and assessed against such corporation, for
each taxable year, a tax equal to twenty-five per centum of the
B. Expenses. The next item involves disallowed expenses incurred in 1953, undistributed portion of its accumulated profits or surplus which shall
broken as follows: be in addition to the tax imposed by section twenty-four, and shall be
computed, collected and paid in the same manner and subject to the
same provisions of law, including penalties, as that tax.1awphl.nt
Miscellaneous expenses P6,759.17
Officer's travelling expenses 2,300.40 The Commissioner found that in violation of the abovequoted section, petitioner
Total had unreasonably accumulated profits as of 1953 in the amount of P347,507.01,
P9,059.57 based on the following circumstances (Examiner's Report pp. 62-68 of BIR
These were disallowed on the ground that the nature of these expenses could not records):
be satisfactorily explained nor could the same be supported by appropriate
papers. 1. Strong financial position of the petitioner as of December 31, 1953.
Assets were P388,617.00 while the liabilities amounted to only
Felix Gulfin, petitioner's accountant, explained the P6,759.17 was actual expenses P61,117.31 or a ratio of 6:1.
credited to the account of the president of the corporation incurred in the interest
of the corporation during the president's trip to Manila (pp. 33-34 of TSN of Dec. 2. As of 1953, the corporation had considerable capital adequate to meet
5, 1962); he stated that the P2,300.40 was the president's travelling expenses to the reasonable needs of the business amounting to P327,499.69 (assets
and from Manila as to the vouchers and receipts of these, he said the same were less liabilities).
made but got burned during the Basilan fire on March 30, 1962 (p. 40 of same
TSN). Petitioner further argues that when it sent its records to Manila in
February, 1959, the papers in support of these miscellaneous and travelling 3. The P200,000 reserved for electrification of drier and mechanization
expenses were not included for the reason that by February 9, 1959, when the and the P50,000 reserved for malaria control were reverted to its
Bureau of Internal Revenue decided to investigate, petitioner had no more surplus in 1953.
obligation to keep the same since five years had lapsed from the time these
expenses were incurred (p. 41 of same TSN). On this ground, the petitioner may 4. Withdrawal by shareholders, of large sums of money as personal
be sustained, for under Section 337 of the Tax Code, receipts and papers loans.
supporting such expenses need be kept by the taxpayer for a period of five years
from the last entry. At the time of the investigation, said five years had lapsed. 5. Investment of undistributed earnings in assets having no proximate
Taxpayer's stand on this issue is therefore sustained. connection with the business as hospital building and equipment
worth P59,794.72.
UNREASONABLY ACCUMULATED PROFITS
6. In 1953, with an increase of surplus amounting to P677,232.01, the
Section 25 of the Tax Code which imposes a surtax on profits unreasonably capital stock was increased to P500,000 although there was no need for
accumulated, provides: such increase.

Sec. 25. Additional tax on corporations improperly accumulating Petitioner tried to show that in considering the surplus, the examiner did not take
profits or surplus (a) Imposition of tax. If any corporation, except into account the possible expenses for cultivation, labor, fertilitation, drainage,
banks, insurance companies, or personal holding companies, whether irrigation, repair, etc. (pp. 235-237 of TSN of Dec. 7, 1962). As aptly answered by
domestic or foreign, is formed or availed of for the purpose of preventing the examiner himself, however, they were already included as part of the working
the imposition of the tax upon its shareholders or members or the capital (pp. 237-238 of TSN of Dec. 7, 1962).
shareholders or members of another corporation, through the medium

115
TAXATION II ATTY. ACOSTA -
CAJUSTIN
In the unreasonable accumulation of P347,507.01 are included P200,000 for Petitioner questions why the examiner covered the period from 1948-1953 when
electrification of driers and mechanization and P50,000 for malaria control which the taxable year on review was 1953. The surplus of P347,507.01 was taken by the
were reserved way back in 1948 (p. 67 of the BIR records) but reverted to the examiner from the balance sheet of petitioner for 1953. To check the figure
general fund only in 1953. If there were any plans for these amounts to be used in arrived at, the examiner traced the accumulation process from 1947 until 1953,
further expansion through projects, it did not appear in the records as was and petitioner's figure stood out to be correct. There was no error in the process
properly indicated in 1948 when such amounts were reserved. Thus, while in 1948 applied, for previous accumulations should be considered in determining
it was already clear that the money was intended to go to future projects, in 1953 unreasonable accumulations for the year concerned. "In determining whether
upon reversion to the general fund, no such intention was shown. Such reversion accumulations of earnings or profits in a particular year are within the reasonable
therefore gave occasion for the Government to consider the same for tax needs of a corporation, it is neccessary to take into account prior accumulations,
purposes. The P250,000 reverted to the general fund was sought to be explained since accumulations prior to the year involved may have been sufficient to cover
as later used elsewhere: "part of it in the Hilano Industries, Inc. in building the the business needs and additional accumulations during the year involved would
factory site and buildings to house technical men . . . part of it was spent in the not reasonably be necessary."12
facilities for the waterworks system and for industrialization of the coconut
industry" (p. 117 of TSN of Dec. 6, 1962). This is not sufficient explanation. Another factor that stands out to show unreasonable accumulation is the fact that
Persuasive jurisprudence on the matter such as those in the United States from large amounts were withdrawn by or advanced to the stockholders. For the year
where our tax law was derived,8 has it that: "In order to determine whether profits 1953 alone these totalled P197,229.26. Yet the surplus of P347,507.01 was left as
were accumulated for the reasonable needs of the business or to avoid the surtax of December 31, 1953. We find unacceptable petitioner's explanation that these
upon shareholders, the controlling intention of the taxpayer is that which is were advances made in furtherance of the business purposes of the petitioner. As
manifested at the time of the accumulation, not subsequently declared intentions correctly held by the Court of Tax Appeals, while certain expenses of the
which are merely the products of after-thought." 9 The reversion here was made corporation were credited against these amounts, the unspent balance was
because the reserved amount was not enough for the projects intended, without retained by the stockholders without refunding them to petitioner at the end of
any intent to channel the same to some particular future projects in mind. each year. These advances were in fact indirect loans to the stockholders
indicating the unreasonable accumulation of surplus beyond the needs of the
Petitioner argues that since it has P560,717.44 as its expenses for the year 1953, a business.
surplus of P347,507.01 is not unreasonably accumulated. As rightly contended by
the Government, there is no need to have such a large amount at the beginning of ALLEGED EXEMPTION
the following year because during the year, current assets are converted into cash
and with the income realized from the business as the year goes, these expenses
may well be taken care of (pp. 238 of TSN of Dec. 7, 1962). Thus, it is erroneous to Petitioner wishes to avail of the exempting proviso in Sec. 25 of the Internal
say that the taxpayer is entitled to retain enough liquid net assets in amounts Revenue Code as amended by R.A. 1823, approved June 22, 1957, whereby
approximately equal to current operating needs for the year to cover "cost of accumulated profits or surplus if invested in any dollar-producing or dollar-
goods sold and operating expenses" for "it excludes proper consideration of funds earning industry or in the purchase of bonds issued by the Central Bank, may not
generated by the collection of notes receivable as trade accounts during the course be subject to the 25% surtax. We have but to point out that the unreasonable
of the year."10 In fact, just because the fatal accumulations are less than 70% of the accumulation was in 1953. The exemption was by virtue of Republic Act 1823
annual operating expenses of the year, it does not mean that the accumulations which amended Sec. 25 only on June 22, 1957 more than three years after the
are reasonable as a matter of law." 11 period covered by the assessment.

Petitioner tried to show that investments were made with Basilan Coconut In resume, Basilan Estates, Inc. is liable for the payment of deficiency income tax
Producers Cooperative Association and Basilan Hospital (pp. 103-105 of TSN of and surtax for the year 1953 in the amount of P88,977.42, computed as follows:
Dec. 6, 1962) totalling P59,794.72 as of December 31, 1953. This shows all the
more the unreasonable accumulation. As of December 31, 1953 already Net Income per return P40,142.90
P59,794.72 was spent yet as of that date there was still a surplus of Add: Over-claimed depreciation 10,500.49
P347,507.01.
Net income per finding
P50,643.39

116
TAXATION II ATTY. ACOSTA -
CAJUSTIN
COLLECTOR OF INTERNAL
REVENUE, petitioner, vs. ESPERANZA A. ZAMORA,
20% tax on P50,643.39 P10,128.67
as Special Administratrix, etc., respondent.
Less: Tax already assessed 8,028.00

Deficiency income tax P2,100.67 There shall be allowed as deductions all the ordinary and necessary expenses
Add: 25% surtax on P347,507.01 86,876.75 paid or incurred during the taxable year, in carrying on any trade or
business. Since promotion expenses constitute one of the deductions in
Total tax due and collectible conducting a business, same must testify these requirements. Claim for the
P88,977.42
deduction of promotion expenses or entertainment expenses must also be
===========
substantiated or supported by record showing in detail the amount and
WHEREFORE, the judgment appealed from is modified to the extent that nature of the expenses incurred.
petitioner is allowed its deductions for travelling and miscellaneous expenses, but
affirmed insofar as the petitioner is liable for P2,100.67 as deficiency income tax
for 1953 and P86,876.75 as 25% surtax on the unreasonably accumulated profit of That to be deductible, said business expenses must be ordinary and necessary
P347,507.01. No costs. So ordered. expenses paid or incurred in carrying on any trade or business; that those
expenses must also meet the further test of reasonableness in amount; that
[G.R. No. L-15290. May 31, 1963.] when some of the representation expenses claimed by the taxpayer were
evidenced by vouchers or chits, but others were without vouchers or chits, the
court should determine from all available data, the amount properly
MARIANO ZAMORA, petitioner, vs. COLLECTOR OF deductible as representation expenses.
INTERNAL REVENUE and COURT OF TAX
APPEALS,respondents.
FACTS:

[G.R. No. L-15280. May 31, 1963.]


Mariano Zamora, owner of the Bay View Hotel and Farmacia Zamora,
Manila, filed his income tax returns the years 1951 and 1952. The Collector of
COLLECTOR OF INTERNAL Internal Revenue found that he failed to file his return of the capital gains
REVENUE, petitioner, vs. MARIANO derived from the sale of certain real properties and claimed deductions which
ZAMORA, respondent. were not allowable. The collector required him to pay the sums of P43,758.50
and P7,625.00, as deficiency income tax for the years 1951 and 1952.

[G.R. No. L-15289. May 31, 1963.]


On appeal by Zamora, the Court of Tax Appeals modified the decision
appealed from and ordered him to pay the reduced total sum of P30,258.00
ESPERANZA A. ZAMORA, as special Administratrix (P22,980.00 and P7,278.00, as deficiency income tax for the years 1951 and
of the Estate of FELICIDAD 1952.
ZAMORA, petitioner, vs.COLLECTOR OF INTERNAL
REVENUE and COURT OF TAX
APPEALS, respondents. Having failed to obtain a reconsideration of the decision, Mariano Zamora
appealed alleging that the Court of Tax Appeals erred (amongst other things,
this being the only relevant to the topic) in disallowing P10,478.50, as
promotion expenses incurred by his wife for the promotion of the Bay View
[G.R. No. L-15281. May 31, 1963.] Hotel and Farmacia Zamora (which is of P20,957.00, supposed business

117
TAXATION II ATTY. ACOSTA -
CAJUSTIN
expenses). shows that she went abroad on a combined medical and business trip, not all of
her expenses came under the category of ordinary and necessary expenses;
part thereof constituted her personal expenses. There having been no means by
Note: He contends that the whole amount of P20,957.00 as promotion which to ascertain which expense was incurred by her in connection with the
expenses in his 1951 income tax returns, should be allowed and not merely business of Mariano Zamora and which was incurred for her personal benefit,
one-half of it or P10,478.50, on the ground that, while not all the itemized the Collector and the CTA in their decisions, considered 50% of the said
expenses are supported by receipts, the absence of some supporting receipts amount of P20,957.00 as business expenses and the other 50%, as her
has been sufficiently and satisfactorily established. For, as alleged, the said personal expenses. We hold that said allocation is very fair to Mariano Zamora,
amount of P20,957.00 was spent by Mrs. Esperanza A. Zamora (wife of there having been no receipt whatsoever, submitted to explain the alleged
Mariano), during her travel to Japan and the United States to purchase business expenses, or proof of the connection which said expenses had to the
machinery for a new Tiki-Tiki plant, and to observe hotel management in business or the reasonableness of the said amount of P20,957.00. While in
modern hotels. The CTA, however, found that for said trip Mrs. Zamora situations like the present, absolute certainty is usually not possible, the CTA
obtained only the sum of P5,000.00 from the Central Bank and that in her should make as close an approximation as it can, bearing heavily, if it chooses,
application for dollar allocation, she stated that she was going abroad on a upon the taxpayer whose inexactness is of his own making.
combined medical and business trip, which facts were not denied by Mariano
Zamora. No evidence had been submitted as to where Mariano had obtained
the amount in excess of P5,000.00 given to his wife which she spent abroad. In the case of Visayan Cebu Terminal Co., Inc. v. Collector of Int. Rev, it was
No explanation had been made either that the statement contained in Mrs. declared that representation expenses fall under the category of business
Zamora's application for dollar allocation that she was going abroad on a expenses which are allowable deductions from gross income, if they meet the
combined medical and business trip, was not correct. The alleged expenses conditions prescribed by law, particularly section 30 (a) [1], of the Tax Code;
were not supported by receipts. Mrs. Zamora could not even remember how that to be deductible, said business expenses must be ordinary and necessary
much money she had when she left abroad in 1951, and how the alleged expenses paid or incurred in carrying on any trade or business; that those
amount of P20,957.00 was spent. expenses must also meet the further test of reasonableness in amount; that
when some of the representation expenses claimed by the taxpayer were
evidenced by vouchers or chits, but others were without vouchers or chits,
documents or supporting papers; that there is no more than oral proof to the
ISSUE: effect that payments have been made for representation expenses allegedly
made by the taxpayer and about the general nature of such alleged expenses;
Whether or not the CTA erred in disallowing P10,478.50 as promotion that accordingly, it is not possible to determine the actual amount covered by
expenses incurred by his wife for the promotion of the Bay View Hotel and supporting papers and the amount without supporting papers, the court
Farmacia Zamora in the absence of receipts proving the same. should determine from all available data, the amount properly deductible as
representation expenses.

HELD: NO
SYLLABUS

Section 30, of the Tax Code, provides that in computing net income, there
shall be allowed as deductions all the ordinary and necessary expenses paid or 1. TAXATION; INCOME TAXES; BUSINESS EXPENSES AS DEDUCTION.
incurred during the taxable year, in carrying on any trade or business. Since Promotion expenses constitute one of the deductions in conducting a business,
promotion expenses constitute one of the deductions in conducting a business, and should satisfy the requirements of Section 30 of the Tax Code, which
same must testify these requirements. Claim for the deduction of promotion provides that in computing net income, there shall be allowed as deductions all
expenses or entertainment expenses must also be substantiated or supported the ordinary and necessary expenses paid or incurred during the taxable year,
by record showing in detail the amount and nature of the expenses incurred in carrying on any trade or business (Vol. 4, Martens, Law of Federal Income
(N.H. Van Socklan, Jr. v. Comm. of Int. Rev.; 33 BTA 544). Considering, as Taxation, sec. 25.03, p. 307).
heretofore stated, that the application of Mrs. Zamora for dollar allocation
2. ID.; ID.; ID.; REQUISITES FOR DEDUCTION OF BUSINESS EXPENSES.

118
TAXATION II ATTY. ACOSTA -
CAJUSTIN
Representation expenses fall under the category of business expenses which
are allowable deductions from gross income, if they meet the conditions
prescribed by law, particularly section 30 (a) (1), of the Tax Code. To be
deductible, they must be ordinary and necessary expenses paid or incurred in
carrying on any trade or business, and should meet the further test of
reasonableness in amount. They should, moreover, be covered by supporting
paper; in the absence thereof the amount properly deductible as representation
expenses should be determined from all available data. (Visayan Cebu
Terminal Co., Inc., vs. Collector of Int. Rev., 108 Phil., 320).
3. ID.; CAPITAL GAINS TAXES; COST BASIS OF PROPERTY ACQUIRED IN
JAPANESE WAR NOTES. The cost basis of property acquired in Japanese
war notes is the equivalent of the war notes in genuine Philippine currency in
accordance with the Ballantyne Scale of values, and the determination of the
gain derived or loss sustained in the sale of such property is not affected by the
decline at the time of sale, in the purchasing power of the Philippine currency.
4. STATUTORY CONSTRUCTION; ANTECEDENTS OR LEGISLATIVE
HISTORY OF STATUTE TO BE CONSIDERED IN ITS INTERPRETATION.
Courts are permitted to look into and investigate the antecedents or the
legislative history of the statutes involved (Director of Lands vs. Abaya, et al.,
63 Phil. 559).

***Fernandez Hermanos Inc. vs CIR

119
TAXATION II ATTY. ACOSTA -
CAJUSTIN

b. Residential house and lot at Wright St., Malate, Manila


After the marriage of Antonio and Eduardo, Jose lived in the house where he
paid rentals of 8K/year to Roxas y Cia

Shares of stocks in different corporations

To manage the properties, Antonio Roxas, Eduardo Roxas and Jose Roxas,
TOPIC: DEDUCTIONS the children, formed a partnership called Roxas y Compania
On 1958, CIR demanded from Roxas y Cia the payment of real estate dealer's
CHARITABLE AND OTHER tax for 1952 amtg to P150.00 plus P10.00 compromise penalty for late
payment, and P150.00 tax for dealers of securities plus P10.00 compromise
CONTRIBUTIONS penalty for late payment.
Basis: house rentals received from Jose, pursuant to Art. 194 of the Tax Code
stating that an owner of a real estate who derives a yearly rental income
therefrom in the amount of P3,000.00 or more is considered a real estate
G.R. No. L-25043 April 26, 1968 dealer and is liable to pay the corresponding fixed tax
The Commissioner further assessed deficiency income taxes against the
ANTONIO ROXAS, EDUARDO ROXAS and ROXAS Y CIA., in their own brothers for 1953 and 1955, resulting from the inclusion as income of Roxas y
respective behalf and as judicial co-guardians of JOSE Cia of the unreported 50% of the net profits derived from the sale of the
ROXAS, petitioners, Nasugbu farm lands to the tenants, and the disallowance of deductions from
vs. COURT OF TAX APPEALS and COMMISSIONER OF INTERNAL gross income of various business expenses and contributions claimed by Roxas
REVENUE, respondents. y Cia and the Roxas brothers
The brothers protested the assessment but was denied, thus appealing to the
Facts: CTA
Don Pedro Roxas and Dona Carmen Ayala, both Spanish, transmitted to their CTA decision: sustained the assessment except the demand for the payment
grandchildren by hereditary succession the following properties: of the fixed tax on dealer of securities and the disallowance of the deductions
Agricultural lands with a total area of 19,000 hectares in Nasugbu, Batangas for contributions to the Philippine Air Force Chapel and Hijas de Jesus' Retiro
Tenants who have been tilling the lands expressed their desire to purchase de Manresa
from Roxas y Cia, the parcels which they actually occupied
The govt, in line with the constitutional mandate to acquire big landed Issue: Should Roxas y Cia be considered a real estate dealer because it
estates and apportion them among landless tenants-farmers, persuaded the engaged in the business of selling real estate
Roxas brothers to part with their landholdings
The brothers agreed to sell 13,500 hec to the govt for P2.079Mn, plus 300K Ruling: NO, being an isolated transaction
survey and subdivision expenses Real estate dealer: any person engaged in the business of buying, selling,
Unfortunately, the govt did not have funds exchanging, leasing or renting property on his own account as principal and
A special arrangement was made with the Rehabilitation Finance holding himself out as a full or part-time dealer in real estate or as an owner of
Corporation to advance to Roxas y Cia the amount of P1.5Mn as loan rental property or properties rented or offered to rent for an aggregate
Under the arrangement, Roxas y Cia. allowed the farmers to buy the lands for amount of three thousand pesos or more a year:
the same price but by installment, and contracted with the RFC to pay its loan Section 194 of the Tax Code, in considering as real estate dealers owners of
from the proceeds of the yearly amortizations paid by the farmers real estate receiving rentals of at least P3,000.00 a year, does not provide any
In 1953 and 1955, Roxas y Cia. derived from said installment payments a net qualification as to the persons paying the rentals
gain of P42,480.83 and P29,500.71. 50% of said net gain was reported for The fact that there were hundreds of vendees and them being paid for their
income tax purposes as gain on the sale of capital asset held for more than one respective holdings in installment for a period of ten years, it would
year pursuant to Sec. 34 of the Tax Code nevertheless not make the vendor Roxas y Cia. a real estate dealer during the

120
TAXATION II ATTY. ACOSTA -
CAJUSTIN
10-year amortization period d. Contribution to Our Lady of Fatima chapel at the FEU
the sale of the Nasugbu farm lands to the very farmers who tilled them for University gives dividends to its stockholders
generations was not only in consonance with, but more in obedience to the Located within the premises of the university, the chapel in question has not
request and pursuant to the policy of our Government to allocate lands to the been shown to belong to the Catholic Church or any religious organization
landless The contributions belongs to the Far Eastern University, contributions to
It was the duty of the Government to pay the agreed compensation after it had which are not deductible under Section 30(h) of the Tax Code for the reason
persuaded Roxas y Cia. to sell its haciendas, and to subsequently subdivide that the net income of said university injures to the benefit of its stockholders
them among the farmers at very reasonable terms and prices. But due to the
lack of funds, Roxas y Cia. shouldered the Government's burden, went out of
its way and sold lands directly to the farmers in the same way and under the SYLLABUS
same terms as would have been the case had the Government done it itself
The power of taxation is sometimes called also the power to destroy.
Therefore it should be exercised with caution to minimize injury to the 1. TAXATION; "POWER TO DESTROY", TO BE EXERCISED FAIRLY,
proprietary rights of a taxpayer. It must be exercised fairly, equally and EQUALLY AND UNIFORMLY. The power of taxation is sometimes called
uniformly also the power to destroy. It should, therefore, be exercised with caution to
Therefore, Roxas y Cia. cannot be considered a real estate dealer for the sale minimize injury to the proprietary rights of a taxpayer. It must be exercised
in question. Hence, pursuant to Section 34 of the Tax Code the lands sold to fairly, equally and uniformly, lest the tax collector kill the "hen that lays the
the farmers are capital assets, and the gain derived from the sale thereof is golden egg".
capital gain, taxable only to the extent of 50%
2. ID.; REAL ESTATE DEALER'S TAX, HELD INAPPLICABLE. Even where
As to the deductions there were hundreds of vendees who paid for their respective holdings in
P40 tickets to a banquet given in honor of Sergio Osmena and P28 San installment for 10 years, such fact did not make the act of subdividing the
Miguel beer given as gifts to various persons representation expenses Nasugbu farm and selling them to the farmers-occupants thereof on
Representation expenses: deductible from gross income as expenditures installment basis a business of selling real estate. This was an isolated
incurred in carrying on a trade or business transaction: the sale of the farm was made in obedience to the request of the
In this case, the evidence does not show such link between the expenses and Government whose policy was to allocate lands to the landless. The
the business of Roxas y Cia Government's duty was to pay the agreed price of the farm lands after it had
b. Contributions to the Pasay police and fire department and other police persuaded the petitioner to sell its hacienda. But the Government lacked funds
departments as Christmas funds and Roxas y Cia, obligingly shouldered the government's burden. It does not
Contributions to the Christmas funds are not deductible for the reason that conform to one's sense of justice for the Government to persuade the taxpayer
the Christmas funds were not spent for public purposes but as Christmas gifts to lend it a helping hand and later to penalize him for doing so. The sale,
to the families of the members of said entities therefore, made by Roxas y Cia, to the farmers of its farmlands does not make
Under Section 39(h), a contribution to a government entity is deductible the company a real estate dealer, and the lands sold to the farmers are capital
when used exclusively for public purposes assets. The gain derived therefrom is capital gain, and is taxable only to the
As to the contribution to the Manila Police trust fund, such is an allowable extent of 50%, not 100%.
deduction for said trust fund belongs to the Manila Police, a government
entity, intended to be used exclusively for its public functions.
3. ID.; TAX DEDUCTIONS; CLAIMS DISALLOWED. Contributions to the
Contributions to the Philippines Herald's fund for Manila's neediest families
Christmas funds of the Pasay City Police, Pasay City Firemen and Baguio City
The contributions were not made to the Philippines Herald but to a group of
Police are not deductible because the Christmas funds were not spent for
civic spirited citizens organized by the Philippines Herald solely for charitable
public purposes but as Christmas gifts to the families of members of said
purposes
entities. Section 39 (h) of the Tax Code provides that a contribution to a
There is no question that the members of this group of citizens do not receive
government entity is deductible only when used exclusively for public
profits, for all the funds they raised were for Manila's neediest families. Such a
purposes. The contribution to the chapel of the FEU located in the premises of
group of citizens may be classified as an association organized exclusively for
said school is not deductible because said chapel was not shown to belong to
charitable purposes mentioned in Section 30(h) of the Tax Code
the Catholic church or any religious organization; on the contrary it was found

121
TAXATION II ATTY. ACOSTA -
CAJUSTIN
At the conclusion of the Second World War, the tenants who have all been tilling
to belong to the FEU contributions to which are not deductible under sec. 30 the lands in Nasugbu for generations expressed their desire to purchase from
(h) of the Tax Code because the net income of said university inures to the Roxas y Cia. the parcels which they actually occupied. For its part, the
benefit of its stockholders. Government, in consonance with the constitutional mandate to acquire big
landed estates and apportion them among landless tenants-farmers, persuaded
the Roxas brothers to part with their landholdings. Conferences were held with
4. ID.; ID.; CLAIMS ALLOWED. Contributions to the Philippines Herald's the farmers in the early part of 1948 and finally the Roxas brothers agreed to sell
fund for Manila's neediest families are allowable deductions because such 13,500 hectares to the Government for distribution to actual occupants for a price
contributions were not made to the Philippines Herald but to a group of civic of P2,079,048.47 plus P300,000.00 for survey and subdivision expenses.
spirited citizens organized by the Herald solely for charitable purposes and
said citizens do not receive profits. Such group of citizens may, therefore, be
classified as an association exclusively organized for charitable purpose It turned out however that the Government did not have funds to cover the
mentioned in sec. 30(h) of the Tax Code. Contributions to the Manila y Police purchase price, and so a special arrangement was made for the Rehabilitation
Trust Fund constitute allowable deductions because the trust fund belongs to Finance Corporation to advance to Roxas y Cia. the amount of P1,500,000.00 as
the Manila Police, a government entity intended to be used exclusively for its loan. Collateral for such loan were the lands proposed to be sold to the farmers.
public functions. Under the arrangement, Roxas y Cia. allowed the farmers to buy the lands for the
same price but by installment, and contracted with the Rehabilitation Finance
Corporation to pay its loan from the proceeds of the yearly amortizations paid by
the farmers.

Don Pedro Roxas and Dona Carmen Ayala, Spanish subjects, transmitted to their In 1953 and 1955 Roxas y Cia. derived from said installment payments a net gain
grandchildren by hereditary succession the following properties: of P42,480.83 and P29,500.71. Fifty percent of said net gain was reported for
income tax purposes as gain on the sale of capital asset held for more than one
(1) Agricultural lands with a total area of 19,000 hectares, situated in the year pursuant to Section 34 of the Tax Code.
municipality of Nasugbu, Batangas province;
RESIDENTIAL HOUSE
(2) A residential house and lot located at Wright St., Malate, Manila; and
During their bachelor days the Roxas brothers lived in the residential house at
(3) Shares of stocks in different corporations. Wright St., Malate, Manila, which they inherited from their grandparents. After
Antonio and Eduardo got married, they resided somewhere else leaving only Jose
in the old house. In fairness to his brothers, Jose paid to Roxas y Cia. rentals for
To manage the above-mentioned properties, said children, namely, Antonio the house in the sum of P8,000.00 a year.
Roxas, Eduardo Roxas and Jose Roxas, formed a partnership called Roxas y
Compania.
ASSESSMENTS
AGRICULTURAL LANDS
On June 17, 1958, the Commissioner of Internal Revenue demanded from Roxas y
Cia the payment of real estate dealer's tax for 1952 in the amount of P150.00 plus
P10.00 compromise penalty for late payment, and P150.00 tax for dealers of
securities for 1952 plus P10.00 compromise penalty for late payment. The
assessment for real estate dealer's tax was based on the fact that Roxas y Cia.
received house rentals from Jose Roxas in the amount of P8,000.00. Pursuant to
Sec. 194 of the Tax Code, an owner of a real estate who derives a yearly rental
income therefrom in the amount of P3,000.00 or more is considered a real estate
dealer and is liable to pay the corresponding fixed tax.

122
TAXATION II ATTY. ACOSTA -
CAJUSTIN
The Commissioner of Internal Revenue justified his demand for the fixed tax on
Contributions to Contribution to
dealers of securities against Roxas y Cia., on the fact that said partnership made
Our Lady of Fatima Chapel, FEU 50.00
profits from the purchase and sale of securities.
ANTONIO ROXAS:
In the same assessment, the Commissioner assessed deficiency income taxes
against the Roxas Brothers for the years 1953 and 1955, as follows: 1953

1953 1955 Contributions to


Antonio Roxas P7,010.00 P5,813.00 Pasay City Firemen Christmas Fund 25.00
Eduardo Roxas 7,281.00 5,828.00
Jose Roxas 6,323.00 5,588.00 Pasay City Police Dept. X'mas fund 50.00
The deficiency income taxes resulted from the inclusion as income of Roxas y Cia.
of the unreported 50% of the net profits for 1953 and 1955 derived from the sale 1955
of the Nasugbu farm lands to the tenants, and the disallowance of deductions
from gross income of various business expenses and contributions claimed by Contributions to
Roxas y Cia. and the Roxas brothers. For the reason that Roxas y Cia. subdivided
its Nasugbu farm lands and sold them to the farmers on installment, the Baguio City Police Christmas fund 25.00
Commissioner considered the partnership as engaged in the business of real
estate, hence, 100% of the profits derived therefrom was taxed. Pasay City Firemen Christmas fund 25.00

Pasay City Police Christmas fund 50.00


The following deductions were disallowed:
EDUARDO ROXAS:
ROXAS Y CIA.:
1953
1953
Contributions to
Tickets for Banquet in honor of
P 40.00 Hijas de Jesus' Retiro de Manresa 450.00
S. Osmea

Gifts of San Miguel beer 28.00 Philippines Herald's fund for Manila's neediest
families 100.00
Contributions to
1955
Philippine Air Force Chapel 100.00
Contributions to Philippines
Manila Police Trust Fund 150.00 Herald's fund for Manila's
neediest families 120.00
Philippines Herald's fund for Manila's neediest
families 100.00 JOSE ROXAS:

1955 1955

Contributions to Philippines 120.00

123
TAXATION II ATTY. ACOSTA -
CAJUSTIN
4. (a) La explotacion de fincas urbanes pertenecientes a la misma o que
Herald's fund for Manila's
pueden pertenecer a ella en el futuro, alquilandoles por los plazos y
neediest families
demas condiciones, estime convenientes y vendiendo aquellas que a
The Roxas brothers protested the assessment but inasmuch as said protest was juicio de sus gerentes no deben conservarse;
denied, they instituted an appeal in the Court of Tax Appeals on January 9, 1961.
The Tax Court heard the appeal and rendered judgment on July 31, 1965 The above-quoted purpose notwithstanding, the proposition of the Commissioner
sustaining the assessment except the demand for the payment of the fixed tax on of Internal Revenue cannot be favorably accepted by Us in this isolated
dealer of securities and the disallowance of the deductions for contributions to the transaction with its peculiar circumstances in spite of the fact that there were
Philippine Air Force Chapel and Hijas de Jesus' Retiro de Manresa. The Tax hundreds of vendees. Although they paid for their respective holdings in
Court's judgment reads: installment for a period of ten years, it would nevertheless not make the vendor
Roxas y Cia. a real estate dealer during the ten-year amortization period.
WHEREFORE, the decision appealed from is hereby affirmed with
respect to petitioners Antonio Roxas, Eduardo Roxas, and Jose Roxas It should be borne in mind that the sale of the Nasugbu farm lands to the very
who are hereby ordered to pay the respondent Commissioner of Internal farmers who tilled them for generations was not only in consonance with, but
Revenue the amounts of P12,808.00, P12,887.00 and P11,857.00, more in obedience to the request and pursuant to the policy of our Government to
respectively, as deficiency income taxes for the years 1953 and 1955, plus allocate lands to the landless. It was the bounden duty of the Government to pay
5% surcharge and 1% monthly interest as provided for in Sec. 51(a) of the the agreed compensation after it had persuaded Roxas y Cia. to sell its haciendas,
Revenue Code; and modified with respect to the partnership Roxas y Cia. and to subsequently subdivide them among the farmers at very reasonable terms
in the sense that it should pay only P150.00, as real estate dealer's tax. and prices. However, the Government could not comply with its duty for lack of
With costs against petitioners. funds. Obligingly, Roxas y Cia. shouldered the Government's burden, went out of
its way and sold lands directly to the farmers in the same way and under the same
Not satisfied, Roxas y Cia. and the Roxas brothers appealed to this Court. The terms as would have been the case had the Government done it itself. For this
Commissioner of Internal Revenue did not appeal. magnanimous act, the municipal council of Nasugbu passed a resolution
expressing the people's gratitude.
The issues:
The power of taxation is sometimes called also the power to destroy. Therefore it
(1) Is the gain derived from the sale of the Nasugbu farm lands an should be exercised with caution to minimize injury to the proprietary rights of a
ordinary gain, hence 100% taxable? taxpayer. It must be exercised fairly, equally and uniformly, lest the tax
collector kill the "hen that lays the golden egg". And, in order to maintain
the general public's trust and confidence in the Government this power must be
(2) Are the deductions for business expenses and contributions used justly and not treacherously. It does not conform with Our sense of justice in
deductible? the instant case for the Government to persuade the taxpayer to lend it a helping
hand and later on to penalize him for duly answering the urgent call.
(3) Is Roxas y Cia. liable for the payment of the fixed tax on real estate
dealers? In fine, Roxas y Cia. cannot be considered a real estate dealer for the sale in
question. Hence, pursuant to Section 34 of the Tax Code the lands sold to the
The Commissioner of Internal Revenue contends that Roxas y Cia. could be farmers are capital assets, and the gain derived from the sale thereof is capital
considered a real estate dealer because it engaged in the business of selling real gain, taxable only to the extent of 50%.
estate. The business activity alluded to was the act of subdividing the Nasugbu
farm lands and selling them to the farmers-occupants on installment. To bolster DISALLOWED DEDUCTIONS
his stand on the point, he cites one of the purposes of Roxas y Cia. as contained in
its articles of partnership, quoted below:
Roxas y Cia. deducted from its gross income the amount of P40.00 for tickets to a
banquet given in honor of Sergio Osmena and P28.00 for San Miguel beer given

124
TAXATION II ATTY. ACOSTA -
CAJUSTIN
as gifts to various persons. The deduction were claimed as representation Lastly, Roxas y Cia. questions the imposition of the real estate dealer's fixed tax
expenses. Representation expenses are deductible from gross income as upon it, because although it earned a rental income of P8,000.00 per annum in
expenditures incurred in carrying on a trade or business under Section 30(a) of 1952, said rental income came from Jose Roxas, one of the partners. Section 194
the Tax Code provided the taxpayer proves that they are reasonable in amount, of the Tax Code, in considering as real estate dealers owners of real estate
ordinary and necessary, and incurred in connection with his business. In the case receiving rentals of at least P3,000.00 a year, does not provide any qualification
at bar, the evidence does not show such link between the expenses and the as to the persons paying the rentals. The law, which states: 1wph1.t
business of Roxas y Cia. The findings of the Court of Tax Appeals must therefore
be sustained. . . . "Real estate dealer" includes any person engaged in the business of
buying, selling, exchanging, leasing or renting property on his own
The petitioners also claim deductions for contributions to the Pasay City Police, account as principal and holding himself out as a full or part-time dealer
Pasay City Firemen, and Baguio City Police Christmas funds, Manila Police Trust in real estate or as an owner of rental property or properties rented or
Fund, Philippines Herald's fund for Manila's neediest families and Our Lady of offered to rent for an aggregate amount of three thousand pesos or
Fatima chapel at Far Eastern University. more a year: . . . (Emphasis supplied) .

The contributions to the Christmas funds of the Pasay City Police, Pasay City is too clear and explicit to admit construction. The findings of the Court of Tax
Firemen and Baguio City Police are not deductible for the reason that the Appeals or, this point is sustained.1wph1.t
Christmas funds were not spent for public purposes but as Christmas gifts to the
families of the members of said entities. Under Section 39(h), a contribution to a To Summarize, no deficiency income tax is due for 1953 from Antonio Roxas,
government entity is deductible when used exclusively for public purposes. For Eduardo Roxas and Jose Roxas. For 1955 they are liable to pay deficiency income
this reason, the disallowance must be sustained. On the other hand, the tax in the sum of P109.00, P91.00 and P49.00, respectively, computed as
contribution to the Manila Police trust fund is an allowable deduction for said follows: *
trust fund belongs to the Manila Police, a government entity, intended to be used
exclusively for its public functions.
ANTONIO ROXAS
The contributions to the Philippines Herald's fund for Manila's neediest families
were disallowed on the ground that the Philippines Herald is not a corporation or Net income per return P315,476.59
an association contemplated in Section 30 (h) of the Tax Code. It should be noted
however that the contributions were not made to the Philippines Herald but to a Add: 1/3 share, profits in
P 153,249.15
group of civic spirited citizens organized by the Philippines Herald solely for Roxas y Cia.
charitable purposes. There is no question that the members of this group of
citizens do not receive profits, for all the funds they raised were for Manila's Less amount declared 146,135.46
neediest families. Such a group of citizens may be classified as an association
organized exclusively for charitable purposes mentioned in Section 30(h) of the
Tax Code. Amount understated P 7,113.69

Rightly, the Commissioner of Internal Revenue disallowed the contribution to Contributions disallowed 115.00
Our Lady of Fatima chapel at the Far Eastern University on the ground that the
said university gives dividends to its stockholders. Located within the premises of
the university, the chapel in question has not been shown to belong to the P 7,228.69
Catholic Church or any religious organization. On the other hand, the lower court
found that it belongs to the Far Eastern University, contributions to which are not Less 1/3 share of 7,042.02 186.67
deductible under Section 30(h) of the Tax Code for the reason that the net income contributions amounting to
of said university injures to the benefit of its stockholders. The disallowance P21,126.06 disallowed from
should be sustained.

125
TAXATION II ATTY. ACOSTA -
CAJUSTIN

partnership but allowed to


partners

Net income per review P315,663.26

Less: Exemptions 4,200.00

Net taxable income P311,463.26

Tax due 154,169.00

Tax paid 154,060.00

Deficiency P 109.00
==========

EDUARDO ROXAS

Net income per return P 304,166.92

Add: 1/3 share, profits in


P 153,249.15
Roxas y Cia

Less profits declared 146,052.58

Amount understated P 7,196.57

Less 1/3 share in


contributions amounting to
P21,126.06 disallowed from
partnership but allowed to
partners 7,042.02 155.55

Net income per review P304,322.47

Less: Exemptions 4,800.00

126
TAXATION II ATTY. ACOSTA -
CAJUSTIN

127
TAXATION II ATTY. ACOSTA -
CAJUSTIN

===========
Net taxable income P299,592.47 WHEREFORE, the decision appealed from is modified. Roxas y Cia. is hereby
ordered to pay the sum of P150.00 as real estate dealer's fixed tax for 1952, and
Tax Due P147,250.00 Antonio Roxas, Eduardo Roxas and Jose Roxas are ordered to pay the respective
sums of P109.00, P91.00 and P49.00 as their individual deficiency income tax all
Tax paid 147,159.00 corresponding for the year 1955. No costs. So ordered.

Deficiency P91.00
===========

JOSE ROXAS

Net income per return P222,681.76

Add: 1/3 share, profits in


P153,429.15
Roxas y Cia. TOPIC: DEDUCTIONS
Less amount reported 146,135.46 ALLOWANCE AND PERSONAL
AND ADDITIONAL EXPENSES
Amount understated 7,113.69

Less 1/3 share of G.R. No. L-12287 August 7, 1918


contributions disallowed
from partnership but allowed VICENTE MADRIGAL and his wife, SUSANA PATERNO, plaintiffs-
as deductions to partners 7,042.02 71.67 appellants,
vs.
JAMES J. RAFFERTY, Collector of Internal Revenue, and VENANCIO
Net income per review P222,753.43 CONCEPCION, Deputy Collector of Internal Revenue, defendants-
appellees.
Less: Exemption 1,800.00

SYLLABUS
Net income subject to tax P220,953.43

Tax due P102,763.00 1. TAXATION; INCOME TAX; PURPOSES. The Income Tax
Law of the United States in force in the Philippine Islands has selected
Tax paid 102,714.00 income as the test of faculty in taxation. The aim has been to mitigate the
evils arising from the inequalities of wealth by a progressive scheme of
Deficiency taxation, which places the burden on those best able to pay. To carry out
P 49.00 this idea, public considerations have demanded an exemption roughly
equivalent to the minimum of subsistence. With these exceptions, the

128
TAXATION II ATTY. ACOSTA -
CAJUSTIN

Income Tax Law is supposed to reach the earnings of the entire non- 8. ID.; ID.; STATUTORY CONSTRUCTION. The Income Tax
governmental property of the country. Law, being a law of American origin and being peculiarly intricate in its
provisions, the authoritative decision of the official charged with enforcing
2. ID.; ID.; INCOME CONTRACTED WITH CAPITAL AND it has peculiar force for the Philippines. Great weight should be given to
PROPERTY. Income as contrasted with capital or property is to be the the construction placed upon a revenue law, whose meaning is doubtful,
test. The essential difference between capital and income is that capital is by the department charged with its execution
a fund; income is a flow. Capital is wealth, while income is the service of
wealth. "The fact is that property is a tree, income is the fruit; labor is a This appeal calls for consideration of the Income Tax Law, a law of American
tree, income the fruit; capital is a tree, income the fruit." (Waring vs. City origin, with reference to the Civil Code, a law of Spanish origin.
of Savannah [1878], 60 Ga., 93.)
STATEMENT OF THE CASE.
3. ID.; ID.; "INCOME:," DEFINED. Income means profits or
gains. Vicente Madrigal and Susana Paterno were legally married prior to January 1,
1914. The marriage was contracted under the provisions of law concerning
4. ID.; ID.; CONJUGAL PARTNERSHIPS. The decisions of conjugal partnerships (sociedad de gananciales). On February 25, 1915, Vicente
this court in Nable Jose vs. Nable Jose [1916], 16 Off. Gaz., 871, and Madrigal filed sworn declaration on the prescribed form with the Collector of
Manuel and Laxamana vs. Losano [1918], 16 Off. Gaz., 1265, approved Internal Revenue, showing, as his total net income for the year 1914, the sum of
and followed. The provisions of the Civil Code concerning conjugal P296,302.73. Subsequently Madrigal submitted the claim that the said
partnerships have no application to the Income Tax Law. P296,302.73 did not represent his income for the year 1914, but was in fact the
income of the conjugal partnership existing between himself and his wife Susana
5. ID.; ID.; ID. M and P were legally married prior to January Paterno, and that in computing and assessing the additional income tax provided
1, 1914. The marriage was contracted under the provisions concerning by the Act of Congress of October 3, 1913, the income declared by Vicente
conjugal partnerships. The claim is submitted that the income shown on Madrigal should be divided into two equal parts, one-half to be considered the
the form presented for 1914 was in fact the income of the conjugal income of Vicente Madrigal and the other half of Susana Paterno. The general
partnership existing between M and P, and that in computing and question had in the meantime been submitted to the Attorney-General of the
assessing the additional income tax, the income declared by M should be Philippine Islands who in an opinion dated March 17, 1915, held with the
divided into two equal parts, one-half to be considered the income of M petitioner Madrigal. The revenue officers being still unsatisfied, the
and the other half the income of P. Held: That P, the wife of M, has an correspondence together with this opinion was forwarded to Washington for a
inchoate right in the property of her husband M during the life of the decision by the United States Treasury Department. The United States
conjugal partnership, but that P has no absolute right to one-half of the Commissioner of Internal Revenue reversed the opinion of the Attorney-General,
income of the conjugal partnership. and thus decided against the claim of Madrigal.

6. ID.; ID.; ID. The higher schedules of the additional tax After payment under protest, and after the protest of Madrigal had been decided
provided by the Income Tax Law directed at the incomes of the wealthy adversely by the Collector of Internal Revenue, action was begun by Vicente
may not be partially defeated by reliance on provisions in our Civil Code Madrigal and his wife Susana Paterno in the Court of First Instance of the city of
dealing with the conjugal partnership. The aims and purposes of the Manila against Collector of Internal Revenue and the Deputy Collector of Internal
Income Tax Law must be given effect. Revenue for the recovery of the sum of P3,786.08, alleged to have been
wrongfully and illegally collected by the defendants from the plaintiff, Vicente
Madrigal, under the provisions of the Act of Congress known as the Income Tax
7. ID.; ID.; ID. The Income Tax Law does not look on the Law. The burden of the complaint was that if the income tax for the year 1914 had
spouses as individual partners in an ordinary partnership. been correctly and lawfully computed there would have been due payable by each
of the plaintiffs the sum of P2,921.09, which taken together amounts of a total of

129
TAXATION II ATTY. ACOSTA -
CAJUSTIN
P5,842.18 instead of P9,668.21, erroneously and unlawfully collected from the From the point of view of test of faculty in taxation, no less than five answers have
plaintiff Vicente Madrigal, with the result that plaintiff Madrigal has paid as been given the course of history. The final stage has been the selection of income
income tax for the year 1914, P3,786.08, in excess of the sum lawfully due and as the norm of taxation. (See Seligman, "The Income Tax," Introduction.) The
payable. Income Tax Law of the United States, extended to the Philippine Islands, is the
result of an effect on the part of the legislators to put into statutory form this
The answer of the defendants, together with an analysis of the tax declaration, the canon of taxation and of social reform. The aim has been to mitigate the evils
pleadings, and the stipulation, sets forth the basis of defendants' stand in the arising from inequalities of wealth by a progressive scheme of taxation, which
following way: The income of Vicente Madrigal and his wife Susana Paterno of the places the burden on those best able to pay. To carry out this idea, public
year 1914 was made up of three items: (1) P362,407.67, the profits made by considerations have demanded an exemption roughly equivalent to the minimum
Vicente Madrigal in his coal and shipping business; (2) P4,086.50, the profits of subsistence. With these exceptions, the income tax is supposed to reach the
made by Susana Paterno in her embroidery business; (3) P16,687.80, the profits earnings of the entire non-governmental property of the country. Such is the
made by Vicente Madrigal in a pawnshop company. The sum of these three items background of the Income Tax Law.
is P383,181.97, the gross income of Vicente Madrigal and Susana Paterno for the
year 1914. General deductions were claimed and allowed in the sum of Income as contrasted with capital or property is to be the test. The essential
P86,879.24. The resulting net income was P296,302.73. For the purpose of difference between capital and income is that capital is a fund; income is a flow. A
assessing the normal tax of one per cent on the net income there were allowed as fund of property existing at an instant of time is called capital. A flow of services
specific deductions the following: (1) P16,687.80, the tax upon which was to be rendered by that capital by the payment of money from it or any other benefit
paid at source, and (2) P8,000, the specific exemption granted to Vicente rendered by a fund of capital in relation to such fund through a period of time is
Madrigal and Susana Paterno, husband and wife. The remainder, P271,614.93 was called an income. Capital is wealth, while income is the service of wealth.
the sum upon which the normal tax of one per cent was assessed. The normal tax (See Fisher, "The Nature of Capital and Income.") The Supreme Court of Georgia
thus arrived at was P2,716.15. expresses the thought in the following figurative language: "The fact is that
property is a tree, income is the fruit; labor is a tree, income the fruit; capital is a
The dispute between the plaintiffs and the defendants concerned the additional tree, income the fruit." (Waring vs. City of Savannah [1878], 60 Ga., 93.) A tax on
tax provided for in the Income Tax Law. The trial court in an exhausted decision income is not a tax on property. "Income," as here used, can be defined as "profits
found in favor of defendants, without costs. or gains." (London County Council vs. Attorney-General [1901], A. C., 26; 70 L. J.
K. B. N. S., 77; 83 L. T. N. S., 605; 49 Week. Rep., 686; 4 Tax Cas., 265. See
further Foster's Income Tax, second edition [1915], Chapter IV; Black on Income
ISSUES. Taxes, second edition [1915], Chapter VIII; Gibbons vs. Mahon [1890], 136 U.S.,
549; and Towne vs. Eisner, decided by the United States Supreme Court, January
The contentions of plaintiffs and appellants having to do solely with the 7, 1918.)
additional income tax, is that is should be divided into two equal parts, because of
the conjugal partnership existing between them. The learned argument of counsel A regulation of the United States Treasury Department relative to returns by the
is mostly based upon the provisions of the Civil Code establishing the sociedad de husband and wife not living apart, contains the following:
gananciales. The counter contentions of appellees are that the taxes imposed by
the Income Tax Law are as the name implies taxes upon income tax and not upon
capital and property; that the fact that Madrigal was a married man, and his The husband, as the head and legal representative of the household and general
marriage contracted under the provisions governing the conjugal partnership, has custodian of its income, should make and render the return of the aggregate
no bearing on income considered as income, and that the distinction must be income of himself and wife, and for the purpose of levying the income tax it is
drawn between the ordinary form of commercial partnership and the conjugal assumed that he can ascertain the total amount of said income. If a wife has a
partnership of spouses resulting from the relation of marriage. separate estate managed by herself as her own separate property, and receives an
income of more than $3,000, she may make return of her own income, and if the
husband has other net income, making the aggregate of both incomes more than
DECISION. $4,000, the wife's return should be attached to the return of her husband, or his
income should be included in her return, in order that a deduction of $4,000 may
be made from the aggregate of both incomes. The tax in such case, however, will

130
TAXATION II ATTY. ACOSTA -
CAJUSTIN
be imposed only upon so much of the aggregate income of both shall exceed The decision of the latter overruling the opinion of the Attorney-General is as
$4,000. If either husband or wife separately has an income equal to or in excess follows:
of $3,000, a return of annual net income is required under the law, and such
return must include the income of both, and in such case the return must be made TREASURY
even though the combined income of both be less than $4,000. If the aggregate DEPARTMENT, Washingt
net income of both exceeds $4,000, an annual return of their combined incomes on.
must be made in the manner stated, although neither one separately has an
income of $3,000 per annum. They are jointly and separately liable for such
return and for the payment of the tax. The single or married status of the person Income Tax.
claiming the specific exemption shall be determined as one of the time of claiming
such exemption which return is made, otherwise the status at the close of the FRANK MCINTYRE,
year." Chief, Bureau of Insular Affairs, War Department,
Washington, D. C.
With these general observations relative to the Income Tax Law in force in the
Philippine Islands, we turn for a moment to consider the provisions of the Civil SIR: This office is in receipt of your letter of June 22, 1915, transmitting
Code dealing with the conjugal partnership. Recently in two elaborate decisions in copy of correspondence "from the Philippine authorities relative to the
which a long line of Spanish authorities were cited, this court in speaking of the method of submission of income tax returns by marred person."
conjugal partnership, decided that "prior to the liquidation the interest of the wife
and in case of her death, of her heirs, is an interest inchoate, a mere expectancy,
You advise that "The Governor-General, in forwarding the papers to the
which constitutes neither a legal nor an equitable estate, and does not ripen into
Bureau, advises that the Insular Auditor has been authorized to suspend
title until there appears that there are assets in the community as a result of the
action on the warrants in question until an authoritative decision on the
liquidation and settlement." (Nable Jose vs. Nable Jose [1916], 15 Off. Gaz., 871;
points raised can be secured from the Treasury Department."
Manuel and Laxamana vs. Losano [1918], 16 Off. Gaz., 1265.)

From the correspondence it appears that Gregorio Araneta, married and


Susana Paterno, wife of Vicente Madrigal, has an inchoate right in the property of
living with his wife, had an income of an amount sufficient to require the
her husband Vicente Madrigal during the life of the conjugal partnership. She has
imposition of the net income was properly computed and then both
an interest in the ultimate property rights and in the ultimate ownership of
income and deductions and the specific exemption were divided in half
property acquired as income after such income has become capital. Susana
and two returns made, one return for each half in the names respectively
Paterno has no absolute right to one-half the income of the conjugal partnership.
of the husband and wife, so that under the returns as filed there would
Not being seized of a separate estate, Susana Paterno cannot make a separate
be an escape from the additional tax; that Araneta claims the returns are
return in order to receive the benefit of the exemption which would arise by
correct on the ground under the Philippine law his wife is entitled to half
reason of the additional tax. As she has no estate and income, actually and legally
of his earnings; that Araneta has dominion over the income and under
vested in her and entirely distinct from her husband's property, the income
the Philippine law, the right to determine its use and disposition; that in
cannot properly be considered the separate income of the wife for the purposes of
this case the wife has no "separate estate" within the contemplation of
the additional tax. Moreover, the Income Tax Law does not look on the spouses as
the Act of October 3, 1913, levying an income tax.
individual partners in an ordinary partnership. The husband and wife are only
entitled to the exemption of P8,000 specifically granted by the law. The higher
schedules of the additional tax directed at the incomes of the wealthy may not be It appears further from the correspondence that upon the foregoing
partially defeated by reliance on provisions in our Civil Code dealing with the explanation, tax was assessed against the entire net income against
conjugal partnership and having no application to the Income Tax Law. The aims Gregorio Araneta; that the tax was paid and an application for refund
and purposes of the Income Tax Law must be given effect. made, and that the application for refund was rejected, whereupon the
matter was submitted to the Attorney-General of the Islands who holds
that the returns were correctly rendered, and that the refund should be
The point we are discussing has heretofore been considered by the Attorney-
allowed; and thereupon the question at issue is submitted through the
General of the Philippine Islands and the United States Treasury Department.

131
TAXATION II ATTY. ACOSTA -
CAJUSTIN
Governor-General of the Islands and Bureau of Insular Affairs for the
advisory opinion of this office. DAVID A. GATES.
Acting Commissioner.
By paragraph M of the statute, its provisions are extended to the
Philippine Islands, to be administered as in the United States but by the In connection with the decision above quoted, it is well to recall a few basic ideas.
appropriate internal-revenue officers of the Philippine Government. You The Income Tax Law was drafted by the Congress of the United States and has
are therefore advised that upon the facts as stated, this office holds that been by the Congress extended to the Philippine Islands. Being thus a law of
for the Federal Income Tax (Act of October 3, 1913), the entire net American origin and being peculiarly intricate in its provisions, the authoritative
income in this case was taxable to Gregorio Araneta, both for the normal decision of the official who is charged with enforcing it has peculiar force for the
and additional tax, and that the application for refund was properly Philippines. It has come to be a well-settled rule that great weight should be given
rejected. to the construction placed upon a revenue law, whose meaning is doubtful, by the
department charged with its execution. (U.S. vs. Cerecedo Hermanos y Cia.
The separate estate of a married woman within the contemplation of the [1907], 209 U.S., 338; In re Allen [1903], 2 Phil., 630; Government of the
Income Tax Law is that which belongs to her solely and separate and Philippine Islands vs. Municipality of Binalonan, and Roman Catholic Bishop of
apart from her husband, and over which her husband has no right in Nueva Segovia [1915], 32 Phil., 634.) We conclude that the judgment should be as
equity. It may consist of lands or chattels. it is hereby affirmed with costs against appellants. So ordered.

The statute and the regulations promulgated in accordance therewith


provide that each person of lawful age (not excused from so doing)
having a net income of $3,000 or over for the taxable year shall make a
return showing the facts; that from the net income so shown there shall
be deducted $3,000 where the person making the return is a single
person, or married and not living with consort, and $1,000 additional
where the person making the return is married and living with consort;
but that where the husband and wife both make returns (they living
together), the amount of deduction from the aggregate of their several
incomes shall not exceed $4,000.

The only occasion for a wife making a return is where she has income
from a sole and separate estate in excess of $3,000, but together they
have an income in excess of $4,000, in which the latter event either the
husband or wife may make the return but not both. In all instances the
income of husband and wife whether from separate estates or not, is
taken as a whole for the purpose of the normal tax. Where the wife has
income from a separate estate makes return made by her husband, while
the incomes are added together for the purpose of the normal tax they
are taken separately for the purpose of the additional tax. In this case,
however, the wife has no separate income within the contemplation of
the Income Tax Law. TOPIC: DEDUCTIONS ITEMS
NOT DEDUCTIBLE
Respectfully,

[G.R. No. L-13325. April 20, 1961.]

132
TAXATION II ATTY. ACOSTA -
CAJUSTIN
SANTIAGO GANCAYCO, petitioner, vs. THE made . . ." (Tax Code, Section 51 [d]). After the expiration of said period,
COLLECTOR OF INTERNAL REVENUE, respondent. income taxes may not be legally and validly collected by distraint and/or levy
(Internal Revenue vs. Avelino, 100 Phil., 327 53 Off. Gaz 546; Collector of
Internal Revenue vs. Zulueta, 100 Phil., 872 53 Off. Gaz., 6532; Sambrano vs.
Gancyaco files his income tax return for the year 1949. Respondent issued a Court of Tax Appeals 101 Phil., 1; 53 Off. Gaz., [15] 4839).
warrant of distraint and levy against the properties of Gancayco for the
2. ID.; ID.; WHEN JUDICIAL ACTION MAY BE RESORTED TO. The
satisfaction of his deficiency income tax liability, and accordingly, the
"judicial action" mentioned in the Tax Code may be resorted to within five (5)
municipal treasurer issued a notice of sale of said property at public auction.
years from the date return has been filed, if there has been no assessment, or
Gancayco filed a petition to cancel the sale and direct that the same be re-
within five (5) years from the date of the assessment made within the statutory
advertised at a future date
period, or within the period agreed upon, in writing, by the Collector of
Internal Revenue, and the taxpayer, before the expiration of said five-year
ISSUE: Whether the sum of PhP 16,860.31 is due from Gancayco as deficiency
period, or within such extension of said stipulated period as may have been
income tax for 1949 hinges on the validity of his claim for deduction:
agreed upon, in writing, made before the expiration of the period previously
a) farming expense PhP 27,459
stipulated, except that in the case of a false or fraudulent return with intent to
b) representation expenses PhP 8,933.45
evade tax or of a failure to file a return, the judicial action may be begun at any
time within ten (10) years after the discovery of the falsity, fraud or omission
HELD:
(Section 331 and 332 of the Tax Code).
a)Farming Expenses - no evidence has been presnted as to the nature of the
said farming expenses other than the care statement of petitioner that they 3. COURT OF TAX APPEALS; JURISDICTION;. Republic Act No. 1125 has
were spent for the development and cultivation of his property. vested the Court of Tax Appeals, not only with exclusive appellate jurisdiction
to review decisions of the Commissioner of Internal Revenue in cases involving
No specification has been made as to the actual amount spent for purchase of disputed assessments, but, also, with the authority to decide "all cases
tools, equipment or materials or the amount spent for improvement. involving disputed assessment of internal revenue taxes or customs duties
pending determination before the Court of First Instance" at the time of the
b) Representation expense approval of said Act, on June 16, 1954 (Section 22, Republic Act No. 1125).

PhP 22, 820 is allowed 4. ID.; ID.; WHAT IT IMPLIES. The jurisdiction of the Court of Tax Appeals
PhP 8,993.45 is disallowed because of the absence of recipt, invoices or to decide all cases involving disputed assessments of internal revenue and
vouchers of the expenditures in question, petitioner could not sspecify the customs duties necessarily implies the power to authorize and sanction the
items constituting the same when or on whom or on what they were incurred. collection of the taxes and duties involved in such assessments as may be
upheld by the Court of Tax Appeals. The same now has the authority formerly
vested in Courts of First Instance to hear and decide cases involving disputed
SYLLABUS assessments of internal revenue taxes and customs duties. Inasmuch as those
cases filed with Courts of First Instance constituted judicial actions, such is,
likewise, the nature of the proceedings before the Court of Tax Appeals, insofar
1. TAXATION; INTERNAL REVENUE TAXES; TWO CIVIL REMEDIES FOR as Sections 316 and 332 of the Tax Code are concerned.
COLLECTION. There are two (2) civil remedies for the collection of internal
revenue taxes, namely; (a) by distraint of personal property; and (b) by
"judicial action" (Commonwealth Act 456, Section 316). The first may not be
availed of except within three (3) years after the "return is due or has been

133

Das könnte Ihnen auch gefallen